Sie sind auf Seite 1von 336

CHAPTER 1

MULTIPLE CHOICE ANSWERS AND SOLUTIONS


1-1: a
1-2: b
1-2: c

Jose's capital should be credited for the market value of the computer contributed by
him.
(40,000 + 80,000) 2/3 = 180,000 x 1/3 = 60,000.

1-3: a
Cash
Land
Mortgage payable

P100,000
300,000
( 50,000)

Net assets (Julio, capital)

P350,000

Total Capital (P300,000/60%)


Perla's interest

P500,000
______40%

Perla's capital
Less:Non-cash asset contributed at market value
Land
P 70,000
Building
90,000
Mortgage Payable
( 40,000)

P200,000

Cash contribution

P 80,000

1-4: b

1-5: d

_120,000

- Zero, because under the bonus method, a transfer of capital is only required.

1-6: b
Reyes

Santos

P200,000

Cash
Inventory
Building
Equipment
Mortgage payable

________

P300,000
150,000
400,000
150,000
( 100,000)

Net asset (capital)

P350,000

P750,000

AA

BB

CC

P55,000

1-7: c
Cash
Property at Market Value
Mortgage payable
Equipment at Market Value

P 50,000
_______

P 80,000
( 35,000)
_______

Capital

P 50,000

P 45,000

P55,000

PP

RR

SS

Cash
Computer at Market Value

P 50,000
__25,000

P 80,000
_______

P 25,000
__60,000

Capital

P 75,000

P 80,000

P 85,000

1-8: a

1-9: c
Maria

Nora

Cash
Merchandise inventory
Computer equipment
Liability
Furniture and Fixtures

P 30,000

200,000

P 90,000
160,000
( 60,000)
________

Total contribution

P230,000

P190,000

Total agreed capital (P230,000/40%)


Nora's interest

P575,000
______60%

Nora's agreed capital


Less: investment

P345,000
190,000

Cash to be invested

P155,000

1-10: d
Roy

Sam

Tim

Cash
Office Equipment
Note payable

P140,000

________

P220,000
_( 60,000)

______

Net asset invested

P140,000

P160,000

Agreed capitals, equally (P300,000/3) =

P100,000

1-11: a
Lara

Mitra

Cash
Computer equipment
Note payable

P130,000

________

P200,000
50,000
_( 10,000)

Net asset invested

P130,000

P240,000

Goodwill (P240,000 - P130,000) =

P110,000

1-12: a
Perez
Cash
Office Equipment
Merchandise
Furniture
Notes payable

P 50,000
30,000

Net asset invested

P 80,000

_______

Reyes
P 70,000

110,000
100,000
( 50,000)
P230,000

Bonus Method:
Total capital (net asset invested)

P310,000

Goodwill Method:
Net assets invested
Add: Goodwill (P230,000-P80,000)

P310,000
_150,000

Net capital

P460,000

1-13: b
Required capital of each partner (P300,000/2)
Contributed capital of Ruiz:
Total assets
P105,000
Less Liabilities
__15,000

P150,000
__90,000

Cash to be contributed by Ruiz

P 60,000

1-14: d
Total assets:
Cash
Machinery
Building
Less: Liabilities (Mortgage payable)

P 70,000
75,000
_225,000

P370,000
__90,000

Net assets (equal to Ferrer's capital account)


Divide by Ferrer's P & L share percentage

P280,000
____70%

Total partnership capital

P400,000

Required capital of Cruz (P400,000 X 30%)


Less Assets already contributed:
Cash
P 30,000
Machinery and equipment
25,000
Furniture and fixtures
__10,000

P120,000

Cash to be invested by Cruz

P 55,000

__65,000

1-15: d
Adjusted assets of C Borja
Cash
P 2,500
Accounts Receivable (P10,000-P500)
9,500
Merchandise inventory (P15,000-P3,000) 12,000
Fixtures
__20,000
Asset contributed by D. Arce:
Cash
P 20,000
Merchandise
__10,000

__30,000

Total assets of the partnership

P 74,000

P 44,000

1-16: a
Cash to be invested by Mendez:
Adjusted capital of Lopez (2/3)
Unadjusted capital
Adjustments:
Prepaid expenses
Accrued expenses
Allowance for bad debts (5% X P100,000)

P158,400
17,500
( 5,000)
_( 5,000)

Adjusted capital

P165,900

Total partnership capital (P165,900/2/3)


Multiply by Mendez's interest

P248,850

Mendez's capital
Less Merchandise contributed

P 82,950
__50,000

Cash to be invested by Mendez

P 32,950

Total Capital:
Adjusted capital of Lopez
Contributed capital of Mendez

P165,900
__82,950

Total capital

P248,850

1-17: d
Moran, capital (40%)
Cash

P 15,000

Furniture and Fixtures


Divide by Moran's P & L share percentage

_100,000

Total partnership capital


Multiply by Nakar's P & L share percentage

P115,000
______40%
P287,500
______60%

Required capital of credit of Nakar:


Contributed capital of Nakar:
Merchandise inventory
Land
Building

P172,500
P 45,000
15,000
__65,000

Total assets
Less Liabilities

P125,000
__30,000

Required cash investment by Nakar

P 95,000
P 77,500

1-18: c
Garcia's adjusted capital (see schedule 1)
Divide by Garcia's P & L share percentage

P40,500
______40%

Total partnership capital


Flores' P & L share percentage

P101,250
______60%

Flores' capital credit


Flores' contributed capital (see schedule 2)

P 60,750
__43,500

Additional cash to be invested by Flores

P 17,250

Schedule 1:
Garcia, capital:
Unadjusted balance
Adjustments:
Accumulated depreciation
Allowance for doubtful account

P 49,500
( 4,500)
( 4,500)

Adjusted balance

P 40,500

Schedule 2:
Flores capital:
Unadjusted balance
Adjustments:
Accumulated depreciation
Allowance for doubtful accounts

P 57,000
( 1,500)
( 12,000)

Adjusted balance

P 43,500

1-19: d
Ortiz

Ponce

Total

( 60%)
( 40%)
P133,000
P108,000
P241,000

Unadjusted capital balances


Adjustments:
Allowance for bad debts
Inventories
Accrued expenses

( 2,700)
3,000
_( 2,400)

Adjusted capital balances

P130,900

( 1,800)
2,000
( 1,600)
P106,000

( 4,500)
5,000
( 4,000)
P237,500

Total capital before the formation of the new partnership (see above) P237,500
Divide by the total percentage share of Ortiz and Ponce (50% + 30%) ______80%
Total capital of the partnership before the admission of Roxas
Multiply by Roxas' interest

P296,875
______20%

Cash to be invested by Roxas

P 59,375

1-20: d
Merchandise to be invested by Gomez:
Total partnership capital (P180,000/60%)

P300,000

Gomez's capital (P300,000 X 40%)


Less Cash investment

P120,000
__30,000

Merchandise to be invested by Gomez

P 90,000

Cash to be invested by Jocson:


Adjusted capital of Jocson:
Total assets (at agreed valuations)
Less Accounts payable
Required capital of Jocson

P180,000
__48,000

P132,000
_180,000

Cash to be invested by Jocson

P 48,000

Unadjusted Ell, capital (P75,000 P5,000)


Allowance for doubtful accounts
Accounts payable

P 70,000
( 1,000)
( 4,000)

Adjusted Ell, capital

P 65,000

Total partnership capital (P113,640/1/3)


Less David's capital

P340,920
_113,640

Cortez's capital after adjustments


Adjustments made:
Allowance for doubtful account (2% X P96,000)
Merchandise inventory
Prepaid expenses
Accrued expenses

P227,280

Cortez's capital before adjustments

P211,200

1-21: b

1-22: c

1,920
( 16,000)
( 5,200)
___3,200

1-23: a
Total assets at fair value
Liabilities
Capital balance of Flor

P4,625,000
(1,125,000)
P3,500,000

Total capital of the partnership (P3,500,000 70%)


Eden agreed profit & loss ratio
Eden agreed capital
Eden contributed capital at fair value
Allocated cash to be invested by Eden

P5,000,000
30%
1,500,000
812,000
P 688,000

1-24: c

1-25: c
__Rey
Contributed capital (assets-liabilities)P471,000
Agreed capital (profit and loss ratio) 382,800
Capital transfer (Bonus)
P 88,200

__Sam_ __Tim
__Total_
P291,000 P195,000 P957,000
382,800 191,400 957,000
P(91,800) P 3,600
-

1-26: d
Total agreed capital (P90,000 40%)
Contributed capital of Candy (P126,000+P36,000-P12,000)

P225,000
150,000

Total agreed capital (P90,000 40%)


Candy, agreed capital interest
Agreed capital of Candy
Contributed capital of Candy
Withdrawal

225,000
60%
135,000
150,000
P 15,000

Total agreed capital (210,000 70%)


Noras interest
Agreed capital of Nora
Cash invested
Cash to be invested by Nora

P300,000
30%
P 90,000
42,000
P 48,000

Contributed capital of May (P194,000 - P56,000)


Agreed capital of May (P300,000 x 70%)
Cash to be invested by May

P138,000
210,000
P 72,000

1-27: a

1-28: a

1-29: c
__Alex_
P100,000
92,000
P( 8,000)

Contributed capital
Agreed capital
Capital invested

_Carlos_
P84,000
92,000
P 8,000

__Total__
P184,000
184,000
-

SOLUTIONS TO PROBLEMS
Problem 1 1
1.

a. Books of Pedro Castro will be retained by the partnership


To adjust the assets and liabilities of Pedro Castro.
1. Pedro Castro, Capital............................................................
Merchandise Inventory.....................................................

600

2. Pedro Castro, Capital............................................................


Allowance for Bad Debts.................................................

200

3. Accrued Interest Receivable.................................................


Pedro Castro, Capital........................................................

35

Computation:
P1,000 x 6% x 3/12 =
P2,000 x 6% x 2/12 =

600
200

P15
_20

Total................................P35
4. Pedro Castro, Capital............................................................
Accrued Interest Payable..................................................
(P4,000 x 5% x 6/12 = P100)

100

5. Pedro Castro, Capital............................................................


Accumulated Depreciation Furniture and Fixtures........

800

6. Office Supplies.....................................................................
Pedro Castro, Capital........................................................

400

100

800
400

To record the investment of Jose Bunag.


Cash. ........................................................................................... 15,067.50
Jose Bunag, Capital..............................................................

15,067.50

Computation:
Pedro Castro, Capital
(1)
P600 P31,400
(2)
200
35 (3)
(4)
100
400 (6)
(5) ___800
P1,700 P31,835
P30,135
Jose Bunag, Capital : 1/2 x P30,135 = P15,067.50
b.

A new set of books will be used


Books of Pedro Castro
To adjust the assets and liabilities.
See Requirement (a).
To close the books.
Notes Payable..............................................................................
Accounts Payable........................................................................
Accrued Interest Payable.............................................................
Allowance for Bad Debts............................................................
Accumulated Depreciation Furniture and Fixtures...................
Pedro Castro, Capital...................................................................
Cash......................................................................................
Notes Receivable..................................................................
Accounts Receivable.............................................................
Accrued Interest Receivable.................................................
Merchandise Inventory.........................................................
Office Supplies.....................................................................
Furniture and Fixtures...........................................................

4,000
10,000
100
1,200
1,400
30,135
6,000
3,000
24,000
7,400
400
6,000

New Partnership Books


To record the investment of Pedro Castro.
Cash ..........................................................................................
Notes Receivable.........................................................................
Accounts Receivable...................................................................
Accrued Interest Receivable........................................................
Merchandise Inventory................................................................
Office Supplies............................................................................
Furniture and Fixtures.................................................................
Notes Payable.......................................................................
Accounts Payable..................................................................
Accrued Interest Payable......................................................
Allowance for Bad Debts......................................................

6,000
3,000
24,000
35
7,400
400
6,000
4,000
10,000
100
1,200

Accumulated Depreciation Furniture and Fixtures.............


Pedro Castro, Capital............................................................

1,400
30,135

To record the investment of Jose Bunag.


Cash. ........................................................................................... 15,067.50
Jose Bunag, Capital..............................................................
2.

15,067.50

Castro and Bunag Partnership


Balance Sheet
October 1, 2008
Assets

Cash ..........................................................................................................
Notes receivable.........................................................................................
Accounts receivable................................................................................... P 24,000
Less Allowance for bad debts..................................................................... ___1,200
Accrued interest receivable........................................................................
Merchandise inventory...............................................................................
Office supplies ..........................................................................................
Furniture and fixtures.................................................................................
6,000
Less Accumulated depreciation.................................................................. ___1,400
Total Assets........................................................................................

P21,067.50
3,000.00
22,800.00
35.00
7,400.00
400.00
__4,600.00
P59,302.50

Liabilities and Capital


Notes payable ..........................................................................................
Accounts payable.......................................................................................
Accrued interest payable............................................................................
Pedro Castro, Capital..................................................................................
Jose Bunag, Capital....................................................................................
Total Liabilities and Capital...............................................................

P 4,000.00
10,000.00
100.00
30,135.00
_15,067.50
P59,302.50

Problem 1 2
Contributed Capitals:
Jose:

Capital before adjustment..................................................... P 85,000


Notes Payable.......................................................................
62,000
Undervaluation of inventory.................................................
13,000
Underdepreciation................................................................. ( 25,000)
Pedro: Cash......................................................................................
Pablo: Cash......................................................................................
11,000
Marketable securities............................................................ _57,500
Total contributed capital.............................................................................
Agreed Capitals:
Bonus Method:
Jose (P231,500 x 50%)................................................................ P115,750
Pedro (P231,500 x 25%).............................................................
57,875
Pablo (P231,500 x 25%).............................................................. __57,875
Total. ........................................................................................... P231,500

P 135,000
28,000
___68,500
P 231,500

Partnership Basic Considerations and Formation

Goodwill Method. To have a goodwill, the only possible base is the capital of Pablo. The
computation is:
Contributed
Capital
Jose
Pedro
Pablo
Total

Agreed
Capital

P135,000
28,000
__68,500
P231,500

Goodwill

P137,000 (50%)
68,500 (25%)
__68,500 (25%)
274,000

2,000
40,500
_____
42,500

Total agreed capital (P68,500 25%) = 274,000


Jose, Pedro and Pablo Partnership
Balance Sheet
June 30, 2008
Assets:
Cash
Accounts receivable (net)
Marketable securities
Inventory
Equipment (net)
Goodwill
Total

Bonus Method

Goodwill Method

P 49,000
48,000
57,500
85,000
45,000
______
P284,500

P 49,000
48,000
57,500
85,000
45,000
__42,500
P327,000

P 53,000
115,750
57,875
__57,875
P284,500

P 53,000
137,000
68,500
__68,500
P327,000

Liabilities and Capital:


Accounts payable
Jose, capital (50%)
Pedro, capital (25%)
Pablo, capital (25%)
Total

Problem 1 3
1.

Books of Pepe Basco


To adjust the assets.
a. Pepe Basco, Capital.....................................................................
Estimated Uncollectible Account..........................................

3,200

b. Pepe Basco, Capital.....................................................................


Accumulated Depreciation Furniture and Fixtures.............
To close the books.

500

Estimated Uncollectible Account.......................................................


Accumulated Depreciation Furniture and Fixtures.........................
Accounts Payable..............................................................................
Pepe Basco, Capital...........................................................................
Cash. ...........................................................................................

4,800
1,500
3,600
31,500

3,200
500

400

Accounts Receivable...................................................................
Merchandise Inventory................................................................
Furniture and Fixtures.................................................................
2.

16,000
20,000
5,000

Books of the Partnership


To record the investment of Pepe Basco.
Cash...................................................................................................
Accounts Receivable.........................................................................
Merchandise Inventory......................................................................
Furniture and Fixtures........................................................................
Estimated Uncollectible account.................................................
Accumulated Depreciation Furniture and Fixtures...................
Accounts Payable........................................................................
Pepe Basco, Capital.....................................................................

400
16,000
20,000
5,000
4,800
1,500
3,600
31,500

To record the investment of Carlo Torre.


Cash...................................................................................................
Carlo Torre, Capital.....................................................................
Computation:
Pepe Basco, capital (Base)..........................................................
Divide by Pepe Basco's P & L ratio.............................................
Total agreed capital.....................................................................
Multiply by Carlo Torre's P & L ratio..........................................
Cash to be invested by Carlo Torre..............................................

47,250
47,250
P31,500
___40%
P78,750
___60%
P47,250

Problem 1 4
a.

Roces' books will be used by the partnership


Books of Sales
1. Adjusting Entries

2.

(a) Sales, Capital........................................................................


Accumulated Depreciation Fixtures..............................

3,200

(b) Goodwill...............................................................................
Sales, Capital....................................................................

32,000

3,200
32,000

Closing Entry
Allowance for Bad Debts............................................................
Accumulated Depreciation Delivery Equipment......................
Accumulated Depreciation Fixtures.........................................
Accounts Payable........................................................................
Notes Payable..............................................................................
Accrued Taxes.............................................................................
Sales, Capital...............................................................................
Cash......................................................................................
Accounts Inventory...............................................................
Merchandise Inventory.........................................................
Prepaid Insurance..................................................................
Delivery Equipment..............................................................

12,800
8,000
91,200
64,000
40,000
8,000
224,000
4,800
72,000
192,000
3,200
48,000

Fixtures.................................................................................
Goodwill...............................................................................

96,000
32,000

Books of Roces (Books of the Partnership)


1.

2.

Adjusting Entries
(a) Roces, Capital.............................................................................
Allowance for Bad Debts......................................................

1,600

(b) Accumulated Depreciation Fixtures.........................................


Roces, Capital.......................................................................

16,000

(c) Merchandise Inventory................................................................


Roces, Capital.......................................................................

8,000

(d) Goodwill.....................................................................................
Roces, Capital.......................................................................

40,000

16,000
8,000
40,000

To record the investment of Sales.


Cash...................................................................................................
Accounts Receivable.........................................................................
Merchandise Inventory......................................................................
Prepaid Insurance..............................................................................
Delivery Equipment...........................................................................
Fixtures..............................................................................................
Goodwill............................................................................................
Allowance for Bad Debts............................................................
Accumulated Depreciation Delivery Equipment......................
Accumulated Depreciation Fixtures.........................................
Accounts Payable........................................................................
Notes Payable..............................................................................
Accrued Taxes.............................................................................
Sales, Capital...............................................................................

b.

1,600

4,800
72,000
192,000
3,200
48,000
96,000
32,000
12,800
8,000
91,200
64,000
40,000
8,000
224,000

Sales' books will be used by the partnership


Books of Roces
1. Adjusting Entries
See Requirement (a).
2. Closing Entry
Allowance for Bad Debts............................................................
Accumulated Depreciation Delivery Equipment......................
Accumulated Depreciation Fixtures.........................................
Accounts Payable........................................................................
Accrued Taxes.............................................................................
Roces, Capital.............................................................................
Cash......................................................................................
Accounts Receivable.............................................................
Merchandise Inventory.........................................................
Prepaid Insurance..................................................................

1,600
12,800
64,000
104,000
6,400
224,000
14,400
57,600
132,800
4,800

Delivery Equipment..............................................................
Fixtures.................................................................................
Goodwill...............................................................................

19,200
144,000
40,000

Books of Sales (Books of the Partnership)


1.

Adjusting Entries
See Requirement (a).

2.

To record the investment of Roces.


Cash...................................................................................................
Accounts Receivable.........................................................................
Merchandise Inventory......................................................................
Prepaid Insurance..............................................................................
Delivery Equipment...........................................................................
Fixtures..............................................................................................
Goodwill............................................................................................
Allowance for Bad Debts............................................................
Accumulated Depreciation Delivery Equipment......................
Accumulated Depreciation Fixtures.........................................
Accounts Payable........................................................................
Accrued Taxes.............................................................................
Roces, Capital.............................................................................

c.

14,400
57,600
132,800
4,800
19,200
144,000
40,000
1,600
12,800
64,000
104,000
6,400
224,000

A new set of books will be opened by the partnership


Books of Roces
1. Adjusting Entries
See Requirement (a).
2. Closing Entry
See Requirement (b).
Books of Sales
1. Adjusting Entries
See Requirement (a).
2. Closing Entry
See Requirement (a).
New Partnership Books
To record the investment of Roces and Sales.
Cash...................................................................................................
Accounts Receivable.........................................................................

19,200
129,600

Merchandise Inventory......................................................................
Prepaid Insurance..............................................................................
Delivery Equipment (net)..................................................................
Fixtures (net).....................................................................................
Goodwill ..........................................................................................
Allowance for Bad Debts............................................................
Accounts Payable........................................................................
Notes Payable..............................................................................
Accrued Taxes.............................................................................
Roces, Capital.............................................................................
Sales, Capital...............................................................................

324,800
8,000
46,400
84,800
72,000
14,400
168,000
40,000
14,000
224,000
224,000

Problem 1 5

1.

To close Magno's books.


Allowance for Bad Debts...................................................................
Accounts Payable..............................................................................
Notes Payable....................................................................................
Accrued Interest Payable...................................................................
R. Magno, Capital..............................................................................
Cash. ...........................................................................................
Accounts Receivable...................................................................
Merchandise Inventory................................................................
Equipment...................................................................................
Other Assets................................................................................

2.

5,000
13,000
12,000
3,000
9,000

To adjust the books of Lagman.


Goodwill............................................................................................
Allowance for Bad Debts............................................................
J. Lagman, Capital.......................................................................

3.

1,000
6,000
10,000
300
24,700

8,000
210
7,790

To record the investment of Magno.


Cash...................................................................................................
Accounts Receivable.........................................................................
Merchandise Inventory......................................................................
Equipment.........................................................................................
Other Assets.......................................................................................
Allowance for Bad Debts............................................................
Accounts Payable........................................................................
Notes Payable..............................................................................
Accrued Interest Payable.............................................................
R. Magno, Capital.......................................................................

5,000
13,000
12,000
3,000
9,000
1,000
6,000
10,000
300
24,700

To adjust the investments of the partners.


Cash...................................................................................................
R. Magno, Capital.......................................................................
(P35,000 P24,700 = P10,300)

10,300

J. Lagman, Capital.............................................................................
Cash. ...........................................................................................

35,790

10,300

23,300

Accounts Payable to J. Lagman...................................................


(P63,000 + P7,790 = P70,790 P35,000 = P35,790)
4.

12,490

Lagman and Magno


Balance Sheet
December 31, 2008
Assets
Cash...................................................................................................
Accounts receivable...........................................................................
Less Allowance for bad debts............................................................
Merchandise inventory......................................................................
Equipment.........................................................................................
Other assets........................................................................................
Goodwill ..........................................................................................
Total Assets.................................................................................

P
P34,000
1,210

32,790
21,000
8,000
46,000
___8,000
P115,790

Liabilities and Capital


Accounts payable...............................................................................
Notes payable....................................................................................
Accrued interest payable....................................................................
Accounts payable to J. Lagman.........................................................
J. Lagman, capital..............................................................................
R. Magno, capital..............................................................................
Total Liabilities and Capital........................................................

P 18,000
15,000
300
12,490
35,000
__35,000
P115,790

Problem 1 6
1.

Books of Toledo
Toledo, Capital............................................................................
Allowance for Bad Debts (15% x P32,000)..........................

4,800
4,800

Books of Ureta

2.

Ureta, Capital..............................................................................
Allowance for Bad Debts (10% x P24,000)..........................

2,400

Cash (90% x P12,000).................................................................


Loss from Sale of Office Equipment...........................................
Office Equipment..................................................................

10,800
1,200

Toledo, Capital (1/4 x P1,200)....................................................


Ureta, Capital..............................................................................
Loss from Sale of Office Equipment.....................................

300
900

2,400

12,000

1,200

New Partnership Books


Cash. ...........................................................................................
Accounts Receivable...................................................................
Merchandise................................................................................
Office Equipment........................................................................
Allowance for Bad Debts......................................................

3,200
32,000
40,000
10,000
4,800

Accounts Payable..................................................................
Notes Payable.......................................................................
Toledo, Capital......................................................................
To record the investment of Toledo.

3.

Cash. ...........................................................................................
Accounts Receivable...................................................................
Merchandise................................................................................
Toledo, Capital............................................................................
Allowable for Bad Debts......................................................
Accounts Payable..................................................................
Ureta, Capital........................................................................
To record the investment of Ureta.

22,800
24,000
36,000
300

Cash...................................................................................................
Ureta, Capital..............................................................................
To record Ureta's cash contribution.

3,400

Computation:
Toledo, capital (P68,400 P300)................................................
Divide by Toledo's profit share percentage..................................
Total agreed capital of the partnership.........................................
Multiply by Ureta's profit share percentage.................................
Agreed capital of Ureta...............................................................
Ureta, capital...............................................................................
Cash contribution of Ureta..........................................................
or
Toledo, capital (P68,400 P300)................................................
Less Ureta, capital.......................................................................
Cash contribution of Ureta..........................................................
4.

10,000
2,000
68,400

2,400
16,000
64,700

3,400

P 68,100
____50%
P136,200
____50%
P 68,100
__64,700
P 3,400
P 68,100
__64,700
P 3,400

Toledo and Ureta Partnership


Balance Sheet
July 1, 2008
Assets
Cash...................................................................................................
Accounts receivable...........................................................................
Less Allowance for bad debts............................................................
Merchandise......................................................................................
Office equipment...............................................................................
Total Assets.................................................................................

P 29,400
P56,000
__7,200

48,800
76,000
__10,000
P164,200

Liabilities and Capital


Accounts payable...............................................................................
Notes payable....................................................................................
Toledo, capital...................................................................................
Ureta, capital.....................................................................................
Total Liabilities and Capital........................................................

CHAPTER 1

P 26,000
2,000
68,100
__68,100
P164,200

MULTIPLE CHOICE ANSWERS AND SOLUTIONS


1-1: a
1-2: b
1-2: c

Jose's capital should be credited for the market value of the computer contributed by
him.
(40,000 + 80,000) 2/3 = 180,000 x 1/3 = 60,000.

1-3: a
Cash
Land
Mortgage payable

P100,000
300,000
( 50,000)

Net assets (Julio, capital)

P350,000

Total Capital (P300,000/60%)


Perla's interest

P500,000
______40%

Perla's capital
Less:Non-cash asset contributed at market value
Land
P 70,000
Building
90,000
Mortgage Payable
( 40,000)

P200,000

Cash contribution

P 80,000

1-4: b

1-5: d

_120,000

- Zero, because under the bonus method, a transfer of capital is only required.

1-6: b
Reyes

Santos

P200,000

Cash
Inventory
Building
Equipment
Mortgage payable

________

P300,000
150,000
400,000
150,000
( 100,000)

Net asset (capital)

P350,000

P750,000

AA

BB

CC

P55,000

1-7: c
Cash
Property at Market Value
Mortgage payable
Equipment at Market Value

P 50,000
_______

P 80,000
( 35,000)
_______

Capital

P 50,000

P 45,000

P55,000

PP

RR

SS

Cash
Computer at Market Value

P 50,000
__25,000

P 80,000
_______

P 25,000
__60,000

Capital

P 75,000

P 80,000

P 85,000

Maria

Nora

1-8: a

1-9: c
Cash

P 30,000

Merchandise inventory
Computer equipment
Liability
Furniture and Fixtures
Total contribution

200,000

P 90,000
160,000
( 60,000)
________

P230,000

P190,000

Total agreed capital (P230,000/40%)


Nora's interest

P575,000
______60%

Nora's agreed capital


Less: investment

P345,000
190,000

Cash to be invested

P155,000

1-10: d
Roy

Sam

Tim

Cash
Office Equipment
Note payable

P140,000

________

P220,000
_( 60,000)

______

Net asset invested

P140,000

P160,000

Agreed capitals, equally (P300,000/3) =

P100,000

1-11: a
Lara

Mitra

Cash
Computer equipment
Note payable

P130,000

________

P200,000
50,000
_( 10,000)

Net asset invested

P130,000

P240,000

Goodwill (P240,000 - P130,000) =

P110,000

1-12: a
Perez
Cash
Office Equipment
Merchandise
Furniture
Notes payable

P 50,000
30,000

Net asset invested

P 80,000

_______

Reyes
P 70,000

110,000
100,000
( 50,000)
P230,000

Bonus Method:
Total capital (net asset invested)

P310,000

Goodwill Method:
Net assets invested
Add: Goodwill (P230,000-P80,000)

P310,000
_150,000

Net capital

P460,000

1-13: b
Required capital of each partner (P300,000/2)
Contributed capital of Ruiz:
Total assets
P105,000
Less Liabilities
__15,000

P150,000

Cash to be contributed by Ruiz

P 60,000

__90,000

1-14: d
Total assets:
Cash
Machinery
Building
Less: Liabilities (Mortgage payable)

P 70,000
75,000
_225,000

P370,000
__90,000

Net assets (equal to Ferrer's capital account)


Divide by Ferrer's P & L share percentage

P280,000
____70%

Total partnership capital

P400,000

Required capital of Cruz (P400,000 X 30%)


Less Assets already contributed:
Cash
P 30,000
Machinery and equipment
25,000
Furniture and fixtures
__10,000

P120,000

Cash to be invested by Cruz

P 55,000

__65,000

1-15: d
Adjusted assets of C Borja
Cash
P 2,500
Accounts Receivable (P10,000-P500)
9,500
Merchandise inventory (P15,000-P3,000) 12,000
Fixtures
__20,000
Asset contributed by D. Arce:
Cash
P 20,000
Merchandise
__10,000

__30,000

Total assets of the partnership

P 74,000

P 44,000

1-16: a
Cash to be invested by Mendez:
Adjusted capital of Lopez (2/3)
Unadjusted capital
Adjustments:
Prepaid expenses
Accrued expenses
Allowance for bad debts (5% X P100,000)

P158,400
17,500
( 5,000)
_( 5,000)

Adjusted capital

P165,900

Total partnership capital (P165,900/2/3)


Multiply by Mendez's interest

P248,850

Mendez's capital
Less Merchandise contributed

P 82,950
__50,000

Cash to be invested by Mendez

P 32,950

Total Capital:
Adjusted capital of Lopez
Contributed capital of Mendez

P165,900
__82,950

Total capital

P248,850

1-17: d
Moran, capital (40%)
Cash
Furniture and Fixtures
Divide by Moran's P & L share percentage

P 15,000
_100,000

P115,000
______40%

Total partnership capital


Multiply by Nakar's P & L share percentage

P287,500
______60%

Required capital of credit of Nakar:


Contributed capital of Nakar:
Merchandise inventory
Land
Building

P172,500
P 45,000
15,000
__65,000

Total assets
Less Liabilities

P125,000
__30,000

Required cash investment by Nakar

P 95,000
P 77,500

1-18: c
Garcia's adjusted capital (see schedule 1)
Divide by Garcia's P & L share percentage

P40,500
______40%

Total partnership capital


Flores' P & L share percentage

P101,250
______60%

Flores' capital credit


Flores' contributed capital (see schedule 2)

P 60,750
__43,500

Additional cash to be invested by Flores

P 17,250

Schedule 1:
Garcia, capital:
Unadjusted balance
Adjustments:
Accumulated depreciation
Allowance for doubtful account

P 49,500
( 4,500)
( 4,500)

Adjusted balance

P 40,500

Schedule 2:
Flores capital:
Unadjusted balance
Adjustments:
Accumulated depreciation
Allowance for doubtful accounts

P 57,000
( 1,500)
( 12,000)

Adjusted balance

P 43,500

1-19: d
Ortiz

Ponce

Total

( 60%)
( 40%)
P133,000
P108,000
P241,000

Unadjusted capital balances


Adjustments:
Allowance for bad debts
Inventories
Accrued expenses

( 2,700)
3,000
_( 2,400)

Adjusted capital balances

P130,900

( 1,800)
2,000
( 1,600)
P106,000

( 4,500)
5,000
( 4,000)
P237,500

Total capital before the formation of the new partnership (see above) P237,500
Divide by the total percentage share of Ortiz and Ponce (50% + 30%) ______80%
Total capital of the partnership before the admission of Roxas
Multiply by Roxas' interest

P296,875
______20%

Cash to be invested by Roxas

P 59,375

1-20: d
Merchandise to be invested by Gomez:
Total partnership capital (P180,000/60%)

P300,000

Gomez's capital (P300,000 X 40%)


Less Cash investment

P120,000
__30,000

Merchandise to be invested by Gomez

P 90,000

Cash to be invested by Jocson:


Adjusted capital of Jocson:
Total assets (at agreed valuations)
Less Accounts payable
Required capital of Jocson

P180,000
__48,000

P132,000
_180,000

Cash to be invested by Jocson

P 48,000

Unadjusted Ell, capital (P75,000 P5,000)


Allowance for doubtful accounts
Accounts payable

P 70,000
( 1,000)
( 4,000)

Adjusted Ell, capital

P 65,000

Total partnership capital (P113,640/1/3)


Less David's capital

P340,920
_113,640

Cortez's capital after adjustments


Adjustments made:
Allowance for doubtful account (2% X P96,000)
Merchandise inventory
Prepaid expenses
Accrued expenses

P227,280

Cortez's capital before adjustments

P211,200

1-21: b

1-22: c

1,920
( 16,000)
( 5,200)
___3,200

1-23: a
Total assets at fair value
Liabilities
Capital balance of Flor

P4,625,000
(1,125,000)
P3,500,000

Total capital of the partnership (P3,500,000 70%)


Eden agreed profit & loss ratio
Eden agreed capital
Eden contributed capital at fair value
Allocated cash to be invested by Eden

P5,000,000
30%
1,500,000
812,000
P 688,000

1-24: c

1-25: c
__Rey
Contributed capital (assets-liabilities)P471,000
Agreed capital (profit and loss ratio) 382,800
Capital transfer (Bonus)
P 88,200

__Sam_ __Tim
__Total_
P291,000 P195,000 P957,000
382,800 191,400 957,000
P(91,800) P 3,600
-

1-26: d
Total agreed capital (P90,000 40%)
Contributed capital of Candy (P126,000+P36,000-P12,000)
Total agreed capital (P90,000 40%)
Candy, agreed capital interest

P225,000
150,000
225,000
60%

Agreed capital of Candy


Contributed capital of Candy
Withdrawal

135,000
150,000
P 15,000

Total agreed capital (210,000 70%)


Noras interest
Agreed capital of Nora
Cash invested
Cash to be invested by Nora

P300,000
30%
P 90,000
42,000
P 48,000

Contributed capital of May (P194,000 - P56,000)


Agreed capital of May (P300,000 x 70%)
Cash to be invested by May

P138,000
210,000
P 72,000

1-27: a

1-28: a

1-29: c
__Alex_
P100,000
92,000
P( 8,000)

Contributed capital
Agreed capital
Capital invested

_Carlos_
P84,000
92,000
P 8,000

__Total__
P184,000
184,000
-

SOLUTIONS TO PROBLEMS
Problem 1 1
1.

a. Books of Pedro Castro will be retained by the partnership


To adjust the assets and liabilities of Pedro Castro.
1. Pedro Castro, Capital............................................................
Merchandise Inventory.....................................................

600

2. Pedro Castro, Capital............................................................


Allowance for Bad Debts.................................................

200

3. Accrued Interest Receivable.................................................


Pedro Castro, Capital........................................................

35

Computation:
P1,000 x 6% x 3/12 =
P2,000 x 6% x 2/12 =

600
200

P15
_20

Total................................P35
4. Pedro Castro, Capital............................................................
Accrued Interest Payable..................................................
(P4,000 x 5% x 6/12 = P100)

100

5. Pedro Castro, Capital............................................................


Accumulated Depreciation Furniture and Fixtures........

800

6. Office Supplies.....................................................................
Pedro Castro, Capital........................................................

400

To record the investment of Jose Bunag.

100

800
400

Cash. ........................................................................................... 15,067.50


Jose Bunag, Capital..............................................................

15,067.50

Computation:
Pedro Castro, Capital
(1)
P600 P31,400
(2)
200
35 (3)
(4)
100
400 (6)
(5) ___800
P1,700 P31,835
P30,135
Jose Bunag, Capital : 1/2 x P30,135 = P15,067.50
b.

A new set of books will be used


Books of Pedro Castro
To adjust the assets and liabilities.
See Requirement (a).
To close the books.
Notes Payable..............................................................................
Accounts Payable........................................................................
Accrued Interest Payable.............................................................
Allowance for Bad Debts............................................................
Accumulated Depreciation Furniture and Fixtures...................
Pedro Castro, Capital...................................................................
Cash......................................................................................
Notes Receivable..................................................................
Accounts Receivable.............................................................
Accrued Interest Receivable.................................................
Merchandise Inventory.........................................................
Office Supplies.....................................................................
Furniture and Fixtures...........................................................

4,000
10,000
100
1,200
1,400
30,135
6,000
3,000
24,000
7,400
400
6,000

New Partnership Books


To record the investment of Pedro Castro.
Cash ..........................................................................................
Notes Receivable.........................................................................
Accounts Receivable...................................................................
Accrued Interest Receivable........................................................
Merchandise Inventory................................................................
Office Supplies............................................................................
Furniture and Fixtures.................................................................
Notes Payable.......................................................................
Accounts Payable..................................................................
Accrued Interest Payable......................................................
Allowance for Bad Debts......................................................
Accumulated Depreciation Furniture and Fixtures.............
Pedro Castro, Capital............................................................

6,000
3,000
24,000
35
7,400
400
6,000
4,000
10,000
100
1,200
1,400
30,135

To record the investment of Jose Bunag.


Cash. ........................................................................................... 15,067.50
Jose Bunag, Capital..............................................................
2.

15,067.50

Castro and Bunag Partnership


Balance Sheet
October 1, 2008
Assets

Cash ..........................................................................................................
Notes receivable.........................................................................................
Accounts receivable................................................................................... P 24,000
Less Allowance for bad debts..................................................................... ___1,200
Accrued interest receivable........................................................................
Merchandise inventory...............................................................................
Office supplies ..........................................................................................
Furniture and fixtures.................................................................................
6,000
Less Accumulated depreciation.................................................................. ___1,400
Total Assets........................................................................................

P21,067.50
3,000.00
22,800.00
35.00
7,400.00
400.00
__4,600.00
P59,302.50

Liabilities and Capital


Notes payable ..........................................................................................
Accounts payable.......................................................................................
Accrued interest payable............................................................................
Pedro Castro, Capital..................................................................................
Jose Bunag, Capital....................................................................................
Total Liabilities and Capital...............................................................

P 4,000.00
10,000.00
100.00
30,135.00
_15,067.50
P59,302.50

Problem 1 2
Contributed Capitals:
Jose:

Capital before adjustment..................................................... P 85,000


Notes Payable.......................................................................
62,000
Undervaluation of inventory.................................................
13,000
Underdepreciation................................................................. ( 25,000)
Pedro: Cash......................................................................................
Pablo: Cash......................................................................................
11,000
Marketable securities............................................................ _57,500
Total contributed capital.............................................................................
Agreed Capitals:
Bonus Method:
Jose (P231,500 x 50%)................................................................ P115,750
Pedro (P231,500 x 25%).............................................................
57,875
Pablo (P231,500 x 25%).............................................................. __57,875
Total. ........................................................................................... P231,500
Goodwill Method. To have a goodwill, the only possible base is the capital of Pablo. The
computation is:
Contributed
Capital

Agreed
Capital

Goodwill

P 135,000
28,000
___68,500
P 231,500

Jose
Pedro
Pablo
Total

P135,000
28,000
__68,500
P231,500

P137,000 (50%)
68,500 (25%)
__68,500 (25%)
274,000

2,000
40,500
_____
42,500

Total agreed capital (P68,500 25%) = 274,000


Jose, Pedro and Pablo Partnership
Balance Sheet
June 30, 2008
Assets:
Cash
Accounts receivable (net)
Marketable securities
Inventory
Equipment (net)
Goodwill
Total

Bonus Method

Goodwill Method

P 49,000
48,000
57,500
85,000
45,000
______
P284,500

P 49,000
48,000
57,500
85,000
45,000
__42,500
P327,000

P 53,000
115,750
57,875
__57,875
P284,500

P 53,000
137,000
68,500
__68,500
P327,000

Liabilities and Capital:


Accounts payable
Jose, capital (50%)
Pedro, capital (25%)
Pablo, capital (25%)
Total

Problem 1 3
1.

Books of Pepe Basco


To adjust the assets.
a. Pepe Basco, Capital.....................................................................
Estimated Uncollectible Account..........................................

3,200

b. Pepe Basco, Capital.....................................................................


Accumulated Depreciation Furniture and Fixtures.............

500

3,200
500

To close the books.


Estimated Uncollectible Account.......................................................
Accumulated Depreciation Furniture and Fixtures.........................
Accounts Payable..............................................................................
Pepe Basco, Capital...........................................................................
Cash. ...........................................................................................
Accounts Receivable...................................................................
Merchandise Inventory................................................................
Furniture and Fixtures.................................................................
2.

Books of the Partnership

4,800
1,500
3,600
31,500
400
16,000
20,000
5,000

To record the investment of Pepe Basco.


Cash...................................................................................................
Accounts Receivable.........................................................................
Merchandise Inventory......................................................................
Furniture and Fixtures........................................................................
Estimated Uncollectible account.................................................
Accumulated Depreciation Furniture and Fixtures...................
Accounts Payable........................................................................
Pepe Basco, Capital.....................................................................

400
16,000
20,000
5,000
4,800
1,500
3,600
31,500

To record the investment of Carlo Torre.


Cash...................................................................................................
Carlo Torre, Capital.....................................................................
Computation:
Pepe Basco, capital (Base)..........................................................
Divide by Pepe Basco's P & L ratio.............................................
Total agreed capital.....................................................................
Multiply by Carlo Torre's P & L ratio..........................................
Cash to be invested by Carlo Torre..............................................

47,250
47,250
P31,500
___40%
P78,750
___60%
P47,250

Problem 1 4
a.

Roces' books will be used by the partnership


Books of Sales
1. Adjusting Entries

2.

(a) Sales, Capital........................................................................


Accumulated Depreciation Fixtures..............................

3,200

(b) Goodwill...............................................................................
Sales, Capital....................................................................

32,000
32,000

Closing Entry
Allowance for Bad Debts............................................................
Accumulated Depreciation Delivery Equipment......................
Accumulated Depreciation Fixtures.........................................
Accounts Payable........................................................................
Notes Payable..............................................................................
Accrued Taxes.............................................................................
Sales, Capital...............................................................................
Cash......................................................................................
Accounts Inventory...............................................................
Merchandise Inventory.........................................................
Prepaid Insurance..................................................................
Delivery Equipment..............................................................
Fixtures.................................................................................
Goodwill...............................................................................
Books of Roces (Books of the Partnership)

1.

3,200

Adjusting Entries

12,800
8,000
91,200
64,000
40,000
8,000
224,000
4,800
72,000
192,000
3,200
48,000
96,000
32,000

2.

(a) Roces, Capital.............................................................................


Allowance for Bad Debts......................................................

1,600

(b) Accumulated Depreciation Fixtures.........................................


Roces, Capital.......................................................................

16,000

(c) Merchandise Inventory................................................................


Roces, Capital.......................................................................

8,000

(d) Goodwill.....................................................................................
Roces, Capital.......................................................................

40,000

16,000
8,000
40,000

To record the investment of Sales.


Cash...................................................................................................
Accounts Receivable.........................................................................
Merchandise Inventory......................................................................
Prepaid Insurance..............................................................................
Delivery Equipment...........................................................................
Fixtures..............................................................................................
Goodwill............................................................................................
Allowance for Bad Debts............................................................
Accumulated Depreciation Delivery Equipment......................
Accumulated Depreciation Fixtures.........................................
Accounts Payable........................................................................
Notes Payable..............................................................................
Accrued Taxes.............................................................................
Sales, Capital...............................................................................

b.

1,600

4,800
72,000
192,000
3,200
48,000
96,000
32,000
12,800
8,000
91,200
64,000
40,000
8,000
224,000

Sales' books will be used by the partnership


Books of Roces
1. Adjusting Entries
See Requirement (a).
2. Closing Entry
Allowance for Bad Debts............................................................
Accumulated Depreciation Delivery Equipment......................
Accumulated Depreciation Fixtures.........................................
Accounts Payable........................................................................
Accrued Taxes.............................................................................
Roces, Capital.............................................................................
Cash......................................................................................
Accounts Receivable.............................................................
Merchandise Inventory.........................................................
Prepaid Insurance..................................................................
Delivery Equipment..............................................................
Fixtures.................................................................................
Goodwill...............................................................................
Books of Sales (Books of the Partnership)

1,600
12,800
64,000
104,000
6,400
224,000
14,400
57,600
132,800
4,800
19,200
144,000
40,000

1.

Adjusting Entries
See Requirement (a).

2.

To record the investment of Roces.


Cash...................................................................................................
Accounts Receivable.........................................................................
Merchandise Inventory......................................................................
Prepaid Insurance..............................................................................
Delivery Equipment...........................................................................
Fixtures..............................................................................................
Goodwill............................................................................................
Allowance for Bad Debts............................................................
Accumulated Depreciation Delivery Equipment......................
Accumulated Depreciation Fixtures.........................................
Accounts Payable........................................................................
Accrued Taxes.............................................................................
Roces, Capital.............................................................................

c.

14,400
57,600
132,800
4,800
19,200
144,000
40,000
1,600
12,800
64,000
104,000
6,400
224,000

A new set of books will be opened by the partnership


Books of Roces
1. Adjusting Entries
See Requirement (a).
2. Closing Entry
See Requirement (b).
Books of Sales
1. Adjusting Entries
See Requirement (a).
2. Closing Entry
See Requirement (a).
New Partnership Books
To record the investment of Roces and Sales.
Cash...................................................................................................
Accounts Receivable.........................................................................
Merchandise Inventory......................................................................
Prepaid Insurance..............................................................................
Delivery Equipment (net)..................................................................
Fixtures (net).....................................................................................
Goodwill ..........................................................................................
Allowance for Bad Debts............................................................
Accounts Payable........................................................................

19,200
129,600
324,800
8,000
46,400
84,800
72,000
14,400
168,000

Notes Payable..............................................................................
Accrued Taxes.............................................................................
Roces, Capital.............................................................................
Sales, Capital...............................................................................

40,000
14,000
224,000
224,000

Problem 1 5

1.

To close Magno's books.


Allowance for Bad Debts...................................................................
Accounts Payable..............................................................................
Notes Payable....................................................................................
Accrued Interest Payable...................................................................
R. Magno, Capital..............................................................................
Cash. ...........................................................................................
Accounts Receivable...................................................................
Merchandise Inventory................................................................
Equipment...................................................................................
Other Assets................................................................................

2.

5,000
13,000
12,000
3,000
9,000

To adjust the books of Lagman.


Goodwill............................................................................................
Allowance for Bad Debts............................................................
J. Lagman, Capital.......................................................................

3.

1,000
6,000
10,000
300
24,700

8,000
210
7,790

To record the investment of Magno.


Cash...................................................................................................
Accounts Receivable.........................................................................
Merchandise Inventory......................................................................
Equipment.........................................................................................
Other Assets.......................................................................................
Allowance for Bad Debts............................................................
Accounts Payable........................................................................
Notes Payable..............................................................................
Accrued Interest Payable.............................................................
R. Magno, Capital.......................................................................

5,000
13,000
12,000
3,000
9,000
1,000
6,000
10,000
300
24,700

To adjust the investments of the partners.

4.

Cash...................................................................................................
R. Magno, Capital.......................................................................
(P35,000 P24,700 = P10,300)

10,300

J. Lagman, Capital.............................................................................
Cash. ...........................................................................................
Accounts Payable to J. Lagman...................................................
(P63,000 + P7,790 = P70,790 P35,000 = P35,790)

35,790

Lagman and Magno


Balance Sheet
December 31, 2008

10,300

23,300
12,490

Assets
Cash...................................................................................................
Accounts receivable...........................................................................
Less Allowance for bad debts............................................................
Merchandise inventory......................................................................
Equipment.........................................................................................
Other assets........................................................................................
Goodwill ..........................................................................................
Total Assets.................................................................................

P
P34,000
1,210

32,790
21,000
8,000
46,000
___8,000
P115,790

Liabilities and Capital


Accounts payable...............................................................................
Notes payable....................................................................................
Accrued interest payable....................................................................
Accounts payable to J. Lagman.........................................................
J. Lagman, capital..............................................................................
R. Magno, capital..............................................................................
Total Liabilities and Capital........................................................

P 18,000
15,000
300
12,490
35,000
__35,000
P115,790

Problem 1 6
1.

Books of Toledo
Toledo, Capital............................................................................
Allowance for Bad Debts (15% x P32,000)..........................

4,800
4,800

Books of Ureta

2.

Ureta, Capital..............................................................................
Allowance for Bad Debts (10% x P24,000)..........................

2,400

Cash (90% x P12,000).................................................................


Loss from Sale of Office Equipment...........................................
Office Equipment..................................................................

10,800
1,200

Toledo, Capital (1/4 x P1,200)....................................................


Ureta, Capital..............................................................................
Loss from Sale of Office Equipment.....................................

300
900

2,400

12,000

1,200

New Partnership Books


Cash. ...........................................................................................
Accounts Receivable...................................................................
Merchandise................................................................................
Office Equipment........................................................................
Allowance for Bad Debts......................................................
Accounts Payable..................................................................
Notes Payable.......................................................................
Toledo, Capital......................................................................
To record the investment of Toledo.

3,200
32,000
40,000
10,000

Cash. ...........................................................................................
Accounts Receivable...................................................................

22,800
24,000

4,800
10,000
2,000
68,400

3.

Merchandise................................................................................
Toledo, Capital............................................................................
Allowable for Bad Debts......................................................
Accounts Payable..................................................................
Ureta, Capital........................................................................
To record the investment of Ureta.

36,000
300

Cash...................................................................................................
Ureta, Capital..............................................................................
To record Ureta's cash contribution.

3,400

Computation:
Toledo, capital (P68,400 P300)................................................
Divide by Toledo's profit share percentage..................................
Total agreed capital of the partnership.........................................
Multiply by Ureta's profit share percentage.................................
Agreed capital of Ureta...............................................................
Ureta, capital...............................................................................
Cash contribution of Ureta..........................................................
or
Toledo, capital (P68,400 P300)................................................
Less Ureta, capital.......................................................................
Cash contribution of Ureta..........................................................

2,400
16,000
64,700

3,400

P 68,100
____50%
P136,200
____50%
P 68,100
__64,700
P 3,400
P 68,100
__64,700
P 3,400

Partnership Basic Considerations and Formation

4.

Toledo and Ureta Partnership


Balance Sheet
July 1, 2008
Assets
Cash...................................................................................................
Accounts receivable...........................................................................
Less Allowance for bad debts............................................................
Merchandise......................................................................................
Office equipment...............................................................................
Total Assets.................................................................................

P 29,400
P56,000
__7,200

48,800
76,000
__10,000
P164,200

Liabilities and Capital


Accounts payable...............................................................................
Notes payable....................................................................................
Toledo, capital...................................................................................
Ureta, capital.....................................................................................

P 26,000
2,000
68,100
__68,100

Total Liabilities and Capital........................................................

P164,200

20
Chapter 2

CHAPTER 2
MULTIPLE CHOICE ANSWERS AND SOLUTIONS
2-1: d
Jordan
P120,000

Pippen
P80,000

( 10,000)

( 10,000)

P110,000

P 70,000

JJ
P18,000

KK

LL

P15,000

P 30,000

)
)
P45,000)

( 6,000)

( 6,000)

( 6,000)

P27,000

P 24,000

P39,000

Allan

Michael

Annual salary
P200,000
Balance, equally
( 20,000)
Total
P180,000
2-2: a
Bonus (.20 X P90,000)
P 18,000
Interest
JJ (.15 X P100,000)
KK (.15 X P200,000)
LL (.15 X P300,000)
90,000
Balance, equally
( 18,000)
Total profit share
P 90,000
2-3: a
2-4: a
Interest
Allan - .10 X (P40,000 + 60,000 /2)
Michael - .10 X (P60,000 + 70,000/2)
P 11,500
Balance, equally
__28,000
Total

P 5,000

)
P 6,500)

_14,000

_14,000

P 19,000

P20,500

Greg
P 6,000

Henry
P 4,000

,000
2-5: a
Interest (.10 of average capital)
P 22,000
Salaries
50,000
Balance, equally
(105,000)
Total
3,000)
2-6: b
Average Capital

Fred
P12,000
30,000

20,000

( 35,000)

( 35,000)

( 35,000)

P 7,000

( P29,000)

(P11,000)

Date
January 1
July 1
August 1

Capital
Balance
140,000
180,000
165,000

Months
Unchanged
6
1
5
12

Peso
Months
P 840,000
180,000
__825,000
P1,845,000

Average capital - P1,845,000/12

P153,750

Interest

P 15,375

(P153,750 X 10%)

Partnership Operations

21

2-7: c
Date
January 1
April 1
June 1
September 1

Capital
Balance
P16,000
17,600
19,200
15,200

Months
Unchanged
3
2
3
4
12

Average Capital(P201,600/12) =

Peso
Months
P 48,000
35,200
57,600
__60,800
P201,600

P16,800

2-8: a
Net profit before bonus
Net profit after bonus (P24,000/120%)
Bonus to RJ
Balance (P24,000-P4,000)X3/5
Total profit share

P 24,000
__20,000
4,000
__12,000
P 16,000

2-9: a
AM
P 3,600
7,500
( 7,720)
P 3,380

Total
P 6,800
22,500
( 19,300)
P 10,000

Net income after salary, interest and bonus


Add back: Salary (P10,000 X 12)
P120,000
Interest (P250,000 X .05)
__12,500
Net income after bonus (80%)
Net income before bonus (P600,000/80%)
Paul's bonus

P467,500

Interest
Salaries
Balance, 3:2
Total

LT
P3,200
15,000
(11,580)
P 6,620

2-10: b
_132,500
P600,000
_750,000
P150,000

2-11: b
CC

DD

P14,000

P 8,400

( 1,500)

__2,100

P12,500

P10,500

Salary

EE Total
P 14,000

000
Balance
28,000
Additional profit to DD
______
Total

5,600
(

600)

P 19,000

,000
Net income
Fees Earned
Expenses
Net Income

P90,000
_48,000
P42,000

22
Chapter 2

2-12: c
Interest

LL
P 2,000

MM
P 1,250

NN Total
P 750

8,500

9,500

5,700

3,800

_____

__7,050

_____

P20,000

P14,000

P 4,550

RR

SS

TT

P15,000

(P10,000)

_47,500

_35,625

_11,875

P62,500

P25,625

P11,875

BB

CC

000
Annual Salary
8,500
Additional profit to give LL, P20,000
19,000*
Additional profit to give MM, P14,000
__7,050
Total
550
*(P9,500/50%) = P19,000
2-13: a
Excess (Deficiency)
RR (P80,000 - P95,000)
SS (P50,000 - P40,000)
P 5,000
Balance 4:3:1
__95,000
Total
P100,000
Net Income (200,000 - 100,000) =
2-14: b
AA - 100,000 X 10%
150,000 X 20%
P 40,000
Remainder, 210,000
BB (60,000 X .05)
CC (60,000 X .05)
6,000
Balance, equally
_204,000
Total
P250,000

Total
)
)

P100,000
AA
P 10,000
30,000

Total
)
)

P 3,000

)
P 3,000

__68,000

_68,000

_68,000

P108,000

P71,000

P71,000

2-15: a
AJ
Bonus to CJ
Net profit before bonus
P44,000
Net profit after bonus (P44,000/110%)P40,000

BJ

CJ
P4,000

Total

P4,000
Interest to BJ
1,000
Salaries
22,000
Balance, 4:4:2
_17,000
Total
P44,000

P1,000

P 10,000

12,000

__6,800

_6,800

__3,400

P 16,800

P7,800

P19,400

2-16: c
Total profit share of Pedro
Less: Salary to Pedro
Interest
Share in the balance (40%)

P200,000
P 50,000
__20,000

Net profit after salary and interest (130,000/40%)


Add: Total Salaries
Total Interest
Total Partnership Income

__70,000
P130,000
P325,000

P150,000
__70,000

_220,000
P545,000

Partnership Operations

23

2-17: c
Net income before extraordinary gain and bonus (69,600-12,000)
Net income after bonus (57,600/120%)
Bonus to RR

P 57,600
_48,000
P 9,600

Distribution of Net Income:


JJ
Bonus
Balance, equally
Net profit before extraordinary gain
Extraordinary gain
Total

P 24,000
P 24,000
__4,800
P 28,800

RR
P 9,600
24,000
P 33,600
__7,200
P 40,800

Total
P 9,600
48,000
P 57,600
_12,000
P 69,600

Interest
Annual Salary
Remainder 60:40
Total

Mel
P 20,000
36,000
__60,000
P116,000

Jay
P 12,000

_40,000
P 52,000

Total
P 32,000
36,000
_100,000
P168,000

DV
P 15,000

JE
P 3,750

FR Total
(P 7,500)

( 36,875)

( 22,125)

( 14,750)

2-18: a

2-19: a
Interest on excess (Deficiency)
P 11,250
Remainder 5:3:2
( 73,750)
Total

(P 21,875)

(P 18,375) (P 22,250)

2,500)
2-20: c
Correction of 1998 profit:
Net income per books
Understatement of depreciation

P 19,500
( 2,100)

Overstatement of inventory, December 31


Adjusted net income

( 11,400)
P 6,000
Pete

Rico Total

Distribution of net income per book:


Equally

P 9,750

P 9,750

Distribution of adjusted net income


Equally

( 3,000)

( 3,000)

P 6,750

P 6,750

500
000)
Required Decrease
P 13,500
2-21: a
Tiger
P 64,000

Salaries
P164,000
Interest
54,000
Bonus (P360,000-P54,000)X.25
76,500
Remainder, 30:70
__65,500
Total
P360,000

Woods Total
P100,000

24,000

30,000

76,500

__19,650

__45,850

P184,150

P175,850

Field

P 25,000
33,600

_23,760
P 82,360

Total
P 20,000
25,000
65,600
30,000
__59,400
P200,000

24
Chapter 2

2-22: a
Salaries
Commission
Interest
Bonus, schedule 1
Remainder, 60:40
Total

Holly
P 20,000

32,000
30,000
__35,640
P117,640

Schedule 1
Net income before salary, commission,
interest and bonus
Less: salaries
Net income before bonus
Net income after bonus (P180,000/120%)
Bonus

P200,000
__20,000
P180,000
_150,000
P 30,000

2-23: a
Capital balance, beginning
P1,000,000
Additional investment
300,000
Capital withdrawal

Mike
P600,000

Tyson
P400,000

100,000

200,000

-200,000

( 100,000)

P500,000

P500,000

300,000
Capital balance before profit and loss distribution
P1,000,000

Net income:
Salary

P200,000

P300,000

__60,000

__40,000

P260,000

P340,000

P760,000

P840,000

( 200,000)

( 300,000)

P560,000

P540,000

0,000
Balance, 3:2
__100,000
Total
0,000
Total
P1,600,000
Drawings
00,000)
Capital balance, end
P1,100,000
Average Capital - King:
Date
January 1
April 1

Capital
Balance
P40,000
55,000

Months
Unchanged
3
9
12

Peso
Months
P120,000
_495,000
P615,000

Months
Unchanged
7
5
12

Peso
Months
P700,000
__650,000
P1,350,000

Average capital P615,000/12 = P51,250


Average Capital - Queen:
Date
January 1
April 1

Capital
Balance
P100,000
130,000

Average capital - P1,350,000 / 12 =P112,500


Partnership Operations

25

2-24: d
Distribution of Net Income - Schedule 1

Interest
Bonus, Schedule 2
Salaries
Residual, 50:50
Total

King
P 5,125
12,725
25,000
( 2,050)
P40,800

Queen
P11,250

30,000
_(2,050)
P39,200

Total
P16,375
12,725
55,000
_(4,100)
P80,000

Schedule 2
Net income before allocation
Less: Interest
Net income before bonus
Net income after bonus (P63,625/125%)
Bonus

P80,000
_16,375
P63,625
_50,900
P12,725

Capital Balance December 31:


Capital balance, January 1

King
P40,000

Queen
P100,000

Total
P140,000

Additional investment
Capital balance before profit and
loss distribution
Net income (Schedule 2)
Drawings (P400 X 52)
Capital balance, December 31

_15,000

__30,000

__45,000

P55,000
40,800
( 20,800)
P75,000

P130,000
39,000
( 20,800)
P148,400

P185,000
80,000
( 41,600)
P223,400

2-25: d
Total receipts (P1,500,000 + P1,625,000)
Expenses
Net income
Distribution to Partners
Red P1,500,000/P3,125,000 X P2,045,000 =
Blue P1,625,000/P3,125,000 X P2,045,000 =

Capital balance of Blue Dec. 31


Capital Balance, Jan. 1
Additional investment
Capital balance before profit and
loss distribution
Profit share
Drawings
Capital balance, Dec. 31

P3,125,000
( 1,080,000)
P2,045,000
P 981,600 (1)
_1,063,400
P2,045,000
P 374,000
___22,000
P 396,000
1,063,400
( 750,000)
P 709,400 (2)

26
Chapter 2

2-26: a
Ray
P150,000

Sam
P180,000

_______

__60,000

150,000

240,000

15,000

20,000

51,000

34,000

66,000

54,000

Total
510,000
Salaries
_42,000

216,000

294,000

_18,000

_24,000

Total
552,000
Drawings
(42,000)

234,000

318,000

(18,000)

(24,000)

Capital balances, March 1


P330,000
Additional investment, Nov. 1
__60,000
Capital balances before salaries, profit and Drawings
390,000
Profit share:
Interest
35,000
Balance, 60:40
85,000
Total
120,000

Capital balances, Feb. 28


P510,000

P216,000

P294,000

Susan
P150,000

Tanny
P30,000

2-27: a
Capital balances, 1/1
P180,000
Additional investment, 4/1
8,000
Capital withdrawals, 7/1
_(6,000)

8,000

Balances before profit distribution


182,000
Profit distribution:
Interest
27,450
Bonus (20% x P30,000)
6,000
Balance, equally
(3,450)

_______

(6,000)

158,000

24,000

23,400

4,050
6,000

Total

(1,725)

(1,725)

21,675

_8,325

179,675

32,325

(12,000)

(12,000)

30,000
Total
212,000
Drawings
(24,000)
Capital balances, 12/31

P167,675

P20,325

P188,000

Partnership Operations

27

2-28: a
Capital balances, beg. 1st year
P300,000
Loss distribution, 1st year:
Salaries
30,000
Interest
30,000
Balance, 5:3:2
(80,000)
Total

Sin
P110,000

Tan
P80,000

20,000

Uy
P110,000
10,000

11,000

8,000

11,000

(40,000)

(16,000)

(24,000)

( 9,000)

( 8,000)

( 3,000)

(20,000)
Total
280,000
Drawings
(30,000)
Capital balances, beg. 2nd year
250,000
Profit distribution, 2nd year:
Salaries
30,000
Interest
25,000
Balance, 5:3:2
(15,000)
Total
40,000
Total
290,000
Drawings
_(30,000)
Capital balances, end of 2nd year
P260,000

101,000

72,000

107,000

(10,000)

(10,000)

(10,000)

91,000

62,000

97,000

20,000

10,000

9,100

6,200

9,700

( 7,500)

( 4,500)

( 3,000)

21,600

_1,700

16,700

112,600

63,700

113,700

_(10,000)

(10,000)

_(10,000)

P102,600

P53,700

P103,700

Jay
P30,000

Kay
P30,000

Loi
P30,000

_(5,000)

_(4,000)

5,000
______

25,000

26,000

35,000

3,000
7,000
_1,000

3,000

3,000

_1,000

_1,000

36,000

30,000

39,000

5,000
______

_(3,000)

_(8,000)

41,000

27,000

31,000

3,600

3,000

3,900

7,000
_1,500

_1,500

_1,500

53,100

31,500

36,400

______

_(4,000)

6,000
_(2,000)

53,100

27,500

40,400

5,310

3,150

3,640

2-29: c
Capital balances, 1/1/06
P90,000
Additional investment, 2006
Capital withdrawal, 2006
_(9,000)
Capital balances
86,000
Profit distribution, 2006:
Interest
Salary
Balance, equally
__3,000
Capital balances, 1/1/07
105,000
Additional investment, 2007
Capital withdrawal, 2002
(11,000)
Capital balances
99,000
Profit distribution, 2007:
Interest
10,500
Salary
Balance, equally
__4,500
Capital balances, 1/1/08
121,000
Additional investment, 2008
Capital withdrawal, 2008
_(6,000)
Capital balances
121,000
Profit distribution, 2008:
Interest

12,100
Salary
Balance, equally
___9,900
Capital balances, 12/31/08 per books
P150,000
Understatement of depreciation
(6,000)
Adjusted capital balances, 12/31/08
P144,000

7,000
__3,300

__3,300

__3,300

P68,710

P33,950

P47,340

(2,000)

(2,000)

P66,710

P31,950

(2,000)
P45,340

28
Chapter 2

2-30: a
Ken
Capital balances, 1/1/07
P300,000
Additional investment, 2007
40,000
Capital withdrawal, 2007
( 20,000)
Balances
320,000
Profit distribution, 2007 (Schedule 1)
Salary
60,000
Balance, beg. Capital ratio
60,000
Capital balances, 1/1/08
440,000
Capital withdrawal, 2008
( 60,000)
Balances
380,000
Profit distribution, 2008:
Salary
60,000
Balance, beg. capital ratio
__60,000
Capital balances, 12/31/08
P500,000

P100,000

Len
P100,000

( 20,000)

_______

_______

80,000

140,000

100,000
60,000

20,000

20,000

20,000

100,000

160,000

180,000

( 20,000)

( 40,000)

_______

80,000

120,000

180,000
60,000

__13,636

__21,818

__24,546

P 93,636

P141,818

P264,546

Total profit for 2 years

2-31: d

P100,000

40,000

Schedule 1 Computation of net profit:


Total capital, 2008 (P647,500 P147,500)
Total capital, 2007 (P300,000 + P40,000 P80,000)

Net profit per year (P240,000 / 2)

Mon

P500,000
_260,000
P240,000

P120,000

Capital balance, 1/1/08


Additional investment
Withdrawals
Cap. bal. before P/L dist.
NP: Salary (16,500 x 12)
Interest on EC (15%)
Balance 25:30:45
Total
Capital balance 12/31/08

_Nardo_
P280,000
96,000
376,000
42,000
( 19,875 )
22,125
P398,125

__Orly
P300,000
60,000
( 90,000 )
270,000
198,000
45,000
( 23,850 )
219,150
P 489,150

__Pedro_
P170,000
( 72,000 )
98,000
25,500
( 35,775 )
( 10,275 )
P 87,72

_Total_
P750,000
156,000
(162,000)
744,000
198,000
112,500
(79,500 )
231,000
P975,000

2-32: d
Sam capital, beginning
Additional investment (Land)
Drawings
Capital balance before net profit (loss)
Capital balance, end
Profit share (40%)
Net profit (P50,000 40%)

P120,000
60,000
( 80,000 )
100,000
150,000
50,000
P125,000

Partnership Operations

29

2-33: a
__Joe__
Capital balance, 1/2/07
P 80,000
Net loss- 2007:
Annual salary
96,000
10% interest on beg. capital
8,000
Bal. beg. cap. ratio: 8:4
( 108,000)
Total
( 4,000)
Capital balance
76,000
Drawings
( 4,000)
Capital balance, 12/31/07
72,000
Net profit- 2008:
Annual salary
96,000
10% interest on BC
7,200
Bonus to JoeNPBB
P 22000
NPAB (22000/110%)20000 2,000
Balance equally
( 67,300)
Total
37,900
Total
109,900
Drawings
( 4,000)
Capital balance, 12/31/08

105,900

__Tom__
P 40,000

__Total__
P120,000

48,000
4,000
( 54,000)
( 2,000)
38,000
( 4,000)
34,000

144,000
12,000
( 162,000)
( 6,000)
114,000
( 8,000)
106,000

48,000
3,400

144,000
10,600

( 67,300)
( 15,900)
18,100
( 4,000)

2,000
( 134,600)
22,000
128,000
( 8,000)

14,100

120,000

2-34: a
Decrease in capital
Drawings
Contribution
Profit share
Net income (45,000 30)

P 60,000
( 130,000)
25,000
45,000
P150,000

30

Chapter 2

SOLUTIONS TO PROBLEMS
Problem 2 1

1.

Castro
Diaz

:
:

(P26,000/P42,500) x
(P16,500/P42,500) x

P23,800
P23,800

=
=

P14,560
__9,240
P23,800

2.

Castro
Diaz

:
:

(P31,250/P50,000) x
(P18,750/P50,000) x

P23,800
P23,800

=
=

P14,875
__8,925
P23,800

Computation of Average Capitals:


Castro:
Capital
Date
Balances
1/1..................................... P26,000
4/10...................................
29,000
5/1.....................................
36,000
8/1.....................................
32,000

Average capital = P375,000 12 months =


Diaz:

Capital
Date
Balances
1/1..................................... P16,500
6/1.....................................
21,500
9/1.....................................
19,500

Average capital = P225,000 12 months =


3.
Interest........................................................
Salaries........................................................
Balance, equally..........................................
Total............................................................

Castro
P 7,500
36,000
( 24,100)
P19,400

Months
Unchanged
3
1
3
5
12

Peso
Months
P 78,000
29,000
108,000
_160,000
P375,000

P31,250
Months
Unchanged
5
3
4
12

Peso
Months
P 82,500
64,500
__78,000
P225,000

P18,750
Diaz
P4,500
24,000
(24,100)
P 4,400

Total
P12,000
60,000
( 48,200)
P23,800

4.
Bonus (a)....................................................
Interest (b)...................................................
Balance, 3:2................................................
Total............................................................

Castro
P 4,760
1,100
_10,764
P16,624

Diaz
P

_7,176
P7,176

Total
P 4,760
1,100
_17,940
P23,800

Partnership Operations

31

Computations:
a. Net profit before bonus................................................
Net profit after bonus (P23,800 125%).....................
Bonus...........................................................................
b.

5.

Castro
Diaz

P23,800
_19,040
P 4,760

Average capital of Castro [(P26,000 + P32,000) 2]............................


Average of Diaz [(P16,500 + P18,500) 2]..........................................
Castro's excess.......................................................................................
Multiply by............................................................................................
Interest...................................................................................................

P29,000
_18,000
P11,000
___10%
P 1,100

:
:

P14,280
__9,520
P23,800

(P3,000/P5,000) x P23,800
(P2,000/P5,000) x P23,800

=
=

Problem 2 2
a.

Average Capital:
Robin:
Date
Jan. 1
Feb. 28
Apr. 30
Sept. 30

Balances
P135,000
95,000
175,000
195,000

Months
Unchanged
2
2
5
3
12

Peso
Months
P270,000
190,000
875,000
__585,000
P1,920,000

Months
Unchanged
3
3
2
2
2
12

Peso
Months
P420,000
600,000
300,000
440,000
__400,000
P2,160,000

Ave. Capital (P1,920,000 12) = P160,000


Hood:

Date

Balances

Jan. 1
Mar. 31
June 30
Aug. 31
Oct. 31

P140,000
200,000
150,000
220,000
200,000

Ave. Capital (P2,160,000 12) = P180,000


Profit Distribution:
Robin : P160,000 P340,000 x P510,000 =

P240,000

Hood

P180,000 P340,000 x P510,000 =

_270,000
P510,000

32

Chapter 2

b.
Interest on ave. capital.........................................
Salaries................................................................
Bonus (P510,000 30,600 160,000) x 25%)....
Balance, equally..................................................
Totals...................................................................
c.
Interest:
Robin (P195,000 P135,000) 10%.............
Hood (P200,000 P140,000) 10%..............
Balance, equally..................................................
Totals...................................................................
d.

Robin
P 14,400
60,000
78,850
_119,775
P274,025

Hood
P 16,200
100,000

_119,775
P235,975

Total
P 30,600
160,000
79,850
_239,550
P510,000

Robin

Hood

Totals

P 6,000
249,000
255,000

P 12,000
498,000
510,000

Hood
P120,000

Total
P200,000
62,000
_248,000
P510,000

P 6,000
249,000
255,000
Robin
P 80,000
62,000
_124,000
P266,000

Salaries................................................................
Bonus (see computations below).........................
Balance, equally..................................................
_124,000
Totals...................................................................
P244,000
Bonus Computations:
Net income before salaries and bonus.....................................................
Less Salaries...........................................................................................
Net income before bonus........................................................................
Net income after bonus (P310,000 125%)............................................
Bonus......................................................................................................

P510,000
200,000
310,000
_248,000
P 62,000

Problem 2 3
a.

De Villa
P 30,000

De Vera

P 20,000
31,200
9,818
__44,182
P105,200

Salaries................................................................
Commission (2% x P1,000,000)..........................
Interest of 8% on average capital.........................
32,800
Bonus (see computations below).........................
9,818
Balance, equally..................................................
__44,182
Total ....................................................................
P116,800
Bonus Computations:
Income before salary, commissions, interest & bonus.............................
Salary and commission (P30,000 + P20,000)..........................................
Interest....................................................................................................
Income before bonus...............................................................................
Income after bonus (P108,000 110%)..................................................
Bonus......................................................................................................
b.

Income Summary.................................................
De Villa, capital...........................................
De Vera, capital...........................................

Partnership Operations

Total
P 30,000
20,000
64,000
19,636
__88,364
P222,000
P222,000
( 50,000)
( 64,000)
108,000
_98,182
P 9,818

P 222,000
116,800
105,200
33

Problem 2 4
a.
Salaries...............................................
Bonus (see computation below)..........
Interest (see computation below)........
Balance, 3:3:4.....................................
Total ...................................................

East
P15,000
3,760
2,800
__3,180
P24,740

North
P20,000

West
P18,000

4,000
__3,180
P27,180

4,800
__4,240
P27,040

Bonus computations:
Net income before bonus..........................................................................
Net income after bonus (P78,960 105%)................................................
Bonus........................................................................................................
Interest computations:
East (10% x P28,000)...............................................................................
North (10% x P40,000).............................................................................
West (10% x P48,000)..............................................................................
Total..........................................................................................................
b.
Interest (see computations below)......
Salaries...............................................
Bonus (see computations below)........
Balance, equally.................................
Total ...................................................
Interest computations:
Average capitals:
East:
Date
1/1
5/1
9/1

Balances
P30,000
36,000
28,000

East
P 3,133
24,000
( 6,056)
P 21,077

North
P 3,633
21,000
4,280
( 6,055)
P 22,858

West
P 5,200
25,000
( 6,055)
P 24,145

Months
Unchanged
4
4
4
12

Total
P53,000
3,760
11,600
_10,600
P78,960
P78,960
_75,200
P 3,760
P 2,800
4,000
__4,800
P11,600
Total
P11,966
70,000
4,280
( 18,166)
P 68,080

Pesos
Months
P120,000
144,000
_112,000
P376,000

Average capital (P376,000 12) ..........................................

P 31,333

North:

Pesos
Months
P80,000
124,000
72,000
_160,000
P436,000

Date
1/1
3/1
7/1
9/1

Balances
P40,000
31,000
36,000
40,000

Months
Unchanged
2
4
2
4
12

Average capital (P436,000 12)...........................................


34

P 36,333
Chapter 2

West:
Date
1/1
4/1
6/1
8/1

Balances
P50,000
57,000
60,000
48,000

Months
Unchanged
3
2
2
5

Pesos
Months
P150,000
114,000
120,000
_240,000

12

P624,000

Ave. capital (P624,000 12)....................................

P 52,000

Interest Computations:
East (10% x P31,333)...........................................................
North (10% x P36,333).........................................................
West (10% x P52,000)..........................................................
Total.....................................................................................

P 3,133
3,633
__5,200
P 11,966

Bonus Computations:
Net income...........................................................................
Less Salary...........................................................................
Net income before bonus......................................................
Net income after bonus (P47,080 110%)...........................
Bonus to North.....................................................................
* To Total
c.

East

West
P 8,990

5,000
__8,237.50
P22,227.50

Total
P 8,990
39,000
12,000
_32,950
P92,940

Bonus Computations:
Net income before salaries & bonus..........................................................
Less Salaries (P21,000 + P18,000)............................................................
Net income before bonus..........................................................................
Net income after bonus (P53,940 120%)................................................
Bonus to West...........................................................................................

P92,940
_39,000
P53,940
_44,950
P 8,990

Bonus (see comp. below)....................


Salaries ..........................................
Interest on beginning capital...............
Remainder, 8:7:5................................
Total ..................................................

North

P 68,000
_21,000
47,080
_42,800
P 4,280

P21,000 P 18,000
3,000
4,000
_13,180 _11,532.50
P37,180 P33,532.50

Problem 2 5
a.

Schedule of Income Distribution:


Salaries..............................................
Interest (see computation on p. 30).....
Balance, equally.................................
Total ..................................................

Maria
P12,000
7,200
__3,133
P22,333

Clara
P10,000
9,600
__3,133
P22,733

Rita
P 8,000
13,800
__3,134
P24,934

Partnership Operations

Interest on Average Capital:


Maria:
P80,000 x 8% x 6 months.......................
P100,000 x 5% x 6 months.....................
Clara:
P120,000 x 8%.......................................
Rita:
P180,000 x 8% x 9 Mos.. . ......................
P150,000 x 8% x 3 Mos.. . ......................
Total ...............................................................

Total
P30,000
30,600
__9,410
P70,000
35

P 3,200
__4,000

P 7,200
9,600

P10,800
__3,000

_13,800
P30,600

b.

Statement of Partners Capital:


Balances, Jan. 1..................................
Additional Investment........................
Capital Withdrawal.............................
Net Income.........................................
Drawings ..........................................
Balance, Dec. 31.................................

Maria
P 80,000
20,000

22,333
( 10,000)
P112,333

Clara
P120,000

22,733
( 10,000)
P132,733

Rita
P180,000

( 30,000)
24,934
( 10,000)
P164,934

Total
P380,000
20,000
( 30,000)
70,000
( 30,000)
P410,000

Benny

Celia

Total
P20,000

Problem 2 6

1.

Allocation of net loss for 2008:


Salary to Alvin....................................
Interests on average capital:
Alvin (P120,000 x 10%)............
Benny (P200,000 x 10%)...........
Celia (P220,000 x 10%).............
Balance, 30:30:40...............................
Total ..................................................

2.

Alvin
P 20,000
12,000

20,000
(29,400)
P 2,600

_(29,400)
P( 9,400)

22,000
_(39,200)
P(17,200)

54,000
_(98,000)
P(24,000)

Benny
P180,000
60,000
________
240,000
__(9,400)
230,600
_______
P230,600

Celia
P220,000
40,000
_(20,000)
240,000
_(17,200)
222,800
_______
P222,800

Total
P520,000
100,000
_(20,000)
600,000
_(24,000)
576,000
_(16,000)
P560,000

Statement of Partnership Capital


Year Ended December 31, 2008
Capitals, January 1, 2008....................
Additional investments.......................
Capital withdrawals............................
Balances. ...........................................
Net loss (see above)............................
Balances. ...........................................
Drawings. ..........................................
Capitals, December 31, 2008..............

Alvin
P120,000
_______
120,000
__2,600
122,600
_(16,000)
P106,600

36

3.

Chapter 2

Correcting entry:
Celia capital........................................
2,400
Alvin capital...............................
2,200
Benny capital.............................
200
To correct capital accounts for error in loss allocation computed as follows:
Alvin
Benny
Celia
Correct loss allocation........................
P2,600
P(9,400) P(17,200)
Actual loss allocation.........................
__(400)
__9,600
__14,800
Adjustment.........................................
P2,200
P 200
P ( 2,400)
Problem 2 7

Capital balances, 1/2/06..............................


Additional investment, 2006.......................

Dino
P45,000
_15,000

Nelson
P45,000
_15,000

Oscar
P45,000
__6,000

Total
P135,000
__36,000

Balances. ....................................................
Net income (Loss) - 2006, equally..............
Withdrawals, 2006......................................
Capital balances, 12/31/06..........................
Additional investment, 2007.......................
Balances. ....................................................
Net income - 2007, 40: 30: 30.....................
Withdrawals, 2007......................................
Capital Balances, 12/31/07..........................
Additional investment, 2008.......................
Balances. ....................................................
Net income, 2008 (schedule 1)....................
Withdrawals, 2008......................................
Capital balances, 12/31/08..........................

60,000
(1,800)
(17,000)
41,200
_____
41,200
10,800
(17,000)
35,000
______
35,000
56,365
(19,000)
P72,365

60,000
( 1,800)
( 7,000)
51,200
_____
51,200
8,100
( 7,000)
52,300
______
52,300
42,272
( 9,000)
P86,572

51,000
( 1,800)
( 3,200)
46,000
__6,000
52,000
8,100
( 3,200)
56,900
___6,000
62,900
20,363
( 3,200)
P80,063

171,000
( 5,400)
( 27,200)
138,400
___6,000
144,400
27,000
( 27,200)
144,200
___6,000
150,200
120,000
( 31,200)
P239,000

Dino
P48,000

3,600
_* 4,765
P56,365

Nelson
P24,000
10,909
3,600
__4,763
P43,272

Oscar
P12,000

3,600
__4,763
P20,363

Total
P84,000
10,909
10,800
__14,291
P120,000

Schedule 1:
Annual salaries...................................
Bonus (see computations below)........
Interest................................................
Balance, equally.................................
Totals..................................................

Bonus computations:
Net income before bonus......................................................................
Net income after bonus (P120,000 110%)..........................._109,091
Bonus to Nelson....................................................................................

P120,000
P 10,909

* To Total
Partnership Operations

37
Problem 2 8
Red, White & Blue Partnership
Statement of Partners' Capital
For Year Ended December 31, 2008

Balances, beginning of year


Add: 20% of fees billed to personal clients
Green's share of fees (Exhibit A)
Remaining net income (Exhibit A)
Subtotals
Less: Withdrawals
Uncollectible accounts identified
with clients of each partner
Excess rent charged to Blue
Total deductions
Balances, end of year

Red
40,200
8,800

White
20,200
4,800

Blue
40,600
4,400

_22,800
_71,800
10,400

_22,800
_47,800
8,800

_11,400
_56,400
11,600

2,400

900

P12,800
P59,000

P 9,700
P38,100

1,800
P13,400
P43,000

Green

3,200
______
__3,200
5,000

Total
P101,000
18,000
3,200
_57,000
179,200
35,800

P 5,000
P (1,800)

3,300
1,800
P 40,900
P138,300

Red, White & Blue Partnership


Exhibit A Computation and Division of Net income
For Year Ended December 31, 2008
Total revenue from fees
Expenses, excluding depreciation and doubtful
accounts expense
Less: Excess rent charged to N ($300 x 6)

P120,000
P38,700
__1,800

Subtotal
Add: Depreciation, computed as follows:
$26,000 x 0.10
$10,000 x 0.10 x 1/2
Total expenses, excluding doubtful accounts expense
Add: Doubtful accounts expense ($3,000 x 0.60)
Total expenses
Net income for year ended Dec. 31, Year 1

36,900
2,600
____500
P40,000
__1,800
41,800

Division of net income:


Fees billed to personal clients:
Red P44,000 x 20%
White P24,000 x 2%
Blue, P22,000 x 20%
Green's share of fees:
Gross fees from new clients after April 1, Year 1
Less: Allocated expenses ($40,000 x $24,000/
$120,000)
Net income from new clients
Green's share (P16,000 x 20%)
Total divided pursuant to special agreement
Balance, divided in income-sharing ratio as follows:
To Red, 40%
To White, 40%
To Blue, 20%
Total
38

P 8,800
48,000
4,400

________
P 78,200

P18,000

24,000
__8,000
P16,000
P 3,200
__21,200
P 57,000
P22,800
22,800
_11,400
P57,000
Chapter 2

Problem 2 9
Allan, Eman and Gino Partnership
Statement of Profit Distribution
Year Ended December 31, 2008
Allan

Eman

Gino

Total

Interest
Commission (P16,120 P5,000) x 10%
Balance, equally

P 4,000

__5,926

P 750
1,112
_5,925

P 250
1,112
_5,925

P 5,000
2,224
_17,776

Total
Adjustments (50% of P25,000 to Allan)

P 9,926
__2,574

P7,787
(1,287)

P7,287
(1,287)

P25,000
_____

Total

P12,500

P6,500

P6,000

P25,000

Sonny

Letty

Total

Problem 2 10
Gary, Sonny, and Letty Partnership
Statement of Partners' Capital Accounts
Year Ended December 31, 2008
Gary
Capital balances, 1/1/08
Additional investments

P210,000
___9,100

P180,000
_______

P 90,000
_______

P480,000
__9,100

Total

_219,100

_180,000

_90,000

489,100

Profit distribution:
Salaries
Interest
Bonus to Gary and Sonny (Schedule 1)
Balance, equally

13,680
25,920

__(9,720)

11,520
21,600

_(9,720)

_(9,720)(29,160)

Total

__29,880

_23,400

_11,720

Total
Drawings
_(48,000)

248,980
_(21,000)

203,400
101,720
(18,000) __(9,000)

Capital balances, 12/31/08

P227,980

P185,400

10,640
10,800

P 92,720

35,840
58,320

_65,000
554,100

P506,100

Schedule 1: Computation of the bonus.


Net profit before interest, salaries and bonus
Less:Salaries
Interest

P 65,000
P35,840
_58,320

Net profit (loss) before bonus

__94,160
P(29,160)

Therefore no bonus is to be given to Gary and Sonny.


Partnership Operations

39

Problem 2 11
a. Entries to record the formation of the partnership and the events that occurred during 2008:
Cash
Inventory
Land
Equipment
Mortgage payable

1,100,000
800,000
1,300,000
1,000,000
500,000

Installment note payable


Kobe, capital (P600,000 + P800,000
+ P1,000,000 P200,000)
Lebron, capital (P500,000 + P1,300,000
- P500,000)
(1)

(2)

(3)

(4)

(5)

Inventory
Cash
Accounts payable

200,000
2,200,000
1,300,000
300,000
240,000
60,000

Mortgage payable
Interest expense
Cash

50,000
20,000

Installment note payable


Interest expense
Cash

35,000
20,000

Accounts receivable
Cash
Sales
Selling and general expenses
Cash
Accrued expenses payable

70,000

55,000
210,000
1,340,000
1,550,000
340,000
278,000
62,000

(6)
(7)

(8)

Depreciation expense
Accumulated depreciation

60,000
60,000

Kobe, drawing
Lebron, drawing
Cash

104,000
104,000
208,000

Sales

1,550,000
Income summary

(9)

1,550,000

Cost of goods sold


Inventory
P900,000 = P800,000 beginning inventory
+ 300,000 purchases
- 200,000 ending inventory

900,000
900,000

40

Chapter 2

Income summary
Cost of good sold
Selling and general expenses
Depreciation expense
Interest expense

1,340,000
900,000
340,000
60,000
40,000

Income summary
Kobe, capital
Lebron, capital

210,000

Kobe, capital
Lebron, capital
Kobe, drawing
Lebron, drawing

104,000
104,000

105,000
105,000

104,000
104,000

Schedule to allocate partnership net income for 2008:


Kobe
Profit percentage
60%
Beginning capital balance
P2,200,000
P3,500,000
Net income (P1,550,000 revenue
- P 1,340,000 expenses)
Interest on beginning capital
balances (3%)
66,000
(105,000)

Lebron
40%
P1,300,000

Total
100%

210,000
39,000
P105,000

Salaries
(240,000)
P(135,000)
Residual deficit
(135,000)
Total
b.

120,000

120,000

(81,000)

(54,000)

P105,000
Kobe-Lebron Partnership
Income Statement

P105,000

-0-

For the Year Ended December 31, 2008


Sales
P1,550,000
Less: Cost of goods sold:
Inventory, January 1
Purchases
Goods available for sale
Less: Inventory, December 31
(900,000)
Gross profit
Less: Selling and general expenses
Depreciation expenses
Operating income
Nonoperating expense- interest
Net income

P800,000
300,000
P1,100,000
(200,000)
P650,000
340,000
60,000

400,000
P250,000
(40,000)
P210,000

Partnership Operations

c.

41

Kobe-Lebron Partnership
Balance Sheet
At December 31, 2008
Assets
Cash
Accounts receivable
Inventory
Land
Equipment (net)
Total assets

P1,589,000
210,000
200,000
1,300,000
940,000
P4,239,000
Liabilities and Capital

Liabilities:
Accounts payable
Accrued expenses payable
Installment note payable
Mortgage payable
Total liabilities
Capital:
Kobe, capital
Lebron, capital
Total capital
Total liabilities and capital

P60,000
62,000
165,000
450,000
P737,000
P2,201,000
1,301,000
3,502,000
P4239,000

42
Chapter 3

CHAPTER 3
MULTIPLE CHOICE ANSWERS AND SOLUTIONS
3-1: c
Implied capital of the partnership (P90,000/20%)
P450,000
Actual value of the partnership
( 420,000)

Goodwill
30,000
HIZON
Capital balances before Goodwill
Goodwill to old partners
_____
Total
Purchase by Hizon (20%)
_90,000
Capital balances after admission
P 90,000

AQUINO

LOCSIN

DAVID

P252,000
__18,000

P126,000
___9,000

P42,000
__3,000

P270,000
( 54,000)

P135,000
( 27,000)

P45,000
( 9,000)

P216,000

P108,000

P36,000

AQUINO

LOCSIN

DAVID

P252,000
( 50,400)

P126,000
( 25,200)

P42,000
( 8,400)

P201,600

P100,800

P33,600

AQUINO

LOCSIN

DAVID

P 50,400

P 25,200

P 8,400

__3,600

__1,800

___600

P 54,000

P 27,000

P 9,000

3-2: b
HIZON
Capital balances before admission
Purchase by Hizon (20%)
_84,000
Capital balances after admission
P 84,000
3-3: d
TOTAL
Capital transferred
P 84,000
Excess divided using profit and loss ratio
__6,000
Cash distribution
P 90,000
3-4: b
Selling price
Interest sold (444,000X1/5)
Combine gain

P132,000
( 88,800)
P 43,200

3-5: b
Implied value of the partnership (P40,000/1/4)
P160,000
Actual value
( 140,000)
Goodwill
20,000
DIAZ
Cash balances
P 20,000
Goodwill, Profit and Loss ratio
__2,000

BERNAL

CUEVAS

P 80,000

P40,000

__12,000

__6,000

Total

P 92,000

P46,000

Capital Transfer (1/4)


( 5,500)
Capital balances after admission
P 16,500

( 23,000)

( 11,500)

P 69,000

P34,500

BANZON

CORTEZ

P 16,000

P 4,000

__6,000

__4,000

P 22,000

P 8,000

PEREZ

CADIZ

P 24,000

P 48,000

5,430

10,860

000

Partnership Dissolution Changes in Ownership

3-6: b
TOTAL
Capital Transfer (20%)
P20,000
Excess, Profit and Loss ratio
_10,000
Cash distribution
P30,000
3-7: d
TOTAL
Capital balances beginning
P 72,000
Net profit, 1:2
16,290
Drawings

( 5,050)

( 8,000)

Capital balances before admission


P 75,240
Capital transfer (squeeze)
(18,810) (1/4)
Capital balances after admission 1:2
P 56,430

P 24,380

P 50,860

( 5,570)

( 13,240)

P 18,810

P 37,620

Capital transfer
P18,810
Excess, 1:2
_11,190
Cash
P30,000

P 5,570

P 13,240

__3,730

__7,460

P 9,300

P 20,700

,050)

3-8: a
Total agreed capital (P150,000/5/6)
Diana's Interest
Cash distribution

P180,000
1/6
P 30,000

Total agreed capital (P36,000/1/5)


Total contributed capital (80,000+40,000+36,000)
Unrecognized Goodwill

P180,000
( 156,000)
P 24,000

3-9: a

3-10: b

Contributed

Agreed

Increase

Capital
P110,000
__40,000
P150,000

Old partners
New partner
Total

Capital
P100,000
__50,000
P150,000

(Dec.)
(P 10,000)
_10,000
P

Ben, capital balance before admission


Bonus share to new partner (10,000X60%)
Ben, capital after admission

P 60,000
( 6,000)
P 54,000

Total agreed capital (P40,000+20,000+17,000)


Pete's interest
Pete's agreed capital balance

P 77,000
1/5
P 15,400

3-11: c

44
Chapter 3

3-12: b
Old partner
New partner
Total

Contributed
Capital
P 65,000
25,000 (1/3)
P 90,000

Agreed
Capital
P60,000
30,000
P90,000

Increase
(Dec.)
(P 5,000)
_5,000
P

Capital balances before admission


Investment by Lory
Bonus to Lory
Capital balances after admission

FRED
P 35,000

(
3,500)
P 31,500

RAUL
P30,000

( 1,500)
P28,500

LORY

25,000
__5,000
P 30,000

3-13: c
Total agreed capital (90,000+60,000+70,000)
Augusts' interest
Agreed capital
Contributed capital
Bonus to June & July

Capital balances before admission


Bonus from August, equally
Capital balances after admission

P220,000
_____1/4
P 55,000
__70,000
P 15,000
JUNE
P90,000
__7,500
P97,500

JULY
P 60,000
__7,500
P 67,500

3-14: a
Total agreed capital (52,000 + 88,000)/80%)
Total capital of Mira & Nina after admission
Cash paid by Elma

P175,000
( 140,000)
P 35,000

3-15: a
Total agreed capital (P41,600/2/3)
Total contributed capital (P23,000+18,600+16,000)

P 62,400
( 57,600)

Goodwill to new partner, Ang

P 4,800
LIM
P23,000

_____
P23,000

Capital balances before admission


Investment by Ang
Goodwill to August
Capital balances after admission

ONG
P 18,600

______
P 18,600

ANG

16,000
__4,800
P20,800

Partnership Dissolution Changes in Ownership

3-16: a
Capital balances before
admission
P1,300,000
Admission by Dong:
By Purchase (1/2)
By Investment
___300,000
Capital balances before
Goodwill and Bonus
P1,600,000
Goodwill to Old Partners (sch. 1)
Bonus to Old Partners
(sch. 1)
________
Capital balances after
admission
P2,000,000

ANG

BENG

CHING

DONG

P600,000

P 400,000

P 300,000

( 300,000)
_______

_______

_______

300,000
_300,000

P300,000

P 400,000

P 300,000

P600,000

150,000
__37,500

150,000
__37,500

100,000
__25,000

( 100,000)

P487,500

P 587,500

P 425,000

Schedule 1:
Old Partners
New Partner
Total

CC
AC
P 1,000,000 P1,500,000
600,000 (25%) __500,000
P 1,600,000 P2,000,000

TOTAL

400,000

P500,000
Inc. (Dec.)
P500,000
( 100,000) Bonus
P400,000 GW

3-17: b
Capital balances before
admission of Alma
P 200,000
Admission of Alma:
Investment
Goodwill to old partner,
70:30 (sch. 1)
___40,000
Capital balances before
admission of Lorna
P 320,000
Admission of Lorna:
Goodwill Written off, 5:3:2
( P40,000)
Investment
Goodwill to old partners,
5:3:2 (sch. 2)
___20,000

MONA

LIZA

ALMA

LORNA

P150,000

P 50,000

80,000

__28,000

___12,000

_______

______

P178,000

P 62,000

P 80,000

(P 20,000)

(P 12,000) (

P8,000)

TOTAL

80,000

75,000

__10,000

____6,000

____4,000

______

75,000

Capital balances after


admission
P 375,000

P168,000

P 56,000

P 76,000

P 75,000

Schedule 1:
Total agreed capital (80,000/25%)
P 320,000
Total capital contributed (200,000+80,000)
( 280,000)
Goodwill to old partners, 70:30
40,000
Schedule 2:
Total agreed capital (75,000/20%)
P 375,000
Total contributed capital (280,000+75,000)
( 355,000)
Goodwill to old partners, 5:3:2
20,000

46
Chapter 3

3-18: c
TOTAL
Unadjusted capital balances
P320,000
Overvaluation of Marketable Securities
( 25,000)
Allowance for Bad Debts
( 25,000)
Adjusted capital balances before admission
P270,000
Total agreed capital (270,000/2/3)
Green's interest
Investment

RED

WHITE

BLUE

P175,000

P100,000

P 45,000

( 12,500)

( 7,500)

( 5,000)

( 12,500)

( 7,500)

( 5,000)

P150,000

P 85,000

P 35,000

P405,000
1/3
P135,000

3-19: b
TOTAL
Capital balances before
admission
P720,000
Capital transfer
to WW (1/6)
______
Balances
P720,000
Equalization of capital
______
Balances
P720,000
Net profit, equally

XX

YY

ZZ

WW

P360,000

P225,000

P135,000

( 60,000)

( 37,500)

( 22,500)

_120,000

P300,000

P187,500

P112,500

P120,000

( 100,000)

__12,500

__87,500

______

P200,000

P200,000

P200,000

P120,000

3,150

3,150

3,150

3,150

12,600
Drawings (2 months)
_( 7,000)
Capital balances before
WWs Investment
P725,600

_( 1,500)

_( 2,000)

_( 1,500)

_( 2,000)

P201,650

P201,150

P201,650

P121,150

Total agreed capital (201,650+201,150+201,650)/2/3


WW's interest
Agreed capital of WW
Contributed capital (see above)
Cash to be invested

P906,675
1/3
P302,225
_121,150
P181,075

3-20: a
Capital balances
P 45,000
Understatement of assets, P12,000
__6,000
Balances before settlement to A
P 51,000
Settlement to A
A's interest (23,750+5,000)
Partial Goodwill to A

A
P 20,750

B
P 19,250

__3,000

__3,000

P 23,750

P 22,250

P 30,250
_28,750
P 1,500

Therefore:
1. Under partial Goodwill method the capital balances of B is P 22,250
2. Under Bonus method the capital balances of B would be:
B, capital balances before settlement to A
P 22,250
Bonus to A (1,500X25/75)
_( 500)
B, capital after retirement of A
P 21,750
Partnership Dissolution Changes in Ownership

3-21: a
Perez
Suarez
Capital balances
P 200,000
Net income, P140,000
28,000
Undervaluation of inventory, P20,000
____4,000
Capital balances before settlement to Perez
P 232,000
Settlement to Perez
Bonus to Perez
_( 6,000)
Capital balances after retirement
P 226,000

Reyes

P 100,000

P 150,000

70,000

42,000

___10,000

____6,000

P 180,000

P 198,000

( 195,000)
___15,000

_(

P 189,000

ELY

FLOR

P 320,000

P 192,000

9,000)

3-22: c
GLOR
Capital balances

P 128,000
Settlement to Ely
Total Goodwill (P40,000/50%)P80,000
___16,000
Capital balances after retirement of Ely
P 144,000

( 360,000)

__40,000 ___24,000
P

P 216,000

3-23: c
Capital balance 3/1/07
Net loss-2007:
Salary (10 months)
Interest (10 months)
Bal. beg. cap. ratio: 48:24
Total
Capital balance
Drawings
Capital balance, 12/31/07
Net profit- 2008:
Salary
Interest
Balance, equally
Total
Capital balance
Drawings
Capital balance 12/31/08

_Alma_
480,000

_Betty_
240,000

480,000
40,000
( 544,000)
( 24,000)
456,000
( 24,000)
432,000

240,000
20,000
( 272,000)
( 12,000)
228,000
( 24,000)
204,000

720,000
60,000
( 816,000)
( 36,000)
684,000
( 48,000)
636,000

576,000
43,200
( 397,800)
221,400
653,400
( 24,000)
629,400

288,000
20,400
( 397,800)
( 89,400)
114,600
( 24,000)
90,600

864,000
63,600
( 795,600)
132,000
768,000
( 48,000)
720,000

Total contributed capital (720,000 + 400,000)


Coras interest
Coras agreed capital
Coras contributed capital
Bonus to Cora, from Alma and Betty 4:2
Therefore entry (c) is correct.

_Total_
720,000

1,120,000
40%
448,000
400,000
48,000

48
Chapter 3

3-24: a
Capital balance, beg. 2007
2007 net profit (90,000 59,000):
Interest
Compensation
Balance, 4:6
Total
Balance
Withdrawal
Repairs (charge to Pete)
Capital balance, 12/31/07

_Pete_
P80,000
8,000
5,000
( 2,000)
11,000
91,000
( 8,000)
( 5,000)
78,000

1/1/08: Admission of Sammy


Total agreed capital (P117,000 +43,000)

_Carlos_
P30,000
3,000
20,000
( 3,000)
20,000
50,000
( 11,000)
39,000

_Total_
P110,000
11,000
25,000
( 5,000)
31,000
141,000
(19,000)
( 5,000)
117,000
P160,000

Sammys interest
Sammys agreed capital
Sammys contributed capital
Bonus to Pete & Carlos, 4:6
Therefore entry (a) is correct.

20%
32,000
43,000
11,000

Partnership Dissolution Changes in Ownership

SOLUTIONS TO PROBLEMS
(a)

Problem 3 1
1. Goodwill Method:
Total agreed capital (P75,000 25%).....................................P300,000
Total contributed capital........................................................ ._275,000
Goodwill to old partners, P/L ratio..........................................P 25,000
Entry
Goodwill...........................................................................
Cash..................................................................................
Red, capital..................................................................
White, capital...............................................................
Blue, capital.................................................................
Green, capital...............................................................

25,000
75,000

2. Bonus Method:
Contributed capital of Green....................................................P 75,000
Agreed capital of Green (P275,000 x 25%).............................._68,750
Bonus to old partners, P/L ratio...............................................P 6,250

5,000
10,000
10,000
75,000

Entry:
Cash..................................................................................
Green, capital...............................................................
Red, capital..................................................................
White, capital...............................................................
Blue, capital.................................................................

75,000
68,750
1,250
2,500
2,500

(b) 1. Implicit Goodwill Method:


Total Implied Capital (P75,000 25)......................................P300,000
Total existing capital.............................................................. ._200,000
Implied Goodwill to old partners............................................P100,000
Entries:
Goodwill...........................................................................
Red, capital..................................................................
White, capital...............................................................
Blue, capital.................................................................

100,000
20,000
40,000
40,000

Red, capital (25% x P80,000)...........................................


White, capital (25% x p120,000)......................................
Blue, capital (25% x P100,000)........................................
Green, capital...............................................................

20,000
30,000
25,000

2. Red, capital (25% x P10,000)......................................................


White, capital (25% x P80,000)..................................................
Blue, capital (25% x P60,000).....................................................
Green, capital........................................................................

15,000
20,000
15,000

75,000

50,000

50
Chapter 3

Problem 3 2
a.

(1) Bonus Method:


Contributed capital of Tomas.............................................................................
Agreed capital of Tomas (P640,000 x 20%).......................................................
Bonus to old partners, P/L ratio..........................................................................
MARIO
P300,000
___3,000
P303,000

TOMAS

_128,000
P128,000

(2) Goodwill Method:


Total agreed capital (P140,000 20%). .........................................
Total contributed capital.................................................................
Goodwill to old partners, P/L ratio.................................................

P700,000
_640,000
P 60,000

Balances before admission.....................


Admission of Tomas...............................
Balances after admission........................

Balances before admission.....................


Admission of Tomas...............................
Balances after admission........................

BRUNO
P200,000
___9,000
P209,000

P140,000
_128,000
P 12,000
TOTAL
P500,000
_140,000
P640,000

BRUNO
P200,000
__45,000
P245,000

MARIO
P300,000
__15,000
P315,000

TOMAS
P

_140,000
P140,000

TOTAL
P500,000
_200,000
P700,000

BRUNO
P245,000
( 36,000)

MARIO
P315,000
( 12,000)

TOMAS
TOTAL
P140,000
P700,000
( 12,000) ( 60,000)

(3) Goodwill with subsequent write-off.


Balances from A-2..................................
Goodwill written off, 6:2:2.....................

Balances..................................................

P209,000

P303,000

P128,000

Balances from A-2..................................


Goodwill written off, 4:4:2.....................
Balances..................................................

BRUNO
P245,000
( 24,000)
P221,000

MARIO
P315,000
( 24,000)
P291,000

TOMAS
TOTAL
P140,000
P700,000
( 12,000) ( 60,000)
P128,000
P640,000

b.

P640,000

Problem 3 3
a.

Total capital after admission (P76,000 + P104,000).........................................................


Total capital before admission (P60,000 + P80,000)........................................................
Goodwill recorded............................................................................................................

P180,000
_140,000
P 40,000

Total capital of the partnership (P180,000 75%)...........................................................


Less: Total capital of old partners plus Goodwill (P140,000 + 40,000)...........................
Cash payment by Barry.....................................................................................................

P240,000
_180,000
P 60,000

Total capital after admission (P52,000 + P68,000)...........................................................


Total capital before admission..........................................................................................
Bonus to Barry..................................................................................................................

P120,000
_140,000
P 20,000

Agreed capital of Barry (P120,000 75%) x 25%...........................................................


Less: Bonus ...................................................................................................................
Cash payment by Barry.....................................................................................................
Partnership Dissolution Changes in Ownership

P 40,000
__20,000
P 20,000

b.

Problem 3 4
a.

Total agreed capital (P60,000 20%)..................................................P300,000


Total contributed capital (P100,000 + P40,000 + P60,000)................ ._200,000
Goodwill to old partners, P/L ratio......................................................P100,000
Entry:
Cash. ..........................................................................................
Goodwill.....................................................................................
Gene, capital.........................................................................
Nancy, capital.......................................................................
Ellen, capital.........................................................................

b.

Cash..................................................................................................
Ellen, capital...............................................................................

60,000
100,000
80,000
20,000
60,000
60,000
60,000

No Goodwill, no bonus because the total agreed capital is equal to the total contributed
capital.
c.

d.

Gene, capital .....................................................................................


Nancy, capital....................................................................................
Ellen, capital...............................................................................

20,000

Cash..................................................................................................
Ellen, capital...............................................................................

32,000

8,000
28,000
32,000

Since the total agreed capital (P172,000) is equal to the total contributed capital (P172,000),
then no Goodwill or bonus is to be recorded.
e.

Total agreed capital (P140,000 80%)................................................P175,000

Total contributed capital (P140,000 + P32,000).................................. ._172,000


Goodwill to new partner......................................................................P 3,000
Entry:
Cash. ..........................................................................................
Goodwill.....................................................................................
Ellen, capital.........................................................................

32,000
3,000
35,000

Problem 3 5
a.
b.

Cash..................................................................................................
Cherry capital..............................................................................

40,000
40,000

Total agreed capital (P120,000 + P50,000)..........................................P170,000


Cherry's interest...................................................................................____25%
Cherry's agreed capital.............................................................................42,500
Contributed capital............................................................................. .__50,000
Bonus to old partners, 70:30................................................................P 7,500

52
Chapter 3

Entry:
Cash. ..........................................................................................
Cherry, capital.......................................................................
Helen, capital........................................................................
Cathy, capital........................................................................
c.

25,000
7,875
3,375
36,250

Total agreed capital (P50,000 25%)..................................................P200,000


Total contributed capital (P120,000 + 50,000).......................................170,000
Goodwill to old partners, 70:30...........................................................P 30,000
Entry:
Cash
.....................................................................................
Goodwill.....................................................................................
Cherry, capital.......................................................................
Helen, capital........................................................................
Cathy, capital........................................................................

e.

42,500
5,250
2,250

Total agreed capital (P120,000 + P25,000)..........................................P145,000


Cherry's interest...................................................................................____25%
Agreed capital of Cherry.........................................................................36,250
Contributed capital............................................................................. .__25,000
Bonus to new partner.......................................................................... .P 11,250
Entry:
Cash. ..........................................................................................
Helen, capital..............................................................................
Cathy, capital...............................................................................
Cherry, capital.......................................................................

d.

50,000

50,000
30,000

Total agreed capital (P120,000 75%)................................................P160,000


Total contributed capital (P120,000 + P25,000).................................. ._145,000
Goodwill to new partner......................................................................P 15,000
Entry:

50,000
21,000
9,000

Cash
.....................................................................................
Goodwill.....................................................................................
Cherry, capital.......................................................................

25,000
15,000
40,000

Problem 3 6
a.

Total agreed capital (P600,000 3/4).................................................................


Santos interest.....................................................................................................
Contribution of Santos........................................................................................

P800,000
_____1/4
P200,000

b.

Total agreed capital (P630,000 3/4).................................................................


Santos' interest....................................................................................................
Contribution of Santos........................................................................................

P840,000
_____1/4
P210,000

Partnership Dissolution Changes in Ownership


c.

d.

e.

Total agreed capital (P624,000 3/4)............................................................................................


Less: Contributed capital of old partners.......................................................................................

P832,000
_600,000

Contributed capital of Santos........................................................................................................

P232,000

Total agreed capital (P600,000 3/4)............................................................................................


Less: Goodwill ............................................................................................................................

P800,000
__10,000

Contributed capital........................................................................................................................
Contributed capital of old partners................................................................................................

790,000
_600,000

Contributed capital of Santos........................................................................................................

P190,000

Total agreed capital (Contributed).................................................................................................


Less: Contributed capital of old partners.......................................................................................

P820,000
_600,000

Contributed capital of Santos........................................................................................................

P220,000

Problem 3 7
a.

b.

c.

Tony, capital
........................................................................................................
Noel, capital......................................................................................................

40,000

Cash

........................................................................................................
Noel, capital......................................................................................................
(P180,000 2/3) x 1/3 = P90,000.

90,000

Cash.........................................................................................................................
Goodwill ..................................................................................................................
Noel, capital......................................................................................................

56,000
4,000

40,000

90,000

60,000

Total agreed capital (P180,000 3/4).............................................................................P240,000


Total contributed capital (P180,000 + P56,000).............................................................._236,000
Goodwill to new partner.................................................................................................P 4,000
d.

Subas, capital
.....
Tony, capital
...
Inventory.............
24,000

14,400
9,600

Cash.........................................................................................................................
52,000
Noel, capital......................................................................................................
Total agreed capital (P52,000 1/4)...............................................................................P208,000
Total capital before inventory write-down (180,000 + 52,000)......................................(232,000)
e.

Write-down to old partners capital.................................................................................( 24,000)


Land.. 92,000
Subas, capital
Tony, capital.

52,000

55,200
36,800

Subas, capital (P155,200 x 1/4).................................................................................


38,800
Tony, capital (P116,800 x 1/4)...................................................................................
29,200
Noel, capital......................................................................................................
Total resulting capital (P68,000 1/4)...........................................................................P272,000
Total capital of old partner (net assets)............................................................................_180,000
Increase in value of land.................................................................................................P 92,000
Capital of old partner after revaluation of land:
Subas (P100,000 + P55,200)..................................................................................P155,200
Tony (P80,000 + P36,800).........................................................................................116,800

68,000

54
Chapter 3

f.

Cash..................................................................................................
Subas, capital.....................................................................................
Tony, capital .....................................................................................
Noel, capital................................................................................

40,000
2,400
1,600
44,000

Agreed capital of Noel (P220,000 x 1/5)..............................................P 44,000


Contributed capital of Noel...................................................................._40,000
Bonus to Noel.......................................................................................P 4,000
g.

Cash..................................................................................................
Goodwill...........................................................................................
Noel, capital................................................................................
Subas, capital (P60,000 x 3/5).....................................................
Tony, capital (P60,000 x 2/5)......................................................

P60,000
60,000
P 60,000
36,000
24,000

Total agreed capital (P60,000 1/5)....................................................P300,000


Total contributed capital (P180,000 + P60,000).................................. ._240,000
Goodwill to old partner, 3:2.................................................................P 60,000
Problem 3 8
a.

b.

c.

Conny, capital....................................................................................
Andy, capital (P8,000 x 3/4)..............................................................
Benny, capital (P8,000 x 1/4).............................................................
Cash. ..........................................................................................

40,000
6,000
2,000

Goodwill...........................................................................................
Conny, capital....................................................................................
Cash. ..........................................................................................

10,000
40,000

Goodwill (P5,000 1/5)....................................................................


Conny, capital....................................................................................
Andy, capital (P25,000 x 3/5)......................................................
Benny, capital (P25,000 x 1/5)....................................................
Cash
.....................................................................................

25,000
40,000

48,000

50,000

15,000
5,000
45,000

Problem 3 9
a.
b.

Spade, capital.....................................................................................
Jack, capital.................................................................................

120,000

Goodwill (P30,000 50%)................................................................


Ace, capital.................................................................................
Jack, capital.................................................................................

60,000

120,000

12,000
18,000

Spade, capital..............................................................................
Spade, capital (P120,000 + P30,000).................................................
Jack, capital.................................................................................

30,000
150,000
150,000

Partnership Dissolution Changes in Ownership

Problem 3-9 (Continued)

c.

d.

e.

f.

g.

Spade, capital.....................................................................................
Cash. ..........................................................................................

180,000

Ace, capital (P60,000 x 2/5)..............................................................


Jack, capital (P60,000 x 3/5)..............................................................
Spade, capital..............................................................................

24,000
36,000

Land.................................................................................................
Ace, capital (20%).......................................................................
Jack, capital (30%)......................................................................
Spade, capital (50%)...................................................................

20,000

Spade, capital.....................................................................................
Ace, capital (P50,000 x .40)..............................................................
Jack, capital (P50,000 x .60)..............................................................
Cash. ..........................................................................................
Land. ..........................................................................................

130,000
20,000
30,000

Goodwill...........................................................................................
Spade, capital.....................................................................................
Cash. ..........................................................................................

30,000
120,000

Goodwill (P30,000 50%)................................................................


Spade, capital.....................................................................................
Ace, capital (P60,000 x 20%)......................................................
Jack, capital (P60,000 x 30%).....................................................
Cash. ..........................................................................................

60,000
120,000

Land.................................................................................................
Ace, capital (20%).......................................................................
Jack, capital (30%)......................................................................
Spade, capital (50%)...................................................................

P40,000

Spade, capital (P120,000 x P20,000).................................................


Ace, capital (P10,000 x 40%)............................................................
Jack, capital (P10,000 x 60%)............................................................
Land. ..........................................................................................
Note payable...............................................................................

140,000
4,000
6,000

56
Chapter 3

180,000

60,000
4,000
6,000
10,000

60,000
120,000

150,000

12,000
18,000
150,000
8,000
12,000
20,000

100,000
50,000

Problem 3 10
Case 1: Bonus of P10,000 to Eddy:
Eddy, capital................................................................................
Charly, capital (P10,000 x 3/5)....................................................
Danny, capital (P10,000 x 2/5)....................................................
Cash .....................................................................................

70,000
6,000
4,000

Case 2: Partial Goodwill to Eddy:


Goodwill.....................................................................................
Eddy, capital................................................................................
Cash .....................................................................................

4,000
70,000

Case 3: Bonus of P5,000 to remaining partner:


Eddy, capital................................................................................
Charly, capital (P5,000 x 3/5)...............................................
Danny, capital (P5,000 x 2/5)................................................
Cash .....................................................................................
Case 4: Total Implied Goodwill of P24,000:
Goodwill.....................................................................................
Eddy, capital................................................................................
Charly, capital (P24,000 x 3/6).............................................
Danny, capital (P24,000 x 2/6)..............................................
Cash .....................................................................................
Case 5: Other assets disbursed:
Eddy, capital................................................................................
Other assets.................................................................................
Charly, capital (P60,000 x 3/6).............................................
Danny, capital (P60,000 x 2/6)..............................................
Cash .....................................................................................
Case 6: Danny purchases Eddy's capital interest:
Eddy, capital................................................................................
Danny, capital.......................................................................

80,000

74,000
70,000
3,000
2,000
65,000
24,000
70,000
12,000
8,000
74,000
70,000
20,000
30,000
20,000
40,000
70,000
70,000

Partnership Dissolution Changes in Ownership

Problem 3 11
a. 1/1/06

Building...............................................................
Equipment...........................................................

52,000
16,000

Cash ....................................................................
Santos capital..............................................
To record initial investment.

12,000

12/31/06 Reyes capital........................................................


Santos capital..............................................
Income summary.........................................
To record distribution of loss as follows:

22,000

1/1/07

40,000

12,000
10,000

Interest.................................................................
Additional profit..................................................
Balance to Reyes.................................................

Santos
P 8,000
4,000
______

Reyes
P

(22,000)

Total
P 8,000
4,000
(22,000

Total ....................................................................

P12,000

P(22,000)

(P10,000)

Cash ....................................................................
Santos capital (15%)............................................
Reyes capital (85%).............................................
Cruz capital.................................................

15,000
300
1,700
17,000

(new investment by Cruz brings total capital to P85,000 after 2006 loss [80,000
10,000 + 15,000]. Cruz's 20% interest is P17,000 [85,000 x 20%] with the extra
P2,000 coming from the two original partners [allocated between them according
to their profit and loss ratio].)
12/31/07 Santos capital.......................................................
Reyes capital........................................................
Cruz capital..........................................................
Santos drawings..........................................
Reyes drawings...........................................
Cruz drawings.............................................

10,340
5,000
5,000
10,340
5,000
5,000

To close drawings accounts for the year based on distributing 20%. Of each
partner's beginning capital balances [after adjustment for Cruz's investment] or
P5,000 whichever is greater. Santos's capital Is P51,700 [40,000 + 12,000 300].)
12/31/07 Income summary.................................................
Santos capital..............................................
Reyes capital...............................................
Cruz capital.................................................
To allocate P44,000 income figure as computed below:
Santos
Interest (20% of P51,700).................................... P10,340
15% of P44,000 income.......................................
6,600
Balance, 60:40..................................................... ______

44,000

Reyes

Cruz

P16,236

P10,824

Total ....................................................................

P16,940

P16,236

P10,824

Initial investment, 2007.......................................


2007 profit...........................................................
Cruz investment...................................................
2007 drawings.....................................................
2007 profit...........................................................

Santos
P40,000
12,000
(300)
(10,340)
_16,940

Reyes
P40,000
(22,000)
(1,700)
(5,000)
_16,236

Capital, 12/31/07.................................................

P58,300

P27,536

16,940
16,236
10,824

58
Chapter 3

Capital balances as of December 31, 2008


Cruz
P17,000
(5,000)
_10,824
P22,824

1/1/08

Cruz capital..........................................................
Diaz capital.................................................
To transfer capital purchase from Cruz to Diaz

22,824
22,824

12/31/08 Santos capital.......................................................


11,660
Reyes capital........................................................
5,507
Diaz capital..........................................................
5,000
Santos drawings..........................................
11,660
Reyes drawings...........................................
5,507
Diaz drawings.............................................
5,000
To close drawings accounts based on 20% of beginning capital Balances (above) or
P5,0000 (whichever is greater).
12/31/08 Income summary.................................................
Santos capital..............................................
Reyes capital...............................................
Diaz capital.................................................
To distribute profit for 2008 computed as follows:

1/1/09

b. 1/1/06

61,000
20,810
24,114
16,076
Reyes

Diaz

Interest (20% of P58,300)....................................


15% of P61,000 profit..........................................
Balance, P40,190, 60:40......................................

Santos
P11,660
9,150
______

P24,114

P16,076

Total ....................................................................

P20,810

P24,114

P16,076

Diaz capital..........................................................
33,900
Santos capital (15%)............................................
509
Reyes capital (85%).............................................
2,881
Cash............................................................
37,290
Diaz capital is [33,900 (P22,824 P5,000 + P16,076)]. Extra 10% is deducted
from the two remaining partners' capital accounts.
Building...............................................................
Equipment...........................................................
Cash ....................................................................
Goodwill..............................................................
Santos capital..............................................
Reyes capital...............................................
To record initial investments. Reyes is credited with goodwill of
Santos investment.

52,000
16,000
12,000
80,000
80,000
80,000
P80,000 to match

Partnership Dissolution Changes in Ownership

12/31/06 Reyes capital..............................................................


30,000
Santos capital..............................................
20,000
Income summary.........................................
10,000
Interest of P16,000 is credited to Santos (P80,000 x 20%) along with a base of
P4,000. The remaining profit is now a P30,000 loss which is attributed entirely to
Reyes.
1/1/07

Cash ....................................................................
15,000
Goodwill..............................................................
22,500
Cruz capital.................................................
37,500
Cash and goodwill contributed by Cruz are recorded. Goodwill is Computed
algebraically as follows:

P15,000 + goodwill =
P15,000 + goodwill =
P15,000 + goodwill =
.80 goodwill
=
goodwill
=

20% (current capital + P15,000 + goodwill)


20% (P150,000 + P15,000 + goodwill)
P33,000 + .20 goodwill
P18,000
P22,500

12/31/07 Santos capital.......................................................


20,000
Reyes capital........................................................
10,000
Cruz capital..........................................................
7,500
Santos drawings..........................................
Reyes drawings...........................................
Cruz drawings.............................................
To close drawings accounts based on 20% of beginning capital
Balances: Santos, p100,000; Reyes, P50,000; and Cruz, P37,500.
12/31/07 Income summary.................................................
Santos capital..............................................
Reyes capital...............................................
Cruz capital.................................................
To allocate P44,000 profit as follows:

20,000
10,000
7,500

44,000
26,600
10,400
6,960
Santos
P20,000
6,600
______

Reyes

Cruz

Interest (20% of P100,000)..................................


15% of P44,000 profit..........................................
Balance of P17,400, 60:40...................................

P10,440

P 6,960

Total ....................................................................

P26,600

P10,440

P 6,960

Capital balances as of December 31, 2004:


Santos
Initial investment, 2006....................................... P80,000
2006 profit allocation...........................................
20,000
Additional investment..........................................
2007 drawings..................................................... (20,000)
2007profit allocation............................................ __26,600

Reyes
P80,000
(30,000)

Capitals, 12/31/07................................................ P106,600

P50,440

(10,000)
_10,440

Cruz
P37,500
(7,500)
__6,960
P36,960

60
Chapter 3
1/1/08

Goodwill.......................................................................
26,588
Santos capital......................................................
3,988
Reyes capital.......................................................
13,560
Cruz capital.........................................................
9,040
To record goodwill implied of Cruz's interest. In effect, the profit Sharing ratio is 15% to
Santos, 51% to Reyes (60% of 85% remaining after Santos's income), and 34% to Cruz
(40% of the 85% remaining after Santos' income). Diaz is paying P46,000, P9,040 in excess
of Cruz's capital (P36,960). The additional payment for this 34% income Interest indicates
total goodwill of P26,588 (P9,040/34%).

1/1/08

Cruz capital..................................................................
Diaz capital.........................................................
To transfer of capital purchase.

46,000

12/31/08 Santos capital................................................................


Reyes capital................................................................
Diaz capital...................................................................
Santos drawings..................................................
Reyes drawings...................................................
Diaz drawings.....................................................
To close drawings accounts based on 20% of beginning capitals.

22,118
12,800
9,200

46,000

22,118
12,800
9,200

12/31/08 Income summary..........................................................


Santos capital......................................................
Reyes capital.......................................................
Diaz capital.........................................................
To allocate profit for 2008 as follows:

61,000
31,268
12,800
9,200
Santos
P22,118
9,150
______

Reyes

Diaz

Interest (20% of P110,588)...........................................


15% of P61,000............................................................
Balance of P29,732, 60:40...........................................

P17,839

P11,893

Totals............................................................................

P31,268

P17,839

P11,893

Santos
P106,600
3,988

Reyes
P50,440
13,560

Diaz

(22,118)
__31,268

(12,800)
_17,839

P119,738

P69,039

Capital balances as of December 31, 2008:


12/31/07 balances.........................................................
Goodwill.......................................................................
Capital purchased.........................................................
Drawings......................................................................
Profit allocation............................................................
12/31/08 balances.........................................................

P46,000
(9,200)
_11,893
P48,693

1/1/09

Goodwill.......................................................................
14,321
Santos capital......................................................
2,148
Reyes capital.......................................................
7,304
Diaz capital.........................................................
4,869
To record implied goodwill. Diaz will be paid P53,562 (110% of the capital balance for his
interest. This amount is P4,869 in excess of the capital account. Since Diaz is only entitled
to a 34% share of profits and losses, the additional P4,869 must indicate that the partnership
as a whole is undervalued by P14,321 (P4,869/34%) which is treated as goodwill.
1/1/09
Diaz capital...................................................................
53,562
Cash....................................................................
53,562
To record settlement to Diaz.
Partnership Dissolution Changes in Ownership

Problem 3 12
Partnership Books Continued as Books of Corporation
Entries in the Books of the Corporation
(1) Inventories..........................................................................................
Land ...................................................................................................
Building. .............................................................................................
Accumulated depreciation bldg.........................................................
Accumulated depreciation equipment...............................................
Equipment...................................................................................
Jack capital.................................................................................
Jill capital...................................................................................
Jun capital...................................................................................
To adjust assets and liabilities of the partnership
to their current fair values.

26,000
40,000
20,000
20,000
30,000

(2) Cash ...................................................................................................


Jack capital.........................................................................................
Jill capital...................................................................................
Jun capital...................................................................................
To adjust capital accounts of the partners to 4:3:3 ratio.

4,000
18,000

(3) Jack capital.........................................................................................

100,000

20,000
58,000
34,800
23,200

20,200
1,800

Jill capital...........................................................................................
Jun capital...........................................................................................
Capital stock...............................................................................
To record issuance of stock to the partners.

75,000
75,000
250,000

New Books Opened for the New Corporation


Entries in the Books of the Partnership
(1) Inventories..........................................................................................
Land ...................................................................................................
Building. .............................................................................................
Accumulated depreciation bldg.........................................................
Accumulated depreciation equipment...............................................
Equipment...................................................................................
Jack capital.................................................................................
Jill capital...................................................................................
Jun capital...................................................................................
To adjust assets and liabilities of the partnership.

26,000
40,000
20,000
20,000
30,000

(2) Cash ...................................................................................................


Jack capital.........................................................................................
Jill capital...................................................................................
Jun capital...................................................................................
To adjust capital accounts of the partners.

4,000
18,000

20,000
58,000
34,800
23,200

20,200
1,800

62
Chapter 3

(3) Stock of JJJ Corporation.....................................................................


Accounts payable.................................................................................
Loans payable Jill.............................................................................
Cash in bank...............................................................................
Accounts payable........................................................................
Inventories..................................................................................
Land...........................................................................................
Building. .....................................................................................
Equipment...................................................................................
To record transfer of assets and liabilities to
The corporation and the receipt of capital stock

250,000
30,000
40,000

(4) Jack capital.........................................................................................


Jill capital...........................................................................................
Jun capital...........................................................................................
Stock of JJJ Corporation.............................................................
To record issuance of stock to the partners.

100,000
75,000
75,000

44,000
26,000
60,000
60,000
70,000
60,000

250,000

Entries in the Books of the Corporation


(1) To record the acquisition of assets and liabilities from the partnership:
Cash in bank. ......................................................................................
Accounts receivable.............................................................................
Inventories..........................................................................................
Land ...................................................................................................
Building (net). .....................................................................................
Equipment (net)...................................................................................
Accounts payable........................................................................

44,000
26,000
60,000
60,000
70,000
60,000
30,000

Loans payable.............................................................................
Capital stock...............................................................................

40,000
250,000

Problem 3 13
a. 1/1/06

12/31/06

Building
1,040,000
Equipment
320,000
Cash
240,000
Lim, capital
800,000
Sy, capital
800,000
(To record initial investment. Assets recorded at market value with two equal
capital balances.
Sy, capital
440,000
Lim, capital
240,000
Income summary
200,000
(The allocation plan specifies that Lim will receive 20% in interest [or 160,000
based on P800,000 capital balance] plus P80,000 more [since that amount is

Partnership Dissolution Changes in Ownership

greater than 15% of the profits from the period]. The remaining P440,000 loss is
assigned to Sy.)
1/1/07

Cash
300,000
Lim, capital (15%)
6,000
Sy, capital (85%)
34,000
Tan, capital
340,000
(New investment by Tan brings total capital to P1,700,000 after 2006 loss
[P1,600,000 P200,000 + P300,000]. Tans 20% interest is P340,000
[P1,700,000 x 20%] with the extra P40,000 coming from the two original
partners [allocated between them according to their profit and loss ratio].)

12/31/07

Lim, capital
206,800
Sy, capital
100,000
Tan, capital
100,000
Lim, drawings
206,800
Sy, drawings
100,000
Tan, drawings
100,000
(To close out drawings accounts for the year based on distributing 20% of each
partners beginning capital balances [after adjustment for Tans investment] or
P100,000 whichever is greater. Lims capital is P1,034,000 [P800,000 +
P240,000 P6,000])

12/31/07

Income summary
880,000
Lim, capital
Sy, capital
Tan, capital
(To allocate P880,000 income figure for 2007 as determined below.)

Interest (20% of P1,034,000


beginning capital balance)
15% of P880,000 income
60:40 split of remaining P541,200 income

Lim

Sy

Tan

P206,800
132,000
-

324,720

216,480

338,800
324,720
216,480

Total

P338,800

P524,720

P216,480

Capital balances as of December 31, 2007:


Initial 2006 investment
2006 profit allocation
Tans investment
2007 drawings
2007 profit allocation
12/31/07 balances
1/1/08

Lim
P800,000
240,000
(6,000)
(206,800)
338,800
P1,166,000

Sy
P800,000
440,000
(34,000)
(100,000)
324,720
P550,720

Tan, capital
456,480
Ang, capital
(To reclassify balance to reflect acquisition of Tans interest.)

Tan
P340,000
(100,000)
216,480
P456,480
456,480

64
Chapter 3

12/31/08

Lim, capital
233,200
Sy, capital
110,140
Ang, capital
100,000
Lim, drawings
233,200
Sy, drawings
110,140
Ang, drawings
100,000
(To close out drawings accounts for the year based on 20% of beginning capital
balances [above] or P100,000 [whichever is greater].)

12/31/08

Income summary
1,220,000
Lim, capital
416,200
Sy, capital
482,280
Ang, capital
321,520
(To allocate profit for 2008 determined as follows)
Lim
Sy
Ang
Interest (20% of P1,166,000 beg. capital)
P233,200
15% of P1,220,000 income
183,000
60:40 split of remaining P803,800
482,280
321,520
Totals
P416,200
P482,280
P321,520

b.

1/1/09

Ang, capital
678,000
Lim, capital (15%)
10,180
Sy, capital 85%)
57,620
Cash
745,800
(Angs capital is P678,000 [P456,480 P100,000 + P321,520]. Extra 10%
payment is deducted from the two remaining partners capital accounts.)

1/1/06

Building
1,040,000
Equipment
320,000
Cash
240,000
Goodwill
1,600,000
Lim, capital
1,600,000
Sy, capital
1,600,000
(To record initial capital investments. Sy is credited with goodwill of P1,600,000
to match Lims investment.)

12/31/06

Sy, capital
Lim, capital
Income summary

600,000
400,000
200,000

(Interest of P320,000 is credited to Lim [P1,600,000 x 20%] along with a base of


P80,000. The remaining amount is now a P600,000 loss that is attributed entirely
to Sy.)
1/1/07

Cash
300,000
Goodwill
450,000
Tan, capital
750,000
(Cash and goodwill being contributed by Tan are recorded. Goodwill must be
calculated algebraically.)

Partnership Dissolution Changes in Ownership

P300,000 + Goodwill = 20% (Current capital + P300,000 + Goodwill)


P300,000 + Goodwill = 20% (P3,000,000 + P300,000 + Goodwill)
P300,000 + Goodwill = P660,000 + .2 Goodwill
.8 Goodwill = P360,000
Goodwill = P450,000
12/31/07

Lim, capital
400,000
Sy, capital
200,000
Tan, capital
150,000
Lim, drawings
400,000
Sy, drawings
200,000
Tan, drawings
150,000
(To close out drawings accounts for the year based on 20% of beginning capital
balances: Lim- P2,000,000, Sy- P100,000, and Tan- P750,000.)

12/31/07

Income summary
Lim, capital
Sy, capital
Tan, capital
(To allocate P880,000 income figure as follows)
Lim

Interest (20% of P2,000,000)


beginning capital balance)
15% of P880,000 income
60:40 split of remaining P348,000
Totals

P400,000
132,000
P532,000

880,000
532,000
208,800
139,200
Sy

Tan

P208,800
P208,800

P139,200
P139,200

Lim
P1,600,000
400,000

Sy
P1,600,000
(600,000)

Tan

(400,000)
532,000
P2,132,000

(200,000)
208,800
P1,008,800

Capital balances as of December 31, 2007:


Initial 2006 investment
2006 profit allocation
Additional investment
2007 drawings
2007 profit allocation
12/31/07 balances
1/1/08

Goodwill
531,760
Lim, capital (15%)
Sy, capital (51%)
Tan, capital (34%)
(To record goodwill indicated by purchase of Tans interest.)

P750,000
(150,000)
139,200
P739,200
79,760
271,200
180,800

In effect, profits are shared 15% to Lim, 51% to Sy (60% of the 85% remaining after Lims
income), and 34% to Tan (50% of the 85% remaining after Lims income). Ang is paying
P920,000, an amount P180,800 in excess of Tans capital (P739,200). The additional payment for
this 34% income interest indicates total goodwill of P531,760 (P180,800/34%). Since Tan is
entitled to 34% of the profits but only holds 19% of the total capital, an implied value for the
66
Chapter 3

company as a whole cannot be determined directly from the payment of P920,000. Thus,
goodwill can only be computed based on the excess payment.
1/1/08

Tan, capital
Ang, capital
(To reclassify capital balance to new partner.)

920,000
920,000

12/31/08

Lim, capital
442,360
Sy, capital
256,000
Ang, capital
184,000
Lim, drawings
442,360
Sy, drawings
256,000
Ang, drawings
184,000
(To close out drawings accounts for the year based on 20% of beginning capital
balances [after adjustment for goodwill].)

12/31/08

Income summary
Lim, capital
Sy, capital
Ang, capital

1,220,000
625,360
356,780
237,860

To allocate profit for 2008 as follows:


Lim
Interest (20% of P2,211,760
beginning capital balance)
P442,360
15% of P1,220,000 income
183,000
60:40 split of remaining P594,640
Totals
P625,360

Sy

Ang

356,780
P356,780

237,860
P237,860

Sy
P1,008,00
271,200
( 256,000)
356,780
P1,380,780

Ang
P739,200
180,800
(184,000)
237,860
P973,860

Capital balances as of December 31, 2008:


12/31/07 balances
Adjustment for goodwill
Drawings
Profit allocation
12/31/08 balances

Lim
P2,132,000
79,760
(442,360)
625,360
P2,394,760

Ang will be paid P1,071,240 (110% of the capital balance) for her interest. This amount is
P97,380 in excess of the capital account. Since Ang is only entitled to a 34% share of profits and
losses, the additional P97,380 must indicate that the partnership as a whole is undervalued by
P286,420 (P97,380/34%). Only in that circumstance would the extra payment to Ang be justified:
1/1/09

Goodwill
Lim, capital (15%)
Sy, capital (51%)
Ang, capital (34%)
(To recognize implied goodwill.)

286,420
42,960
146,080
97,380

Partnership Dissolution Changes in Ownership

1/1/09

Ang, capital
Cash
(To record final distribution to Ang.

1,071,240
1,071,240

68
Chapter 4

CHAPTER 4
MULTIPLE CHOICE ANSWERS AND SOLUTIONS
4-1: a
Capital balances before realization
Loss on liquidation, P40,000
Cash distribution

PAR
P 20,000
( 20,000)
P

BOOGIE
P 16,000
( 12,000)
P 4,000

BIRDIE
P 10,000
( 8,000)
P 2,000

Capital balances before liquidation


Gain of P10,000 (150,000-140,000)
Cash distribution

PING
P 50,000
__6,000
P 56,000

PANG
P 50,000
__2,000
P 52,000

PONG
P 10,000
__2,000
P 12,000

Capital balances before liquidation


Loss of P40,000 (P140,000-P100,000)
Cash distribution

PING
P 50,000
( 24,000)
P 26,000

PANG
P 50,000
( 8,000)
P 42,000

PONG
P 10,000
( 8,000)
P 2,000

Capital balances before liquidation


Loss of P70,000 (P140,000-P70,000)
Balances
Absorption of Pong's deficiency, 6:2
Cash distribution

PING
P 50,000
( 42,000)
P8,000
( 3,000)
P 5,000

PANG
P 50,000
( 14,000)
P 36,000
( 1,000)
P 35,000

PONG
P 10,000
( 14,000)
( 4,000)
__4,000

COLT
MARK
Capital balances before liquidation (net of loans)P290,000 P200,000
Loss of P130,000, 4:3:3
( 52,000)
( 39,000)
Cash distribution
P238,000
P161,000

CLOCK
P220,000
( 39,000)
P181,000

4-2: c

4-3: b

4-4: a

4-5: b

4-6: c
Capital balances before liquidation
Loss of P60,000, 40:50:10
Cash distribution

JONAS
P160,000
( 24,000)
P136,000

CARLOS
P 45,000
( 20,000)
P 25,000

TOMAS
P 55,000
( 6,000)
P 49,000

Partnership Liquidation

4-7: a
Capital balances before liquidation
Loss of P100,000, 4:3:3
Cash distribution

ARIEL
P40,000
( 40,000)
P

BERT
P180,000
( 30,000)
P150,000

NORY
P23,000

OSCAR
P 13,500

15,000
( 30,900)
P 7,100

( 20,600)
( P7,100)

CESAR
P 30,000
( 30,000)
P

4-8: b
Capital balances before realization
Additional investment by Nory for
the unpaid liabilities (33,000-18,000)
Loss on realization (schedule 1)
Payment by Oscar to Nory
Schedule 1
Total capital before liquidation
Unpaid liabilities
Total loss on realization

P 36,500
15,000
P 51,500

4-9: d
Capital balances before liquidation (net)
Loss on realization (schedule 1) P27,500
Balances, cash distribution

BLACK
P99,000
( 13,750)
P85,250

Schedule 1:
Capital balances of white (net)
Cash received by White
White's share of total loss (30%)

WHITE
P 91,500
( 27,500)
P 64,000

GREEN
P138,000
_( 5,500)
P132,500

P 91,500
_83,250
P 8,250

Total loss on realization (P8,250/39%)

P 27,500

4-10: c
Capital balances before liquidation (net)
Loss on realization, P63,600
Balances
Unrecorded liabilities, P500
Balances
Elimination of Nora's deficiency
Payment to partners

ANA
P27,000
( 25,320)
P 1,680
( 200)
P 1,480
( 1,380)
P 100

EVA
P 43,000
( 25,320)
P 17,680
( 200)
P 17,480
( 1,380)
P 16,100

NORA
P 10,000
( 12,660)
( 2,660)
( 100)
( 2,760)
__2,760
P

Capital balances before liquidation (net)


Loss on realization (schedule 1) P45,000
Payment to partners

ARIES
P33,500
( 22,500)
P11,000

LEO
P 49,000
( 13,500)
P 35,500

TAURUS
P 36,500
( 9,000)
P 27,500

4-11: d

70
Chapter 4

Schedule 1:
Taurus capital (net)
Payment to Taurus
Share of total loss (20%)

P36,500
( 27,500)
P 9,000

Total loss on realization (9,000/20%)

P45,000

4-12: c
OLGA
Capital balances, June 11
P 4,200
Net loss from operation (squeeze)
( 2,800)
Capital balances, August 30 before
liquidation (48,500-25,600)
P 1,400
Loss on realization (47,500-30,000)
( 5,000)
Balances

TOTAL

MONA

NORA

P32,700

P15,000

P13,500

( 9,800)

( 4,200)

( 2,800)

P22,900

P10,800

P10,700

( 17,500)

( 7,500)

( 5,000)

P 5,400

P 3,300

P 5,700

_1,500

_____

_____

P 6,900

P 3,300

P 5,700

00)
Additional investment by Olga
_1,500
Balances
00)
Elimination of Olga's deficiency
_2,100
Payment to partners

______

( 1,260)

( 840)

P 6,900

P 2,040

P 4,860P

Capital balances before liquidation


Operating loss, P21,000
Drawings
Loans
Loss on realization, P12,000
Balances
Absorption of Tita's deficiency
Payment to Nora

RITA
P49,000
( 3,500)
( 10,000)

( 2,000)
P33,500
__1,500
P32,000

SARA
P18,000
( 7,000)
( 15,000)
8,000
( 4,000)
P

_____
P

TITA
P10,000
( 10,500)
( 20,000)
25,000
( 6,000)
( 1,500)
_1,500
P

CLARO

PEDRO

P45,000

P27,000

( 24,000)

( 24,000)

P21,000

P 3,000

4-13: b

4-14: a
ANDRO
Capital balances before liquidation
P50,000
Loss on realization
Accounts Receivable (P50,000 X 40%)
Investment (P30,000 - P20,000)
Equipment (P60,000-P30,000)
Total

P20,000
10,000
_30,000
P60,000

000)
Payment to partners

P38,000
4-15: c
Capital balances before liquidation (inclusive loans)
P19,000
Loss on realization, (squeeze)
( 15,400)
Capital balances - cash distribution

TOTAL
P47,500

MONA
P28,500

( 38,500)

( 23,100)

P 9,000

P 5,400P

3,600
Partnership Liquidation

Cash after realization


Less Liabilities (P36,000-P7,500)
Total capital after realization

P 37,500
( 28,500)
P 9,000

4-16: a
FF capital before distribution of net loss
Add: share of net loss (P10,000 X 40%)
FF capital before liquidation
Cash settlement to FF
FF share of total loss on realization (40%)

P100,000
_( 4,000)
96,000
( 80,000)
P 16,000

Total loss on realization (P16,000/40%)

P 40,000

Total capital before liquidation (P260,000-P10,000)


Add: Liabilities
Total assets
Cash before liquidation
Non-cash assets
Loss on realization
Cash to be realized

P250,000
_100,000
P350,000
( 50,000)
P300,000
( 40,000)
P260,000

4-17: d
Capital balances before realization (net)
P62,500
Loss on realization (squeeze)
( 25,000)
Capital balances after realization
(liabilities-unpaid)
P37,500
Elimination of CC's deficiency
( 19,000)
Balances
P18,500
Investment by DD
_____
Payment to EE
P18,500
4-18: d

TOTAL
P100,000

CC
P 15,000

DD
P22,500

( 125,000)

( 62,500)

( 37,500)

(P 25,000)

( 47,500)

( 15,000)

_______

__47,500

( 28,500)

(P43,500)

(P 25,000)
__43,500

______

_43,500

P 18,500

Total capital before liquidation


Liabilities
Total assets
Less: Cash balance before realization
Cash after payment of liabilities
payment of liabilities
Cash realized
Non-cash asset
Less: cash realized
Loss on realization

P 30,000
__1,500
P 31,500
P 11,100
1,500
( 11,600)

__1,000
P 30,500
_11,600
P 18,900

72
Chapter 4

4-19: d
TOTAL
Capital balances
P 80,000
Salary of LL (P600 X 8 months)
___4,800
Capital balances before liquidation
P 84,800
Loss on realization
Balances
Additional investment by NN
Payment to partners

LL

MM

NN

P 50,000

P 20,000

P 10,000

__4,800

_______

_______

P 54,800

P 20,000

P 10,000

( 44,880)
P 9,920
______
P 9,920

( 14,960)
P 5,040
_____
P 5,040

( 14,960)
(P 4,960)
__4,960
P

4-20: b
KK's total interest (P60,000-P10,000)
Less: Cash to be paid to KK
Share of total loss (1/3)

P 50,000
__10,000
P 40,000

Total loss on realization (P40,000/1/3)

P120,000

Total assets:
Total interest of the partners before liquidation:
JJ (P70,000+P30,000+P10,000)
KK (P60,000-P10,000)
LL (P30,000+P10,000)
Divide by
Total
Loss on realization
Cash to be realized

P110,000
50,000
__40,000

P200,000
______50%
P400,000
_120,000
P280,000

4-21: a
Capital balances, July 1
P 25,000
Advances to NN, August 1
OO Loan, September 1

TOTAL
P 75,000

NN
P 25,000

( 10,000)
20,000

( 10,000)

OO
P 25,000

20,000

Interest, December 31 (6%)


NN (5 mos.)
OO (4 mos.)
Compensation to PP
___2,500
Capital balances before liquidation
P 27,500
Loss on realization (squeeze)
( 17,550)
Cash distribution

250)
400
__2,500

250)

_______

400
_______

P 87,650

P 14,750

P 45,400

_56,250

( 17,550)

( 17,550)

P 35,000

( 2,800)

P 27,850

9,950
NN should pay P2,800 and this is to be divided to OO & PP equally or P1,400 each.

Partnership Liquidation

4-22: a
Capital balances before realization
P350,000
Loss on realization (squeeze)
_500,000
Capital balances after realization
(unpaid liabilities)
( 150,000)
Elimination of AS's deficiency
P150,000
Cash to be absorbed

TOTAL
P 950,000

PG
P350,000

JR
P250,000

( 1,000,000)

__20,000

( 200,000)

(P 50,000)

P 50,000

P 50,000

_______

( 90,000)

( 60,000)

(P 40,000)

(P 10,000)

RM
P500,000
( 490,000)
P 10,000

ST
P825,000
( 735,000)
P 90,000

TOTAL
P 27,500

LT
P 20,000

AM
P 5,000

__37,500

_18,750

__-9,375

P 65,000

P 38,750

P 14,375

AG
P 420,000

BM
P375,000

CP
P205,000

( 300,000)

( 300,000)

(200,000)

P 120,000

P 75,000

P 5,000

4-23: a
Capital balances before realization (net)
Loss on realization, P1,225,000
Payment to Partners
4-24: a
Capital balances before realization (net)
P 2,500
Gain on realization (squeeze)
__9,375
Capital balances after realization
P 11,875
4-25: c
Capital balances before realization (net)
P150,000
Loss on realization, P1,000,000
(200,000)
Balances

P(50,000)
Additional investment by DJ
50,000
4-26: a
Settlement to Uy
Uy capital before liquidation (net):
Uy capital
Receivable from Uy
Loss of Uy (50%)

P351,500
P553,500
( 132,000)

Total loss on realization (P70,000 50%)


__Uy__
553,500
(132,000)

CB before liquidation
Receivable from Uy
Loan to Wi
Salary payable to Vi
Interest before realization
Loss on realization
Settlement to partners

P140,000
__Vi__
452,500

__Wi__
486,000
( 40,500)

135,000
587,500
( 42,000)
545,500

421,500
( 70,000)
351,500

421,500
P 70,000

445,500
( 28,000)
417,500

__Total__
1,492,000
(132,000)
(40,500)
135,000
1,454,500
( 140,000)
1,314,500

74
Chapter 4

SOLUTIONS TO PROBLEMS
Problem 4 1
Case 1
Rivas and Briones
Statement of Liquidation
December 31, 2008
Partners'
Capitals
Assets

Rivas,

Briones,

Rivas

Loan

Loan

(90%)

Briones
Cash
(10%)
Balances before liquidation. . .
P10,000
Realization of assets and
distribution of loss...........
( 6,600)
Balances................................
3,400
Payment of liabilities.............
______
Balances................................
3,400
Offset Rivas' loan against his
capital deficiency............
______

Others Liabilities

P 20,000 P200,000 P132,000 P 18,000 P 20,000

P40,000

_134,000 ( 200,000) _______ _______ _______

( 59,400)

154,000

132,000

( 132,000) ______ ( 132,000)


22,000

18,000

20,000

( 19,400)

______ _______ _______


18,000

20,000

_______ _______ _______ ( 18,000) _______

( 19,400)
_18,000

Balances................................
22,000

20,000 ( 1,400)
3,400
Additional loss to Briones...... _______ _______ _______ _______ _______ __1,400
( 1,400)
Balances................................
22,000

20,000

2,000
Payment to partner................. P(22,000)

P(20,000)

P(2,000)
Case 2
Rivas and Briones
Statement of Liquidation
December 31, 2008
Partners'
Capitals
Assets

Rivas,

Briones,

Rivas

Cash
Others Liabilities
Loan
Loan
(30%)
Balances before liquidation. . . P20,000 P200,000 P132,000 P 18,000 P 20,000
P10,000
Realization of assets and
distribution of loss........... 134,000 ( 200,000) _______ ______ _______
( 19,800)
Balances................................ 154,000
132,000
18,000
20,000
9,800
Payment of liabilities............. ( 132,000) _______ ( 132,000) ______ _______
______
Balances................................
22,000

18,000
20,000
9,800
Offset loan against capital
deficiency........................ ________ _______ _______ ( 6,200) ( 9,800)
__9,800
Balances................................
22,000

11,800
10,200
Payment to partner................. P(22,000)

P(11,800) P(10,200)

(70%)

Briones

P40,000
( 46,200)
( 6,200)
_______
( 6,200)
__6,200

Partnership Liquidation
Case 3
Rivas and Briones
Statement of Liquidation
December 31, 2008
Partners'
Capitals
Assets

Rivas,

Briones,

Rivas

Briones
(50%)
Balances before liquidation........
P10,000
Realization of assets and
distribution of loss...............

Cash

Others

Liabilities

Loan

Loan

(50%)

P 20,000

P200,000

P132,000

P 18,000

P20,000

P40,000

_134,000 ( 200,000) _______

_______

______

( 33,000)

( 33,000)
Balances......................................
154,000

132,000
18,000
20,000
23,000)
Payment of liabilities.................. ( 132,000) _______ ( 132,000)
__
Balances......................................
22,000

18,000
20,000
23,000)
Offset Briones'' loan against
his capital deficiency........... _______ _______ _______ _______ ( 20,000)
_20,000
Balances......................................
22,000

18,000

3,000)
Additional loss to Rivas.............. _______ _______ _______ _______ _______
__3,000
Balances......................................
22,000

18,000

Payment to partner...................... P(22,000)

P(18,000)

( 7,000)
_
_______
( 7,000)
______
7,000
( 3,000)
4,000
P( 4,000)

Journal Entries
Case 1:
Cash..............................................................................................................
Rivas, Capital.................................................................................................
Briones, Capital..............................................................................................
Other Assets............................................................................................
Liabilities. .....................................................................................................
Cash........................................................................................................
Rivas, Loan....................................................................................................
Rivas, Capital..........................................................................................
Briones, Capital..............................................................................................
Rivas, Capital..........................................................................................
Briones, Loan.................................................................................................
Briones, Capital..............................................................................................
Cash ....................................................................................................
Case 2:
Cash..............................................................................................................
Rivas, Capital.................................................................................................
Briones, Capital..............................................................................................
Other Assets............................................................................................
Liabilities. .....................................................................................................
Cash........................................................................................................
Rivas, Loan....................................................................................................
Briones, Loan.................................................................................................
Rivas, Capital..........................................................................................
Briones, Capital......................................................................................
Rivas, Loan....................................................................................................
Briones, Loan.................................................................................................
Cash........................................................................................................
76
Chapter 4

Case 3:
Cash...................................................................................................
Rivas, Capital....................................................................................
Briones, Capital.................................................................................
Other Assets................................................................................
Liabilities...........................................................................................
Cash. ...........................................................................................
Briones, Loan....................................................................................

134,000
59,400
6,600
200,000
132,000
132,000
18,000
18,000
1,400
1,400
20,000
2,000
22,000
134,000
46,200
19,800
200,000
132,000
132,000
6,200
9,800
6,200
9,800
11,800
10,200
22,000

134,000
33,000
33,000
200,000
132,000
132,000
20,000

Briones, Capital...........................................................................
Rivas, Capital....................................................................................
Briones, Capital...........................................................................
Rivas, Loan........................................................................................
Rivas, Capital....................................................................................
Cash. ...........................................................................................

20,000
3,000
3,000
18,000
4,000
22,000

Problem 4 2
Blando and Castro
Statement of Liquidation
April 30, 2008
Partners'
Capitals
A s s e t s

Accounts

Blando,

Blando

Others

Payable

Loan

(60%)

Castro
Cash Receivables Inventory
(40%)
Balances before
liquidation.................... P 18,000
P99,000
Collection of
receivables and
distribution of loss........ _37,500
( 15,000)
Balances............................
84,000
Realization of
inventory and
distribution of
loss...............................
,000)
Balances............................
60,000
Realization of other
assets and distribution
of loss...........................
( 17,600)

P75,000

P90,000

P84,000

P42,000

P 24,000

P102,000

( 75,000)

_______ _______

_______

_______

( 22,500)

84,000

42,000

24,000

( 90,000) _______

_______

_______

42,000

24,000

_______

_______

42,000

24,000

17,100

_______

_______

24,000

17,100

55,500

_30,000

_______

85,500

_40,000

_______

_______

Balances............................ 125,500
42,400
Payment of accounts
payable......................... ( 42,000)
_______
Balances............................
83,500
42,400
Payments to partners.. P(83,500)
P(42,400)

_______

90,000

84,000

( 84,000)

_______ _______

( 42,000)

Partnership Liquidation

Problem 4 3
a.

Electric Company
Statement of Partnership Realization and Liquidation
June 30, 2008

79,500

( 36,000)
43,500

( 26,400)

P(24,000) P( 17,100)

Balances
14,000
Sale of
assets at a loss
( 8,000)
6,000
Payment to
creditors
______

Capital Balances
Volt Watt
30% 20%

Cash

Amp.
Loan

Noncash
Assets

Liabilities

Volt,
Loan

Amp
50%

20,000

15,000

135,000

30,000

10,000

80,000

36,000

_95,000

______

(135,000)

______

______

(20,000)

(12,000)

115,000

15,000

-0-

30,000

10,000

60,000

24,000

_(30,000)

______

_______

(30,000)

______

_______

______

10,000

60,000

24,000

85,000
15,000
-06,000
Offset Amp,
receivable
(15,000)
Payments to partners:
Loan
(10,000)
Capitals
_(75,000) ______
_______
,000)
Balances
-0-0-0-0b. (1) Cash
Amp, Capital
Volt, Capital
Watt, Capital
Noncash Assets
Sell noncash assets at a loss of P40,000.

-0-

(15,000)
_______
-0-

(10,000)
______

(45,000)

(24,000)

-0-

-0-

-0-

95,000
20,000
12,000
8,000
135,000

(2) Liabilities
Cash
Pay creditors.

30,000

(3) Amp, Capital


Amp, Loan
Offset receivable from Amp against his capital credit.

15,000

(4) Volt, Loan


Amp, Capital
Volt, Capital
Watt, Capital
Cash
Final lump-sum distribution to partners.

10,000
45,000
24,000
6,000

30,000

15,000

85,000

Note: All partners permitted Amp to offset his receivable against his capital credit. Alternatively, Amp
could be required to pay the partnership the P15,000 receivable; the partnership would then pay him an
additional P15,000 for his capital credit. In this case, an offset of the receivable against the capital credit is
reasonable, provided the receivable is not interest-bearing, Amp has a sufficient capital credit, Amp is
personally solvent, and the note is not secured against specific assts of Amp. The offset is not automatic,
but must be determined by the terms of the initial note, and by the partners.

78
Chapter 4

Problem 4 4

a.

b.

Bina, capital before liquidation.........................................................................................


Payment to Bina................................................................................................................
Loss absorbed by Bina (40%)...........................................................................................

P320,000
_128,000
P192,000

Loss on realization (P192,000 40%)..............................................................................

P480,000

AIDA, BINA & CELIA


Statement of Partnership Liquidation
January 1, 2008

Balances before liquidation.


Realization & dist. of loss. . .
Balances.............................
Settlement to partners..........

Cash

Other Assets

P80,000
240,000
320,000
(320,000)

P720,000
( 720,000)

_______

Aida
(5)
P320,000
( 240,000)
80,000
( 80,000)

Capital
Bina
Celia
(4)
(1)
P320,000
P160,000
( 192,000) ( 48,000)
128,000
112,000
( 128,000) ( 112,000)

Problem 4 5
a.

b.

LL, capital before liquidation...........................................................................................


Settlement to LL...............................................................................................................
Gain realized by LL (20%)...............................................................................................

P 70,000
__98,000
P 28,000

Total gain on realization (P28,000 20%).......................................................................


Other assets sold...............................................................................................................
Selling price ...................................................................................................................

P140,000
_500,000
P640,000

JJ, KK & LL
Statement of Liquidation

Balances before liquidation....


Realization & Dist. of gain....
Balances................................
Payment of liabilities.............
Payment to Partners...............
( 98,000)

Cash
P50,000
640,000
690,000
( 60,000)
(630,000)

Other
Assets
Liabilities JJ (4)
P500,000
P60,000 P180,000
( 520,000) _______ __56,000

60,000
236,000
( 60,000)
_______ _______ ( 236,000)

Partnership Liquidation

Problem 4 6
a.

BB.................................................... P160,000
CC.................................................... P20,000
DD................................................... P60,000

Capital
KK(4)
(LL(2)
P240,000
P70,000
__56,000
_28,000
296,000
98,000
( 296,000)

EE....................................................
b.

P 0

BB, CC, DD, & EE


Statement of Liquidation
Cash Liabilities
P
0
P60,000

Balances before liquidation....


P(180,000)
Advances by BB to pay liabilities
Deposit by DD....................... 60,000
________
Balances................................ 60,000
0)
Elimination of EE's deficiency
Elimination of DD's deficiency
Payment to partners................ 60,000

C a p i t a l
CC (10%)DD (20%)
EE (40%)
P80,000 (P120,000)

BB (30%)
P160,000

( 60,000)
60,000
______ _______

_______

220,000

( 90,000)
______ __( 90,000)

40,000

__60,000

80,000

( 60,000)

( 180,00

( 30,000)
( 30,000)
20,000

( 60,000)
120,000

180,000

Problem 4 7
Sayson and Company
Statement of Liquidation
Date

Assets

Liabilities
Accounts
Notes

Pea

P a r t n e r s' C a p i t a l s
Sayson
Zobel
Ayala

Pea
Cash

Noncash

Payable

Payable

Loan

(45%)

(30%)

(15%)

Balances before liquidation....


P1,650
Realization of assets and
distribution of gain...........
______

P 15,000

P155,250

P11,250

P9,000

P 1,500

P 75,345

185,000

( 155,250)

_______

______

______

17,850

11,900

______

Balances.................................
1,650
Payment of liabilities.............
______

200,000

11,250

9,000

1,500

93,195

98,398

( 14,993)

( 11,250)

( 9,000)

______

______

______

_______

1,500

93,195

98,398

( 14,993)

( 7,937)

( 5,292)

14,993
-

(10%)

Balances.................................
1,650
Additional loss to Sayson,
Zobel and Pea;
45:30:10............................
1,764)
Balances.................................
(114)
Offset Pea's loan against
his capital deficiency........
114
Balances.................................

( 20,250) ________
179,750

_______ ________ ________


179,750

_______ ________ ________


179,750

Payments to partners.............. P(179,750)

80

______

______

P 86,498 P(14,993)

1,500

85,258

93,106

______

( 114)

______

______

1,386

85,258

93,106

P(1,386) P(85,258) P(93,106)

_______
-

Chapter 4

Problem 4 8
a.

Art, Bea and Cid Partnership


Statement of Liquidation
June 4, 2008

(20%)
Balances before liquidation
(including Bea loan, P4,000)......
P10,000
Realization of assets
at a loss of P63,300...................
Unrecorded accounts payable..........
(100)
Payment to creditors........................
______
Balances.........................................
(2,760)
Eliminate Cid's deficit.....................
_2,760
Balances.........................................
Payment to Partners.........................
b.
2008
July 5

c.

Cash

Assets
Other

Liabilities

Partners' Capital
Art (40%) Bea (40%) Cid

P 6,000

P94,000

P20,000

P27,000

P43,000

30,000

( 94,000)

(25,320)
500

(25,320)
(200)

(12,660)
(200)

(20,500)

______

(20,500)

______

______

16,200

1,480

17,480

______

______

______

(1,380)

(1,380)

16,200
(16,200)

100
_( 100)

16,100
( 16,100)

Cash .............................................................................................
Art capital (P63,300 x 40%)...........................................................
Bea capital (P63,300 x 40%)..........................................................
Cid capital (P63,300 x 20%)...........................................................
Other assets...........................................................................
To record realization of other assets at a loss of P63,300.

30,700
25,320
25,320
12,660

Art capital (P500 x 40%)................................................................


Bea capital (P500 x 40%)...............................................................
Cid capital (P500 x 20%)................................................................
Liabilities..............................................................................
To record trade accounts payable.

200
200
100

Liabilities. ......................................................................................
Cash. .....................................................................................
To record payment of liabilities.

20,500

Art capital.......................................................................................
Bea capital......................................................................................
Cid capital.............................................................................
To eliminate Cid's capital deficit.

1,380
1,380

Art capital.......................................................................................
Bea capital......................................................................................
Cid capital. .....................................................................................
Cash. .....................................................................................
To record payments to partners to complete liquidation.

100
4,000
12,100

94,000

500

20,500

2,760

16,200

Cid's loss must be limited to P5,000, or P25,000 for the partnership (P5,000 / 20% = P25,000).
Because the liquidation of liabilities results in a loss of P500, only P24,500 may be lost on the
realization of other assets. This requires that other assets realize P69,500 (P94,000 24,500) to

enable Cid to receive P5,000 from the partnership to pay personal creditors in full.

Problem 4 9
KGB Partnership
Statement of Realization and Liquidation
Lump-sum Liquidation on June 30, 2008
-

Preliquidation balances
Sale of assets
and distribution
of 430,000 loss
Cash contributed
by B
Distribution of deficit
of insolvent partner:
20/60 (P2,000)
40/60 (P2,000)
Offset deficit with loan
Contribution by G
Payment of creditors
Distribution to K
Postliquidation
balances

Capital Balances
K
G
20%
40%
(240,000) (100,000)

B
40% (120,000)

(60,000)

86,000
(154,000)

172,000
72,000

172,000
52,000

(60,000)

(154,000)

72,000

50,000
2,000

Cash
50,000

Noncash
Assets
950,000

Liabilities
(480,000)

G
Loan
(60,000)

520,000
570,000

950,000
-0-

(480,000)

(480,000)

50,000
620,000

-0-

(2,000)
666
620,000
620,000
13,334
633,334
(480,000)
153,334
(153,334)
-0-

-0-0-

(480,000)
(480,000)

(60,000)
60,000
-0-

(153,334)
(153,334)

(480,000)
480,000
-0-

-0-

(153,334)
(153,334)
153,334

-0-0-0-

-0-

-0-

-0-

-0-

1,334
73,334
(60,000)
13,334
(13,334)
-0-

-0-0-0-

-0-

-0-

-0-

-0-

82
Chapter 4
KGB Partnership
Schedule of Distribution of Personal Assets
June 30, 2008
.
Personal assets, excluding partnership
capital and loan interests
Personal liabilities
Personal net worth, excluding
partnership capital and loan
interests
Contribution to partnership
Distribution from partnership
Personal capacity

500,000
(460,000)

600,000
(480,000)

700,000
(650,000)

40,000

120,000
(13,334)
-0- 106,666

50,000

153,334
193,334

-0- -0- -

Partnership Liquidation by Installment

CHAPTER 5
MULTIPLE CHOICE ANSWERS AND SOLUTIONS
5-1: b
Capital balances before liquidation
Loan balances
Total interest
Possible loss (40,000+10,000)
Balances
Additional loss to RJ & SJ, 5:3
Cash distribution

RJ
P22,000
_10,000
32,000
( 25,000)
7,000
( 1,250)
P 5,750

SJ
P30,000
______
30,000
( 15,000)
15,000
( 750)
P14,250

TJ
P 8,000
______
8,000
( 10,000)
( 2,000)
__2,000
P

AR
P 5,500

BR
P 5,150

CR
DR
P 6,850

_1,000

_____

_____

6,500
( 6,800)

5,150
( 5,100)

6,850
( 3,400)

( 300)
___300

50
( 150)

3,450
( 100)

_____

( 100)
___100

3,350
_( 67)

P 3,283

5-2: a
Capital balances
P 4,500
Loan balances
_____
Total interest
Possible loss (23,000-6,000)
( 1,700)
Balances
Additional loss to BR, CR, DR, 3:2:1
( 50)
Balances
Additional loss to CR & DR, 2:1
_( 33)
Payment to partners
P 2,717
Total liabilities
Total Capital
Total Assets

5-3: c

P 1,000
_22,000
P23,000

Capital balances
P25,000
Loan balances
Advances

DD
P40,000

BALANCES
EE
FF
GG
P30,000
P15,000

5,000
_____

10,000
_____

( 4,500)

45,000

40,000

10,500

____50%

____30%

____10%

90,000

133,333

105,000

_____
90,000

_____
133,333

( 91,667)
105,000

_____

( 28,333)

_____

90,000

105,000

105,000

_____

(15,000)

( 15,000)

P90,000

P90,000

P90,000

500)
Total interest
22,500
Divided by P/L Ratio
____10%
Loss Absorption balances
225,000
PI - TO GG
Balances
133,333
PII - TO EE & GG, 30:10
( 28,333)
Balances
10,500
PIII - TO EE, FF, GG, 3:1:1
( 15,000)
Balances
P90,000
PIV - P/L Ratio

__ __

84
Chapter 5

DD
PI - To GG
P 9,167
PII - To EE (28,833 X 30%)
GG (28,833 X 10%)
PIII To EE (15,000 X 30%)
FF (15,000 X 10%)
GG (15,000 X 10%)
__1,500

_____

P 8,433

4,500

_____

1,500
_____

P12,933

P 1,500

EE

FF

Total
P13,500
PIV - P/L Ratio
DD
Distribution of P18,000
PI - TO GG
P 9,167
PII - TO EE & GG, 3:1, P8,833
__2,208
Cash distribution
P11,375

CASH PAYMENT
EE
FF
GG

GG

_____

_6,625

_____

P 6,625

5-4: a
Capital balances before liquidation
Loss on realization, P40,000
Capital balances before cash distribution

TAN
P40,000
( 16,000)
24,000

LIM
P65,000
( 16,000)
49,000

WAN
P48,000
( 8,000)
40,000

Possible loss, P90,000


Balances
Additional loss to Lim & Wan, 4:2
Cash distribution

( 36,000)
( 12,000)
_12,000
P

( 36,000)
13,000
( 8,000)
P 5,000

( 18,000)
22,000
( 4,000)
P18,000

Capital balances before cash distribution


Possible loss (90,000+3,000)
Balances
Additional loss to Lim & Wan, 4:2
Cash distribution

TAN
P24,000
( 37,200)
( 13,200)
_13,200
P

LIM
P49,000
( 18,600)
30,400
( 8,800)
P21,600

WAN
P40,000
( 18,600)
21,400
_( 4,400)
P17,000

CARPIO
P72,000
( 5,000)
67,000
( 55,000)
12,000
( 6,000)
P 6,000

LOBO
P54,000
( 5,000)
49,000
( 55,000)
( 6,000)
__6,000
P

JACOB

SANTOS

P40,000

P72,000

( 15,000)

( 9,000)

( 1,000)

( 600)

24,000

62,400

__8,000

_____

32,000

62,400

( 45,000)

27,000

( 13,000)

35,400

_13,000

( 7,800)

P27,600

A
P16,200

B
P12,000

C
P37,700

_____

___160

___240

5-5: b

5-6: d
Tan (14,000 X 40%)
Lim (14,000 X 40%)
Wan (14,000 X 20%)

P5,600
P5,600
P2,800

5-7: a
Capital balances before liquidation
Goodwill written-off
Cash balance
Possible loss (100,000+10,000), 110,000
Capital balances before liquidation
Additional loss to Carpio
Cash distribution
Partnership Liquidation by Installment

5-8: d
HERVAS
Capital balances before liquidation
P 7,000
Loss on realization (120,000-90,000)
( 6,000)
Liquidation expenses, P2,000
( 400)
Capital balances before cash distribution
63,600
Loan balances
_____
Total interest
63,600
Possible Loss (210,000-120,000)
( 18,000)
Balances
45,600
Additional loss to Santos & Hervas
( 5,200)
Cash distribution
P40,400

5-9: d
Capital balances before liquidation
P17,700
Salary payable

_______
Balances

16,200

12,000

37,860

( 600)

( 600)

( 600)

15,600

11,400

37,260

( 150)

( 150)

( 150)

15,450

11,250

37,110

12,000

14,400

_____

27,450

25,650

37,110

( 27,000)

( 27,000)

( 27,000)

( 1,350)

10,110

( 780)

__1,350

( 780)

( 330)
___330

_____

9,330
( 330)

40)
Loss on realization (P2,400)
( 600)
Balances
17,340
Liquidation expenses (P600)
( 150)
Balances
17,190
Loan balances
__9,600
Total interest
26,790
Possible Loss (126,000-18,000)
( 27,000)
Balances

450

0)
Additional loss to A & C
____210
Balances
Additional loss to C
_____
Cash distribution

P 9,000

5-10: a

Total interest
Profit and Loss ratio
Loan absorption balances
Priority I - to Sy
Balances
Priority II - to Sy & Less
Total

DY
P22,000
2/4
44,000
_____
44,000
_____
P44,000

DY
Priority I - to Sy (6,000 X 1/4)
Priority II - to Sy (12,000 X 1/4)
to Lee (12,000 X 1/4)
Total

_____
P

BALANCES
SY
P15,500
1/4
62,000
( 6,000)
56,000
( 12,000)
P44,000

LEE
P14,000
1/4
56,000
_____
56,000
( 12,000)
P44,000

CASH PAYMENTS
SY
LEE
1,500

3,000

_____
_3,000
P 4,500
P 3,000

86
Chapter 5
Further cash distribution, profit and loss ratio
Cash distribution to Dy
Divided by Dy's Profit and Loss ratio
Amount in excess of P7,560
Total payment under priority I & II
Total cash distribution to partner

P 6,250
2/4
12,500
__7,500
P20,000

5-11: d
Cash before liquidation
Cash realized

P12,000
_32,000

Total
Less:

44,000

Payment of liquidation expense


Payment of liability
Payment to partners (Q 5-10)
Cash withheld

P 1,000
5,400
20,000

_26,400
P17,600

5-12: c
Loss absorption balances:
Cena (18,000/50%)
Batista (27,000/30%)
Excess of Batista
Multiply by Batista's Profit & Loss ratio
Priority I to Batista

P36,000
90,000
54,000
____30%
P16,200

5-13: c
Capital balances
Loan balances
Total interest
Divided by Profit and Loss Ratio
Loss Absorption balances
Priority I to CC
Balances
Priority II to BB & CC, 2:1
Total interest

AA
P15,000
10,000
25,000
2/5
62,500
_____
62,500
_____
P62,500

AA
Priority I to CC (12,500 X 1/5)
Priority II to BB (25,000 X 2/5)
to CC (25,000 X 1/5)
Total
Priority III P/L Ratio
Cash distribution to CC:
Priority I
Priority II (12,000-2,500) X 1/3
Total cash paid to CC
Partnership Liquidation by Installment

____
P

BALANCES
BB
P30,000
_5,000
35,000
2/5
87,520
_____
87,520
( 25,000)
P62,500

CC
P10,000
10,000
20,000
1/5
100,000
( 12,500)
100,000
( 25,000)
P62,500

CASH PAYMENTS
BB
CC

2,500
10,000

_____
_5,000
P10,000
P 7,500
P2,500
3,167
P5,667

5-14: c

Capital balances
P 30,000
Loan balances
______
Total interest
_30,000
Divided by Profit and Loss Ratio
_____10%
Loss Absorption balances
300,000
Priority I to LL
______
Balances

JJ
P 60,000

BALANCES
KK
LL
MM
P 64,500
P 54,000

_18,000

_30,000

______

_78,000

_94,500

_54,000

____40%

_____35%

_____15%

195,000

270,000

360,000

______

______

( 60,000)

195,000

270,000

300,000

300,000
Priority II to LL, MM, 15:10
( 30,000)
Balances
270,000
Priority II to KK, LL, MM, 35:15:10
( 75,000)
Total
P195,000

______

______

( 30,000)

195,000

270,000

270,000

______

( 75,000)

( 75,000)

P195,000

P195,000

P195,000

______

CASH PAYMENT
KK
LL
MM

9,000

4,500

1,750

11,250
______
______

P 1,750

LL
P 9,000

4,500

3,000

_____

__7,350

___3,150

__2,100

P 7,350

P 16,650

P 5,100

ARCE
P 20,000
_10,000
_32,000
_____50%
64,000
______
64,000
______
P 64,000

BALANCES
BELLO
P 24,900
______
_24,900
_____30%
83,000
( 8,000)
75,000
( 11,000)
P 64,000

CRUZ
P 15,000
______
_15,000
_____20%
75,000
______
75,000
( 11,000)
P 64,000

JJ
Priority I to LL (30,000 X 15%)
Priority II to LL (30,000 X 15%)
to MM (30,000 X 10%)
Priority II to KK (75,000 X 35%)
to LL (75,000 X 15%)
to MM (75,000 X 10%)
___7,500
Total
P 10,500

P 24,750

Further cash distribution, Profit and Loss ratio


Cash distribution to Partners (P38,100-9,000), P29,100

Priority I to LL
P 9,000
Priority II to LL, MM, 15:10
Priority II to KK, LL, MM, 35:15:10
(29,100-16,500), 12,600
__12,600
Cash distribution
P 29,100

JJ

KK

MM TOTAL

5-15: a

Capital balances
Loan balances
Total interest
Divided by Profit and Loss Ratio
Loss Absorption balances
Priority I to Bello
Balances
Priority II to Bello & cruz, 3:2
Total
88
Chapter 5

ARCE
P - I to Bello (8,000 X 30%)
P - II to Bello (11,000 X 30%)
to Cruz (11,000 X 20%)

_____

CASH PAYMENTS
BELLO
CRUZ
2,400

3,300

_____
_2,200

Total

P 5,700

P2,200

Further Cash distribution, Profit and Loss ratio


Based on the above cash priority program, the P2,000 is only a partial payment to Bello who
is entitled to a maximum of P2,400 under Priority I. Only after satisfying Priority I, Cruz will
receive payment and only after P7,900 has been distributed to Bello and Cruz will Arce receive
payment. Therefore no payments are made to Arce and Cruz.

5-16: a
Cash paid to Arce
Divide by Profit & Loss ratio
Amount in excess of P7,900
Add: cash paid under PI and PII
Total cash distribution to partners
Cash paid to Creditor (30,000-10,000)
Total
Less cash before realization
Cash realized from sale of asset

P2,000
_____5%
40,000
_7,900
47,900
20,000
67,900
_6,000
P61,900

Cash distribution to Cruz


Divide by profit and loss ratio
Cash distribution under Priority II
Multiply by Bello's Profit and Loss ratio
Cash distribution to Bello under Priority II
Cash distribution to Bello under Priority I
Total cash distribution to Bello

P 6,200
2/5
15,500
3/5
9,300
__2,400
P11,700

5-17: b

5-18: b
BALANCES
MONZON
NIEVA
NIEVA
Total Interest
Profit and Loss ratio
Loss absorption balances
Priority I - to Nieka
_2,500
Total
P2,500

CASH PAYMENT
MONZON

P22,500
_____60%
37,500
______

P17,500
_____40%
43,750
( 6,250)

_____

P37,500

P37,500

Further cash distribution - Profit and Loss ratio


All the P2,000 should be paid Nieva, since she is entitled to P2,500 under Priority I
Partnership Liquidation by Installment

5-19: b
Cash distribution
PI to Nieva (2,500-2,000)
Balances, 6:40
Cash distribution
5-20: a

CASH MONZON
P12,500

( 500)

_12,000
__7,200
P

P 7,200

NIEVA

500
_4,800
P5,300

Cash before liquidation


June: Cash realized
Payment to creditor
Payment to Partners
Cash balances, June 30
July: Cash realized
Payment of liquidation expense
Payment to Partners
Cash balances, July 31
Aug: Cash realized
Cash distribution for August,
Profit and Loss ratio

P 5,000
18,000
( 20,000)
__2,000
1,000
12,000
( 500)
( 12,500)

_22,500
P22,500

Distribution to Partners - August


Monzon (22,500 X 60%)

P13,500

Nieva (22,500 x 40%)

P 9,000

90
Chapter 5

SOLUTIONS TO PROBLEMS
Problem 5 1
Suarez, Tulio and Umali
Statement of Liquidation
January 1 to april 31, 2008
Assets
Cash

Others Liabilities

Tulio,
Loan

Umali,
Loan

Umali (25%)
Balances before liquidation. P 2,000.00 P46,000.00 P6,000.00 P5,000.00 P2,500.00
P7,500.00
January Installment:
Realization of assets and

Partners' Capitals
Suarez (40%) tulio (35%)
P14,450.00 P12,550.00

distribution of loss.... 10,500.00


( 375.00)
Balances......................... 12,500.00
7,125.00
Payment of expenses of
realization and distribution
to partners...................... ( 500.00)
( 125.00)
Balances......................... 12,000.00
7,000.00
Payment of liabilities..... ( 6,000.00)
_______
Balances......................... 6,000.00
7,000.00
Payments to partners
(Schedule 1).............. ( 4,000.00)
_______
Balances......................... 2,000.00
7,000.00
February Installment:
Realization of assets and
distribution of loss.... 6,000.00
( 250.00)
Balances......................... 8,000.00
6,750.00
Payment of expenses of
realization and distribution
to partners...................... ( 750.00)
( 187.50)
Balances......................... 7,250.00
6,562.50
Payments to partners
(Schedule 2).............. ( 6,000.00)
_______
Balances......................... 1,250.00
6,562.50
March Installment:
Realization of assets and
distribution of loss.... 10,000.00
( 1,250.00)
Balances......................... 11,250.00
5,312.50
Payment of expenses of
realization and distribution
to partners...................... ( 600.00)
( 150.00)
Balances......................... 10,650.00
5,162.50
Payments to partners,
P & L ratio................( 10,150.00)
( 2,037.50)
Balances.........................
500.00
3,125.00
April Installment:
Realization of assets and
distribution of loss.... 4,000.00
( 2,000.00)
Balances......................... 4,500.00
1,125.00
Payment of expenses of
realization and distribution
to partners...................... _(400.00)
( 100.00)
Balances......................... 4,100.00
1,025.00

( 12,000.00) _______

_______

______

600.00) (

525.00)

34,000.00

6,000.00

5,000.00

2,500.00

13,850.00

_______

_______

_______

_______

34,000.00

6,000.00

5,000.00

2,500.00

13,650.00

11,850.00

_______ ( 6,000.00)

_______

_______

_______

________

34,000.00

5,000.00

2,500.00

13,650.00

11,850.00

_______

_______
11,850.00

_______
34,000.00

_______

( 7,000.00) _______
27,000.00

_______ ______
27,000.00

_______ ______
27,000.00

( 3,812.50) ( 187.50)
2,312.50

13,650.00

_______

_______

__(400.00) (

1,187.50

2,312.50

13,250.00

_______

2,312.50

12,950.00

_______
1,187.50

( 1,187.50) ( 1,812.50)

500.00

( 15,000.00)______

______

______

500.00

_______ ______

______

_______

500.00

______

______

( 12,000.00)______

______

______

12,000.00
______
12,000.00

200.00) (

1,187.50

12,000.00

12,025.00

( 500.00)

______

______

______

______

175.00)

350.00)

11,500.00

300.00) (

262.50)

11,237.50

( 1,650.00) ( 1,350.00)
11,300.00

9,887.50

( 2,000.00) ( 1,750.00)
9,300.00

240.00) (
9,060.00

8,137.50

210.00)
7,927.50

( 4,060.00) ( 3,552.50)
5,000.00

4,375.00

( 3,200.00) ( 2,800.00)
1,800.00

___(160.00) (
1,640.00

1,575.00

140.00)
1,435.00

Final Payments to partnersP(41,100.00)_____

_____

_____

_____

P( 1,640.00) P( 1,435.00)

P(1,025.00)

Partnership Liquidation by Installment

Schedule 1

Capital balances......................................
Loan balances.........................................
Total interests..........................................
Possible loss (P2,000 + P34,000)...........
Balances..................................................
Additional loss to Tulio and Umali 35:25
Payments to partners...............................
Apply to loan..........................................

Suarez (40%)
P13,650.00
_____ _
13,650.00
( 14,400.00)
( 750.00)
___750.00

__ __

Tulio (35%)Umali (25%)


P11,850.00 P7,000.00
__5,000.00 _2,500.00
16,850.00
9,500.00
( 12,600.00) ( 9,000.00)
4,250.00
500.00
( 437.50) ( 312.50)
P 3,812.50 P 187.50
P 3,812.50 P 187.50

Schedule 2
Capital balances......................................
Loan balances.........................................
Total........................................................
Possible loss (P1,250 + P27,000)...........
Payments to partners...............................
Apply to loan..........................................
Apply to capital......................................

92
Chapter 5

Suarez (40%)
P12,950.00

12,950.00
( 11,300.00)
P 1,650.00

P 1,650.00

Tulio (35%)Umali (25%)


P11,237.50 P6,562.50
__1,187.50 _2,312.50
12,425.00
8,875.00
( 9,887.50) ( 7,062.50)
P 2,537.50 P1,812.50
_1,187.50 _1,812.50
P 1,350.00
P

Problem 5 2
Miller and Bell Partnership
Statement of Partnership Realization and Liquidation

Cash
25,000
40,000

Balances
Sale of inventory
Payment to
creditors
(10,000)
55,000
Payments to
partners
(Schedule 1)
(50,000)
5,000
Sale of inventory 30,000
Payment to
creditors
( 5,000)
30,000
Offset deficit
with loan
______
30,000
Payments to
partners:
Loan
( 6,000)
Capitals
(24,000)
Balances
0

Inventory
120,000
( 60,000)

Accounts
Payable
15,000

Bell
Loan
60,000

Capital
Miller
Bell
80%
20%
65,000
5,000
(16,000) (4,000)

______
60,000

(10,000)
5,000

______
60,000

______
49,000

______
60,000
( 60,000)

______
5,000

(49,000)
11,000

_(1,000)
______
48,000
1,000
(24,000) 6,000)

______
0

( 5,000)
0

______
11,000

______
______
24,000 (5,000)

______
0

______
0

( 5,000)
6,000

______ (5,000)
24,000

______
0

______
0

( 6,000)
______
0

______
1,000

(24,000)
0

______
0

Schedule 1:
Miller and Bell Partnership
Schedule of Safe Payments to Partners

Capital and loan balances


Possible loss of 60,000 on remaining inventory
Safe payment

Partnership Liquidation by Installment

Problem 5 3
HORIZON PARTNERSHIP
Statement of realization and Liquidation

Miller
80%
49,000
(48,000)
1,000

Bell
20%
61,000
(12,000)
49,000

May July, 2008


Assets

Partners Capital
TT
PP
(1/3) (1/3)

Cash

Other

Liabilities

SS
(1/3)

Balances before liquidation


90,000
May sale of assets at a loss of P30,000
(10,000)

20,000

280,000

80,000

60,000

70,000

75,000

(105,000)

______

(10,000)

(10,000)

Balances
80,000
Payment to creditors

95,000

175,000

80,000

50,000

60,000

(80,000)

______

(80,000)

______

______

15,000

175,000

50,000

60,000

(15,000)

______

______

______

175,000

50,000

60,000

(12,000)

(12,000)

38,000

48,000

______

(10,000)

Balances
80,000
Payments to PP (Exhibit A)
(15,000)
Balances
65,000
June sale of assets at a loss of P36,000
(12,000)
Balances
53,000
Payment to partners (Exhibit A)
(15,000)
Balances
38,000
July sale of remaining assets at a loss of
P33,000
(11,000)
Balances
27,000
Payment to partners
(27,000)

______

25,000

(61,000)

25,000

114,000

(25,000)

______

114,000

38,000

38,000

(114,000)

(11,000)

(11,000)

81,000

27,000

27,000

(81,000)

(27,000)

(27,000)

SS

TT

60,000

70,000

1
60,000

1
70,000

______

______

60,000

70,000

______

(10,000)

60,000

60,000

81,000

______

______

______

Exhibit A Cash distributions to partners during liquidation:

Capital account balances before liquidation


90,000
Income sharing ratio
Loss absorption balances
90,000
Required reduction to bring
capital account balance for PP
to equal the next highest balance for TT PI.
(20,000)
Balances
70,000
Required reduction to bring the balances for
TT and PP to equal the balance for SS PII.
(10,000)
Balances
60,000
Summary of cash distribution program:
To creditors before partners receive anything
To partners:
(1) First distribution to PP
20,000
(2) Second distribution to TT and PP equally
10,000
(3) Any amount in excess of $120,000
to the three partners in incomesharing ratio

PP

80,000
20,000
20,000

10,000

1/3

1/3

1/3

b. After the cash distribution in June, the partners capital accounts had balances corresponding to the income-sharing
ratio (38,000 each). From this point on any cash payments to partners may be made in the income-sharing ratio or
equally in this problem. In other words, after the creditors are paid and TT and PP receive 10,000 and 30,000,
respective, any additional cash that becomes available may be paid to the three partners equally.

94
Chapter 5

Problem 5 4
1. X, Y and Z
Cash Priority Program
January 1, 2008

X
Total

Balances
Y

Capital balances..................................
Loan balances.....................................

P60,000
22,5000

P45,000
15,000

P20,000
6,500

Total interests......................................

P82,500

P60,000

P26,500

Loss absorption balances.................... P165,000


Priority I to Y...................................
............................................................P10,500

P200,000
(35,000)

P132,500

Balances............................................. 165,000
Priority II to X and Y.......................
(32,500)
............................................................26,000

165,000
(32,500)

Total.................................................... P132,500
............................................................P36,500

P132,500

X (50%)

Cash Payments
Y (30%) Z (20%)

P10,500

132,500
________

P16,250

9,750

P132,500

P16,250

P20,250

Any amount in excess of P36,500.......


............................................................ 100%

50%

30%

20%

2. January
Available for distribution...............................
Priority I to Y..............................................

Cash
P 7,500
( 7,500)

Payment to partner.........................................
February.......................................................
Available for distribution...............................
Priority I to Y (P10,500 P7,500)..............
Priority II to X and Y; 5:3...........................

Cash
P20,000
( 3,000)
( 17,000)P10,625

Payments to partners......................................

P 7,500

Y
P 3,000
6,375

P10,625 P 9,375
Cash
P45,000
( 9,000)
( 36,000)

Payments to partners......................................
April..............................................................
Available for distribution...............................
Excess; 5:3:2..................................................

P 7,500

Payments to partners......................................
March............................................................
Available for distribution...............................
Priority II to X and Y; 5:3
(P26,000 P17,000).................................
Excess; 5:3:2..................................................

Cash
P15,000
( 15,000)

P 5,625
18,000

P 3,375
10,800

P7,200

P23,625

P14,175

P7,200

P 7,500

P 4,500

P3,000

P 7,500

P 4,500

P3,000

Partnership Liquidation by Installment

Problem 5 5
AB, CD & EF Partnership
Statement of Partnership Realization and Liquidation
Able
Loan

Cash
20%
Balances before liquidation
18,000 30,000
74,000
January transactions:
1. Collection of accounts
receivable at loss
of 15,000
51,000
( 3,000)
2. Sale of inventory at
loss of 14,000
38,000
( 2,800)
3. Liquidation expenses paid ( 2,000)
( 400)
4. Share of credit memorandum
600
5. Payments to creditors
( 50,000) _____
______
55,000 30,000
68,400
Sale payments to partners
(Schedule 1
( 45,000) ______
(18,400)
10,000 30,000
50,000
February transactions:
6. Liquidation expenses paid ( 4,000) ______
( 800)
6,000 30,000
49,200
Safe payments to partners
(Schedule 2)
-0- _____
0
6,000 30,000
49,200
March transactions:
8. Sale of mac. & equip. at a
loss of 43,000
146,000
( 8,600)
9. Liquidation expenses paid ( 5,000) ______
( 1,000)
147,000 30,000

Other
Assets
307,000

Accounts CD
Payable Loan

AB
50%

53,000 20,000 118,000

Capital
CD EF
30%
90,000

( 66,000)

( 7,500) ( 4,500)

( 52,000)

( 7,000) ( 4,200)
( 1,000) ( 600)
( 3,000)

1,500

900

______

_____

-0- 20,000 104,000

81,600

______ (50,000) _____


189,000

_____ ______ (20,000) ______ ( 6,600)


189,000

-0-

-0- 104,000

______ ______ ______


189,000

-0-

-0-

( 2,000) ( 1,200)

-0- 102,000

73,800

-0- 102,000

73,800

______ ______ ___


189,000

75,000

(189,000)

( 21,500) (12,900)

_______ ______ ______

( 2,500) ( 1,500)

-0-

-0-

-0-

78,000

59,400

39,600
10. Offset AB's loan
receivable against capital
(30,000)
( 30,000)
Payments to partners
(147,000) ______ _______ ______ ______ ( 48,000) (59,400)
(39,600)
Balances at end of liquidation
0
0
0
0
0
0
0

96
Chapter 5

Partnership
Schedules of Safe Payments to Partners
Schedule 1: January
Capital and loan balancesa
Possible loss:
Other assets (189,000) and possible liquidation
costs (10,000)
Balances
Absorption of AB's potential deficit balance
CD : (25,500 x 3/5 = 15,300)
EF : (25,500 x 2/5 = 10,200)
Safe payment
a = (104,000) capital less 30,000 loan receivable
= (81,600) capital plus 20,000 loan payable
= (68,400) capital
Schedule 2: February
Capital and loan balancesb
Possible loss:
Other assets (189,000) and possible liquidation
costs (6,000)
Absorption of AB's potential deficit balance
CD : (25,500 x 3/5 = 15,300)
EF : (25,500 x 2/5 = 10,200)
Safe payment
b = (102,000) capital less 30,000 loan receivable
= (73,800) capital
= (49,200) capital

AB
50%

CD
30%

EF
20%

P74,000

P101,600

P68,400

( 99,500)
( 25,500)
25,500

( 59,700)
41,900

( 39,800)
28,600

( 15,300)
_______
P 26,600

( 10,200)
P 18,400

______
P
-0-

72,000
( 97,500)
( 25,500)
25,500
_______
0

73,800

49,200

( 58,500)
15,300

( 39,000)
10,200

( 15,300)
________
0

( 10,200)
0

Partnership Liquidation by Installment

Problem 5 6
1.

M, N, O and P
Cash Priority Program
January 1, 2008
M

Total
Capital balances. .P 70,000
Loan balances... 20,000
Total interests......P 90,000
Loss absorption
balances..........P240,000
Priority I to O... _______
P20,000
Balances.............. 240,000
Priority II to O
and P............... _______
10,000
Balances.............. 240,000
Priority III to
M, O and P.....( 40,000)
25,000
Total....................P200,000
P55,000

Balances
N
O

Cash Payments
M (3/8) N (3/8) O (1/8)
P (1/8)

P 70,000 P 30,000
5,000
25,000
P 75,000 P 55,000

P 20,000
15,000
P 35,000

P200,000 P440,000 P280,000


_______ ( 160,000) ________
200,000

280,000

240,000

P20,000

5,000

P5,000

280,000

_______ ( 40,000) ( 40,000)


200,000

240,000

_______ ( 40,000) ( 40,000)P15,000


P200,000 P200,000

P200,000P15,000

Any amount in excess of P55,000


8/8

3/8

5,000

5,000

P30,000

P10,000

3/8

1/8

1/8

2.
Schedule 1

Available for distribution......................


Priority I to O....................................
Priority II to O and P; 1:1..................
Payments to partners............................
Apply to loan........................................
Apply to capital....................................

Cash
P25,000
( 20,000)
( 5,000)

________

_______

P20,000
2,500
P2,500
P22,500
( 22,500) ( 2,500)

Schedule 2
Cash

Available for distribution......................


P40,000
Priority II to O and P; 1:1..................
( 5,000
Priority III to M, O and P; 3:1:1........ ( 25,000)
Excess, 3:3:1:1.....................................
( 10,000)
..............................................................1,250
Payments to partners............................
Apply to loan........................................
Apply to capital

P15,000
3,750

P3,750

18,750
( 18,750)

P3,750
( 3,750)

P 2,500
5,000
1,250

P2,500

8,750
( 2,500) ( 8,750)
P 6,250

98
Chapter 5

Problem 5 7

Bronze, Gold & Silver


Cash Distribution Plan
June 30, 2008
Loss Absorption Balances
Bronze
Gold
Silver
Profit and loss ratio
20%
Pre-liquidation capital and
loan balances
P24,000
Loss absorption balances
(Capital and loan
balances/P& L ratio)
P110,000
Decrease highest LAB
to next highest:
Gold: (30,000 x .30) _______
______
110,000
24,000
Decrease LAB's
to next highest:
Gold: (10,000 x .30)
Silver: (10,000 x .20) _______
_( 2,000)
P110,000
22,000

Capital and Loan Accounts


Bronze
Gold
Silver
50%
30%

P55,000

P45,000

P150,000

P120,000

( 30,000)

_______

______

( 9,000)

120,000

120,000

55,000

36,000

( 10,000)
________

( 10,000)

_______

( 3,000)
_______

P110,000

P110,000

P 55,000

P 33,000

Accounts

Bronze

Summary of Cash Distribution


(If Offer of P100,000 is Accepted)
Gold Silver

Payable
Cash available
First
Next
Next
0
Additional paid in P&L ratio
15,000

P106,000
( 17,000)
( 9,000)
( 5,000)

50%

30%

P 17,000
P 9,000
3,000

( 75,000)

_______

P37,500

22,500

P 17,000

P37,500

P34,500

-0-

20%

P17,000

Partnership Liquidation by Installment

Problem 5 8
Part A
Balances
South
East

North
West
Total Interest (capital and loan
balances
P120,000
P 88,000
Divided by P/L ratio
30%
10%
Loss absorption potential
P400,000 P880,000
Priority II To South
(335,000)
Balances
400,000
545,000
Priority II To South and East, 10:20
(145,000)
Balances
400,000
400,000
Priority III To North, South, and
east 30:10:20
(250,000) (250,000)
_____
Total
150,000
150,000

Cash Payments
North South
East

West

P109,000 P 60,000
20%
40%
P545,000 P150,000
________
545,000
150,000
(145,000)
400,000
150,000

33,500
14,500 29,000

(250,000)

______

75,000 25,000 50,000

150,000

150,000

75,000 73,000 79,000

Further cash distribution P/L ratio


Part B
(1) Cash
65,600
North capital (30% of P16,400 loss)
4,920
South capital (10%)
1,640
East capital (20%)
3,280
West capital (40%)
6,560
Accounts receivable
To records collection of receivables with losses allocated to partners.
(2)

(3)

Cash
North capital (30% x P103,000)
South capital (10%)
East capital (20%)
West capital (40%)
Property and equipment
To record sale of property and equipment.
North capital

82,000

150,000
30,900
10,300
20,600
41,200
253,000
31,800

South capital
58,600
East capital
35,000
West capital
15,200
Cash
140,600
To record cash installment to partners of P230,600 based on the cash distribution plan in Part A.
First P90,000 is held to pay liabilities (P74,000) and estimated liquidation expenses of P16,000.
Next P33,500 goes entirely to South.
Next P43,500 is split between to South (P14,500) and East (P29,000).
Remaining P63,600 is allocated to North (P31,800), South (P10,600) and East (P21,200)
(4)

Liabilities
Cash
To record payment of liabilities.

74,000
74,000

100
Chapter 5

(5)

Cash
North capital (30% of P30,000 loss)
South capital (10%)
East capital (20%)
West capital (40%)
Inventory
To record inventory sold.

71,000
9,000
3,000
6,000
12,000
101,000

(6)

North capital
35,500
South capital
11,833
East capital
23,667
Cash
71,000
To record distribution of cash according to cash distribution plan. Although P87,000 cash is
being held, P16,000 must be retained to pay liquidation expenses. The Remaining P71,000
is divided among North, South, and East on a 30:20 basis.

(7)

North capital (30% of expenses)


South capital (10%)
East capital (20%)
West capital (40%)
Cash
To record liquidation expenses paid.

(8)

3,300
1,100
2,200
4,400
11,000

North capital (30/60 of deficit)


2,080
South capital (10/60)
693
East capital (10/60)
1,387
West capital
To eliminate capital deficiency of West as computed below:

Capital balances, beginning


Loss on accounts receivable
Loss on property and equipment
Cash distribution
Liquidation expenses
Subtotal
Elimination of West deficiency
Capital balances

4,160

North
P120,000
(4,920)
(30,900)
(31,800)
( 3,300)

South
P88,000
( 1,640)
(10,300)
(58,600)
( 1,100)

East
P109,000
( 3,280)
(20,600)
(50,200)
( 2,200)

West
P60,000
( 6,560)
(41,200)
0
( 4,400)

4,580
( 2,090)

1,527
( 693)

3,053
( 1,666)

( 4,160)
4,160

P 2,500

P 834

P 1,666

P 0

(9)

North capital
South capital
East capital
Cash
To record final cash distribution.

2,500
834
1,666
5,000

Partnership Liquidation by Installment

Problem 5 9
DR Company
Schedule of Safe Payments to Partners

Capital and loan balances, August 1, 2008


Write-off of P24,000 in goodwill
Write-off of P12,000 of receivables
Gain of P6,000 on sale of P32,000 of
inventory (one-half of P64,000 book
value)
Capital and loan balances, August 31, 2008
Possible loss of P16,000 for remaining
receivables and P32,000 for
remaining inventory
Possible liquidation costs of P4,000
Balances (* = deficit)
Distribute Bens potential deficit
To Dan: P7,600 x 40/70
To Red: P7,600 x 30/70
Safe payments to partners

Dan
(40%)

Red
(30%)

Ben
(30%)

(42,000)
9,600
4,800

(45,000)
7,200
3,600

(17,000)
7,200
3,600

(2,400)
(30,000)

(1,800)
(36,000)

(1,800)
(8,000)

19,200
1,600
(9,200)

14,400
1,200
(20,400)

14,400
1,200
7,600*
(7,600)

4,343
(4,857)

3,257
(17,143)

-0-

Of the P84,000 in cash at the end of August, P58,000 will be required to liquidate the debts to
outside creditors, and P4,000 must be held in reserve to pay possible liquidation costs. Thus, a
total of P22,000 in cash can be safely distributed to partners as of August 31, 2008.

Problem 5 10
(1)

Journal entry to record Jennys contribution:


Cash
Equipment
Jenny, capital

40,000
60,000
100,000

Journal entry to record Kennys contribution:


Cash
Inventory
Equipment
Notes payable

60,000
10,000
180,000
50,000

Kenny, capital

200,000

102
Chapter 5

(2)

Capital balances of Jenny and Kenny before admission of Lenny:


Beginning capital balance
Interest on beginning capital balance
Annual salary
Remainder
Ending capital balance

Jenny
P100,000
10,000
15,000
48,000
P173,000

Kenny
P200,000
20,000
20,000
72,000
P312,000

Explanation:
Each partner receives 10% on beginning capital balance. Each partner receives
her respective income (P15,000 to Jenny and P20,000 to Kenny). The amount distributed
thus far is P65,000. The remainder to be distributed is P120,000 (P185,000 30,000
35,000). Two-fifths of this remainder of P129,000 (48,000) is allocated to Jenny; 3/5 x
P120,000 (72,000) is allocated to Kenny. The total income allocated to Jenny and Kenny
is P73,000 and P112,000 respectively.
The admission of Lenny can now be recorded by the following entry:
Cash

175,000
Lenny, capital
Jenny, capital
Kenny, capital

110,000
26,000
39,000

Explanation:
The book value of the partnership after the income distribution in 2006 was
P485,000 (P173,000 + P312,000). After Lennys contribution, the value of the
partnership is P485,000 + P175,000 = P660,000. A one-sixth interest in the partnership is
P660,000 x 1/6 = P110,000. Using the bonus method, we compute a bonus of P175,000
P110,000 = P65,000. Using the 2:3 profit sharing ratio, the amount allocated to Jenny is
P26,000 (2/5 x P65,000) and the amount allocated to Kenny is P39,000 (3/5 x P65,000).
(3)

Schedule of Safe Payments


Capital balances
Partners loan
Gain on realization
Possible loss
Safe payments to partners

Jenny
P200,000
9,000
(156,000)
P 53,000

Kenny
P400,000
(50,000)
15,000
(260,000)
P105,000

Lenny
P200,000
6,000
(104,000)
P102,000

Explanation:
The sale of assets realized a gain of P30,000 (P210,000 P180,000) which is
distributed to the partners on the new profit sharing ratio: 30% to Jenny, 50% to Kenny,
and 20% to Lenny. Liabilities are paid. A possible loss on the unsold assets (P520,000) is
distributed to partners in their profit and loss ratio of 30:50:20 to Jenny, Kenny and
Lenny respectively.
Joint Venture

103

CHAPTER 6
SOLUTIONS TO MULTIPLE CHOICES

6-1: a
Assets per Jessica Company- balance sheet
Jessicas proportionate interest in assets of JV (50%)
Total assets of Jessica
6-2: a

P3,550,000
1,000,000
P4550,000

Total liabilities only of Jenny Co.

6-3: b
6-4: b
Investment of Heart
Profit share:
Sales
Cost of sales (150,800 125%)
Gross profit
Expenses
Net Profit
Profit/loss ratio
Balance of investment in JV

P80,000
150,800
120,640
30,160
10,000
20,160
x 40%

8,064
P88,064

6-5: a
Cash
Merchandise inventory
Accounts receivable
Total assets
Sweet Cos, proportionate interest
Sweet Companys share in total asset

P190,000
29,360
150,800
370,160
x 60%
P222,096

6-6: a
Sales
Cost of sales
Purchases
Merchandise inventory, end (50% of P10,000)

7,200
P10,000
__5,000

_5,000

Gross profit
Expenses

2,200
___500

Net profit

P 1,700

104
Chapter 6

6-7: b
Original investment (cash)
Profit share (P1,700 / 2)

P10,000
___850

Balance of Investment account

P10,850

6-8: a
Joint venture account before profit distribution (credit balance)
Unsold merchandise

P 9,000
__2,500

Joint venture profit before fee to Salas

P11,500

Joint venture profit after fee to Salas (P11,500 / 115%)

P10,000

6-9: b
Fee of Salas (P10,000 x 15%)
Profit share of Salas (P10,000 x 25%)

P 1,500
_2,500

Total

P 4,000

6-10: b
Salas

Salve

Balance before profit distribution


Profit share:Sabas (P10,000 x 40%)
Salve (P10,000 x 35%)

P 500 (dr) P 2,000 (cr)


4,000
______
_3,500

Balance

P 3,500 (cr) P 5,500 (cr)

6-11: d
Joint venture account balance before profit distribution (debit)
Joint venture profit (P4,500 x 3)

P 6,000
_13,500

Cost of unsold merchandise (inventory) taken by Dante

P19,500

Edwin Capital:
Debits: Balance before profit distribution
Credits: Profit share

P14,000
__4,500

6-12: b

Due from Edwin (debit balance)

P 9,500

Joint Venture

105

Settlement to Ferdie (Balance of capital account)


Debits:
Credits: Balance before profit distribution
Profit share

P 0
P16,000
__4,500

Due to Ferdie (credit balance)


Settlement to Dante (balance of JV Cash account)
Debits: Balance before cash settlement
Due from Edwin

_20,500
P20,500

P30,000
__9,500

P39,500

Credits: Due to Ferdie

_20,500

Balance

P19,000

6-13: a
JV account balance before profit distribution (cr)
Unsold merchandise (required dr balance after profit distribution)

P 4,600
__2,000

Joint venture profit before fee to Jerry


Joint venture profit after fee (P6,600 / 110%)
Fee to Jerry

P 6,600
__6,000
P 600

6-14: d
Harry Capital
Balances before profit distribution
Profit distribution:
Harry P6,000 x 50%)
Isaac (P6,000 x 20%)

(P 200)

Cash settlements

P 2,800

Isaac Capital
P 1,800

3,000
1,200
P 3,000

6-15: b
Sales
Cost of sales:
Merchandise inventory, beg (contributions)
Freight
Purchases

P14,000
P14,000
300
__4,000

Goods available for sale


Merchandise inventory, end (P8,300/2)

P18,300
__4,150

14,150

Gross profit (loss)


Expenses (P400 + P200)

(150)
__600

Net profit (loss)

P( 750)

6-16: c
Contributions to the Joint Venture (P5,000 + P8,000)
Loss share (P750 x 50%)
Unsold merchandise taken (withdrawal)

P13,000
( 375)
( 4,150)

Final settlement to jack

P 8,475

106
Chapter 6

SOLUTIONS TO PROBLEMS
Problem 6 1
Books of Blanco (Manager)
JV Cash
Joint Venture
Cash
Ablan Capital
Joint Venture
JV cash

Books of Ablan

100,000
90,000

Investment in JV
Merchandise inventory
100,000
90,000

60,000
60,000

90,000
90,000

Joint Venture
JV cash
JV cash
Joint Venture

20,000
20,000
200,000
200,000

Computation of JV Profit
Total debit to JV
Total credit to JV

P170,000
P200,000

Credit balance (Profit)

P 30,000

Distribution
Joint Venture
Profit from JV
Ablan capital
Ablan capital
JV cash
Cash
JV cash

30,000
15,000
15,000
105,000
105,000

Investment in JV
Profit from JV

15,000
15,000

Cash
Investment in JV

105,000
105,000

155,000
155,000

Joint Venture

107

Problem 6 2
Books of the Joint Venture
1.

2.

3.
4.

Computer equipment
Ella capital
Fabia capital

105,000
60,000
45,000

Purchases
Supplies
Diaz capital

80,000
2,000

Expenses
Diaz capital

9,000

Cash

82,000
9,000
150,000

Sales
5.

Expenses
Cash

150,000
30,000
30,000

6.
7.
8.

Merchandise inventory
Ella capital

20,000

Fabia capital
Cash

10,000

20,000
10,000

Adjusting and closing entries:


(a)
(b)

Expenses
Supplies
Sales

500
500
150,000

Income summary

150,000

Income summary
Merchandise inventory
Purchases

77,500
2,500

Income summary
Expenses

39,500

Distribution of profit:
Income summary
Diaz capital
Ella capital
Fabia capital

80,000
39,500
33,000
11,000
11,000
11,000

108
Chapter 6

Books of Diaz
(1)
(2)
(3)

Investment in Joint Venture


Cash

82,000

Investment in Joint Venture


Cash

9,000

82,000
9,000

To record profit share:


Investment in Joint Venture
Profit from Joint Venture

11,000
11,000

Books of Ella:
(1)
(2)
(3)

Investment in Joint Venture


Computer equipment

60,000

Investment in Joint Venture


Merchandise inventory

20,000

60,000
20,000

To record profit share:


Investment in Joint Venture
Profit from Joint Venture

11,000
11,000

Books of Fabia:
(1)
(2)

Investment in Joint Venture


Computer equipment

45,000

Cash

10,000

45,000

Investment in Joint Venture


(3)

10,000

To record profit share:


Investment in Joint Venture
Profit from Joint Venture

11,000
11,000

Joint Venture

109

Problem 6 3
(1)

No Separate Set of Joint Venture Books is Used

Books of Duran (Manager)


May 1:

7:

Joint Venture
Castro capital
Cash

12,500

JV cash
Bueno capital

10,000

26: Joint Venture


JV cash

12,000
500
10,000
9,500
9,500

30: JV accounts receivable


Joint Venture

16,000

June 30: JV cash


JV accounts receivable

15,000

27: JV cash
Joint Venture

16,000
15,000
9,000
9,000

30: To record unsold merchandise taken by Duran:


Merchandise inventory
Joint Venture

3,000
3,000

To record profit distribution:


Joint Venture
Profit from JV
Bueno capital
Castro capital

6,000
2,000
2,000
2,000

To record settlements:
Bueno capital
Castro capital
JV cash
Cash
Accounts receivable
JV accounts receivable

12,000
14,000
24,500
1,500
1,000
1,000

110
Chapter 6

Books of Bueno
May 7:

Investment in Joint Venture


Cash

June 30: Investment in Joint Venture


Profit from Joint Venture
Cash

10,000
10,000
2,000
2,000
12,000

Investment in Joint Venture

12,000

Books of Castro
May 1:

Investment in Joint Venture


Merchandise inventory

June 30: Investment in Joint Venture


Profit from Joint Venture
Cash

12,000
12,000
2,000
2,000
14,000

Investment in Joint Venture


(2)

14,000

A Separate Set of Books is used:

Books of the Joint Venture


May 1:

7:

Merchandise inventory
Castro capital
Duran capital

12,500

Cash

10,000

12,000
500

Bueno capital
26: Purchases
Cash

10,000
9,500
9,500

30: Accounts receivable


Sales
June 20: Cash

16,000
16,000
15,000

Accounts receivable
27: Cash

15,000
9,000

Sales

9,000

Joint Venture

111

June 30: Closing entries:


Sales

25,000
Income summary

Income summary
Merchandise inventory, end
Merchandise inventory
Purchases

25,000
19,000
3,000
12,500
9,500

Distribution of profit:
Income summary
Bueno capital
Castro capital
Duran capital

6,000
2,000
2,000
2,000

Settlements to Venturers:
Bueno capital
Castro capital
Duran capital
Merchandise inventory
Accounts receivable
Cash

12,000
14,000
2,500
3,000
1,000
24,500

Books of Duran (Manager/Operator)


May 1:

Investment in Joint Venture


Cash

June 30: Investment in Joint Venture


Profit from Joint Venture
Cash

500
500
2,000
2,000
2,500

Investment in Joint Venture


Books of Bueno and Castro (Same as in No. 1 requirement)

2,500

112
Chapter 6

Problem 6 4
(1)

Books of Seiko (Manager/Operator)

April1:

May:

June:

JV Cash
Notes payable PNB
Roles capital
Timex capital

August:

34,000
34,000
34,000

Joint venture
Cash
Rolex capital

64,100

Rolex capital
JV cash

30,000

Joint venture
Cash
Rolex capital
Timex capital
July:

102,000

16,300
7,800
30,000
111,400
37,400
64,700
9,300

Cash
Rolex capital
Timex capital
JV cash

40,000
15,000
10,000

Joint venture
Cash
Rolex capital
Timex capital

55,770

Cash
Rolex capital
Timex capital
JV cash

45,000
67,000
13,500

Joint venture
Cash
Rolex capital
Timex capital

30,600

65,000
13,970
31,240
10,560

125,500
9,730
16,560
4,310

To record sales:
JV cash (P421,000 x 96%)
Joint venture

404,160
404,160

Joint Venture

113

To record payment of loan to PNB:


Notes payable PNB
Rolex capital
Timex capital
Joint venture (Interest expense)
JV cash

34,000
34,000
34,000
8,000
110,000

To record distribution of profit:


Joint venture
Gain from JV (30%)
Rolex capital (60%)
Timex capital (10%)

134,290
40,287
80,574
13,429

Computed as follows:
Total debits tot he JV account
Total credits to the JV account

P269,870
_404,160

Gain (credit balance)

P134,290

To record settlement:
Cash
Rolex capital
Times capital
JV cash

32,687
128,874
14,099
175,660

Computations:
Settlement to Rolex - Balance of capital account:
Debits: June
July
August
Payment of note payable

P30,000
15,000
67,000
_34,000

P146,000

Credits: April 1
May
June
July
August
Profit share

P34,000
47,800
64,700
31,240
16,560
_80,574

__274,874

Credit balance

114
Chapter 6

Settlement to timex Balance of capital account

P 128,874

Debits: July
August
Payment of loan

P 10,000
13,500
__34,000

P 57,500

Credits: April 1
June
July
August
Profit share

P 34,000
9,300
10,560
4,310
__13,429

_71,599

Credit balance

P 14,099

Settlement to Seiko Balance of JV cash account


Debits: April 1
Loan proceeds

P102,000
_404,160

P506,160

Credits: June
July
August
Payment of loan

P 30,000
65,000
125,500
_110,000

_330,500

Balance of JV cash
Less:Settlement to Rolex
Settlement to Timex

175,660
P128,874
__14,099

Settlement to Seiko
(2)

_142,973
P 32,687

Partial Balance Sheet


June 30, 2008
Books of Seiko (Manager/operator)
Current assets:
Investment in joint Venture:
Joint Venture assets:
Cash
Joint Venture
Less:Equity of other venturers
(P116,500 + P43,300)

P 72,000
_175,500

P247,500
_159,800

Current liabilities:
Notes payable PNB

87,700
34,000

Joint Venture

115

Computation of balances as of June 30, 2008:


JV Cash
April 1 P102,000
Balance P 72,000

P30,000

Joint Venture
June

May
June

P 64,100
_111,400

Balance P175,500

Notes Payable
P34,000

Rolex capital
April

June

P 30,000
_______

P 34,000
47,800
__64,700

P 30,000

P146,500

April 1
May
June

P116,500
Timex capital
P34,000
__9,000

April
June

P43,300
Problem 6 5
Consolidated Balance Sheet
Cash
Receivables
Inventory
Other assets

P 61,000
122,000
102,500
__40,500

Total assets

P326,000

Accounts payable
Other liabilities
Capital stock
Retained earnings

P 61,000
96,500
50,000
_118,500

Total liabilities and stockholders' equity

P326,000

Consolidated Income Statement


Sales
Cost of sales

P246,750
_124,750

Gross profit
Operating expenses

122,000
__58,250

Consolidated net income

P 63,750

116
Chapter 6

Problem 6 6
(a)

Journal entries on venture books


June 15:

Cash

1,000,000

MacDo
Initial contribution at 6%
July 1:

Land

1,000,000
2,400,000

Mortgage payable
Cash

1,650,000
750,000

Purchased land for cash and 6% mortgage.


Aug 1:

Cash

1,100,000

MacDo
Additional contribution at 6%.
Land

1,100,000
950,000

Cash
Paid for improvements.
Sept 30:

Oct 31:

Nov 30:

Dec 31:

950,000

Mortgage payable
Interest expense- Mortgage
Cash
Reduced mortgage and paid interest.

250,000
3,750

Mortgage payable
Interest expense- Mortgage
Cash
Reduced mortgage and paid interest.

400,000
8,000

Mortgage payable
Interest expense- Mortgage
Cash
Reduced mortgage and paid interest.

300,000
7,500

253,750

408,000

307,500

Mortgage payable
200,000
Interest expense- Mortgage
21,000
Cash
Reduced mortgage and make semi-annual
interest payment.

221,000

Joint Venture

31:

117

Cash

2,600,000

Sales
Sales to date.
31:

31:

31:

2,600,000

Commissions
Cash
P2,600,000 x 5%

130,000

Expenses
Cash
Paid expenses

628,100

Interest expense- Venturer


MacDo
6% on P1,000,000 from June 15 to

130,000

628,100
60,000
60,000

December 31, and on P1,100,000


from August 1 to December 31.
31:

Sales

2,600,000

Land (cost of land sold)


Expenses
Commissions
Interest expense- mortgage
Interest- venturer
Income summary
To close income and expense accounts.
31:

31:

1,145,000
628,100
130,000
40,250
60,000
596,650

Income summary
MacDo
MacEn
To divide gain, 60:40.

596,650

MacDo

801,650

596,650
238,660

Cash
Payment on account.
(b)

801,650

Journal entries on MacDos books:


June 15:

Aug 1:

Investment in Joint Venture


Cash
Initial contribution.

1,000,000

Investment in Joint Venture


Cash
Additional contribution.

1,100,000

1,000,000

1,100,000

118
Chapter 6

Dec 31:

31:

31:

Investment in Joint Venture


Interest income
Interest earned on cash advanced.

60,000
60,000

Investment in Joint Venture


Gain on Joint Venture
60% of gain on venture.

357,990

Cash

801,650

357,990

Investment in Joint Venture


Repayment in part of advances.
(c)

801,650

MacDo and MacEn Joint Venture


Income Statement
For the period from June 15 to December 31, 2008
Sales
Cost of land sold:
Land
Improvements
Total

P2,600,000
P2,400,000
950,000
P3,350,000

Unsold land
Gross profit
Expenses:
Advertising and office expenses
Interest on mortgage
Interest on advances
Commissions
Net gain

2,205,000
P 628,100
40,250
60,000
130,000

Distributions:
MacDo (P596,650 x 60%)
MacEn (P596,650 x 40%)

1,145,000
1,455,000

858,350
P 596,650
P 357,990
238,660

Mac Do and MacEn Joint Venture


Balance Sheet
December 31, 2008
Assets
Cash
Land
Total Assets

P 250,000
2,205,000
P2,455,000

Liabilities and equity:


Mortgage payable
MacDo
MacEn
Total liabilities and equity

P 500,000
1,716,340
238,660
P2,455,000

Joint Venture

119

Venturers equity (interest)


Invested
Shares:
Gain
Interest on advances
Commissions
Total
Balances
Withdrawn
Equity (interests)

MacDo
P2,100,000

MacEn

Total
P2,100,000

P 357,990
60,000

P238,660

P 596,650
60,000
130,000
786,650
2,886,650
(931,650)
P1,955,000

417,990
2,517,990
(801,650)
P1,716,340

130,000
368,660
368,660
(130,000)
P238,660

120
Chapter 7

CHAPTER 7
MULTIPLE CHOICE ANSWERS AND SOLUTIONS
7-1: c
Amount realized secured by inventory
Unsecured claim (P10,000 x 25%)

P 30,000
__2,500

Total amount received

P 32,500

7-2: d

7-3: d

Amount realized secured by inventory


Unsecured claim (P88,000 x 75%)

P120,000
__66,000

Total amount received

P186,000

(P15,000,000 + P200,000)

7-4: a
Realizable value:
Current assets
Land and building
Less mortgage payable

P 50,000
P240,000
_200,000

__40,000

Total
Less accounts payable

90,000
_160,000

Estimated deficiency to unsecured creditors

P 70,000

7-5: c
Total realizable value to unsecured creditors (P90,000)/total unsecured
Claims (P160,000) = 56.25%
7-6: a
Free assets:
Current assets
Buildings and equipment
Total
Liabilities with priority:
Administrative expenses
Salary payable
Income taxes
Total

P 33,000
_110,000
P143,000
P 20,000
6,000
__8,000
P 34,000

Corporation in Financial Difficulty Liquidation

Free assets after payment of liabilities with priority:


(P143,000 P34,000)
Unsecured liabilities
Notes payable
Accounts payable
Bonds payable
Total

P109,000
P 30,000
83,000
__70,000
P183,000

Percentage of Unsecured liabilities to be paid: P109,000 / P183,000 = 60%


Payment of notes payable:
Value of security (land)
60% of remaining P30,000
Total collected

P 90,000
__18,000
P108,000

7-7: c
Free assets:
Other assets
Excess from assets pledged with secured
Creditors (P116,000 P70,000)
Total
Liabilities with priority
Free assets after payment of liabilities with priority
(P126,000 P42,000)
Unsecured liabilities:
Excess of partially secured liabilities over pledge
Assets (P130,000 P50,000)
Unsecured creditors
Total

P 80,000
__46,000
P126,000
P 42,000
P 84,000
P 80,000
_200,000
P280,000

Recovery percentage: P84,000 / P280,000 = 30%


Payment of partially secured debt:
Value of pledged assets
30% of remaining P80,000
Total collected

P 50,000
__24,000
P 74,000

122
Chapter 7

7-8: a
The holder of Debt Two will receive P100,000 from the sale of the pledged
asset. Since the holder wants to receive P142,000 out of the total debt of
P170,000, the company must be able to generate enough cash to pay off
60% of the unsecured liabilities (P42,000/P70,000) after paying 100% of
the liabilities with priority (P110,000).
Unsecured liabilities:
Unsecured creditors
Excess liability of Debt One in excess of pledged
Asset (P210,000 P180,000)
Excess liability of Debt Two in excess of pledged
Asset (P170,000 P100,000)

P230,000
30,000
__70,000

Total unsecured liabilities


Necessary percentage

P330,000
____60%

Cash needed for these liabilities

P198,000

In order for the holder of Debt Two to received exactly P142,000, the other free assets
must be sold for P308,000. With that much money, the liabilities with priority

(P110,000) can be paid with the remaining P198,000 going to the unsecured debts of
P330,000. This 60% figure would insure that the holder of Debt Two would get
P100,000 from the pledged asset and P42,000 (P70,000 x 60%) from the free assets.
7-9: c
Estate equity, beg. (P100,000 P85,000)
Loss on realization (P100,000 P75,000)
Unrecorded liabilities:
Interest expense
Administrative expense

P 15,000
( 25,000)
P

250
4,000

( 4,250)

Estate deficit

P( 14,250)

7-10: c
Total assets at net realizable value
Fully secured liabilities
Estimated administrative expense

P 75,000
(40,000)
_( 4,000)

Estimated amount available


Unsecured claims (P45,000 + P250)

P 31,000
(45,250)

Estimated deficiency to unsecured creditors

P 14,250

Corporation in Financial Difficulty Liquidation

7-11: b
Assets pledged with fully secured creditors
Fully secured creditors
Free assets
_160,000
Total free assets
Less: Liabilities with priority
Available to unsecured non-priority claims

P185,000
_130,000

55,000

215,000
__35,000
P180,000

7-12: b
Machinery
Recoveries of unsecured claims (50,000 - 10,000) X .50
Amount to be realized

P 10,000
__20,000
P 30,000

7-13: b
Notes Payable
Less: Inventories
Unsecured Liabilities
% of recovery
Recovery
Add: Inventories
Amount to be received by Wood
7-14: a - P7,000

P 23,940
19,200
4,740
____78%
3,697
_19,200
P 22,897

7-15: a - P30,000
7-16: b - P57,200 [52,000 + (8,000 X .65)]
7-17: d - P72,800 (112,000 X .65)
7-18: d
Estimated loss:
Account Receivable
Inventories (28,000 - 18,500)
Building (59,000 - 22,000)
Equipment (5,600 - 2,000)
Goodwill
Prepaid expenses
Less: Stockholder's equity
Common stock
Deficit
Estimated deficiency

P 8,160
9,500
7,000
3,600
5,650
___430
P 72,000
( 16,660)

P 64,340
_55,340
P 9,000

124
Chapter 7

7-19: d
Accounts Receivable (39,350 - 16, 110)
Notes Receivable (18,500 - 12,500)
Inventories (87,850 - 45,100)
Prepaid expenses
Equipment (48,800 - 9,000)
Total estimated loss

P 23,240
600
42,750
950
__39,800
P112,740

7-20: b P33,750 (95,000 - 61,250) on Land and Building


7-21: d
Total Free Assets:
Balance of Assets Pledged to
Fully Secured Creditor (95,000 - 90,000)
Free Assets:
Cash
Accounts Receivable
Inventories
Equipment
Total
Less: Unsecured liabilities with priority (1,850 + 4,650)
Net Free Assets
Divide by Unsecured creditors:
Balance of Partially Secured Creditor
Notes Payable - PNB
P 15,000
Notes Receivable
__12,500
Accounts Payable
52,500
Notes Payable
__51,250

P 5,000
P 2,700
16,110
45,100
__9,000

__72,910
77,910
___6,500
P 71,410

2,500
103,750 P106,250

Estimated recovery %

67%

7-22: d
Fully secured (Notes Payable)
Partially secured:
Notes Payable - PNB
Add (2,500 X 67%)
Unsecured Creditor with Priority
Unsecured Creditor without Priority (103,750 X 67%)
Total

P 90,000
P12,500
__1,675

14,175
6,500
__69,513
P180,188

Corporation in Financial Difficulty Liquidation

7-23: a
Unsecured creditors without priority
Estimated deficiency to unsecured creditors:
Loss on realization
Estimated liquidation expenses
Total
Stockholders equity
Net free assets
Liabilities with priority
Free assets

P1,102,500
551,250
55,125
606,375
441,000

165,375
937,125
122,500
P 1,059,625

7-24: a
Estimated net gain (loss) on realization:
Gain on realization
Loss on realization
Estimated claims
Total
Stockholders equity
Estimated deficiency

78,750
(336,700)

(257,950)
( 43,750)
(301,700)
295,750
P( 5,950)

7-25: b
Notes payable (175,000 140,000)
Unsecured liabilities (420,000 52,500)
Total
Free assets (157,500 + 210,000)
Estimated deficiency

P 35,000
367,500
402,500
367,500
35,000

7-26: a
Old receivable (net)
Marketable securities
Old inventory
Depreciable assets- net
Total assets to be realized
7-27: a

P 38,000
12,000
60,000
96,000
P206,000

Old receivable
New receivable
Marketable securities
Sales of inventory
Total asset realized

P 21,000
47,000
10,500
75,000
P153,500

7-28: a
Gain on sale of inventory (P75,000 60,000)
Loss on realization:
Marketable securities (12,000 10,500)
Trustees expenses
Depreciation
Net loss

15,000
1,500
4,300
16,000

126
7

(21,800)
P( 6,800)
Chapter

SOLUTIONS TO PROBLEMS
Problem 7 1
(A)

Laguna Company
Statement of Affairs
October 31, 2008

Book
Value

Estimated
Assets
Realizable Value
Assets pledge for fully secured creditors:
P107,000 .... Plant assets.................................................. P67,400
Less; Fully secured liabilities......................_ 50,400
Assets pledged for partially secured creditors:
39,000. .... Inventories.................................................. P18,000
Free Assets:
4,000. ..... Cash............................................................ P 4,000
46,000. ..... Accounts, receivable...................................
46,000
2,000. ..... Supplies....................................................... __1,500
Total free assets...............................................
Less: Unsecured liabilities with priority..........
Net Free Assets................................................
Estimated deficiency to unsecured creditors (to balance)
P198,000
Book
Creditors'
Value
Liabilities & Stockholders' Equity
Claim
Fully secured liabilities:
P50,400....... Mortgage payable (including interest, P400) P50,400
Partially secured liabilities:
21,000.. ..... Notes payable..............................................
P21,000
Less: Inventory............................................
_18,000
Unsecured creditors with priority:
5,800.. ..... Wages payable
P 5,800
1,200.. ..... Property taxes payable................................
_1,200
Total............................................................
P 7,000
Unsecured creditors without priority:
60,000.. ..... Accounts payable........................................
19,000.. ..... Notes payable..............................................
Stockholders' Equity........................................
P198,000
(B)
Creditor Group
Amount of

Free Assets
P17,000

_51,500
P68,500
__7,000
P61,500
_20,500
P82,000
Unsecured
Liabilities

P 3,000

60,000
19,000
_____
P82,000
Amount to

Percentage
Claim

be Paid

P7,000
50,400
21,000
79,000

P7,000
50,400
20,250 *
59,250

to be

paid
Unsecured liabilities with priority....................................
Fully secured creditors.....................................................
Partially secured creditors................................................
Unsecured creditors without priority................................
* P18,000 + (P3,000 X 0.75) = P20,250
(C) See statement of affairs in requirement (A)
Corporation in Financial Difficulty Liquidation

127

Problem 7 2
VC Corporation
Statement of Realization and Liquidation
Month Ended January 31, 2008
Assets to be realized:
Land........................ P10,000
Building.................. 43,000
Equipment.............. 28,000
Patents.................... __4,400
P20,800
Assets Acquired...............

P85,400
0

Assets realized:
land............................. P
0
Building......................
0
Equipment...................
8,800
Patents......................... _12,000
Assets not realized:
Land............................ P10,000
Building...................... 43,000
Equipment................... _13,000

66,000
Liabilities Liquidated:
Account payable..... P14,000
Loans payable......... __7,000
120,000
Liabilities not Liquidated:
Accounts payable....
Loans payable.........

66,000
33,000

21,000

Liabilities to be Liquidated:
Accounts payable........ P80,000
Loans payable............. _40,000

99,000

Gain on realization......................... ___7,600


___6,200
Total ............................................... P213,000
P213,000

Loss on realization.....................
Total...........................................

VC Corporation
Balance Sheet
January 31, 2008
Cash ...............................................
P 66,000
Land ...............................................
33,000
Building.........................................
( 26,300)
Equipment......................................

P 6,700

Accounts payable.........................

10,000

Loans payable..............................

43,000

Estate deficit................................

_13,000

100.0%
100.0%
96.4%
75.0%

Total ............................................... P 72,700


00

VC Corporation
Estate Deficit
January 31, 2008
Gain on realization....................................................................
Loss in realization ....................................................................
Trustee's expenses ....................................................................
Net gain on realization..............................................................
Estate deficit, January 1, 2008..................................................
Estate deficit, January 31, 2008.................................................

P 7,600
( 6,200)
( 1,300)
P 100
( 26,400)
P(26,300)

128

Chapter 7
Problem 7 3

Rizal Corporation
Statement of Affairs
Book
Values
Assets
Assets pledged to fully secured creditors:
P 80,000........... Land and building...............................................
Less: Mortgage payable......................................
50,000........... Finished Goods...................................................
Less: Loan payable.............................................
32,000...........
12,000...........

4,000...........
8,000...........
36,000...........
1,000...........
8,000...........
45,000...........
16,000...........

Estimated
Realizable Value

Assets pledged to partially secured creditors:


Accounts receivable (80% x 30,000)..................
Trucks.................................................................
Totals...................................................................
Free Assets:
Cash....................................................................
AR (20% x 30,000).............................................
Inventory Materials..........................................
Prepaid expense..................................................
Trucks.................................................................
Equipment...........................................................
Intangible............................................................_______
Total Free Assets......................................................
Less: Unsecured liability with priority (12,000 + 8,000)
Net free assets..........................................................
Estimated deficiency to unsecured creditors (to Balance)

P102,000
43,000
P 55,000
50,000

Liabilities and Equity


Fully secured creditors:
P 43,000........... Mortgage payable...............................................
50,000........... Loans payable.....................................................
Total....................................................................
Partially secured creditors':
Bank Loan...........................................................
Less: Receivable (80% x 30,000)........................
5,000........... Truck Loan..........................................................
Less: trucks.........................................................

25,000...........

P 59,000

24,000
3,500
27,500
4,000
6,000
27,000
0
2,500
25,000
64,500
P128,500
20,000
108,500

________
81,000
P 292,000...........Total unsecured liabilities........................................
Book
Values

Free
Assets

P189,500
Creditors'
Claim

Unsecured
Liabilities

94,000
50,000
144,000
25,000
24,000
5,000
3,500

12,000...........
8,000...........

Unsecured creditors with Priority:


Wages payable....................................................
Taxes payable......................................................
Totals...................................................................

12,000
8,000
20,000

Unsecured creditors:
77,000........... Accounts payable................................................
110,000........... Stockholder Loan................................................
( 38,000)...........Stockholder Equity..................................................
P 292,000
Total.........................................................................
Corporation in Financial Difficulty Liquidation

77,000
110,000

187,000
P189,500
129

Problem 7 4
Mapayapa Corporation
Statement of Affairs
November 1
Book
Value

Estimated
Assets
Realizable Value
Assets pledged to fully secured creditors:
P60,000........ Investments................................................. P 69,000
180,000........ Accounts receivable....................................
171,000
Total............................................................
240,000
Less: Note payable......................................
210,000
Free assets:
66,000........ Cash............................................................ P 66,000
248,000........ Accounts receivable.................................... 193,500
291,000........ Merchandise inventory................................ 180,000
870,000........ Plant & equipment...................................... 330,000
114,000........ Notes receivable.......................................... 108,300
........ Patent.......................................................... __12,000
Total free assets...........................................
Less: Unsecured liabilities with priority..........
Net free asset...............................................
_________
Estimated deficiency (to balance)....................
P1,839,000
Total................................................................
Book
Value

Liabilities & Equity


Fully secured creditors:
P 210,000........ Notes payable..............................................
Unsecured creditor with priority:
Accrued wages............................................
Accrued property tax...................................
Total............................................................
Unsecured creditor:
960,000........ Account payable..........................................
Accrued expenses........................................
300,000........Capital stock
__369,000........Retained earnings............................................
P1,839,000
Total................................................................

Creditor's
Claim

Free
Assets

P 30,000

_889,800
919,800
__13,800
906,000
60,300
P966,300
Unsecured
Liabilities

P210,000
P 7,200
___6,600
P 13,800
P960,000
6,300
_______
P966,300

130

Chapter 7

Problem 7 5
a.

b.

Total fair value of assets (estimated proceeds)..........................


Less:Fully and partially secured creditors claim:
Notes payable, interest (secured by receivable and
inventory)................................................................... 125,000
Bonds payable (secured by land & building).................... 231,000
Available to unsecured creditors...............................................
Less:Unsecured creditors with priority:
Wages payable..................................................................P 9,500
Taxes payable...................................................................__14,000
Amount available to unsecured creditors..................................

P471,000

__23,500
P 91,500

Unsecured portion of notes payable and interests (P195-P125)


Accounts payable......................................................................
Total claims of unsecured creditors...........................................

P 70,000
__95,000
P165,000

356,000
115,000

P91,500
= 55.45%
P165,000
c.

Distribution of P471,000:
Creditors
Accounts payable
Wages payable
Taxes payable
Notes payable & interests

Amount
P 95,000....
9,500.....
14,000.....
125,000.....
70,000
Bonds payable & interests
231,000.....
Total estimated payment........................................

Corporation in Financial Difficulty Liquidation

Percent
Realized
55.45%
100%
100%
100%
55.45%
100%

Total
Payment
P 52,678
9,500
14,000
125,000
38,815
_231,000
P470,993

131

Problem 7 6
1.

Evergreen Company
Statement of Affairs
June 30, 2008
Book
Values

P460,000
80,000
140,000
100,000
120,000
100,000

Estimated
Realizable
Values

ASSETS
Pledged with fully secured creditors:
Land and building.....................................
P340,000
Less: Mortgage payable (including accrued interest)
(330,000)
Free Assets:
Cash .........................................................
P 80,000
Accounts receivable net.........................
126,000
Inventories................................................
84,000
Machinery net........................................
40,000
Goodwill...................................................
_ _____0_

Available for
Unsecured
Creditors
P 10,000

330,000

Total free assets............................................................


Less: liabilities with priority........................................

340,000
_140,000

Net free assets..............................................................


Estimated deficiency (Squeeze figure).........................

200,000
_130,000

P1,000,000

P330,000
LIABILITIES AND STOCKHOLDERS' EQUITY
Secured &
Priority
Claims

P120,000
20,000

Liabilities with priority


Wages payable..........................................
Property taxes payable..............................

300,000
30,000

Total .........................................................
Fully secured creditors
Mortgage payable.....................................
Interest on mortgage payable....................

220,000
100,000
10,000

Total .........................................................
P330,000
Unsecured creditors
Accounts payable.........................................................
Note payable-unsecured...............................................
Interest payable-unsecured...........................................

Stockholders' Equity
400,000 Capital stock.............................................
(200,000) Retained earnings (deficit)...........................................

Unsecured
Non-priority
Liabilities

P120,000
__20,000
P140,000
300,000
__30,000

P220,000
100,000
10,000
___
P330,000

P1,000,000
2.

Settlement per peso of unsecured creditors is P.6250 (P200,000/P320,000). No payment is


made for the P10,000 unsecured interest claim.
132 ____
Chapter 7
Problem 7 7
1.

Entries on trustee's books.


2008

March 1:

Cash.......................................................P8,000
Accounts receivable net.......................16,000
Inventories..............................................72,000
Land.......................................................40,000
Buildings net.....................................200,000
Intangible assets.....................................52,000
Accounts payable........................................
Note payable...............................................
Deferred revenue........................................
Wages payable............................................
Mortgage payable.......................................
Estate equity...............................................
To record custody of Kimerald Corporation.

March 1 to 31:Cash.......................................................15,200
Estate equity................................................800
Accounts receivable-net..............................
To record collection of receivables and recognize loss.
Cash.......................................................38,800
Estate equity...........................................33,200
Inventories..................................................
To record sale of inventories at a loss.
Cash.....................................................180,000
Estate equity...........................................60,000
Land............................................................
Buildings-net..............................................
To record sale of land and buildings at a loss.
Estate equity...........................................52,000
Intangible assets..........................................
To write off intangible assets.
Estate equity...................................................16,400
Administrative expenses payable.......................

P100,000
80,000
2,000
6,000
160,000
40,000

16,000

72,000

40,000
200,000

52,000

16,400

To accrue trustee expenses.

Corporation in Financial Difficulty Liquidation

2.

133

Financial Statements
Kimerald Corporation in Trusteeship
Balance Sheet
March 31, 2008
Assets
Cash .........................................................................................
Liabilities and Deficit

P242,000

Accounts payable. ....................................................................


Note payable-unsecured............................................................
Revenue received in advance....................................................
Wages payable..........................................................................
Mortgage payable. ....................................................................
Administrative expense payable-new........................................

P100,000
80,000
2,000
6,000
160,000
__16,400

Total liabilities..........................................................................
Less: Estate deficit....................................................................

P364,400
_122,400

Total liabilities net of deficit.....................................................

P242,000

Kimerald Corporation in Trusteeship


Statement of Cash Receipts and Disbursements
March 1 to 31, 2008
Cash balance, March 1, 2008....................................................
Add: Cash receipts
Collections of receivables..................................P 15,200
Sale of inventories.................................................38,800
Sale of land and buildings...................................180,000

P 8,000

Total .........................................................................................
Less: Cash disbursements.........................................................

242,000
____0

Cash balance, March 31, 2008..................................................

P242,000

_234,000

Kimerald Corporation in Trusteeship


Statement of Changes in Estate Equity
March 1 to 31, 2008
Estate equity, March 1..............................................................
Less:Loss on uncollectible receivables.........................P 800
Loss on sale of inventories....................................33,200
Loss on sale of land and buildings.........................60,000
Loss on write off of intangibles.............................52,000
Administrative expenses....................................._16,400

P 40,000

Estate deficit, March 31............................................................

P122,400

_162,400

134

3.

Chapter 7

Entries on trustee's books:


2008
April: Mortgage payable...........................................160,000
Cash.....................................................................
To record payment of secured creditors from
proceeds from sale of Land and buildings.
Administrative expenses payable-new..............16,400
Deferred revenue.................................................2,000
Wages payable....................................................6,000
Cash.....................................................................
To record payment of priority liabilities.
Accounts payable..............................................32,000

160,000

24,400

Note payable-unsecured....................................25,600
Cash.....................................................................
To record payment of P.32 per peso to unsecured
creditors (available Cash of P57,600 divided by
unsecured claims of P180,000).
Accounts payable..............................................68,000
Note payable-unsecured....................................54,400
Estate equity........................................................
To write-off remaining liabilities and
close trustee's records.

57,600

122,400

Reorganization and Troubled Debt Restructuring

135

CHAPTER 8
MULTIPLE CHOICE ANSWERS AND SOLUTIONS
8-1: a
Trade accounts payable (P52,000 + P62,700)
P114,700
12% preferred stock (5,000 x P1)
Paid in capital in excess of par (5,000 x P9)
Cash (P62,700 x P0.80)
_100,160

P 5,000
45,000
_50,160

Gain from discharge of indebtedness


14,540
8-2: c
8-3: c
8-4: b
Carrying value of the note payable:
Principal
Interest
__60,000 P660,000
Restructured value:
Principal
Interest
_110,000 _510,000
Gain on debt restructuring
P150,000
8-5: d
Other income:
Fair value of land
P450,000
Books value of land
_360,000
Other income
90,000
Extraordinary gain:

P600,000

P400,000

Book value of note payable


Principal
Interest
P560,000
Fair value of land
_450,000

P500,000
__60,000

Extraordinary gain
P110,000
8-6: a
Book value of bonds payable
P500,000
Par value of preferred stock (5,000 shares x P100)
_500,000
No gain no loss
0

136
Chapter 8

8-7: a
Book value of notes payable:
Principal
Interest
___500 P 3,000
Par value of common stock issued (200 shares x P5)
__1,000
Additional paid in capital
P 2,000
Add gain on payment of accounts payable:
Book value
Payment
__2,000

P 2,500

P 10,000
__8,000

Total gain on debt discharge


P 4,000
8-8: a
Carrying value of debt:
Note payable
Interest payable
P112,000
Fair value machinery
_(36,000)

P100,000
__12,000

Balance of debt

76,000
Restructured debt:
Note payable
Interest (P50,000 x .08 x 2)
__58,000
Restructuring difference (gain)

P 50,000
___8,000

P 18,000
8-9: d
Principal
P300,000
Interest payable (300,000 x 10%)
__30,000
Carrying value
P330,000
8-10: c
Should be P310,600
Restructured principal of note payable
P260,000
Interest payable:
On book value (P300,000 x 10% 30%)
On restructured (P260,000 x 8% x 2)
__50,600

P 9,000
_41,600

Future cash flows to liquidate the debt


P310,600
8-11: d
8-12: d
Loss on transfer of land:
Original cost
Market value
P 20,000

P290,000
_270,000

Gain on restructuring of debt:


Carrying value of debt
Market value of land
P 30,000

P300,000
_270,000

Reorganization and Troubled Debt Restructuring

137

8-13: a
Transfer gain (loss):
Carrying amount of equipment
Fair value of equipment
Transfer loss

P80,000
75,000
P(5,000)

Restructuring gain:
Carrying amount of the debt
Fair value of equipment transferred
Restructuring gain

P100,000
75,000
P 25,000

Carrying amount of real estate transferred


Fair value of real estate
Loss on restructuring of payables

P100,000
90, 000
P(10,000)

Carrying amount of liability


Fair value of real estate transferred
Restructuring gain

P150,000
90,000
P 60,000

8-14: d

8-15: d

8-16: c
Gain on revaluation of land (120,000 85,000)
Gain on the extinguishment of debt (185,000 120,000)
Total gain

P 35,000
65,000
P100,000

Carrying value of debt (P800,000 + 80,000)


Total future payments (P700,000 + 80,000)
Restructuring gain

P880,000
780,000
P100,000

8-17: a

8-18: a
First determine the expected future cash flows as follows:
70,000 x .79719
=
P55,803
5,600 x 1.69005
=
9,464
Present value of future cash flow
P65,267
The interest revenue can be computed using the effective interest method
as follows:
Present value at 12/31/06
P65,267
Interest income at 12/31/07 (65,267 x 12%)
7,832
Interest receivable at 12/31/07 (70,000 x 8%)
5,600
2,232
Present value at 12/31/07
P67,499
Interest income at 12/31/08 (67,499 x 12%)

P 8,100

138
Chapter 8

SOLUTIONS TO PROBLEMS
Problem 8 1
Journal entries for company emerging from bankruptcy using fresh start
accounting:

Receivables.......................................................................................10,000
Inventory. .........................................................................................10,000
Building...............
100,000
Reorganization value in excess of amount
Allocable to tangible assets.........................................................60,000
Additional paid in capital......................................................
180,000
To adjust accounts to market value as part of fresh start accounting. Since the company has
a reorganization value of P760,000 but the assets have a market value of only P700,000
(P90,000 + P210,000 + P400,000), and account entitled Reorganization Value in Excess of
Amount Allocable to Tangible Assets must be recorded for P60,000.
Liabilities.............
300,000
Common stock (P330,000 x 80%)...............................................
Gain on debt discharge................................................................
To record settlement of liabilities.
Problem 8 2

264,000
36,000

2008
July 14: Costs of reorganization................................................................50,000
Cash with escrow agent........................................................

50,000

Common stock
580,000
Common stock (60,000 x P1)...............................................
Additional paid in capital......................................................

60,000
520,000

Note payable 10%


120,000
Interest payable (P120,000 x 10% x 3/12)................................... 3,000
Note payable 12%..............................................................

123,000

Trade accounts payable


100,000
Cash P100,000 x 0.80)..........................................................
Gain on debt discharge..........................................................

80,000
20,000

Additional paid in capital


290,000
Gain on debt discharge
20,000
Retained earnings..................................................................
Costs of reorganization.........................................................

260,000
50,000

Reorganization and Troubled Debt Restructuring

139

Problem 8 3
Jade Corporation
Balance Sheet
December 31, 2008
ASSETS
Current assets:
Cash ................................................................................... P 23,000
Inventory............................................................................. __45,000
Property and equipment:
Land ................................................................................... 140,000
Buildings.............................................................................
Equipment........................................................................... _154,000

220,000
_514,000

Total assets..........................................................................

P582,000

LIABILITIES AND STOCKHOLDERS' EQUITY


Liabilities not subject to compromise
Current liabilities:
Accounts payable................................................................. P 60,000
Long-term liabilities:
Note payable (2006)......
P100,000
Note payable (2003)......
_100,000. ._ 200,000
Liabilities subject of compromise
Accounts payable................................................................. 123,000
Accrued expenses................................................................
30,000
Income taxes payable...........................................................
22,000
Note payable (due 2008)....................................................... _170,000
Total liabilities.....................................................................
Stockholders' Equity

P 68,000

P260,000

_345,000
605,000

Common stock. ................................................................... 200,000


Retained earnings (deficit).................................................... (223,000)

_(23,000)

Total liabilities and stockholders' equity (deficit)..................

P582,000

Problem 8 4
Preliminary computations:
Book values prior to reorganization:
Total assets (P100,000 + P112,000 + P420,000 + P78,000)...............
Total liabilities (P80,000 + p35,000 + P100,000 + P200,000 +
P185,000 + P200,000).................................................................
Common stock (given)......................................................................
Deficit (given)
.............................................................................

P710,000
P800,000
P240,000
P330,000

140
Chapter 8
Book values after reorganization:
Total assets (reorganization value).................................................................
Total liabilities (P5,000 + P4,000 + P100,000 + P50,000 +
P71,000 + P110,000)..............................................................................
Common stock (returned shares are reissued)...............................................
Deficit (eliminated) ......................................................................................
Additional paid in capital (squeeze)..............................................................

P780,000
P340,000
P240,000
0
P200,000

Since the company will have 30,000 shares outstanding after the reorganization, the additional paid in
capital equals P6.66 per share.
Because the company has a reorganization value of P780,000 but the assets have a market value of only
P735,000, an account entitled Reorganization Value in Excess of Amount allocable to Tangible Assets must
be recognized for P45,000.
JOURNAL ENTRIES:
1.
Land and buildings ......................................................................................80,000
Reorganization Value in excess of amount
allocable to tangible assets......................................................................45,000
Accounts receivable.........................................................................
Inventory ......................................................................................
Equipment ......................................................................................
Additional paid in capital................................................................
To adjust accounts to market value as part of fresh start accounting.

20,000
22,000
13,000
70,000

2.

Common stock...............................................................................................144,000
Additional paid in capital........................................................................
144,000
To record shares turned in to the company by the owners as part of the reorganization plan. 18,000
shares at P8 par value.

3.

Accounts payable........................................................................................... 80,000


Note payable...........................................................................................
Common stock, P8 par value..................................................................
Additional paid in capital (P6.66 per share)...........................................
Gain on debt discharge...........................................................................
To record settlement of accounts payable.

5,000
8,000
6,666
60,334

Accrued expenses..........................................................................................35,000
Note payable...........................................................................................
Gain on debt discharge...........................................................................
To record settlement of accrued expenses.

4,000
31,000

4.

5.

Note payable............
200,000
Note payable...........................................................................................
Common stock, P8 par value..................................................................
Additional paid in capital (P6.66 per share)...........................................
Gain on debt discharge...........................................................................
To record settlement of note payable due in 2007

Note payable............
185,000
Note payable...........................................................................................
Common stock, P8 par value..................................................................
Additional paid in capital, P6.66 per share.............................................
Gain on debt discharge...........................................................................
To record settlement of note payable due in 2008
Reorganization and Troubled Debt Restructuring

50,000
80,000
66,667
3,333

6.

71,000
56,000
46,667
11,333
141

Problem 8 5
7.

8.

Note payable. ....................................................................................200,000


Note payable. ..............................................................................
Gain on debt discharge................................................................
To record settlement of note payable due in 2009

110,000
90,000

Additional paid in capital (P334,000 P200,000).............................134,000


Gain on debt discharge......................................................................196,000
Retained earnings (deficit)..........................................................
330,000
To adjust additional paid in capital to appropriate balance, close out gain, and eliminate
deficit balance as part of fresh start accounting.

Since the Company has a reorganization value of P800,000 but only P653,000 can be assigned to
specific assets based on market value, the remaining P147,000 is reported as a Reorganization
Value in Excess of Amount Allocable to Identifiable Assets.
Sun Corporation
Balance Sheet Fresh Start Accounting
December 31, 2008
ASSETS
Current assets
Accounts receivable...................................................................................
Inventory...................................................................................................
Property and equipment
Land and buildings. ...................................................................................
Machinery.................................................................................................
Intangible assets
Patents ...................................................................................................
Reorganization value in excess of amount allocable To identifiable assets

P 18,000
_111,000

P129,000

278,000
_121,000

399,000

125,000
_147,000

_272,000

Total assets................................................................................................

P800,000

LIABILITIES AND STOCKHOLDERS' EQUITY


Current liabilities
Accounts payable.......................................................................................
Long-term liabilities
Note payable (due in 2 years)..................................................................... P 35,000
Note payable (due in 5 years).....................................................................
50,000
Note payable (due in 8 years)..................................................................... _100,000

_185,000

Total liabilities...........................................................................................

P282,000

P 97,000

Stockholders' Equity:
Common stock. ......................................................................................... P500,000
Additional paid in capital (squeeze)........................................................... __18,000
Total liabilities and stockholders' equity.................................................................

_518,000
P800,000

142
Chapter 9

CHAPTER 9
MULTIPLE CHOICE ANSWERS AND SOLUTIONS
9-1: d
Deferred gross profit, Dec. 31 (before adjustment)
P1,050,000
Less: Deferred gross profit, Dec. 31 (after adjustment)
Installment accounts receivable, Dec. 31
P1,500,000
Gross profit rate
____
25% __375,000
Realized gross profit, 2008
P 675,000
OR
Installment Sales (P1,050,000 25%)
P4,200,000
Less: Installment account receivable, Dec. 31
__1,500,00
Collection
P2,700,000
Gross profit rate
___X 25%
Realized gross profit, 2008
P 675,000
9-2: a
Deferred gross profit, before adjustment
Deferred gross profit, end
2006 (6,000 X 35%)
2007 (61,500 X 33%)
2008 (195,000 X 30%)
Realized gross profit, December 31, 2008
(Total P107,235)

2006
P7,230

2007
P 60,750

2008
P 120,150

2,100
20,295
P5,130

P 40,455

___58,500
P 61,650

9-3: c
Deferred gross profit balance, end
Divide by Gross profit rate based on sales (25% 125%)
Installment Accounts Receivable, end
Collection
Installment Sales

P 202,000
____
20%
P1,010,000
___440,000
P1,450,000

Sales
Cost of installment sales
Deferred gross profit
Less: Deferred gross profit, end
Installment accounts receivables, 12/31
(1,000,000-400,000)
Gross profit rate (300,000 1,000,000)
Realized gross profit
Operating expenses

P1,000,000
__700,000
P 300,000

9-4: b

P 600,000
___X 30%

__180,000
P 120,000
___80,000

Operating income
Interest and financing charges
Net income

40,000
__100,000
P 140,000

Installment Sales

9-5: a
Market value of repossessed merchandise
(before reconditioning cost)
Less: unrecovered cost
Unpaid balance (80,000-30,000)
Less: Deferred gross profit (50,000X20%)
Loss on repossession

P 30,000
P 50,000
___10,000

__40,000
(P 10,000)

9-6: a
Installment sales
Less: collection on installment sales
Installment account receivables, 12/31/08
Gross profit rate (500,000 1,000,000)
Deferred gross profit, 12/31/08

P1,000,000
__200,000
800,000
___X 50%
P 400,000

OR
Deferred gross profit (1,000,000-500,000)
Less: Realized Gross Profit (200,000 X 50%)
Deferred gross profit, 12/31/08

P500,000
_100,000
P400,000

Fair value of repossessed merchandise


Less: unrecovered cost
Unpaid balance
Less: Deferred gross profit (200,000 X 32.5%)
Loss on repossession

P120,000

9-7: d
P 200,000
___65,000

_135,000
(P 15,000)

9-8: b
Realized gross profit:
Collections:
Downpayment
Installment received (205,000-200,000)
Total
Gross Profit Rate (150,000 240,000)
Realized gross profit
Gain (loss) on repossession:
Appraised value of repossessed merchandise
Less:unrecovered cost
unpaid balance
less: deferred gross profit (200,000 X 62.5%)
Gain on repossession

144
Chapter 9

P 35,000
___5,000
40,000
_X 62.5%
P 25,000
P165,000
P 200,000
__125,000

__75,000
P 90,000

9-9: b
Sch.1
Date
Apr-1
Apr-1
May-1
Jun-1
Jul-1
Aug-1

Collection
750
625
625
625
625

Applying
to
Interest

Applying
to
principal

125.00
115.00
104.80
__94.40
P439.20

750.00
500.00
510.00
520.20
___530.60
P2,810.80

Gain (loss) on repossession:


Market value of repossessed merchandise
Less:unrecovered cost
unpaid balance of principal (sch. 1)
less: deferred gross profit (4,189 X 35%)
Loss on repossession (rounded)

Balance
of
principal
P7,000.00
6,250.00
5,750.00
5,240.00
4,719.80
4,189.00

P 1,875
P 4,189
__1,466

Realized gross profit:


Collection applying to principal (sch. 1)
Gross profit rate
Realized gross profit

___2,723
(P 848)
P2,810.80
__X 35%
P 983.78

9-10: c
Year of Sales
2007
2008
Deferred gross profit (Sales X Gross Profit Rate)
2007 (P300,000 X 30%)
2008 (P450,000 X 40%)
2007: Accounts written-off (P25,000 X 30%)
Realized gross profit (P100,000 X 30%)
2008: Accounts written-off, 2007 (P75,000 X 30%)
Accounts written-off, 2008 (P50,000 X 40%)
Realized gross profit, 2007 (P50,000 X 30%)
Realized gross profit, 2008 (P150,000 X 40%)
Deferred gross profit, 12/31/08 (P75,000)

( 60,000)
( 15,000)
________
( 60,000)
P 15,000 P 60,000

Deferred gross profit, 2007 (P1,050,000 - 735,000)


Realized gross profit, 2007 (P150,000 X 30%)
Deferred gross profit, 12/31/07
Realized gross profit, 2008 (P390,000-90,000) X 30%
Deferred gross profit, 12/31/08

P 315,000
( 45,000)
270,000
( 90,000)
P 180,000

P 90,000
P 180,000
( 7,500)
( 30,000)
( 22,500)

9-11: a

Installment Sales

9-12: a
Deferred gross profit (Sales - Cost of Installment Sales)
Realized gross profit, 2007 (P630,000 X 40%)

2007
P 480,000
( 252,000)

2008
P450,000

Realized gross profit, 2007 (P450,000 X 40%)


Realized gross profit, 2008 (P900,000 X 30%)
Deferred gross profit, 12/31/08 (P228,000)

( 180,000)
_______
P 48,000

( 270,000)
P180,000

9-13: c
Trade-in value
Less: Actual value
Estimated selling price
Less:reconditioning cost
normal gross profit (25,000 X 15%)
Overallowance
Realized gross profit:
Collection:
Downpayment
Actual value of merchandise-Trade In
Installment collected (5,000 X 3)

P 30,000
P 25,000
P 1,250
__3,750

___5,000

P 5,000
20,000
_15,000

Gross Profit Rate:


Sales
Overallowance
Net Sales
Cost of Installment Sales
Gross Profit
Gross Profit Rate (15,000 75,000)
Realized Gross Profit

__20,000
P 10,000

P 40,000
P 85,000
( 10,000)
P 75,000
_60,000
P 15,000
_X 20%
P 8,000

9-14: c
Collection excluding interest (P900,000-P300,000)
Gross profit rate (P1,200,000 P3,600,000)
Realized Gross Profit, December 31, 2008
Add Interests
Total Revenue

P 600,000
X 33 1/3%
200,000
__300,000
P 500,000

9-15: a
Wholesale value of repossessed merchandise
Less: unrecovered cost
Unpaid balance:
Sales, 10/1/07
P 24,000
Collection, 2007 (6,000 2,000)
( 8,000)
Collection, 2008 (1,000 X 7)
( 7,000)
Deferred gross profit (9,000 X 25%)
Loss on repossession

P 9,000
__2,250

4,000

___6,750
(P 2,750)

146
Chapter 9

9-16: a
Trade-in Value (P300 X 6)
Less: Actual value
Estimated selling price (P315 X 6)
Less:Reconditioning cost (P25 X 6)
Gross Profit (P1,890 X 10%)
Over-allowance
9-17: a

P 1,800
P 1,890
P150
_189

___339

___1,551
P
249

Deferred gross profit, before adjustment


Deferred gross profit, end
2007: P32,500 X (30% 130%)
2008: P180,000 X (33 1/3% 133 1/3%)
Realized gross profit on installment sales

P 76,000
P 7,500
_45,000

__52,500
P 23,500

9-18: d
Unpaid balance (P27,000 - P16,000)
Multiply by gross profit rate (P734,400 P2,160,000)
Deferred gross profit to be cancelled on repossession

P 11,000
___X 34%
P 3,740

Collection:
2007 Downpayment
2008 Installment collection
Interest
Total

P 600,000
600,000
__540,000
P1,740,000

Cost to be recovered

P4,000,000

9-19: b

Since cost is not yet fully recovered, then no gross profit is to be recognized in 2008.
9-20: d
Regular Sales
Cost of regular sales
Gross profit on regular sales
Add: Realized gross profit on installment sales
2007 (25,000 X 50%)
2008 (62,500 X 55%)
Total realized gross profit
Operating expenses
Net income, 12/31/08

P 187,500
__112,500
P 75,000
P12,500
_34,375

__46,875
121,875
___31,250
P 90,625

Installment Sales

9-21: a
Installment sales 2007
Collections:
Down payment (20% x 785,000)
Installment (40% x 628,000)
Installment accounts receivable 2007, 12/31/07
Gross profit rate on sales
Deferred gross profit- 2007, 12/31/07

P785,000
P157,000
251,200

408,200
376,800
35/135
P 97,689

9-22: a
Regular sales
Cost of regular sales
Gross profit on regular sales
Realized gross profit on installment sales:
Installment sales (1,093,750 x 240%)

P1,575,000
1,050,000
525,000
2,625,000

Installment accounts receivable-12/31/08


Collections
Gross profit on rate on sales
Total realized gross profit
Operating expenses (1,137,500 x 70%)
Net income

1,575,000
1,050,000
140/240

612,500
1,137,500
796,250
P

341,250
9-23: a
Regular sales
Cost of regular sales
Gross profit on regular sales
Realized gross profit on installment sales:
Collections excluding Interest (312,000 24,000)288,000
Gross profit rate (270,000/900,000)
30%
Total realized gross profit
Loss on repossession
Fair value of repossessed merchandise
54,000
Less: Unrecovered cost (100,000 x 70%)
70,000
Total realized GP after loss on repossession
Less: Operating expenses
72,000
Installment accounts written-off (44,000 x .70) 30,800
Net operating income
Interest income
Net income

P375,000
215,000
160,000
86,400
246,400
( 16,000)
230,400
102,800
127,600
24,000
P151,600

148_

Chapter 9

SOLUTIONS TO PROBLEMS
Problem 9 1
Journal Entries:
2006
Installment A/R2006................ 104,000
Installment A/R2007................

Installment A/R2008................

Installment Sales..................
104,000
Cost of Installment Sales............
Inventory.............................

64,480

Cash...........................................
Installment A/R2006
Installment A/R2007..........

66,980

2007

116,000

116,000

121,000
121,000

68,440
64,480

73,810
68,440

125,520
57,200

2008

73,810
145,460

29,120
71,920

15,000
26,680

Installment A/R2008..........
Interest Revenue..................

9,780

Installment Sales........................ 104,000


Cost of Installment Sales.....
64,480
Deferred Gross Profit2006.
39,520
Deferred Gross Profit2007.

Deferred Gross Profit2008.

Deferred Gross Profit2006.......


Deferred Gross Profit2007.......
Deferred Gross Profit2008.......
Realized Gross Profit...........

21,736

21,736

Computations:
2006: P57,200 X .38 =

P21,736

2007: P29,120 X .38 =


P71,920 X .41 =
Total RGP

P11,066
29,987
P40,553

2008: P15,000 X .38 =


P26,680 X .41 =
P76,230 X .39 =
Total RGP

P 5,700
10,939
29,730
P46,369

_
24,480
116,000

76,230
27,550
121,000

68,440

47,560

11,066
29,487

40,553

73,810

47,190
5,700
10,939
29,730
46,369

Installment Sales
149
2007:

2008:

Problem 9 2
Inventory.................................................................................................45,200
Cash.................................................................................................
Notes Receivable 2007 (P32,000 + P62,000 + 3,600)............................97,600
Unearned Interest Revenue (P7,167 + P3,600)...............................
Installment Sales..............................................................................
Cost of Installment Sales (P45,200 P2,000 inventory increase)..........43,200
Inventory..........................................................................................
Cash........................................................................................................35,600
Notes Receivable 2007....................................................................
Unearned Interest Revenue 2007............................................................3,600
Interest Revenue..............................................................................
Installment Sales.....................................................................................86,833
Cost of Installment Sales.................................................................
Deferred Gross Profit on Installment Sales2007...........................
Deferred Gross Profit on Installment Sales2007..................................16,080*
Realized Gross Profit on Installment Sales.....................................
*Gross profit percentage: 50.25% (P43,633 P86,833)
.5025 x 32,000 = P16,080
Inventory.................................................................................................52,020
Cash.................................................................................................

45,200
10,767
86,833
43,200
35,600
3,600
43,200
43,633
16,080

52,020

Notes Receivable2008..........................................................................89,5001
Unearned Interest Revenue..............................................................
Installment Sales..............................................................................

11,9552
77,545

160,000 + (P50,000 + P5,500) P26,000* = 89,500


*2007 Notes receivable collected in 2008
2Interest revenue from 2007 notes: P7,167 P5,579 = P1,588
Interest revenue from 2008 notes: P5,500 P1,588 = P3,912
Discount on notes receivable at end of 2008..........................................P 8,043
Interest revenue from 2008 notes (see above)........................................ 3,912
Total discount at time of sale..................................................................P11,955
Cost of Installment Sales (P52,020 P8,000)........................................44,020
Inventory..........................................................................................
Cash........................................................................................................55,500
Notes Receivable2007 (P62,000 P36,000).................................
Notes Receivable2008...................................................................
* P89,500 P60,000 = P29,500
Discount on Notes Receivable2007......................................................1,588
Discount on Notes Receivable2008......................................................3,912
Interest Revenue..............................................................................
Installment Sales.....................................................................................77,545
Cost of Installment Sales.................................................................
Deferred Gross Profit on Installment Sales2008...........................
Deferred Gross Profit on Installment Sales2007 (P26,000
P1,588 = P24,412; P24,412 x .5025)...................................................12,267
Deferred Gross Profit on Installment Sales2008..................................11,062*
Realized Gross Profit on Installment Sales.....................................
profit percentage: 43.23% (P33,525 P77,545)
.4323 x (P29,500 P3,912) = P11,062
150
Problem 9 3
1.

2.

2006: Gross profit rate

Deferred gross profit, 1/1


=
Install. contracts rec'l, 1/1

P24,000
=
P60,000

40%

2007: Gross profit rate

Deferred gross profit, 1/1


P24,000
= =
Install. contracts rec'l, 1/1
P140,000

42%

2008: Gross profit rate

Gross profit
=
Installment sales

P86,000
=
P200,000

Journal Entries:
Accounts Receivable......................................................................................
Sales.......................................................................................................
Installment Contracts Receivable 2008......................................................
Installment Sales.....................................................................................
Cost of Installment Sales...............................................................................
Shipments on Installment Sales..............................................................
Purchases.......................................................................................................
Cash........................................................................................................
Selling Expenses............................................................................................
Cash........................................................................................................
Cash..............................................................................................................
Accounts Receivable..............................................................................
Installment Contracts Receivable 2006...............................................
Installment Contracts Receivable 2007...............................................
Installment Contracts Receivable 2008...............................................
Adjusting Entries:
Installment Sales............................................................................................

44,020
26,000
29,500*

5,500
44,020
33,525

23,329
Chapter 9

43%
600,000
600,000
200,000
200,000
114,000
114,000
476,000
476,000
210,000
210,000
790,000
560,000
40,000
80,000
110,000

200,000

Cost of Installment Sales........................................................................


Deferred Gross Profit on Installment sales 2008.................................
Deferred Gross Profit 2006 (P40,000 x 40%).............................................
Deferred Gross Profit 2007 (P80,000 x 42%).............................................
Deferred Gross Profit 2008 (P110,000 x 43%)...........................................
Realized Gross Profit..............................................................................
Doubtful Accounts Expense (1/4 x 1% x P600,000)......................................
Allowance for Doubtful Accounts..........................................................
Closing Entries:
Sales.............................................................................................................
Merchandise Inventory, December 31...........................................................
Shipments on Installment Sales.....................................................................
Merchandise Inventory, January 1..........................................................
Purchases................................................................................................
Selling Expenses.....................................................................................
Doubtful Accounts Expense...................................................................
Income Summary....................................................................................
Realized Gross profit.....................................................................................
Income Summary....................................................................................
Income Summary...........................................................................................
Retained Earnings...................................................................................
Installment Sales
151
3.
Good Buy Mart
Income Statement
Year Ended December 31, 2008
Sales.............................................................................................................
Cost of sales:
Merchandise inventory, January 1..........................................................
Purchases................................................................................................
Cost of goods available for sale..............................................................
Less Shipments on installment sales.......................................................
Cost of goods available for regular sales................................................
Less Merchandise inventory, December 31............................................
Gross profit on regular sales..........................................................................
Add Realized gross profit on installment sales (Schedule 1)........................
Total realized gross profit..............................................................................
Operating expenses:
Selling expenses......................................................................................
Doubtful accounts expense.....................................................................
Net income ....................................................................................................

114,000
86,000
16,000
33,600
47,300
96,900
1,500
1,500
600,000
260,000
114,000
240,000
476,000
210,000
1,500
46,500
96,900
96,900
143,400
143,400

P600,000
P240,000
476,000
716,000
114,000
602,000
260,000

210,000
1,500

342,000
258,000
96,900
354,900
211,500
P143,400

Schedule 1

Collections ...........................................
Multiply by Gross profit rate................
Realized gross profit.............................
4.

2006
P40,000
40%
P16,000

Years of Installment Sales


2007
2008
P80,000
P110,000
42%
43%
P33,600
P 47,300

Total

P 96,900

Good Buy Mart


Balance Sheet
December 31, 2008
A s s e t s
Cash..............................................................................................................
Merchandise inventory...................................................................................
Accounts receivable.......................................................................................
Allowance for doubtful accounts...................................................................

P144,000
260,000
P 62,000
3,500

58,500

Installment contracts receivable 2006.........................................................


Installment contracts receivable 2007.........................................................
Installment contracts receivable 2008.........................................................
Other assets....................................................................................................
Total Assets.............................................................................................

20,000
60,000
90,000
200,000
P832,500

Liabilities and Equity


Liabilities:
Accounts payable....................................................................................
Deferred gross profit on installment sales 2006..................................
Deferred gross profit on installment sales 2007..................................
Deferred gross profit on installment sales 2008..................................
Total Liabilities.......................................................................................
Equity:
Capital stock...........................................................................................
Retained earnings....................................................................................
700,600
Total Liabilities and Equity....................................................................

P 60,000
8,000
25,200
38,700
131,900
P406,000
294,600
P832,500
Chapter 9

152
Problem 9 4
1.

2007: GP rate =
30%

Deferred gross profit, 1/1

=
=

P21,600 + P1,200

Install. contracts rec'l, 1/1


2008: GP rate =
35%

Gross profit

P24,000 + P52,000
=

P150,000 P97,500

Installment sales
2.

=
=

P150,000

Installment Sales............................................................................................
Cost of Installment Sales........................................................................
Deferred Gross Profit, 2008....................................................................
Deferred Gross profit, 2007...........................................................................
Deferred Gross Profit, 2008...........................................................................
Realized Gross Profit..............................................................................

P22,800

P76,000
=

P52,500

P150,000
150,000
97,500
52,500
14,400
25,900
40,300

Computation:

Installment contracts receivable, 1/1.....................


Less Installment contracts receivable, 12/31........
Total credit for the period......................................
Less Credit representing repossession..................
Credit representing collections..............................
Multiply by Gross profit rate................................
Realized gross profit.............................................

2007
Sales
P76,000
24,000
52,000
4,000
P48,000
30%
P14,400

Sales.............................................................................................................
Realized Gross Profit.....................................................................................
Loss on Repossession.............................................................................
Cost of Sales...........................................................................................
Selling and Administrative Expenses.....................................................
Income Summary....................................................................................
Income Summary...........................................................................................
Retained Earnings...................................................................................

2008
Sales
P150,000
76,000
74,000

P 74,000
35%
P 25,900

Total

P 40,300

212,000
40,300
400
165,000
66,000
20,900
20,900
20,900

3.

Apple Company
Income Statement
Year Ended December 31, 2008

Sales................................................................................................................................
Cost of sales.......................................................................................................
Gross profit on regular sales.............................................................................................
Add Realized gross profit on installment sales (Schedule 1)............................
Total realized gross profit.................................................................................................
Less Loss on repossession.............................................................................
Total realized gross profit after adjustment for loss on repossession...............................
Selling and administrative expenses...................................................................
Net income .......................................................................................................................
Installment Sales
153

P212,000
165,000
47,000
40,300
87,300
400
86,900
66,000
P 20,900

Schedule 1

Installment contracts receivable, 1/1........................


Less Installment contracts receivable, 12/31............
Total credit for the period.........................................
Less Credit representing repossession......................
Credit representing collections.................................
Multiply by Gross profit rate....................................
Realized gross profit................................................

2007
Sales
P76 000
24,000
52,000
4,000
P48,000
30%
P14,400

2008
Sales
P150,000
76,000
74,000

P 74,000
35%
P 25,900

Total

P40,300

Problem 9 5
1.

Cost of Installment Sales...................................................................


Shipments on Installment Sales...................................................

54,400

Installment Sales................................................................................
Cost of Installment Sales.............................................................
Deferred Gross Profit, 2008........................................................

80,000

54,400
54,400
25,600

Gross profit = P25,600 P80,000 = 32%


Deferred Gross Profit, 2007...............................................................
Deferred Gross Profit, 2008...............................................................
Realized Gross Profit..................................................................

14,000
8,000
22,000

Computation:

Installment contracts receivable, 1/1.............


Less Installment contracts receivable, 12/31.
Total credit for the period..............................
Less Credit representing repossession...........
Credit representing collections......................
Multiply by Gross profit rate.........................
Realized gross profit.....................................

2007
Sales
P82,000
_ 36,000
46,000
__6,000
P40,000
__35%*
P14,000

2008
Sales
P 80,000
_55,000
25,000
___
P 25,000
___32%
P 8,000

Total

P 22,000

DGP, 1/1
P28,700 (26,600 + 2,100)
*2007 Gross profit rate= =
=
ICR, 1/1
P82,000 (36,000 + 40,000 + 6,000)

2.

Sales..................................................................................................
Merchandise Inventory, December 31...............................................
Shipments on Installment Sales.........................................................
Merchandise Inventory, January 1...............................................
Purchases.....................................................................................
Repossessed Merchandise...........................................................
Loss on Repossession..................................................................
Operating Expenses.....................................................................
Income Summary........................................................................

200,000
52,000
54,400

Realized Gross Profit.........................................................................


Income Summary........................................................................

22,000

Income Summary...............................................................................
Retained Earnings.......................................................................

31,500

35%

60,000
180,000
3,000
900
53,000
9,500
22,000
31,500

PPG Discount Center, Inc.


Income Statement
Year Ended December 31, 2008

Sales...........................................................
Cost of sales:
Inventory, January 1............................. P 60,000
Purchases.............................................. 180,000
Repossessed merchandise..................... __3,000
Cost of goods available for sale............ 243,000
Less Shipments on installment sales..... _54,400
Cost of goods available for regular sales 188,600
Less Inventory, December 31............... _52,000
Gross profit.................................................
Less Deferred gross profit on installment
sales, 2008............................................
Realized gross profit, 2008..........................
Add Realized gross profit on 2007
installment sales....................................
Total realized gross profit............................
Less Loss on repossession...........................
Total realized gross profit after adjustment
for loss on repossession........................
Operating expenses.....................................
Net income..................................................

Regular
P200,000

Installment
P80,000

Total
P280,000

_136,600
P 63,400

54,400
25,600

191,000
89,000

17,600
8,000

17,600
71,400

14,000
22,000
___900

14,000
85,400
__900

P21,100

84,500
_53,000
P31,500

Problem 9 6
1.

London Products
Schedule of Cost of Goods Sold
Year Ended December 31, 2008
Merchandise inventory, January 1....................................................................................
Purchases .......................................................................................................................
Freight-in .......................................................................................................................
Repossessed merchandise..................................................................................
Cost of goods available for sale........................................................................................
Less Merchandise inventory, December 31.........................................................
Cost of goods sold.............................................................................................................

2.

London Products
Schedule of Allocation of Cost of Goods Sold
Year Ended December 31, 2008

Cash sales ...................


Charge sales..................
Installment sales...........

Amount
P60,000
120,000
300,000

120%
125%

On Cash
Price Basis
P 60,000
100,000
240,000

Ratio to
Total
60/400
100/400
240/400

P 400,000
3.

P 48,000
238,000
12,000
14,000
312,000
52,000
P260,000

Allocated
Cost
P 39,000
65,000
156,000
P260,000

London Products
Income Statement
Year Ended December 31, 2008

Sales.................................................
Cost of goods sold..............................
Gross profit........................................
Less Unrealized gross profit:
On installment contracts
receivable,12/31 (192,000 x 144/300)
Realized gross profit..........................
Add Realized gross profit on
prior years' sales (Schedule 1):
2006.....................................
19,200
2007.....................................
14,700
Total realized gross profit..................
Less Loss on repossession
(Schedule 2)................................
Total realized gross profit after
adjustment for loss on
repossession................................
Less Operating expenses....................
Net income ........................................

Installment
Charge
Cash
Total
Sales
Sales
Sales
P480,000 P 300,000
P120,000
P 60,000
260,000
156,000
65,000
39,000
P 220,000 P 144,000P 55,000P 21,000
92,160
127,840

92,160
51,840

33,900
161,740

33,900
85,740

10,200

10,200

151,540
93,000
P 58,540

P 75,540

Schedule 1
2006
Installment contracts receivable, January 1:
2006 P32,000 40%................................................................

P80,000

2007

2007 P56,000 35%................................................................


Less Installment contracts receivable, December 31.........................
Total credits.......................................................................................
Less Credit representing repossession...............................................
Total collections.................................................................................
Multiply by Gross profit rate.............................................................
Realized gross profit..........................................................................

_22,000
58,000
_10,000
P48,000
___40%
P19,200

P160,000
__90,000
70,000
28,000
P 42,000
___35%
P 14,700

2006
P 2,000

2007
P12,000

Total
P 14,000

10,000

28,000

38,000

Schedule 2

Fair market value of repossessed merchandise....


Less Unrecovered cost:
Unpaid balance..............................................
Less Unrealized profit
2006 P10,000 x40%.............................
2007 P28,000 x35%.............................
Balances ............................................................
Gain (loss) on repossession..................................

4,000
__6,000
P(4,000)

9,800
13,800
18,200
__24,200
P( 6,200) P( 10,200)

Problem 9 7
1.

2007
2007
2007 installment sales (P400,000 x 42%*)..................................P 168,000
2008:
2007 installment sales (P173,000 x 42%)....................................
2008 installment sales (P560,000 x 38.5%*)............................... ________
Deferred gross profit.......................................................................... P 168,000

2008

P 72,660
__215,600
P 288,260

*Computation of Gross profit percentages (see next page)


2007
Installment sales................................................................................P2,210,000
Less Trade-in allowances (P226,000 P158,000)............................. _______
Adjusted installment sales................................................................. 2,210,000
Cost of sales:
Inventories, January 1 (new).......................................................

Purchases (new).......................................................................... 1,701,800


Repossessed merchandise............................................................

Cost of goods available for sale................................................... 1,701,800


Less: Inventories, December 31
New merchandise.................................................................. 420,000
Repossessed merchandise..................................................... _______
Total...................................................................................... 420,000
Cost of sales................................................................................ 1,281,800
Gross profit........................................................................................ P 928,200
Gross profit percentages....................................................................
*2007 : P195,000 x 20%=P39,000
2008 : P110,000 x 40% =_44,000
P83,000

42%

2008
P3,100,000
____68,000
_3,032,000
420,000
1,767,000
_83,000*
2,270,000

358,820
____46,500
405,320
_1,864,680
P1,167,320
38.5%

Uncollectible installment contracts expense, per books.


Correct Uncollectible installment contracts expense:
Fair market value of repossessed merchandise
2007 sales (P195,000 x 20%)........................... P 39,000
2008 sales (P110,000 x 40%)............................ __44,000
Unrecovered cost
2007 sales [P105,000 x (100% 42%)]............ 60,900
2008 sales [P82,000 x (100% 38.5%)]........... __50,430
Adjustment to Uncollectible installment contracts expense

P 99,000

P 83,000

__111,330

__28,330
P 70,670

Installment
Sales
P3,032,000
_1,864,680
1,167,320

Total
Sales
P3,237,000
_2,022,680
1,214,320

__247,170
920,150

__247,170
967,150

___51,240
971,390
___28,330
P 943,060

___51,240
1,018,390
___28,330
990,060
__592,960
P 397,100

Fortune Sales Corporation


Income Statement
Year Ended December 31, 2008
Cash
Sales
Sales
..................................................................... P205,000
Cost of sales.................................................................. _158,000
Gross profit................................................................... P 47,000
Less Unrealized gross profit on 2008 installment
sales (Schedule 1)...................................................
Realized gross profit on 2008 sales...............................
Add Realized gross profit on 2007 installment
sales (Schedule 2)...................................................
Total realized gross profit..............................................
Less Uncollectible installment contracts expense..........
Total realized gross profit after adjustment...................
Operating expenses.......................................................
Net income....................................................................

158

Chapter 9

Schedule 1
Installment contracts receivable 2008, December 31....................
Installment contracts receivable 2008 defaulted...........................
Total............................................................................................
Multiply by 2008 gross profit percentage.....................................
Unrealized gross profit on 2008 installment sales........................

P 560,000
___82,000
P 642,000
___38.5%
P 247,170

Schedule 2
Installment contracts receivable 2007, January 1...............................
Less Installment contracts receivable 2007, December 31.................
Total credits for the period.................................................................
Less Installment contracts receivable 2007 defaulted........................
Total collections.................................................................................
Multiply by 2007 gross profit percentage..........................................

P 400,000
__173,000
227,000
__105,000
P 122,000
_____42%

Realized gross profit on 2007 installment sales.................................

51,240

9-8
1.

Apportionment of cost (P600,000) to Lots 1, 2 and 3:


Lot 1 :2/3 x P360,000....................................
Lot 2 :2/3 x P240,000....................................
Lot 3 :1/3......................................................
1/3 x P240,000........................................
Total cost.......................................................

P 240,000
160,000
P120,000
__80,000

__200,000
P 600,000

Journal Entries for 2007


March 31
Cash..............................................................................................
36,000.00
Notes Receivable (Lot 2)............................................................... 364,000.00
Lot 2 .....................................................................................
Deferred gain on Sale of Land................................................
June 30
Cash.............................................................................................. 120,000.00
Notes Receivable (Lot 3)............................................................... 720,000.00
Lot 3.......................................................................................
Deferred Gain on Sale of Land...............................................
Cash..............................................................................................
16,000.00
Interest Income (P364,000 x 12% x 3/12)...............................
Notes Receivable (Lot 2)........................................................
September 30
Cash..............................................................................................
Interest Income (P358,920 x 12% x 3/12)...............................
Notes Receivable (Lot 2)........................................................

160,000.00
240,000.00

200,000.00
640,000.00
10,920.00
5,080.00

16,000.00
10,767.60
5,232.40

Installment Sales
159

October 31
Cash..............................................................................................
72,000.00
Notes Receivable (Lot 1)............................................................... 288,000.00
Lot 1.......................................................................................
Deferred Gain on Sale of Land...............................................
December 31
Cash..............................................................................................
Notes Receivable (Lot 1)........................................................
Notes Receivable (Lot 2)........................................................
Notes Receivable (Lot 3)........................................................
Interest Income.......................................................................

240,000.00
120,000.00

78,000.00
6,240.00
5,389.37
6,800.00
59,570.63

Computation:
Total
Collections...................................... P78,000.00
Apply to interest:
Lot 1 P288,000.00 x 12% x 2/12
Lot 2 P353,687.60 x 12% x 3/12 59,570.63
Lot 3 P720,000.00 x 12% x 6/12 _________
Apply to principal........................... P18,429.37

Lot 1
P12,000.00

Lot 2
P16,000.00

Lot 3
P50,000.00

10,610.63
_________
P 5,389.37

_43,200.00
P 6,800.00

5,760.00
_________
P 6,240.00

2.

Deferred Gain on Sale of Land (Lot 1).............................................. 26,080.00


Deferred Gain on Sale of Land (Lot 2).............................................. 31,021.06
Deferred Gain on Sale of Land (Lot 3).............................................. 96,368.00
Realized Gain on Sale of Land....................................................

153,469.06

Computation:
Lot 1
P78,240.00

Collections applied to principal.......


P126,800.00
Multiply by Gross profit rates:
Lot 1 P120,000 P360,000.....
Lot 2 P240,000 P400,000.....
Lot 3 P640,000 P840,000.....
Realized gain...................................
3.

Lot 2
P51,701.77

Lot 3

33.33%
_________
P26,080.00

60%
_________
P31,021.06

Lot 3 (80% x P200,000).....................................................................160,000.00


Deferred Gain on Sale of Land (Lot 3) (P640,000 P96,368)..........543,632.00
Loss on Repossession........................................................................ 9,568.00
Notes Receivable (Lot 3) (P720,000 P6,800)...........................

_____76%
P96,368.00

713,200.00

Problem 9 9
Galaxy Investment Company
Income Statement
Year Ended December 31, 2008
Sales Schedule 1) ....................................................................................................
Cost of sales (Schedule 2)........................................................................................
Gross profit............................................................................................................
Less Sales commissions..........................................................................................
221,000
Gross profit............................................................................................................
Less Deferred gross profit
Installment Notes Balance P5,370,000
=
Installment Sales
P8,060,000

P 8,060,000
1,612,000
6,448,000

6,227,000

=67% x P6,227,000

4,172,090

Realized gross profit................................................................................................


Expenses:
Advertising and promotion............................................................................ P 730,000
Sales manager's salary...................................................................................
120,000
General office expenses (1/4 x P236,000)..................................................... 59,000
Net profit ...............................................................................................................

2,054,910

909,000
P 1,145,910

Schedule 1

A lots : 26 @ P150,000................................................
B lots : 32 @ P100,000................................................
C lots : 12 @ P80,000..................................................
.........................................................

Total
Sales Price
P3,900,000
3,200,000
960,000
P8,060,000

Cash
Received
P1,650,000
800,000
240,000
P2,690,000

Installment
Notes Balance
P 2,250,000
2,400,000
720,000
P 5,370,000

Schedule 2
Class

Number of
Lots

Unit
Price

Total
Sales Value

A........................................................................
B........................................................................
C........................................................................
Total.. ..........................................................

80
100
120
300

P150,000
100,000
80,000

Cost of tract:
Cost of land....................................................................................................
Legal fees, etc................................................................................................
Grading contract.............................................................................................
Water and sewerage system contract.............................................................
Paving contract...............................................................................................
General office expenses (3/4 x P236,000).....................................................
Total..............................................................................................................

P12,000,000
10,000,000
9,600,000
P31,600,000
P 4,800,000
600,000
225,000
184,900
266,300
177,000
P 6,253,200

P6,253,200
Cost rate : = 20% (rounded off)
P31,600,000
Cost of sales (P8,060,000 x 20%)...........................................................................
Installment Sales
161

P 1,612,000

Problem 9 10
Rizal Company
Income Statement
Year Ended December 31, 2008
Installment sales [(P14,300 x 7) + (P725 x 4)]...........................................
Cost of goods sold on installment (schedule 1)..........................................
Gross profit. ...............................................................................................
Less Deferred gross profit on sales
(P103,000 P21,000 = P82,000 x 23%*).........................................
Realized gross profit on 2008 sales............................................................
Add Realized gross profit on prior years' sales
2006 : P60,000 x 33-1/3*...................................................................
2007 : P115,000 x 35%*....................................................................
Total realized gross profit...........................................................................
Less Loss on repossession (Schedule 4).....................................................
Total realized gross profit after adjustment................................................
General and administrative expenses..........................................................
Net income (loss).......................................................................................

P103,000
__79,310
23,690
__18,860
4,830
P20,000
_40,250

__60,250
65,080
__33,100
31,980
__50,000
P(18,020)

*See Schedule 3
Schedule 1
Purchases (P10,500 x 8).............................................................................
Repossessed merchandise...........................................................................
Cost of goods available for sale..................................................................
Less Inventory, December 31
Number of units on hand...................................................................
Multiply by average unit cost (Schedule 2)........................................
Cost of goods sold on installment...............................................................
Schedule 2

P 84,000
___2,520
86,520
1
P 7,210

___7,210
P 79,310

Purchases during 2008 (P10,500 x 8).........................................................


Add Repossessed merchandise...................................................................
Total ..........................................................................................................
divide by Number of units (8 + 4)..............................................................
Average unit cost........................................................................................

P 84,000
___2,520
P 86,520
_____
P 7,210

162

Chapter 9

Schedule 3
.......................................................
Sales
2006 : P15,000 x 10.......................................
2007 : P14,000 x 20.......................................
2008 : P14,300 x 7.........................................
P725 x 4..............................................
Sales
.......................................................
Cost of goods sold:
Inventory, January 1........................................
Purchases .......................................................
Repossessed merchandise................................
Cost of goods available for sale.......................
Less Inventory, December 31..........................
Cost of goods sold...........................................
Gross profit. ............................................................
Gross profit rates......................................................

2006

2007

2008

P150,000
P280,000
_______
150,000

_______
280,000

100,100
__2,900
103,000

120,000
_____
120,000
_20,000
100,000
P 50,000
33-1/3%

20,000
162,000
_____
182,000
_____
182,000
P 98,000
35%

84,000
_2,520
86,520
_7,210
79,310
P23,690
23%

Schedule 4
Fair market value of repossessed merchandise...........................................
Less Unrecovered cost
Unpaid balance:
Original sales amount (P14,000 x 4)........................................... P 56,000
Collections prior to repossession................................................. __1,200
Total. ...........................................................................................
54,800
Less Unrealized profit (P54,800 x 35%)............................................ _19,180
Loss on repossession..................................................................................
Long-Term Construction Contracts

CHAPTER 10
MULTIPLE CHOICE ANSWERS AND SOLUTIONS
10-1: a
Percentage of Completion Method:

P 2,520

_35,620
P33,100

Contract Price
Less:Total estimated cost
Cost incurred
Estimated remaining cost
Gross profit estimated
% of completion (200,000/600,000)
Gross profit to be recognized
Zero Profit Method:

P1,000,000
P 200,000
_400,000

__600,000
400,000
__33 1/3%
P 133,333
0

10-2: a P100,000
Contract Price
Less: Total estimated cost
Estimated gross profit
% of completion:
2007 (3,900,000/7,800,000)
2008(6,300,000/8,100,000)
Gross profit earned to date
Less: Gross profit earned in prior year
Gross profit earned each year

2007
2008
P9,000,000 P9,000,000
_7,800,000 _8,100,000
1,200,000
900,000
50%
_________ ______78%
600,000
700,000
________ ___600,000
P 600,000 P 100,000

10-3: a
Contract Price
Less: Total estimated cost (3,600,000 + 1,200,000)
Estimated gross profit
% of completion (3,600,000/4,800,000)
Gross profit earned to date
Less: Gross profit earned in 2007
Gross profit earned in 2008

P6,000,000
_4,800,000
1,200,000
_____75%
900,000
__600,000
P 300,000

Contract Price
Less: Total estimated cost (930,000 + 2,170,000)
Loss

P3,000,000
_3,100,000
(P 100,000)

10-4: b

164
Chapter 10

10-5: b
Total cost to date, 2008 (4,800,000 X 60%)
Less: Cost incurred in 2007 (4,500,000 X 20%)
Cost incurred in 2008

P2,880,000
__900,000
P1,980,000

Percentage of Completion Method:


Contract Price
Less: Total estimated cost (900,000/1,800,000)
Estimated gross profit
% of completion (900,000/2,700,000)

P3,000,000
_2,700,000
300,000
___33.33%

10-6: a

Gross profit recognized, 2007


Add: Cost Incurred
Construction in Progress - 2007
Zero Profit Method:
Cost incurred to Construction in Progress - 2007

100,000
___900,000
P 1,000,000
P 900,000

10-7: a
Contract Price
Less: Total estimated cost
Estimated gross profit
% of completion
Gross Profit earned to date
Gross Profit earned in prior year
Gross Profit earned this year

2007
2008
P4,200,000 P4,200,000
_3,000,000 _3,750,000
1,200,000
450,000
_____20% ____100%
240,000
450,000
_______ __240,000
P 240,000 P 210,000

10-8: b
Collections:
Contract Billings
Less: Accounts receivable
Collections
Initial Gross Profit:
Contract Price
Gross Profit rate:
Income recognized
Divide by Construction in Progress
Initial Gross Profit

P 47,000
___15,000
P 32,000
P 800,000
10,000
50,000 =_____20%
P 160,000

Long-Term Construction Contracts

10-9: a
Gross profit (loss) earned in 2008
Gross profit earned in prior years
Gross profit earned to date - 2008
Divide by percentage of completion - 2008
Estimated gross profit - 2008
Less: Contract price
Total estimated cost
Less: Cost incurred - 2008
Cost incurred to date - 2007
Less: Cost incurred - 2006
Cost incurred in 2007
10-10: b

(P 20,000)
_180,000
160,000
___100%
160,000
2,000,000
1,840,000
_820,000
1,020,000
__360,000
P 660,000

Gross profit earned to date - 2007 (P40,000 + P140,000)


Divide by estimated gross profit - 2007:
Contract price
P2,000,000
Gross profit rate [180,000/(1,020,000 + 180,000)]
___X 15%
Percentage of completion - 2007

P 180,000
__300,000
60%

10-11: a, Refer to Q 10-10 solutions.


10-12: d
Contract price
Estimated gross profit - 2007 (Refer to Q 10-10)
Total estimated cost
Less: Cost incurred to date - 2007 (refer to Q 10-9)
Estimated cost to complete - 2007

P2,000,000
__300,000
1,700,000
1,020,000
P 680,000

2007: Construction in progress


Less: Construction costs
Gross profit recognized - 2007

P 244,000
__210,000
P 34,000

2008: Construction in progress (P728,000-P244,000)


Less: Construction costs
Gross profit recognized - 2008

P 484,000
__384,000
P 100,000

10-13: d

166
Chapter 10

10-14: d
Project 1
Project 2
Percentage of Completion Method:
Contract price
P 300,000
Less: Total estimated cost
Cost incurred to date - 2008
P 280,000
Estimated cost to complete
___70,000
Total
__350,000
Estimated gross profit (Loss)
(50,000)
Percentage of completion
_______
Profit (loss) to be recognized
(P 50,000)
Total is (P10,000)

P 420,000
P 240,000
__120,000
__360,000
60,000
__66.67%
P 40,000

Zero Profit Method - The loss (P50,000) for project 2 only.


10-15: a
Contract price (cost X 120%)
P3,744,000
Less: Total estimated costs
(1) Cost incurred to date
3,120,000
Estimated cost to complete
________
(2) Total
_3,120,000
Estimated gross profit
624,000
Percentage of completion (1 2)
____100%
Gross profit earned to date
624,000
Gross profit earned in prior years
__477,360
Gross profit earned this year

2006
2007 2008
P3,744,000 P3,744,000
546,000

1,544,400

_2,054,000 _1,315,000
_2,600,000 _2,860,000
1,144,000

884,000

_____20%

_____54%

240,240

477,360

_______

__240,240

P 240,240

P 237,120

146,640
10-16: d
Contract price
P6,300,000
Less: Total estimated cost
Cost incurred to date
3,040,000
Estimated cost to complete
_1,960,000
Total
P5,000,000
Estimated gross profit
1,300,000
Percentage of completion:
2007 (1,425,000 - 50,000) 5,500,000
2008 (3,040,000 - 50,000) 5,000,000
__59.80%
Profit earned to date
777,400
Less: Gross profit earned in prior year
__200,000
Gross profit earned this year
P 577,400

Long-Term Construction Contracts

10-17: a

2007 2008
P6,300,000
1,425,000
_4,075,000
P5,500,000
800,000
25%
________
200,000
________
P 200,000

Cash collections:
Progress billings
Less: Accounts receivable, end
Collection

P1,500,000
__500,000
P1,000,000

Cost incurred to date:


Construction in Progress
Less: Gross profit earned
Cost incurred to date

P1,600,000
__200,000
P1,400,000

10-18: d
Percentage of Completion Method:
Apartment A
2007
2008
1,620,000 1,620,000

Contract price
2,520,000
Less: Total Estimated Costs
(1)
Cost incurred to date
P2,310,000
Estimated cost to complete
(2)
Total estimated cost
2,310,000
Estimated Gross Profit
210,000
Percentage of completion (1 2)
_100.00%
Gross profit earned to date
210,000
Less: Gross profit earned in Prior years
__187,200
Gross Profit earned this year
22,800
Total Gross Profit 20 (P75,000 + P22,800)

Apartment B
2007 2008
2,520,000

P 600,000 P1,200,000 P1,560,000


840,000
1,440,000

240,000
1,440,000

690,000
2,250,000

180,000

180,000

270,000

_41.67%

_83.33%

_69.33%

75,000

150,000

187,200

_______

___75,000

_______

P 75,000

P 75,000

P 187,000

P97,800

Zero Profit Method - P210,000 gross profit earned in 2008 for Apartment B.
10-19: d
2007
Contract price:
2007
2008 (P6,000,000-P50,000)
P5,950,000
Less: Total estimated costs
(1) Cost incurred to date
2,650,000
Estimated cost to complete
(2) Total estimated cost
2,650,000
Estimated Gross Profit
3,300,000
Percentage of completion (1 2)

P6,000,000
_________
2,340,000
260,000
2,600,000
3,400,000
____90%

2008

___100%
Gross profit earned to date
3,300,000
Less: Gross profit earned in Prior year
3,060,000
Gross Profit earned this year

3,060,000
_______
P3,060,000

240,000
168
Chapter 10

10-20: a
(1)Cost incurred to date
P6,150,000
(2)Estimated cost to complete
(3)Total Estimated Costs
6,150,000

2006
P3,400,000
1,600,000
5,000,000

Percentage of completion (1 3)
Contract price
P6,000,000
Less: Total estimated cost
6,150,000
Estimated Gross Profit
(150,000)
Percentage of completion
Gross profit earned (loss) to date
(150,000)
Add: Cost incurred to date
6,150,000
Construction in Progress
6,000,000
Less: Contract billings
6,000,000
Balance

68%
P6,000,000
5,000,000

2007 2008
P5,950,000
150,000
6,100,000
98%
P6,000,000
6,100,000

1,000,000

(100,000)

68%
680,000

100%
(100,000)

3,400,000
4,080,000

5,950,000
5,850,000

3,200,000

5,200,000

P 880,000

P 650,000

10-21: c
Construction in Progress:
Cost incurred to date, 2007
Gross profit earned, 2007 (Schedule 1)
P2,725,000
Less: Contract billings, 2006 (P3,250,000 x 75%)
2,437,500

P2,625,000
100,000

Excess of Construction in Progress over Contract Billings (CA)


P 287,500
Schedule 1 Computation of gross profit earned, 2006
Contract price
P3,250,000
Total estimated cost:
Cost to date
2,625,000
Estimated cost to complete
750,000
Total
3,375,000

2006
P3,250,000
1,075,000
1,612,500
2,687,500

Estimated gross profit (loss)


(125,000)
% of completion

562,500

Gross profit (loss) to date


(125,000)
Gross profit earned in prior years
225,000

225,000

40%

Gross profit earned this year


P 100,000

P 225,000

10-22: a
2005
2006
P2,800,000 P2,800,000

Contract price
P2,800,000
Estimated cost:
Cost to date
2,440,000
Estimated costs to complete
380,000

1,300,000
1,360,000

780,000

Total
2,820,000

2,660,000

Estimated gross profit


(20,000)

140,000

% of completion
Long-Term Construction Contracts

1,960,000

2,740,000
60,000

48.87%

71.53%

10-23: b
2007

Project A

Project B

P2,900,000

P3,400,000

1,680,000
1,120,000

1,440,000
1,760,000

Project

C
Contract price
P 1,700,000
Estimated costs:
Cost to date
Estimated cost to complete
960,000
Total
1,280,000

2,800,000

3,200,000

Estimated gross profit


% of completion

100,000
60%

200,000
45%

Gross profit earned this year (P255,000)


P 105,000

P 60,000

P 90,000

Project A

Project B

Project C

P2,900,000

P3,400,000

P1,700,000

2,120,000
0

1,183,000
1,360,000

560,000
117,000

2,640,000

3,480,000

1,300,000

2008

320,000

420,000

Project

D
Contract price
P 2,000,000
Estimated costs
Cost to date2,640,000
Estimated costs to complete
1,040,000
Total
1,600,000
Estimated gross profit (loss)
% of completion

260,000
100%

Gross profit (loss) to date


Gross profit earned in prior year

260,000
60,000

(80,000)

400,000
91%

400,000

(80,000)
364,000
90,000
105,000

140,000

Gross profit earned this year(P609,000)


P 140,000

P 200,000

P 10,000

2007
P 255,000

Gross profit earned


P 609,000
General and administrative expenses
Net income

P 259,000

120,000
P 135,000

120,000

P 489,000

10-24: c
Contract price
P10,000,000
Gross profit earned to date, 2008 (P900,000 P100,000)

800,000

Total cost to date, 2008


9,200,000
Less: cost incurred in 2008
4,100,000
Cost to date, 2007
P 5,100,000
Gross profit earned to date

900,000
Divided by % of completion:
(P5,100,000 + P900,000) / P10,000,000
Estimated gross profit, 2007
P 1,500,000

10-25: d
Construction in progress:
Cost incurred to date
P 440,000
Gross profit earned to date (P2,500,000 P2,000,000)

110,000

Total
Less: Contract billings (P2,500,000 x 30%)

550,000
750,000

Excess of contract billings over construction in progress (CL)


P( 200,000)
170
Chapter 10

10-26: a
Contract price
Total estimated cost:
Cost incurred to date:
Site labor cost
Cost of construction materials
Depreciation of special plant & equip
Total
Estimated cost to complete
Estimated gross profit
Percentage of completion (45/100)
Gross profit to be recognized

P120,000,000
10,000,000
30,000,000
5,000,000
45,000,000
55,000,000

Cost incurred to date- 2007


Total estimated cost (8,000,000 / 40%)
Estimated cost to complete
Cost incurred in 2007

20,000,000
8,000,000

100,000,000
20,000,000
45%
P 9,000,000

10-27: a
P12,000,000
3,700,000

3,700,000
Cost incurred in 2006
Estimated cost at completion- 2006
Total estimated cost- 2006

8,300,000
12,450,000
P20,750,000

Percentage of completion- 2006 (8,300,000/ 20,750,000) = 40%


10-28: a
Contract price
Total estimated cost:
Cost incurred to date
Estimated cost to complete
Total estimated cost
Estimated gross profit
Percentage of completion
Gross profit recognized

2007
Contract 1
Contract 2
P600,000
P450,000
150,000
150,000
300,000
300,000
50%
P150,000

87,500
162,500
250,000
200,000
35%
P70,000

CIP-2007
P237,500

P220,000

Contract 1

2008
Contract 2

Contract

600,000
350,000
250,000
80%
200,000
150,000
50,000

450,000
300,000
150,000
60%
90,000
70,000
20,000

900,000
500,000
400,000
36%
144,000
144,000

3
Contract price
Total estimated cost
Estimated gross profit
Percentage of completion
Gross profit earned to date
Gross profit earned in 2007
Gross profit earned this year

Long-Term Construction Contracts

10-29: a
Bicol
Contract price
P875,000
Total estimated cost
Cost incurred
656,250
Est. cost to complete
Total estimated cost
656,250
Estimated gross profit
218,750
Percentage of completion
100%
Gross profit earned
P218,750
Total cost incurred
Total gross profit earned
Construction in progress
Less: Billings

Davao
Aklan
P1,225,000 P437,500
175,000
700,000
875,000
350,000
20%
P 70,000

Percentage of completion
1,006,250
332,500
1,338,750
1,312,500

175,000
175,000
350,000
87,500
50%
P43,750

Total
1,006,250

332,500

Zero Profit
1,006,250
218,750
1,225,000
1,312,500

Due from (to)

26,250

(87,500)

10-30: a
Contract price
Total estimated cost:
Cost incurred
Estimated cost to complete
Estimated gross profit
Percentage of completion
Gross profit recognized

P40,825,000
8,475,000
28,400,000

172

36,875,000
3,950,000
22.983%
P 907,830

Chapter 10

SOLUTIONS TO PROBLEMS
Problem 10 1
(a)
Contract Price
P 450,000
Less: Total estimated cost
(1) Cost incurred to date
Estimated costs to complete
_______
(2) Total
_320,000
Estimated gross profit
Percentage of completion (1 2)
___100%
Estimated gross profit to date
Less: Gross profit earned in prior year
__100,000
Gross profit earned this year
30,000

2007
P 450,000
200,000
__100,000

2008

320,000

__300,000
150,000
______2/3

130,000

100,000
_______

130,000

P 100,000P

(b)

Contract Price
P 450,000
Less: Total cost incurred
__320,000
Gross profit
P 130,000

(c)

2007: Construction in Progress


Cost of construction
Construction Revenue
2008: Construction in Progress
Cost of Construction
Construction Revenue

100,000
200,000
300,000
30,000
320,000
350,000
Problem 10 2

(a)

Construction Revenue
P1,250,000
Less: Cost incurred
_1,250,000
Gross profit 2008

0
Construction in Progress (cost incurred)
P1,250,000
Less: Contract billings (P5,800,000 x 30%)
_1,740,000
Billings in excess of related costs
P(490,000)
(b)

Contract price
P5,800,000
Less: Total estimated costs
Cost incurred to date
Estimated costs to complete
5,000,000
Estimated gross profit
Percentage of Completion (P1,250,000 500,000)
_____25%
Gross profit
P 200,000

P1,250,000
3,740,000
800,000

Construction on Progress (P1,250,000 + P200,000)


P1,450,000
Less: Contract billings
_1,740,000
Billings in excess of related costs
P(290,000)
Long-Term Construction Contracts

(a)
Contract Price
P55,000,000
Less: Total estimated costs

173

Problem 10 3
2005
P55,000,000

2006
P55,000,000

2007
P55,000,000

2008

(1) Cost incurred to date


50,000,000
Estimated costs to complete
________
(2) Total
50,000,000
Estimated gross profit
5,120,000
Percentage of completion (1 2)
____100%
Gross profit earned to date
5,000,000
Gross profit earned in prior yr(s)
_3,500,000
Gross profit earned the year
P 1,500,000

15,000,000

25,000,000

35,000,000

_35,000,000

25,000,000

15,000,000

_50,000,000

50,000,000

50,000,000

5,000,000

5,000,000

5,000,000

______30%

_____50%

_____70%

1,500,000

2,500,000

3,500,000

________

_1,500,000

_2,500,000

P 1,500,000

P 1,000,000

P 1,000,000

(b)

2007
(1) Construction in Progress
Cash or Payable
15,000,000

15,000,000

(2) Accounts Receivable


Contract Billings
20,000,000

15,000,000

(3) Cash
Accounts Receivable
25,000,000

12,000,000

(4) Construction in Progress


Cost of Construction
Construction Revenue
16,500,000

1,000,000
15,000,000

2008
15,000,000

15,000,000
20,000,000
15,000,000
25,000,000
12,000,000
1,500,000
15,000,000
16,000,000
Problem 10 4

(a)
Cost incurred to date
P10,000,000
Divide by total estimated cost
_12,000,000
Percentage of Completion

(b)

Contract Price
P15,000,000
Less: Total Estimated Cost
Cost incurred to date
10,000,000
Estimated costs to complete
__2,000,000
Total
_12,000,000

2006
P 1,000,000

2007
P 5,500,000

P 9,000,000

P11,000,000

11.11%

50%

2006
P15,000,000

2007
P15,000,000

1,000,000

5,500,000

__8,000,000

__5,500,000

__9,000,000

_11,000,000

2008

83.33%
2008

Estimated gross profit


3,000,000
Percentage of completion
___83.33%
Gross profit earned to date
9,500,000
Less: Gross profit earned in prior yrs.
_2,000,000
Gross profit earned this year
P 500,000

6,000,000

4,000,000

___11.11%

______50%

666,600

2000,000

________

___666,600

P 666,600

P 1,333,400

174
10

(c)

Chapter

(1) Construction in progress (cost incurred)


Cash
1,000,000

1,000,000

(2) Accounts Receivable


Contract Billings
1,325,000

1,325,000

(3) Cash
Accounts Receivable
1,200,000

1,200,000

(4) Construction in progress (gross profit)


Cost of construction
Construction Revenue
1,666,600

666,600
1,000,000

Problem 10 5
(1)

2005
P14,000,000

Contract Price
P14,000,000
Less: Total Estimated Cost
Cost incurred to date
6,500,000
13,900,000
Estimated cost to complete
__6,800,000
________
Total
_13,300,000
13,900,000
Estimated gross profit
700,000
Percentage of completion
___48.87%
____100%
Gross profit (loss) to date
342,090
Less: Gross profit (loss) in prior yrs. ________
( 100,000)
Gross profit (loss) this year
P 342,090
200,000
(2)

2005
Cost of construction
6,500,000
Construction in progress 342,090

2006
P14,000,000

2007
P14,000,000

9,800,000

12,200,000

_3,900,000

_1,900,000

13,700,000

14,100,000

300,000
___71.53%

( 100,000)
_____100%

100,000

214,590
___342,090

( 100,000)
___214,590

100,000

P( 127,500)

P( 314,590)

2006
3,300,000
127,500

2007
2,400,000
314,590

2008

2008
1,700,000
200,000

Construction Revenue
1,900,000

6,842,090

3,172,500

2,085,410

Problem 10 6
(1)
Contract Price
Less: Total estimated costs
Cost incurred to date
Estimated costs to complete
Total
Estimated gross profit
Percentage of completion
Gross profit (loss) to date
Gross profit (loss) in prior yrs.
Gross profit (loss) this year

2005
P 6,000,000

2006
P 6,000,000

2007
P 6,000,000

3,400,000
_2,100,000
_5,500,000
500,000
___61.82%
309,100
________
P 309,100

5,950,000
___150,000
_6,100,000
( 100,000)
_______
( 100,000)
__309,100
P 409,100

6,150,000
________
_6,150,000
( 150,000)
________
( 150,000)
( 100,000)
P 50,000

Long-Term Construction Contracts


175

(2)
Cost of construction
Construction in progress
50,000
Construction Revenue
150,000
(3)

Cash
Accounts Receivable
Contract Billings
Construction in progress

2005
3,400,000
309,100

2006
2,550,000
409,100

3,709,100

2007
200,000

2,140,900

400,000
400,000
6,000,000
6,000,000
Problem 10 7

(1)

2006
P16,000,000

Contract Price
Less:Total Estimated Cost
Cost incurred to date
4,600,000
Estimated costs to complete
__9,640,000
Total
_14,240,000
Estimated gross profit
1,760,000
Engineer's estimate of comp.
______31%
Gross profit to date
545,600
Less: Gross profit earned in prior yrs. ________
Gross profit earned this yr.
P 545,600
(2)
(a) Construction on progress
Cash
5,250,000
(b) Accounts receivable
Contract billings
5,000,000

2007
P16,000,000

2008
P16,000,000

9,100,000
__5,100,000
_14,200,000
1,800,000
______58%
1,044,000
__545,600
P 498,410

14,350,000
_________
_14,350,000
1,650,000
_____100%
1,650,000
_1,044,000
P 606,000

2006
4,600,000

2007
4,500,000
4,600,000

5,000,000

2008
5,250,000
4,500,000

6,000,000
5,000,000

5,000,000
6,000,000

(c) Cash
Accounts receivable
6,100,000

4,500,000

5,400,000
4,500,000

6,100,000
5,400,000

(d) Cost of constructions


4,600,000
4,500,000
5,250,000
Construction in progress
545,600
498,400
606,000
Construction revenue
5,145,600
4,998,400
5,856,000
(e) Contract billings
16,000,000
Construction on progress
16,000,000
(3)

Zero Profit Method: 2008 Entres


(a) Construction in progress
Cash / accounts payable

5,250,000
5,250,000

(b) Accounts receivable


Contract billings

5,000,000
5,000,000

176
Chapter 10

(c) Cash
Accounts receivable

6,100,000

(d) Cost of construction


Construction in progress
Construction revenue

5,250,000
1,650,000

6,100,000

6,900,000

(e) Contract billings


Construction in progress
(4)

16,000,000
16,000,000

The following entry would be the only one different from (2).
*

Cost of construction
Construction in progress
Construction revenue
6,720,000
*

2006
2007
2008
4,414,400
3,821,600
6,114,000
545,600
498,400
606,000
4,960,000
4,320,000

Total estimated costs x estimated percentage of completion.


Problem 10 8

(1)
Contract Price
P6,500,000
Less:Total Estimated Costs
Cost incurred to date
6,850,000
Estimated costs to complete
________
Total
_6,850,000
Estimated gross profit (loss)

2006
P6,500,000

2007
P6,500,000

2,150,000

5,250,000

_3,850,000

_1,500,000

_6,000,000

_6,750,000

500,000

(250,000)

2008

(350,000)
Less: Gross profit (loss) in prior yrs.
_(250,000)
Gross profit (loss) this years
P( 600,000)
(2)

________

___520,000

P 520,000

P( 250,000)

In 2008 when the project is completed.

Franchise Accounting

CHAPTER 11
MULTIPLE CHOICE ANSWERS AND SOLUTIONS
11-1: b
No revenue is to be reported. Because the franchisor fails to render substantial
services to the franchisee as of December 31, 2008.
11-2: c
Initial franchise fee
Less: Cost of franchise
Net income

P5,000,000
____50,000
P4,950,000

11-3: a
The total initial franchise fee of P500,000 is to be recognized as earned because the
collectibility of the note for the balance is reasonably assured.
11-4: b
Cash downpayment
Collection of note applying to principal
Revenue from initial franchise fee

P 100,000
__200,000
P 300,000

Cash downpayment, January 2, 2008


Collection applying to principal, December 31, 2008
Total Collection
Gross profit rate [(5,000,000-500,000) 5,000,000]
Realized gross profit, December 31, 2008

P2,000,000
_1,000,000
3,000,000
_____90%
P2,700,000

Face value of the note (P1,200,000 - P400,000)


Present value of the note (P200,000 X 2.91)
Unearned interest income, July 1, 2008

P 800,000
__582,000
P 218,000

Initial franchise fee


Less: unearned interest income
Deferred revenue from franchise fee

P1,200,000
__218,000
P 982,000

11-5: a

11-6: b

11-7: d

11-8: d
Initial franchise fee
Continuing franchise fee (P400,000 X .05)
Total revenue
Cost

P 500,000
___20,000
520,000
___10,000

Net income

P 510,000

178
Chapter 11

11-9: b
Deferred Revenue from franchise fee:
Downpayment
Present value of the note (P1,000,000 X 2.91)
P8,910,000
Less: Cost of franchise fee
_2,000,000

P6,000,000
2,910,000

Deferred gross profit


P6,910,000
Gross profit rate (6,910,000 8,910,000)

77.55%

Downpayment (collection during 2008)


P6,000,000
Gross profit rate
___77.55%
Realized gross profit from initial franchise fee
P4,653,000
Add: Continuing franchise fee (5,000,000 X .05)
__250,000
Total
P4,903,000
Less: Franchise expense
___50,000
Operating income
P4,853,000
Interest income, 12/31/05 (P2,910,000 X 14%) X 6/12
__203,700
Net income
P5,056,700
11-10: b
Face value of the note receivable
P1,800,000
Present value of the note receivable
1,263,900
Unearned interest income
536,100
Initial franchise fee
P3,000,000
Less: Unearned interest income
536,100
Deferred revenue from franchise fee
P2,463,900
11-11: a
Revenues from:

Initial franchise fee


P1,000,000
Continuing franchise fee (P2,000,000 X .05)
100,000
Total revenue from franchise fees
P1,100,000
11-12: d
Realized gross profit from initial franchise fee [(350,000 + 90,000) x 37%]
P 162,800
Continuing franchise fee (P121,000 + P147,500) x 5%
___13,425
Total revenue
Expenses
___42,900

176,225

Net operating profit


Interest income (P900,000 x 15%) x 6/12
___67,500

133,325

Net income
P 200,825

Franchise Accounting

11-13: c
Cash down-payment

95,000
Present of the note (P40,000 x 3.0374)
__121,496
Total
496
11-14: a
Initial franchise fee
P 50,000
Continuing franchise fee (P400,000 x 5%)
__20,000
Total revenue
P 70,000
11-15: c
Should be P80,000
Initial franchise fee down-payment (P100,000 / 5)
P 20,000
Continuing franchise fee (P500,000 x 12%)
__60,000
Total earned franchise fee
P 80,000
11-16: a
The unearned interest credited is the difference between the face value and the
present value of the notes receivable (900,000 720000).

The down payment of P600,000 is recognized as revenue since it is a fair


measure of the services already performed by the franchisor.
11-17: b
Cora (P100,000 + P500,000)
Dora (P100,000 + P500,000)
Total

P 600,000
600,000
P1,200,000

Down payment (3,125,000 x 40%)


Present value of notes receivable ( 1,875,000/4) 468,750 x 3.04
Adjusted sales value of initial franchise fee
Direct cost of services
Gross profit

P1,250,000
1,425,000
2,675,000
802,500
1,872,500

11-18:

Gross profit rate (1,872,500 2,675,000)

70%

180
Chapter 11

Date
Collection
Interest
Principal
1/1
6/30
468,750
171,000
297,750
12/30
468,750
135,270
333,480
Total collection applying to principal 631,230
Down payment
1,250,000
Total collection
1,881,230
Gross profit rate
70%
Realized gross profit on
initial franchise fee
1,316,861
11-19: c

Balance of PV of NR
P1,425,000
1,127,250
793,770

Franchise Accounting

SOLUTIONS TO PROBLEMS
Problem 11 1
a.

The collectibility of the note is reasonably assured.


Jan. 2:

July 31:

Cash...................................................................................12,000,000
Notes receivable................................................................ 8,000,000
Deferred Revenue from IFF.........................................

20,000,000

Deferred cost of Franchises............................................... 2,000,000


Cash..............................................................................

2,000,000

Nov. 30: Cash/AR...........................................................................


Revenue from continuing franchise fee (CFF)..............

29,000

Dec. 31: Cash / AR.........................................................................


Revenue from CFF.......................................................

36,000

29,000
36,000

Cash.................................................................................. 2,800,000
Notes receivable...........................................................
Interest income (P8,000,000 x 10%).............................

2,000,000
800,000

Adjusting Entries:
(1)
Cost of franchise revenue........................................... 2,000,000
Deferred cost of franchises..................................

2,000,000

(2)

Deferred revenue from IFF........................................20,000,000


Revenue from IFF..................................................
To recognize revenue from the initial franchise fee.

b.

The collectibility of the note is not reasonably assured.

20,000,000

Jan. 2 to Dec. 31 = Refer to assumption a.


Adjusting entry: to recognized revenue from the initial franchise fee (installment method)
(1)

(2)

To defer gross profit:


Deferred Revenue from IFF.......................................20,000,000
Cost of Franchise Revenue..................................
Deferred gross profit Franchises.......................
GPR = P18,000 P20,000,000 = 90%
To recognize gross profit:
Deferred gross profit Franchises.............................12,600,000
Realized gross profit............................................
(P14,000,000 X 90%)

2,000,000
18,000,000

12,600,000

182
Chapter 11

a.

Problem 11 2
Collection of the note is reasonably assured.
Jan. 5: Cash. .................................................................................... 600,000
Notes Receivable.................................................................. 1,000,000
Unearned interest income.................................................
Deferred revenue from F.F................................................
Face value of NR.............................................................................
Present value (P200,000 x P2,9906)................................................
Unearned interest.............................................................................

179,718

Dec. 31: Cash / AR.........................................................................


Revenue from CFF.......................................................
(P80,000 X 5%)

4,000

Cash..................................................................................
Notes Receivable..........................................................

200,000

2) Cost of Franchise..............................................................
Deferred cost of Franchise.........................................

b.

1,000,000
__598,120
401,880

Nov. 25: Deferred cost of Franchise................................................


Cash..............................................................................

Adjusting Entries:
1) Unearned interest income.................................................
Interest income...........................................................
P598,120 x 20%

179,718
4,000

200,000
119,624
119,624
179,718
179,718

3) Deferred revenue from FF................................................ 1,198,120


Revenue from FF.......................................................
Collection of the note is not reasonably assured.
Jan. 5 to Dec. 31 before adjusting entries Refer to Assumption a.
Dec. 31: Adjusting Entries:
1) Unearned interest income.................................................
Interest income..........................................................

401,880
1,198,120

1,198,120

119,624
119,624

2) Cost of franchise...............................................................
Deferred cost of franchise..........................................

179,718
179,718

3) Deferred revenue from FF................................................ 1,198,120


Cost of Franchise.......................................................
Deferred gross profit Franchise...............................
GPR = 1,018,402 1,198,120 = 85%)

179,718
1,018,402

4) Deferred gross profit Franchise.....................................578,319.60


Realized gross profit Franchise...............................
(P600,000 + P200,000- P119,624) x 85%

578,319.60

Franchise Accounting

Problem 11 3
2007
July 1:

Cash. .......................................................................................... 120,000


Notes Receivable......................................................................... 320,000
Unearned interest income.....................................................
Deferred revenue from FF....................................................
Face value of NR......................................................................... P320,000
Present value (P80,000 x 3.1699)................................................ _253,592
Unearned interest income............................................................ P 66,408

Sept. 1 to
Nov. 15: Deferred cost of franchise...........................................................
Cash. ....................................................................................
(P50,000 + P30,000)
Dec. 31: Adjusting Entry:
Unearned interest income............................................................
Interest income.....................................................................
(P253,592 x 10% x 1/2)

80,000
80,000

12,680
12,680

2008
Jan. 10: Deferred cost of franchise...........................................................
Cash. ....................................................................................

50,000

July 1:

80,000

Cash. ..........................................................................................
Note receivable.....................................................................

Dec. 31: Adjusting Entries:


(1) Cost of franchise...................................................................
Deferred cost of franchise.................................................

66,408
373,592

50,000
80,000
130,000
130,000

(2) Deferred revenue from FF....................................................


Revenue from FF..............................................................

373,592

(3) Unearned interest income.....................................................


Interest income.................................................................

25,360

373,592
25,360

184
Chapter 11

Problem 11 4
2008
Jan. 10: Cash. .......................................................................................... 6,000,000
Deferred revenue from FF.....................................................

6,000,000

Jan. 10 to
July 15: Franchise expense....................................................................... 2,250,000
Cash. ....................................................................................

2,250,000

Deferred revenue from FF........................................................... 4,000,000


Revenue from FF..................................................................
Initial Franchise fee....................................................................P6,000,000
Deficiency
Market value of costs (P180,000 90%) x 10 yrs.................( 2,000,000)
Adjusted initial fee (revenue).......................................................P4,000,000
July 15: (a) Continuing expenses.............................................................
Cash / Accounts payable...................................................

a)

b)

4,000,000

180,000

(b) Deferred revenue from FF.................................................... 200,000


Revenue from CFF...........................................................
(P180,000 90%)
Problem 11 5
Adjusted initial franchise fee:
Total initial F.F............................................................................
Less: Face Market value of kitchen equipment...........................
Adjusted initial FF.......................................................................
Revenues:
Initial FF. ....................................................................................
Sale of kitchen equipment...........................................................
Continuing F.F. (P2,000,000 x 2%).............................................
Total. ..........................................................................................
Expenses:
Initial expenses............................................................................ P 500,000
Cost of kitchen equipment........................................................... 1,500,000
Net income........................................................................................

180,000
200,000

P4,500,000
_1,800,000
P2,700,000
P2,700,000
1,800,000
___40,000
4,540,000
_2,000,000
P2,540,000

Journal Entries:
Jan. 2: Cash. .................................................................................... 1,500,000
Notes receivable.................................................................... 3,000,000
Deferred revenue from FF (adjusted SV).........................
Revenue from FF (Market value of equipment)................

2,700,000
1,800,000

Cost of kitchen equipment.................................................... 1,500,000


Kitchen equipment............................................................

1,500,000

Franchise Accounting

Jan. 18: Franchise expense.......................................................................


Cash..................................................................................

500,000
500,000

April 1: Cash .........................................................................................2,000,000


Notes receivable...............................................................

2,000,000

Dec. 31: Cash .........................................................................................1,000,000


Notes receivable...............................................................

1,000,000

Cash / Account receivable...........................................................


Revenue from continuing FF............................................

40,000
40,000

Deferred revenue from FF........................................................... 2,700,000


Revenue from FF..............................................................

2,700,000

Problem 11 6
Recognition of initial franchise fee (IFF) (6 mos. after opening)
Revenue from initial FF:
Total initial FF...................................................................................P2,500,000
Less: Deficiency in continuing FF (Sch. 1)........................................ 160,000
Expense (costs of initial services)..............................................................
Net income................................................................................................
Schedule 1 Estimated deficiency in CFF
(1)
Yr. of
Estimated
Contract
Continuing FF
1
P220,000
2
220,000
3
220,000
4
220,000
5
220,000
6
150,000
7
150,000
8
150,000
9
90,000
10
90,000

(2)
Market Value
of Continuing Services
P250,000
250,000
250,000
125,000
125,000
125,000
125,000
125,000
125,000
125,000

2,340,000
__700,000
P1,640,000

(Excess of 2 over 1)
Deficiency
P 30,000
30,000
30,000

35,000
__35,000
P160,000

Recognition of revenue from CFF and costs:


Years 1-3
Revenue from CFF......................... P250,000
Expenses....................................... _200,000
Net income.................................... P 50,000
186
Chapter 11

Years 4-5
P220,000
_100,000
P120,000

Years 6-8
P150,000
_100,000
P 50,000

Years 9-10
P125,000
_100,000
P 25,000

Problem 11 7
Revenues:
Initial FF (Sch. 1)
Interest income
Continuing FF
Others
Expenses:
Initial expenses
Continuing expense
Others
Net Income

1/12/2008

6/1/2008

7/1/2008

62,500

80,000

287,200

( 50,000) ( 68,000)
P 12,500
P 12,000

( 70,000)

P217,200

6/30/2009
45,490*
48,000

( 36,000)
P 57,490

* P454,900 x 10% = P45,490


Schedule 1: Computation of initial FF to the recognized:
Total initial fee ...........................................................................................................
Less:
Interest unearned on the note.........................................................................
A
Market value of inventory..............................................................................
B
Market value of equipment............................................................................
B
Deficiency in continuing costs.......................................................................
C
Adjusted initial FF.......................................................................................................
A.

B.

Unearned Interest:
Face value of the note...........................................................................................
Present value (120,000 x 3.7908)..........................................................................
rounded
Unearned interest..................................................................................................
Market value of equipment and inventory:
Equipment (P50,000 80%).................................................................................
Inventory..............................................................................................................

P750,000
( 145,100)
( 80,000)
( 62,500
( 175,200)
P287,200
P600,000
454,900
P145,100
P 62,500
80,000

Income from Sales:


Equipment
P62,500
50,000

Inventory
P80,000
68,000

Total
P142,500

P12,500

P12,000

P 24,500

Analysis of Continuing costs:


Market value of costs is P4,000/Mo. or P48,000 / yr.
Continuing Fees:
Years 1-4
Gross revenues ...........................................
P330,000/mo.
Gross fees per month...................................
P 2,475/mo.

Years 5-16
P450,000/mo.
P 3,375/mo.

Years 17-20
P500,000/mo.
P 3,750/mo.

P 40,500
( 48,000)
(
7,500)
x 12
P( 90,000)

P 45,000
( 48,000)
(
3,000)
x4
P( 12,000)

Sales Price. ..................................................


Cost.............................................................
118,000
Net income ..................................................
C.

Gross fees per year......................................


Market value of continuing costs................
Deficiency per year......................................
Number of years..........................................
Deficiency
...........................................
Total deficiency for 20 years is P175,200
Franchise Accounting

P 29,700
( 48,000)
( 18,300)
x4
P( 73,200)

Dates of Revenue Recognition:......................................................


January 12, 2008............................................................
June 1, 2008...................................................................
July 1, 2008....................................................................
June 30, 2009.................................................................

Types of Revenue
Sale of equipment
Sale of inventory
Initial FF (as adjusted0
Interest income and
continuing revenue.

CHAPTER 12
MULTIPLE CHOICE
12-1:

d. This is recorded when the working fund is replenished.

12-2:

c.
Sales
Cost of goods sold:
Purchases
Merchandise inventory, end
Gross profit
Expenses
Net income (loss)

12-3:

P800,000
180,000

620,000
P 80,000
198,000
P (118,000)

b
Sales
Cost of goods sold (P70,000 / 140%)
Gross profit
Less: Samples (P8,000 P6,000)
Expenses
Net income

12-4:

P 700,000

P 70,000
50,000
P 20,000
P 2,000
2,800

4,800
P 15,200

a
Sales
Cost of goods sold
Gross profit
Expenses (P9,000 + P4,500)
Net income

12-5:

12-6:

12-7:

12.8

a
Shipment of merchandise to home office
Equipment sent to home office
Expenses assigned to branch by the home office
Cash remittance to home office

P 100,000
72,000
P 28,000
13,500
P 14,500

P 80,000
50,000
8,000
(40,000)

Home office account balance

P 98,000

12-9: d
12-10: a
Home Office account balance before closing, Dec. 32, 1008
P 35,000
Net income (loss)
Sales
P147,000
Cost of cost goods sold
Shipment to branch
P135,000
Inventory, 12/31
18,500 116,500
Gross profit
P 30,500
Expenses
13,500
17,000
Home Office account balance (Investment in Branch account balance) P 52,000
Shipment to Branch account has no beginning balance, because this was closed at the end
of 2008.
12-11: b
Petty cash fund
Accounts receivable
Inventory
Home Office account balance

Jan. 1, 2008
P 6,000
86,000
74,000
P166,000

Jan. 1, 2009
P 6,000
98,000
82,000
P186,000

12-12: d

Unadjusted balances, Dec. 31


Remittance in transit
Shipment in transit
Cash collections of home office
Adjusted balances, Dec. 31

(Branch Books)
Home Office
P 21,320
7,280
( 400)
P 28,200

(Home Office Books)


Investment in Branch
P 38,600
(10,400)
P 28,200

12-13: a
Unadjusted balance Investment in Branch account, 12/31
Charge for advances by president
Erroneous entry for merchandise allowance
Share in advertising expense
Unadjusted balance Home Office account, 12/31

12-14: a

P430,000
(5,500)
( 600)
(9,000)
P414,900

Unadjusted balances, 12/31


Shipment in transit
Collection of HO A/R by branch
Error in recording of branch profit
Returns of merchandise in transit
Adjusted balances, 12/31

(Branch Books)
(Home Office Books)
Home Office Investment in Branch
P 97,350
P 84,000
6,150
25,000
900
( 6,400)
P103,500
P103,500

12-15: a
(Branch Books)
Home Office
Unadjusted balances
P25,550
Error in recording shipment to Cavity branch
(12,000)
Error in recording shipment to Tagaytay branch 15,000
Branch AR collected by home office
Merchandise returns in transit
( 1,200)
Error in recording branch profit
( 3,600)
Adjusted balances
P23,750

(Home Office Books)


Investment in Branch
P27,350
(3,000)
P23,750

12-16: c
Unadjusted balance- Investment in Branch account
Remittance in transit
Shipment in transit
Expenses allocated
Error in recording remittance
Error in recording shipments
Unadjusted balance Home Office account

Unadjusted balances,
Remittance in transit
Shipment in transit
Expenses allocated
Unrecorded HO collection of AR
Error in recording shipments
Adjusted balances
12.17

P 85,000
(10,000)
(20,000)
( 5,000)
3,000
( 9,000)
P 44,000

( Branch Books)
Home Office
P 44,000

(HomeOffice Books)
Investment in Branch
P 85,000
(10,000)

20,000
5,000
(3,000)
9,000
P 75,000

P 75,000

(Branch Books)
Home Office

(Home Office Books)


Investment in

Branch
Unadjusted balances
Branch AR collected by Home Office
Shipments in transit
Acquisition of furniture
Merchandise returns
Cash remittance in transit
Adjusted balances

P 440,000
( 8,000)
32,000

P 464,000

PROBLEMS
Problem 12-1
Home Office Books

Branch Books

P 496,000
(12,000)
(15,000)
( 5,000)
P 464,000

1. Investment in branch
Cash

30,000

2. Investment in branch
Shipment to branch

75,000

Cash
30,000
75,000

3. No entry
4. No entry

30,000
Home office

30,000

Shipment from home office


Home office

75,000

Purchases
Accounts payable

10,000

75,000
10,000

Accounts receivable
Sales

5. Shipment to branch
Investment in branch

2,000
2,000

6. No entry

125,000
125,000

Home office
2,000
Shipment from home office
Cash

105,000
Accounts receivable

7. No entry

2,000
105,000

Accounts payable

7,000
Cash

7,000
8. No entry

Salaries
Rent
Utilities
Other operating expenses

10,000
5,000
2,000
12,000
Cash

29,500
9. Investment in branch
Accumulated depn

10. Cash
Investment in branch

7,500
7,500

65,000

Depreciation
Rent
Insurance
Home office

1,500
5,000
1,000
7,500

Home office

65,000

65,000

Cash
65,000

11. Cash

3,000
Investment in branch

12. Investment in branch


Branch income

3,000
10,000
10,000

Home office
Accounts receivable
Sales
Inventory, end

3,000
3,000

125,000
5,000
Shipment from

HO

73,000
Purchases

10,000
Salaries

10,000
Rent
Utilities

10,000
2,000
Other operating expenses

12,500
Home office

Problem 12-2
a.

Books of the Branch


1.

Cash

200,000

10,000

2.
3.

Merchandise inventory
Home office

350,000

Merchandise inventory
Accounts payable

400,000

Accounts receivable
Sales

650,000

Cost of goods sold


Merchandise inventory

425,000

Cash

600,000

550,000
400,000
650,000
425,000

Accounts receivable
4.

5.

b.

Advertising expense
Sales commission
Other expense
Cash

40,000
65,000
45,000
150,000

Accounts payable
Home office
Cash

370,000
120,000
490,000

Manila Sales Naga Branch


Income Statement
Year Ended December 31, 2008
Sales
Cost of goods sold
Gross profit
Expenses:
Advertising expense
Sales commissions
Other expenses
Net income

c.

600,000

P650,000
425,000
225,000
P40,000
65,000
45,000

150,000
P 75,000

Manila Sales Naga Branch


Balance Sheet
December 31, 2008
Cash
Accounts receivable
Merchandise inventory
Total assets

P160,000
50,000
325,000
P535,000

Accounts payable
Home office

30,000
505,000

Total liabilities and capital

P535,000

Problem 12-3
Home Office Books

Branch Books
(1) Adjusting Entries

a. Investment in branch
Cash

63,750

b. Investment in branch
Shipment to branch

75,300

Cash

63,750

63,750

Home office

73,300

Shipment from HO
Home office

63,750
75,300
75,300

c. Accounts receivable
Sales

157,500

d. Purchases
Accounts payable

183,750

e. Cash

170,400

157,500

Accounts receivable
Sales

183,750

Purchases
33,750
Accounts payable
33,750
Cash

Accounts receivable

f. Accounts payable
Cash
g. Expenses
Cash

170,400

186,000
186,000
39,900

99,000
99,000

80,100
Accounts receivable

Home office
Cash

80,100

Accounts payable
Cash

18,375

80,100
80,100
18,375

39,900

Furniture & fixtures branch


Investment in branch
h. Cash

12,000
12,000
80,100

Investment in branch

Home office
Cash

12,000

80,100
Expenses
Cash

i. Retained earnings
Cash

12,000

27,000
27,000

15,000
15.000

(2) Adjusting Entries


j. Expenses
Acc. Depreciation

1,750
1,750

k. Investment in branch
Acc. Depn Br. F & F

975

l. Prepaid expenses
Expenses

375

m. Expenses
Accrued expenses

150

975

Expenses
Home office

375

Prepaid expenses
Expenses

150

Expenses
Accrued expenses

975
975
1,125
1,125
450
450

Closing Entries
Home Office Books
n. Sales
Shipments to branch
Merchandise inv., 12/31
Merchandise inv. 1/1
Purchases
Expenses
Income summary

157,500
75,300
72,750

o. Branch loss
Investment in branch

2,100

p. Income summary
Branch loss

2,100

q. Income summary

18,075

Branch Books

60,180
183,750
41,445
20,175
2,100
2,100

Sales
99,000
Merchandise inv., 12/31 35,250
Income summary
2,100
Purchases
33,750
Shipment from HO
75,300
Expenses
27,300
Home office
2,100
Income summary

2,100

Retained earnings

3.

18,075

Individual Financial Statements


Cebu Company Home Office
Income Statement
Year Ended December 31, 2008
Sales
Cost of sales
Merchandise inventory, 1/1
Purchases
Goods available for sale
Shipment to branch
Goods available for own sale
Merchandise inventory, 12/31
Gross profit
Expenses
Net operating income
Branch income (loss)
Net income

P157,500
P 60,180
183,750
P243,930
( 75,300)
P168,630
( 72,750)

95,880
P 61,620
41,445
P 20,175
( 2,100)
P 18,075

Cebu Company Branch


Income Statement
Year Ended December 31, 2008
Sales
Cost of sales
Purchases
Shipments from home office
Goods available for sale
Merchandise inventory, 12/31
Gross profit
Expenses
Net income (loss)

P 99,000
P 33,750
75,300
P109,050
35,250

73,800
P 25,200
27,300
P( 2,100)

Cebu Company Home Office


Balance Sheet
December 31, 2008
Assets
Cash
Accounts receivable
Merchandise inventory, 12/31
Prepaid expenses
Furniture and fixtures
Less: Accumulated depreciation
Branch furniture and fixtures
Less: Accumulated depreciation
Investment in branch
Total assets
Liabilities and Stockholders Equity
Liabilities
Accrued expenses
Accounts payable
Total liabilities
Stockholders Equity
Capital stock

P 34,800
28,575
72,750
3,075
P30,000
8,370

21,630
P12,000

975

11,025
45,825
P217,680

P 2,025
31,950
P 33,975
P 75,000

Retained earnings
Total liabilities and stockholders equity

108,705

183,705
P217,680

Cebu Company Branch


Balance Sheet
December 31, 2008

4.

Assets
Cash
Accounts receivable
Merchandise inventory, 12/31
Prepaid expenses
Total assets

P 6,375
18,000
35,250
1,125
P61,650

Liabilities and Capital


Accounts payable
Home office
Total liabilities and capital

P 450
15,375
P61,650

Combined Financial Statements


Cebu Company
Combined Income Statement
Year Ended December 31, 2008
Sales
Cost of sales
Merchandise inventory, 1/1
Purchases
Goods available for sale
Merchandise inventory, 12/31
Gross profit
Expenses
Combined net income

P256,500
P 60,180
217,500
P277,680
108,000

169,680
P 86,820
68,745
P 18,075

Cebu Company
Balance Sheet
December 31, 2008
Assets
Cash
Accounts receivable
Merchandise inventory
Prepaid expenses
Furniture and fixtures
Less: accumulated depreciation
Total assets
Liabilities and Stockholders Equity
Accrued expenses
Accounts payable

P 41,175
47,475
108,000
4,200
P42,000
9,345

32,655
P233,505
P 2,475
47,325

Capital stock
Retained earnings
Total liabilities and stockholders equity

75,000
108,705
P233,505

Problem 12-4
Branch Books

Home Office Books


(a) and (b) Closing Entries

Sales
145,000
Inventory, 12/31
60,000
Inventory, 1/1
Shipments from HO
Expenses
Income summary
Income summary
Home office

Sales
560,000
Inventory, 12/31
90,000
Shipments to branch
145,000
Inventory, 1/1
Purchases
Expenses
Income summary

18,000
145,000
20,000
23,000

22,000

Investment in branch
Branch income

22,000

45,000
540,000
90,000
120,000

22,000
22,000

Branch income
22,000
Income summary
22,000
Income summary
142,000
Retained earnings
142,000

CG Corporation
Combined Statement Working Paper
Year Ended December 31, 2008
Eliminations
Home
Office

Debits
Cash
Accounts receivable
Inventory, 1/1
Investment in branch
Equipment (net)
Purchases
Shipments from HO
Expenses
Total debits

36,000
54,000
45,000
70,000
95,000
540,000
90,000
930,000

Branch

Debit

Credit

7,000
29,000
18,000

Income
Statement
Dr (Cr)

Balance
Sheet
43,000
83,000

63,000
(2) 70,000
95,000
540,000

145,000
20,000
219,000

(1)145,000
110,000

Inventory 12/31 (BS)


Total assets

150,000
371,000

Credits
Accounts payable
Home Office
Capital stock
Retained earnings, 1/1
Sales
Shipments to branch
Total credits

54,000
144,000
560,000
145,000
930,000

145,000

Inventory, 12/31 (IS)

90,000

60,000

27,000

4,000
70,000

31,000
(2) 70,000
54,000
144,000
(705,000)
(1)145,000

219,000
(150,000)

215,000

215,000

Net income

142,000

Total liabilities & equity


1.
2.

142,000
371,000

To eliminate shipments to branch and shipments from HO


To eliminate reciprocal accounts.

Problem 12-5
(1)

Oro Company
Working Paper for Combined Statements
Year Ended December 31, 2008
Home
Office

Branch

Debits
Cash
Notes receivable
Accounts receivable (net)
Inventories
Furniture & fixtures (net)
Investment in Branch
Cost of goods sold
Operating expenses

63,000
10,500
120,600
143,700
72,150
124,050
300,750
104,250

128,700
32,850

Totals

939,000

275,700

Credits
Accounts payable
Common stock
Retained earnings
Home Office
Sales
Totals

Eliminations
Debit
Credit

Income
Statements
Dr (CR)

21,900
55,950
36,300

84,900
10,500
176,550
45,000
72,150

(2)135,000
(1)124,050
(2)135,000

564,050
137,100
389,100

61,500
300,000
37,500

61,500
300,000
37,500
124,050
151,650

540,000
939,000

275,700

Net Income

(1)124,050
(691,650)
289,050

289,050
9,900

(1) To eliminate shipments


(2) To eliminate reciprocal accounts.

Closing Entries
2.

Branch Books

Sales
Income Summary
Cost of goods sold

Balance
Sheet

3. Home Office Books


151,650
9,900
128,700

(9,900)
389,100

Operating expenses
Home Office
Income summary

32,850
9,900
9,900

Branch loss
9,900
Investment in Branch

9,900

Income summary
Branch loss

9,900

9,900

Problem 12-6
a.

b.

Investment in Branch account (Home Office Books)


Unadjusted balance
Error in recording cash transfer, April 8
Cash transfer recorded in subsequent year, Dec. 31
Error in recording allocated depreciation, Dec. 31
Adjusted balance

P138,200
( 45,000)
( 15,000)
6,000
P 84,200

Home Office account (Branch Books)


Unadjusted balance
Error in recording salary allocation, April 5
Error in recording inventory transfer, July 6
Unrecorded allocated depreciation, Dec. 31
Adjusted balance

P(93,000)
( 200)
12,000
( 3,000)
P(84,200)

Adjusting Entries
Home Office Books

Branch Books

Other income
Investment in branch
Rizal

45,000

Cash

15,000

200
200

45,000

Investment in branchRizal
Investment in branch
Accumulated depn

Salary expense
Home office

Home office
12,000
Shipments from HO

12,000

Depreciation expense
Home office

3,000

15,000
6,000
6,000

3,000

Problem 12-7
a.

b.

Investment in Branch account (Home Office Books)


Unadjusted balance, Dec. 31
Cash remittance in transit
Merchandise returns in transit
Adjusted balance, Dec. 31

P166,400
(30,000)
(12,000)
P124,400

Home Office account (Branch Books)


Unadjusted balance, Dec. 31
Error in recording expense
Shipment in transit
Supplies charged to branch
Collection of branch receivable
Adjusted balance, Dec. 31

P103,200
7,200
24,000
8,000
( 18,000)
P124,400

Adjusting Entries

Home Office Books


Cash
30,000
Shipment to branch
12,000
Investment in branch
42,000

Branch Books
Shipment from HO
24,000
Supplies
8,000
Expenses
7,200
Accounts receivable
18,000
Home office
21,200

Problem 12-8
(1)

Reconciliation Statement
(Home Office Books) (Branch Books)
Investment in Branch
Home Office
Unadjusted balances, 1/31
Advertising charged to branch
Home office AR collected by branch
Shipment in transit
Error in recording receipt of merchandise
Understatement of depreciation
Remittance in transit, 1/31

(2)

P59,720

P 43,268
480

600
( 180)
( 432)
(12,800)
P47,088

P 47,088

Adjusting Entries
Home Office Books
Retained earnings
Cash
Accounts receivable
Investment in branch

Branch Books
432
12,800

Advertising
480
Shipments from HO
3,520
600
Shipment from HO
12,632
Home office

Problem 12-9
(1)

Branch Books
Adjusting Entries
Shipment from home office
Operating expenses (P4,200 + P3,900)
Home office

57,600
8,100
65,700

Closing Entries

(2)

Sales
Inventory, 12/31 (P64,580 + P57,600)
Inventory, 1/1
Shipment from HO (P623,200 + P57,600)
Operating expenses
Income summary

778,200
122,180

Income summary
Home office

116,990

Home Office Books

47,800
680,800
54,790
116,990
116,900

180
3,820

Accounts receivable
Investment in branch
Cash (P20,000 + P19,200)

470
330
800

Investment in branch
Branch income
(3)

116,990
116,900

Reconciliation Statement

Unadjusted balances, 12/31


Error in recording remittance to branch
Shipment in transit
Expenses charged to branch
Branch net income
Freight erroneously charged to branch
Cash remittance in transit to HO
Adjusted balances, 12/31

Home Office Books


Branch Books
(Investment in Branch) (Home Office)
P 206,344 P 140,974
20,000
57,600
8,100
116,990
116,990
( 470)
( 19,200)
P 323,664 P 323,664

Problem 12-1111
a.

P 2,000
Sales (P 27,000 + P 33,000 + P 26,000) .
Cost of Goods Sold (P 36,000 + P 18,000) .
Gross Profit
Rent Expense ..
Property Tax Expense
Depreciation Expense
Miscellaneous Expense .
General Corporate Expense
Net Income

b.

P 4,000
5,000
4,000
11,000
6,000

(30,000)
P 2,000

P 180,000
Initial Transfers .
June Inventory Shipment ..
Property Tax Payment ..
September Inventory Shipment
Expense Allocation ..
Cash Transfer ...
Balance in Home Office/Branch Accounts (correct) ..

c.

P 86,000
(54,000)
P 32,000

P 188,000
18,000
5,000
26,000
6,000
(63,000)
P 180,000

Journal Entries Tarlac Branch


1/10/08

Cash .
Inventory .
Equipment
Home Office

30,000
36,000
122,000
188,000

1/20/08
2/1/08

4/1/08

5/1/08
6/5/08
7/6/08
9/9/08

Rent Expense
Cash .
Cash ..
Sales
Cost of Goods Sold ..
Inventory .
Cash .
Sales ...
Cost of Goods Sold .
Inventory
Miscellaneous Expenses .
Cash ...
Inventory .
Home office ...
Property Tax Expense .
Home Office ..
Inventory
Home Office .

10/1/08

Cash
Sales .
Cost of Goods Sold ..
Inventory ..
11/1/08
Miscellaneous Expenses ...
Cash .
12/22/08 Home Office
Cash .
12/31/08 Depreciation Expense .
Accumulated depreciation ..
12/31/08 General Corporate Expenses
Home Office ..

d.

4,000
4,000
27,000
27,000
18,000
18,000
33,000
33,000
18,000
18,000
7,000
7,000
18,000
18,000
5,000
5,000
26,000
26,000

26,000
26,000
18,000
18,000
4,000
4,000
63,000
63,000
4,000
4,000
6,000
6,000

TARLAC BRANCH
Balance Sheet
December 31, 2008
Assets
Cash .
Inventory .
Equipment ...
Accumulated Depreciation .
Total Assets

P 38,000
26,000
P 122,000
(4,000)

118,000
P 182,000

Equity
Home Office* ..

P 182,000

*Home office balance is P 180,000 as computed in Part b plus the P 2,000 net
income for the period.

CHAPTER 13
MULTIPLE CHOICE
13-1:

13-2:

a
Goods available for sale:
At billed price (P30,000 + P180,000)
At cost (P210,000 / 120%)
Balance of Allowance for Overvaluation account before adjustment

13-3:

c
Inter-company inventory profit (IIP) before closing
Less: IIP from shipment from home office
Billed price
Cost (P300,000 / 120%)
IIP from beginning inventory at billed price
Divided by
Cost of branchs beginning inventory

13-4:

P210,000
175,000
P 35,000

P 66,000
P300,000
250,000

50,000
P 16,000
20%
P 80,000

Billed Price %
Cost
Overvaluation
Beginning inventory from HO
P15,000
150% P10,000
P 5,000
Shipments
110,000
150% 73,333
36,667
Balance before adjustment
P41.667
Ending inventory from HO
5,000
150%
3,333
1,667
Required adjustments
13-5:

b
Shipment to branch, at billed price
Shipping cost
Total cost
Sold (50%)
Inventory

13-6:

P40,000

P375,000
2,000
P377,000
188,500
P188,500

a
Shipment to branch, at cost
Shipping cost

P312,500
2,000

Billed price
Sold (50%)
Inventory, at billed price

13-7:

P314,500
157,250
P157,250

c
Home office account balance after closing branch profit
Less: branch profit
Investment in branch account balance before closing branch profit

13-8:

d
Branch ending inventory, at billed price
Acquired from home office, at billed price:
Cost (P6,000 / 20%)
Mark-up
Purchased from outsiders

13-9:

P765,000
130,000
P635,000

P 50,000
P30,000
6,000

36,000
P 14,000

b
Cost of goods sold Home office
Cost of goods sold Branch:
Billed price
Less: overvaluation (P110,000 P90,000)
Combined cost of goods sold

P590,000
P300,000
20,000

280,000
P870,000

13-10: c
13-11: d
Overvaluation of branch ending inventory acquired from HO:
Billed price
Cost (P28,600 / 130%)
Adjusted balance of allowance for overvaluation account

P 28,600
22,000
P 6,600

13-12: b
Shipment from home office
Expenses
Cash remittance to home office
Home Office account balance before closing

P 90,000
17,000
(70,000)
P 37.000

13-13: b
Shipment to branch, at cost
Ending inventory, at cost (P70,000 / 30%)
Cost of goods sold
Freight (P6,000 x P50,400/P72,000)
Total
13-14: b (20% of P30,000)
13-15: b (P151,200 / 140%)

P 72,000
( 21,600)
P 50,400
4,200
P 54,600

13-16: c
Sales
Cost of goods sold
Shipments from home office (P151,200/140%) P108,000
Inventory, 1/1 (P28,350 / 140%)
20,250
Inventory, 12/31 (P25,200 / 140%)
( 18,000)
Gross profit
Expenses
Branch profit as far as the home office is concerned

P270,000

110,250
P159,750
90,000
P 69,750

13-17: c
Unsold merchandise
Less: Merchandise acquired from home office, at billed price
Merchandise acquired from outsiders
Merchandise acquired from home, at cost (P7,500 / 20%)
Branch inventory at cost, 12/31

P 60,000
45,000
P 15,000
37,500
P 52,500

13-18: a
Branch inventory, 1/1
Acquired from home office at billed price:
Overvaluation [P99,900 (P390,000 P300,000)]
Cost (P9,900 / 30%)
Purchases from outsiders

P 54,600
P 9,900
33,000

42,900
P 11,700

13-19: c
Acquired from home office [(P60,000 x 80%) 120%]
Acquired from outsiders (P60,000 x 20%)
Branch inventory, 12/31 at cost

P 40,000
12,000
P 52,000

13-20: b
Sales (P148,000 + P144,000)
Cost of sales at cost to home office:
Shipment from home office (P108,000 / 120%) P90,000
Purchases
52,000
Inventory, 12/31 (no. 19 above)
(52,000)
Gross profit
Expenses (P76,000 + P24,000)
Branch net income (actual)

P192,000

90,000
P102,000
100,000
P 2,000

13-21: b
Allowance for overvaluation account balance
Overvaluation on the shipment (P200,000 x 25%)
Overvaluation on the branch beginning inventory
Cost of branch beginning inventory (P7,500 / 25%)
Branch beginning inventory at billed price
13-22: b
Sales
Cost of goods sold cost to home office

P 57,500
50,000
P 7,500
30,000
P 37,500
P400,000

Beginning inventory
P 30,000
Shipment from home office
200,000
Ending inventory (P40,000 / 125%)
( 32,000)
Gross profit
Expenses
Branch net income as far as the home office is concerned

198,000
P202,000
100,000
P102,000

13-23: b
Branch inventory, 1/1
Acquired from home- at billed price
Overvaluation [P24,000 (P80,000 P60,000)] P 4,000
At cost [(P4,000 (P20,000 / P60,000)]
12,000
Acquired from outsiders

P 20,000
16,000
P 4,000

13-24: a
Sales
Cost of sales (at cost to home office)
Inventory, 1/1 (P12,000 + P4,000)
P16,000
Shipments from home office
60,000
Purchases
30,000
Inventory, 12/31 [(P20,000133 1/3%) +P6,000] (21,000)
Gross profit
Expenses
Branch net income (actual)

P200,000

85,000
P115,000
60,000
P 55,000

13-25: a
Inventory, 1/1
Shipments from home office
Overvaluation
Cost of goods available for sale

P 75,000
360,000
( 72,500)
P362,500

Percentage of mark-up (P72,500 / P362,500)

20%

13-26: b

13-27: a
Billing percentage above cost (P20,000 / P80,000)
Branch inventory, 6/1 at cost (P12,000 / 125%)
Home office inventory, 6/1
Purchases
Goods available for sale
Inventory, 6/30 at cost:

25%
P 9,600
40,000
160,000
P209,600

Branch (P10,000 / 125%)


Home office
Combined cost of goods sold

P 8,000
60,000

68,000
P141.600

13-28: d
Sales
Cost of goods sold
Gross profit
Expenses
Combined net income

P450,000
141,600
P308,400
150,000
P158,400

13-29: d
Sales
Cost of goods sold:
Inventory, 1/1: Home office
P57,500
Branch (P22,250 / 125%) 17,800 P 75,300
Purchases
410,000
Goods available for sale
P 485,300
Inventory, 12/31: Home office
P71,250
Branch (P29,250/120%) 24,375
95,625
Gross profit
Expenses
Combined net income

P687,500

389,675
P297,825
241,750
P 56,075

13-30: a

Sales
P669,000
Cost of goods sold:
Inventory, 1/1:
Home office
P160,000
Branch [P15,000 + (P49,000 / 122.5%)]
55,000
P215,000
Purchases
460,000
Goods available for sale
P675,000
Inventory, 12/31:
Home office
P110,000
Branch [P11,000 + (P52,000 / 133 1/3%)]
50,000
160,000
515,000
Gross profit
P154,000
Expenses
145,000
Combined net income
P
9,000
13-31: a
The entries made by the branch to record the interbranch transfer of merchandise
are:
Books of Branch 1:
Home office
19,500
Freight in
3,500
Shipment from home office
16,000
Books of Branch 3:
Shipment from home office
16,000
Freight in
4,000

Cash
Home office

2,500
17,500

Therefore the home office would make the following entry:


Investment in Branch 3
17,500
Excess freight
2,000
Investment in Branch 1

19,500

13-32: a
(Home office books)
Investment in branch
77,000

Unadjusted balances
Error in recording shipment
Error in recording expense
Unrecorded cash remittance
Adjusted balances

(Branch books)
Home office
61,000
(10,000)
5,000
46,000

(31,000)
46,000

13-33: c
13-34: a

Home office books

Cebu branch books

Bacolod branch

books
Inv in Bacolod 25,000

Home office 25,000

Cash

25,000
Inv in Cebu

25,000

Cash

25,000

Home office

25,000
Inv in Bacolod 34,300

Home office 34,300

Cash

34,300
Inv in Cebu

34,300

SD

700

Home office

34,300
AR
Inv in Bacolod 62,500
Expenses
62,500
Expenses
150,000
Home office
62,500
Inv in Cebu
212,500

35,000
Home office 212,500
Expenses
Cash

37,500
250,000

Inv in Cebu 253,000


Freight in
3,000
S to branch
200,000 S from HO 250,000
Allowance
50,000 Home office
253,000
Cash
3,000
Inv in Bacolod 252,700
Home office 253,000
Excess freight 300
S from H
253,000
Inv in Cebu
253,000

(Home office books)


Investment in Cebu Branch

(Bacolod branch books)


Home Office

25,000
34,300
212,500
253,000
253,000 524,800
271,800

25,000
34,300
62,500
252,700
374,500

Problem 13-1
(a)

Journal Entries

Home Office Books

Branch

Books
(1) Investment in branch
Cash
(2) Investment in branch
Cash
(3) Investment in branch
Shipment to branch
Allowance for overValuation
(4)

18,000 Equipment
18,000
Home office
3,000
3,000

Rent expense
Home office

3,000
3,000

20,000
Operating expenses
Cash
Cash
Sales

(b)

18,000

100,000 Shipment from HO 100,000


80,000
Home office
100,000

No entry

(5) Cash
Investment in branch

18,000

60,000
Home office
60,000
Cash

11,000
11,000
105,000
105,000
60,000
60,000

Working Paper Elimination Entries

(1)

(2)

Home office
61,000
Investment in branch
To eliminate reciprocal accounts computed
as follows:
Equipment purchased
P 18,000
Rent paid
3,000
Inventory shipped
100,000
Cash transfer
( 60,000)
Balance
P 61,000

61,000

Shipment to branch
80,000
Allowance for overvaluation of branch inventory 20,000
Shipment from home office
100,000
To eliminate inter-company shipments

(3)

(c)

Inventory, 12/31 (Income statement)


Inventory, 12/31 (Balance Sheet)
To reduce inventory, 12/31 to cost.

5,000
5,000

Closing Entries Branch Books


Sales
Inventory, 12/31
Rent expense
Shipment from home office
Operating expenses
Income summary
Income summary
Home office

105,000
25,000
3,000
100,000
11,000
16,000
16,000
16,000

Problem 13-2
a.

Branch Books

Equipment
Shipment from home office
Cash
Home office

50,000
60,000
10,000

Purchases
Cash or accounts payable

30,000

Prepaid rent
Home office

10,000

Cash
Accounts receivable
Sales

40,000
50,000

Advertising expense
Salary expense
Cash

8,000
5,000

Home office
Cash

120,000
30,000
10,000

90,000

13,000
10,000
10,000

Home office
Accounts receivable

3,000

Rent expense
Prepaid rent

5,000

3,000
5,000

Home Office Books


-

Investment in branch
120,000
Equipment
Shipment to branch
Allowance for overvaluation of branch inventory
Cash
To record assets sent to branch
Investment in branch
Cash
To record rent expense of the branch

10,000

Cash

10,000
10,000

Cash

3,000

Investment in branch
To record collection of branch receivable.
b.

20,000
10,000

10,000

Investment in branch
To record cash remittance from branch
-

50,000
40,000

3,000

Income Statement
Sales
Cost of goods sold
Shipment from home office at cost
Purchases
Goods available for sale
Ending inventory:
From home office (1/3)
From outsiders (1/4)
Gross profit
Expenses:
Advertising expense
Salary expense
Rent expense
Net income

P90,000
P40,000
30,000
70,000
P13,333
7,500

(20,833)
P 8,000
5,000
5,000

49,167
P40,833

18,000
P22,833

Problem 13-3
a.

Investment in Branch account beginning balance


Cash transfer
Inventory transfer
Rent allocated
Expenses allocated
Inventory transfer
Transportation allocated
Unadjusted balance Investment in Branch account

P 86,000
( 32,000)
34,500
1,000
3,000
46,000
3,000
P141,500

b.

Home Office account beginning balance


Inventory transfer

P 54,000
34,500

Rent allocated
Expenses allocated
Inventory transfer (error made)
Cash transfer
Home Office account unadjusted balance
c.

1,000
3,000
64,000
( 74,000)
P 82,500

Reconciliation Statement
Unadjusted balances, 1/31
Unrecorded cash transfer
Error in recording transfer (overstated)
Expense allocation not recorded
Adjusted balances, 1/31

Investment in Branch
P141,500
( 74,000)

Home Office
P 82,500
18,000
( 3,000)

P 67,500

P 67,500

Problem 13-4
a.

b.

Books of Branch X
Shipment from home office
Freight-in
Home office

5,000
300

Home office
Shipment from office

5,800
5,800

Books of Branch Y
Shipment from home office
Freight-in
Home office

c.

5,300

5,000
600
5,600

Books of the Home Office


Investment in branch X
Shipment to branch X
Cash

5,300

Investment in branch Y
Inter-branch freight expense
Investment in branch X

5,000
600

Shipment to branch X
Shipment to branch Y

5,000

Malakas Company
Combination Worksheet

5,000
300

5,600
5,000

Year Ended December 31, 2008

Malakas

Davao

Debits
Cash
Accounts receivable
Inventory, 12/31
Investment in branch
Land, bldg, and equipment
Shipment from office
Purchases
Depreciation expense
Advertising expense
Rent expense
Miscellaneous expense
Inventory, 1/1
Total debits

25,000
108,000
209,000
207,000
340,000
348,000
25,000
36,000
12,000
40,000
175,000
1,525,000

18,000
25,000
42,000
112,000
96,000
8,000
15,000
5,000
20,000
35,000
376,000

Credits
Accumulated depreciation
Accounts payable
Notes payable
Home office

80,000
37,000
220,000
-

16,000
15,000
176,000

Common stock
Retained earnings, 1/1
Sales
Shipment to branch
Inventory, 12/31

100,000
240,000
529,000
110,000
209,000

127,000
42,000

Adjustments and
Eliminations
Debit
Credit

(4) 14,000

(5) 16,000
(7)207,000

(3) 14,000

(6)110,000

43,000
133,000
249,000
452,000

(1) 9,000
(1) 6,000
(1) 2,000
(2) 10,000

348,000
33,000
60,000
23,000
62,000
200,000
877,000

(7)207,000

96,000
52,000
220,000
-

(1) 17,000
(3) 14,000

100,000
(2) 10,000

(230,000)
(655,000)

(6)110,000
(5) 16,000

(4) 14,000

Combined net income

(249,000)
(179,000)

Combined retained earnings


Totals

Income Retained
Statement Earnings Balance
Dr (Cr)
Dr (Cr)
Sheet

(179,000)
(409,000)

1,525,000

376,000

388,000

388,000

877,000

Adjustments and Elimination Entries


(1)

(2)

(3)

(4)

(5)

Advertising expense
Rent expense
Miscellaneous expenses
Home office
Unrecorded expenses allocated to the branch

(409,000)

9,000
6,000
2,000
17,000

Retained earnings, 1/1


10,000
Inventory, 1-1
To eliminate unrealized inventory profit of preceding year

10,000

Shipment from home office


Home office
Unrecorded shipments

14,000
14,000

Inventory, 12/31 (debits)


Inventory (credits)
Shipment not yet received by the branch

14,000

Inventory, 12/31 (debits)


Inventory (credits)

16,000

14,000

16,000

To reduce ending inventory to cost


(6)

(7)

Shipment to branch
Shipment from home office
To eliminate inter-company shipments

110,000

Home office
Investment in branch
To eliminate reciprocal accounts

207,000

110,000

207,000

Problem 13-6
a.

Eliminating Entries
(1)
(2)
(3)

(4)
(5)

Home office
Investment in branch Silver

395,000

Home office
Investment in branch Opal

260,000

395,000
260,000

Unrealized intra-company profit Silver


Unrealized intra-company profit Opal
Inventory from home office

20,000
16,000

Inventory
Inventory from home office

90,000

Unrealized intra-company profit Silver


Equipment

40,000

Ginto Company

36,000
90,000
40,000

Balance Sheet Working Paper


December 31, 2008

Cash
Accounts receivable
Inventory
Inventory from home office
Land
Buildings and equipment
Investment in branch Silver
Investment in branch Opal
Total debits
Accumulated depreciation
Accounts payable
Bonds payable
Common stock
Retained earnings
Home office
Unrealized intra-company profit
Silver
Opal
Total credits

b.

Home
Office
81,000
100,000
260,000

Silver
Branch
20,000
40,000
50,000
70,000

Branch
15,000
25,000
44,000
56,000

Opal

70,000
700,000
395,000
260,000
1,866,000

30,000
350,000

20,000
200,000

560,000

360,000

2,055,000

280,000
110,000
400,000
300,000
700,000
-

120,000
45,000

80,000
20,000

480,000
175,000
400,000
300,000
700,000

395,000

260,000

60,000
16,000
1,866,000

560,000

360,000

Eliminations
Credit

Debit
(4) 90,000

Combined
116,000
165,000
444,000

( 3) 36,000
(4) 90,000
(5) 40,000
(1)395,000
(2)260,000

120,000
1,210,000

(1)395,000
(2)260,000
(3) 20,000
(5) 40,000
(3) 16,000
821,000

821,000

2,055,000

Ginto Company
Combined Balance Sheet
December 31, 2008

Assets
Cash
Accounts receivable
Inventory
Land
Buildings and equipment
Less: Accumulated depreciation
730,000
Total assets
Liabilities and Stockholders Equity
Liabilities
Accounts payable
175,000
Bonds payable
Total liabilities
575,000
Stockholders Equity
Common stock
Retained earnings
1,000,000
Total liabilities and stockholders equity
P1,575,000

P 116,000
165,000
444,000
120,000
P1,210,000
480,000
P1,575,000

P
400,000
P
P 300,000
700,000

Problem 13-7
a.

b.

Books of Branch P
Shipment from home office
Freight-in
Home office

8,000
50

Home office
Shipment from home office
Freight-in
Cash

8,120
8,000
50
70

Books of Branch Q
Shipment from home office
Freight-in
Home office

c.

8,050

8,000
80
8,080

Books of Home Office


Investment in branch P
Shipment to branch P
Cash

8,050

Investment in branch Q
Inter-branch freight expense
Investment in branch P

8,080
40

Shipment to branch - P
Shipment to branch Q

8,000

8,000
50

8,120
8,000

Problem 13-8
Debits:
Cash = P36,000 (add the book values and include the P9,000 transfer in transit)
Accounts receivable = P118,000
Inventory, 12/31 = P151,000 (branch balance would be P81,000 when the shipment in transit is
included. This balance must be adjusted to cost of P54,000
(P81,000 150%) and then add to home office balance of P97,000.
Investment in branch = 0 (eliminated)
Land, buildings and equipment = P460,000
Shipment from home office = 0 (eliminated)
Purchases = P429,000
Depreciation expense = P28,000 (add the two book values and the year-end allocation)
Advertising expense = P58,000 (add the two book values and the year-end allocation)
Rent expense = P30,000 (add the two book values and the year-end allocation)
Miscellaneous expense = P100,000 (add the two book values and the year-end allocation)
Inventory, 1/1 = P145,000 (branch balance is adjusted to cost of P24,000 (P36,000 / 150%),
and then added to home office balance.
Total debits = P1,555,000 (add the above totals)
Credits

Accumulated depreciation = P108,000


Accounts payable = P104,000
Notes payable = P180,000
Home office = 0 (eliminated)
Common stock = P60,000 (home office balance)
Retained earnings, 1/1 = P248,000 (home office balance after reduction of P12,000 unrealized
profit in beginning inventory of branch. Cost is P24,000
(P36,000 / 150%) which indicates the P12,000 unrealized.
Sales = P704,000
Shipment to branch = 0 (eliminated)
Inventory, 12/31 = P151,000
Total credits = P1,555,000 (add the above totals)
Reconciliation Statement
Investment in Branch account balance (Home office books)
Unrecorded cash transfer
Adjusted balance

P177,000
( 9,000)
P168,000

Home Office account balance (Branch books)


Inventory transfer in transit
Expense allocated not yet recorded
Adjusted balance

P123,000
21,000
24,000
P168,000

Problem 13-9
Home Office Books

Case A
C
(1)
Investment in 60,000
branch
Shipment to
branch
Unrealized 61,200
inventory profit
(2) Cash
Investment in
branch
Closing entries:
(3) Sales
Inventory, 12/31
Shipment to branch
Purchases
Expenses
Income
summary
(4)
Investment in
branch
Branch income
summary
Branch income
summary
Investment in
branch

Case B

75,000
60,000
-

90,000
60,000
15,000

61,200
61,200

130,000
8,000
60,000

Case
60,000
30,000
61,200

61,200
130,000
8,000
60,000

150,000
17,200
30,800

61,200
130,000
8,000
60,000

150,000
17,200
30,800

150,000
17,200
30,800

13,000
13,000
500

14,000
500

13,500

14,000
27,000

500
13,000
43,800

43,800
43,800

14,000
13,000
43,800

43,800

43,800

Unrealized
inventory profit
Branch income
summary
Income
summary
Income summary
Retained
earnings
Ilocos Branch Books

Case A

Case B

Case

C
(1) Shipment from home 60,000
office
Home office
81,000
(2) Accounts receivable
Sales
64,000
(3) Cash
Accounts
receivable
14,000

75,000
60,000

75,000
81,000

81,000

81,000

64,000
64,000

81,000
64,000

64,000
14,000

61,200

90,000

81,000

14,000
(4) Expenses
Cash

90,000

64,000
14,000

14,000
61,200

61,200

14,000
61,200

61,200

61,200

(5) Home office


Cash
Closing entries

81,000
6,000

(6) Sales
Inventory 12/31
Shipment from
HO
Expenses
13,000
Income
summary

81,000
7,500
60,000
14,000
13,000

81,000
9,000
75,000
14,000

500

14,000

13,000
500

(7) Income summary


Home office
Home office
Income
summary

90,000
14,000

14,000
500

14,000

Working Paper for Combined Financial Statements


December 31, 2008

Eliminations
Home
Office
Income Statement
Sales
Merchandise inventory,
12/31
Shipment to branch
Total credits
Shipment from home
office
Purchases
Expenses
Total debits
Net income(loss) carried
forward
Retained
Earnings
Statement
Net income (loss) from
above
Retained earnings, 12/31 Carried forward

130,000
8,000
60,000
198,000

Branch

Debit

Credit

81,000
9,000 (3)
3,000
(2)
60,000
90,000
90,000

Combine
d
211,000
14,000
225,000

(2)
90,000

150,000
17,200
167,200
30,800

14,000
104,000
(14,000)

150,000
31,200
181,200
43,800

30,800

(14,000)

43,800

30,800

(14,000)

43,800

Balance Sheet
Cash (overdraft)
Accounts receivable
Merchandise inventory,
12/31
Investment in branch
Total debits

39,000
45,000
8,000

(3)
3,000
(1)
28,800

28,800
120,800

Accounts payable
Unrealized
inventory
profit
Capital stock
Retained earnings, from
above
Home office
Total credits

(11,200)
17,000
9,000

14,800

20,000
30,000
40,000
30,800

103,800
20,000
-

(2)
30,000

40,000
43,800

(14,000)

120,800

27,800
62,000
14,000

28,800 (1)
28,800
14,800
121,8
121,80
00
0

103,800

Problem 13-10
(1)

Consolidated Working Paper


Home
Office

Branch A

Branch B

Debits
Cash
Inventories

33,000
70,000

22,000
21,000

13,000
15,000

Other current assets


Investment in Branch A
Investment in Branch B
Cost of sales *

50,000
45,000
42,000
80,000

25,000

23,000

57,000

45,000

Expenses

90,000

25,000

20,000

Adj. & Elim.


(dr) Cr

Income
Statement

Balance
Sheet
68,000

A (12,000)
B 8,000
D 45,000
D 42,000
B (8,000)
C 25,000

110,000
98,000
(165,000)
(135,000)

Credits
Current liabilities
Capital stock
Retained earnings, Jan. 1
Home Office
Allow. for overvaluation of
Branch inv. Branch A
Allow. for overvaluation of
Branch inv. Branch B
Sales

410,000

150,000

116,000

276,000

40,000
100,000
50,000

15,000

11,000

66,000
100,000
50,000

45,000

30,000

13,000
12,000
195,000
410,000

A 12,000
D (87,000)
C (13,000)
C (12,000)

90,000
150,000

75,000
116,000

360,000

Net income

60,000

60,000
276,000

Book value of cost of sales from home office and branches

Home Office
Inventory, January 1,
Purchases
Shipment to branch
Shipment from home office
Goods available for sale
Inventory, Dec. 31
Cost of sales

P 80,000
160,000
( 90,000)
P150,000
( 70,000)
P 80,000

Investment in
Branch A

Investment in
Branch B

P 18,000

P24,000

60,000
P 78,000
( 21,000)
P 57,000

36,000
P 60,000
(15,000)
P 45,000

(2) Reconciliation of Home Office and Investment in Branch accounts.


Books of Home Office

Books of

Books of

Unadjusted balances, Dec.31

Investment
In Branch
A
P 45,000

Investment
In Branch
B
P 42,000

Branch A
Home
Office
P 45,000

Shipments in transit to Branch B


Branch Profit (Schedule 1)
Adjusted balances, December 31

Branch B
Home
Office
P 30,000
12,000

8,000
P 53,000

10,000
P 52,000

8,000

10,000

P 53,000

P 52,000

Schedule 1:
Sales
Cost of sales:
Beginning inventory
Shipment from home office
Goods available for sale
Ending inventory
Cost of sales
Gross profit
Expenses
Net profit

Branch A
P90,000
P18,000
60,000
78,000
21,000
57,000
33,000
25,000
P 8,000

Branch B
P75,000
P24,000
48,000
72,000
27,000
45,000
30,000
20,000
P10,000

CHAPTER 14
MULTIPLE CHOICE
14-1:

a
Purchase price (8,000 shares x P30)
Direct acquisition cost
Contingent consideration
Acquisition cost

14-2:

a
Purchase price
Direct acquisition cost
Acquisition cost
Less: Fair value of net assets acquired
Goodwill

14-3:

14-5:

P250,000
50,000
P300,000
180,000
P120,000

c
Purchase price (100,000 shares x P36)
Direct acquisition cost
Contingent consideration
Acquisition cost

14-4:

P240,000
4,000
5,000
P249,000

P3,600,000
100,000
20,000
P3,720,000

b
Purchase price (600,000 shares x P50)
Direct acquisition cost
Acquisition cost
Less: goodwill recorded
Fair value of net assets acquired

P30,000,000
300,000
P30,300,000
6,120,000
P24,180,000

Capital stock issued (at par)

P30,000,000

Purchase price
Legal fees
Acquisition cost
Less: Fair value of net assets acquired
Current assets
Plant assets
Liabilities
Income from acquisition
14-6:

a (at fair value at date of acquisition)

14-7:

P2,550,000
25,000
P2,575,000
P1,100,000
2,200,000
( 300,000)

3,000,000
P( 425,000)

Abel net income, January to December (P80,000 + P1,320,000) P1,400,000


Cain net income, April to December
400,000
Total net income
P1,800,000
14-8:

a
Acquisition cost
Less: Fair value of net assets acquired
Cash
Inventory
Property, plant and equipment
Liabilities
Income from acquisition

14.9

P 800,000
P 160,000
380,000
1,120,000
( 360,000)

1,300,000
P (500,000)

a
Acquisition cost
Less: Fair value of net assets acquired (P600,000 P188,000)
Goodwill
Avons assets
Bells assets at fair value
Total assets

P 700,000
412,000
P 288,000
2,000,000
600,000
P2,888,000

14-10: b
Debit to Investment in Stock
Brokers fee
Pre-acquisition audit fee
Legal fees for the combination
Total

P 50,000
40,000
32,000
P 122,000

Debit to expenses:
General administrative costs
Other indirect costs
Total

P 15,000
6,000
P 21,000

Debit to APIC
Audit fee for SEC registration of stock issue
SEC registration fee for stock issue
Total

P 46,000
5,000
P 51,000

14-11: d
Acquisition costs:
Cash
Stocks issued at fair value
Contingent liabilities
Total
Less: fair value of net assets acquired:
Cash
Inventories
Other current assets
Plant assets (net)
Current liabilities
Other liabilities
Goodwill

P200,000
330,000
70,000
P600,000
P40,000
100,000
20,000
180,000
(30,000)
(40,000)

Total assets after combination:


Total assets before combination
Cash paid
Registration and issuance costs of shares issued
Polos assets after combination
Assets acquired at fair values
Goodwill
Total assets after combination

270,000
P330,000
P 760,000
(200,000)
( 30,000)
P 530,000
340,000
330,000
P1,200,000

14-12: d
Acquisition cost
Less: Fair value net assets acquired
Goodwill

P1,400,000
1,350,000
P 50,000

14-13: a
Acquisition cost
Less: Fair value of net identifiable assets acquired:
Current assets
P 80,000
Non-current assets
120,000
Liabilities
( 20,000)
Income from acquisition

P160,000

Non- current assets

P120,000

180,000
P(20,000)

14-14: c
Acquisition cost
Less: Fair value of identifiable assets acquired:
Cash
P 60,000
Merchandise inventory
142,500
Plant assets (net)
420,000
Liabilities
(135,000)

P600,000

487,500

Goodwill

P112,500

14-15: b
Acquisition cost
Less: Fair value of identifiable assets acquired
Goodwill
MMs net assets at book value
PPs net assets at fair value
Total assets after combination

P1,000,000
800,000
P 200,000
1,200,000
800,000
P2,200,000

14-16: c, Under the purchase method assets are recorded at their fair values (P225.000)
14-17: d
Capital stock issued at par (10,000 shares x P10)
APIC (10,000 shares x P40)
Total

P100,000
400,000
P500,000

14-18: d, net assets are recorded at their fair values.


14-19: a
Income from acquisition
Fair value of net assets acquired P2,000,000 P400,000)
Acquisition cost

P 100,000
1,600,000
1,500,000

Shares to be issued (P1,500,000 P40)

37,500 shares

14-20: d
Goodwill
Fair value of net assets acquired
Acquisition cost

P 200,000
1,600,000
P1,800,000

Shares to be issued (P1,800,000 P40)

45,000 shares

Total assets of Pablo before acquisition at book value


Total assets acquired from Siso at fair value (100,000 +440,000)
Total assets
Less: cash paid (15,000 + 25,000)
Total assets after cash payment
Goodwill to be recognized (Sched 1)
Total assets after combination

P 700,000
540,000
1,240,000
40,000
1,200,000
195,000
1,395,000

Sched 1: Acquisition cost:


Purchase price (30,000 shares x P20) 600,000
Direct cost
25,000
Contingent consideration
50,000
Fair value of net assets acquired (540,000 60,000)
Goodwill

675,000
480,000
195,000

Stockholders equity before acquisition


Capital stock issued at par (30,000 shares x P10)

650,000
300,000

14-21:

14-22:

APIC (50,000 +300,000) 15,000


Stockholders equity after acquisition

335,000
1,285,000

14-23: a
Acquisition cost
Less: fair value of net assets acquired
Goodwill

B Company
P4,400,000
4,150,000
P 250,000

C Company
P638,000
370,000
P268,000

Total goodwill recorded (250,000 + 268,000)


14-24: a
A Company
B Company
C Company
Cash paid for combination expenses
Goodwill (see 14-23)
Total assets after combination

518,000
5,250,000
6,800,000
900,000
(30,000)
518,000
13,438,000

14-25: a
Stockholders equity before acquisition
Capital stock issued at par (229,000 shares x P10)
Additional paid-in-capital [(229,000 x 12) 10,000]
Indirect cost (reduction from retained earnings)
Stockholders equity after acquisition

P1,300,000
2,290,000
2,738,000
(20,000)
6,308,000

PROBLEMS
Problem 14-1
1.

Books of Big Corporation


Accounts receivable
Inventories
Property, plant and equipment
Current liabilities
Income from acquisition
Cash
To record acquisition of net assets of Small.

120,000
140,000
300,000
50,000
5,000
505,000

Computation of Income from Acquisition:


Acquisition cost (P500,000 + P5,000)
Less: Fair value of net identifiable assets acquired:
Accounts receivable
P120,000
Inventories
140,000
Property, plant and equipment
300,000
Current liabilities
( 50,000)
Income from acquisition
2.

Books of Small Corporation


Cash
Current liabilities

500,000
50,000

P505,000

510,000
P( 5,000)

Accounts receivable
Inventories
Property, plant and equipment
Retained earnings
To record sale of net assets to Big.
Common stock
Retained earnings
Cash
To record liquidation of the corporation.

120,000
100,000
280,000
50,000
200,000
300,000
500,000

Problem 14-2
Cash
Inventory
Building and equipment net
Patent
Accounts payable
Cash
Income from acquisition
To record acquisition of the net assets at fair values.

50,000
150,000
300,000
200,000
30,000
570,000
100,000

Computation of Income from Acquisition


Acquisition cost (P565,000 + P5,000)
Less: Fair value of net identifiable assets acquired
Total assets
P700,000
Accounts payable
( 30,000)
Income from acquisition

P570,000
670,000
P(100,000)

Problem 14-3
Cash and receivables
Inventory
Building and equipment
Goodwill
Accounts payable
Common stock, P10 par value
Additional paid-in capital
Cash
To record acquisition of net assets acquired.

50,000
200,000
300,000
65,000

Computation of Goodwill
Purchase price (6,000 shares x P90)
Direct acquisition cost
Acquisition cost
Less: fair value of net identifiable assets acquired
Total assets
P550,000
Accounts payable
( 50,000)
Goodwill

50,000
60,000
480,000
25,000

P540,000
25,000
P565,000
500,000
P 65,000

Problem 14-4
(1)

Cash
Accounts receivable
Inventory
Land
Building and equipment
Bond discount
Goodwill
Accounts payable
Bonds payable
Common stock, P10 par value
Additional paid-in capital
Cash (P10,000 + P3,000)
To record purchase of net assets of Tan.

60,000
100,000
115,000
70,000
350,000
20,000
108,000
10,000
200,000
120,000
480,000
13,000

Computation of Goodwill
Purchase price (12,000 shares x P50)
Professional fees (P10,000 + P3,000)
Acquisition cost
Less: Fair value of net identifiable assets acquired
Total assets
P695,000
Total liabilities
( 190,000)
Goodwill
(2)

P600,000
13,000
P613,000
505,000
P108,000

Additional paid-in capital


6,000
Cash
To record costs of issuing and registering of shares issued
(P5,000 + P1,000)

(3)

Expenses
Cash
To record indirect acquisition costs.

6,000

9,000
9,000

Problem 14-5
1.
2.
3.
4.
5.
6.
7.

Common stock:: P200,000 + (8,000 shares x P10)


Cash and receivables: P150,000 + P40,000
Land: P100,000 + P85,000
Building and equipment net: P300,000 + P230,000
Goodwill: (8,000 shares x P50) - P355,000
APIC: P20,000 + (8,000 shares x P40)
Retained earnings

P280,000
190,000
185,000
530,000
45,000
340,000
330,000

Problem 14-6
Combined Balance Sheet
After acquisition
Based on P40/share

Based on

P20/share
Cash and receivables
Inventory
Building and equipment
Accumulated depreciation
Goodwill
Total assets

P 350,000
645,000
1,050,000
(200,000)
180,000
P2,025,000

P 350,000
645,000
1,050,000
(200,000)
P1,845,000

Accounts payable
Bonds payable
Common stock P10 Par value
Additional paid-in capital
Retained earnings(including income from acquisition)
Total liabilities and stockholders equity

P 140,000
485,000
450,000
550,000
400,000
P2,025,000

P 140,000
485,000
450,000
250,000
520,000
P1,845,000

Computation of Goodwill Based on P40 per share:


Acquisition cost (15,000 shares x P40)
Less: Fair value of net identifiable assets (P545,000 P125,000)
Goodwill
Computation of Income from Acquisition Based on P20 per share:
Acquisition cost (15,000 shares x P20)
Less: Fair value of net identifiable assets
Income from acquisition (added to retained earnings of Red)

P600,000
420,000
P180,000
P300,000
420,000
P(120,000)

Problem 14-7
(a)

Combined Balance Sheet


January 1, 2008

ASSETS
Cash and receivables
Inventory
Land
Plant and equipment
Less: Accumulated depreciation
Goodwill
Total assets
LIABILITIES AND STOCKHOLDERS EQUITY
Current liabilities
Capital stock, P20 par value
Capital in excess of par
Retained earnings
Total liabilities and stockholders equity
Computation of Goodwill
Acquisition cost

P 110,000
142,000
115,000
P540,000
150,000
390,000
13,000
P 770,000
P 100,000
214,000
216,000
240,000
P 770,000
P210,000

Less: Fair value of net identifiable assets acquired


(P217,000 P20,000)
Goodwill
(b)

197,000
P 13,000

Stockholders Equity section


(1) With 1,100 shares issued
Capital stock: P200,000 + (1,100 shares x P20)
Capital in excess of par: P20,000 + (1,100 x P280)
Retained earnings
Total

P222,000
328,000
240,000
P790,000

(2) With 1,800 shares issued


Capital stock: P200,000 + (1,800 shares x P20)
Capital in excess of par: P20,000 + (1,800 x P280)
Retained earnings
Total

P 236,000
524,000
240,000
P1,000,000

(3) With 3,000 shares issued


Capital stock: P200,000 + (3,000 shares x P20)
Capital in excess of par: P20,000 + (3,000 x P280)
Retained earnings
Total

P260,000
860,000
240,000
P1,360,000

Problem 14-8
Revenue
Net income
Earnings per share
(a)
(b)
(c)
(d)

(e)

2007 (a)
P1,400,000
500,000
P 5.00

2008
P1,800,000 (b)
545,000
P 4.84 (d)

Separate figures for Dollar Transport only.


P2,000,000 P200,000
P620,000 - P55,000
P545,000 / 112,000 shares (100,000 + 125,000) 2
P700,000 / 125,000 shares

Problem 14-9
a.

Books of Peter Industries


Cash
Accounts receivable
Inventory
Long-term investments

28,000
258,000
395,000
175,000

2009
P2,100,000
700,000
P 5.60 (e)

Land
100,000
Rolling stock
63,000
Plant and equipment
2,500,000
Patents
500,000
Special licenses
100,000
Discount on equipment trust notes
5,000
Discount on debentures
50,000
Goodwill
244,700
Allowance for bad debts
Current payables
Mortgage payables
Premium on mortgage payable
Equipment trust notes
Debenture payable
Common stock
APIC common
Cash (direct acquisition cost)
To record acquisition of assets and liabilities at fair values.
Computation of Goodwill
Purchase price (180,000 shares x P14)
Direct acquisition cost
Acquisition cost
Less: fair value of net identifiable assets acquired
Total assets
P4,112,500
Total liabilities
(1,702,200)
Goodwill
Expenses
Cash
To record indirect cost.
b.

6,500
137,200
500,000
20,000
100,000
1,000,000
180,000
2,298,000
135,000

P2,520,000
135,000
P2,655,000
2,410,300
P 244,700

42,000
42,000

Books of HCC:
Common stock
APIC Common
Treasury stock
To record retirement of treasury stock.
P7,500 = P5 x 1,500 shares
P4,500 = P12,000 P7,500
Investment in stock - Peter
Allowance for bad debts
Accumulated depreciation
Current payable
Mortgage payable
Equipment trust notes
Debentures payable
Discount on bonds payable
Cash
Accounts receivable
Inventory
Long-term investments
Land
Rolling stock

7,500
4,500
12,000

2,520,000
6,500
614,000
137,200
500,000
100,000
1,000,000
40,000
28,000
258,000
381,000
150,000
55,000
130,000

Plant and equipment


Patents
Special licenses
Gain on sale of assets and liabilities
To record sale of assets and liabilities to Peter.

2,425,000
125,000
95,800
1,189,900

Common stock
592,500
APIC Common
495,500
APIC Retirement of preferred
22,000
Retained earnings
1,410,000
Investment in stock Peter
To record retirement of HCC stock and distribution of
Peter Industries stock:
P592,500 = P600,000 - P7,500
P495,500 = P500,000 P4,500
P1,410,000 = P220,000 + P1,189,900

2,520,000

Problem 14-10
a.

Increase in capital stock (P240,00 P200,000)


Increase in APIC (P420,000 P60,000)
Value of shares issued

P 40,000
360,000
P 400,000

b.

Total assets after combination


Total assets of Subic before combination
Total fair value of assets of Clark before combination

P1,130,000
650,000
P 480,000

Total liabilities after combination


P220,000
Total liabilities of Subic before combination
(140,000)
Fair value of Clarks net assets (including goodwill)
Less: Goodwill
Fair value of Clarks net assets before combination

( 80,000)
P 400,000
55,000
P 345,000

c.

Par value of common stock after combination


Par value of common stock before combination
Increase in par value
Divided by par value per share
Number of shares issued

P 240,000
200,000
P 40,000
P5
8,000 shares

d.

Value of shares computed in (a)


Number of shares issued computed in
Market price per share

P 400,000
8,000
P
50

Problem 14-11
a.

Inventory reported by Son at date of combination was P70,000


(325,000 P20,000 P55,000 P140,000 P40,000)

b.

Fair value of total assets reported by Son:


Fair value of cash
Fair value of accounts receivable
Fair value of inventory
Buildings and equipment reported following purchase
Buildings and equipment reported by Papa

P 20,000
55,000
110,000
P570,000
(350,000)

220,000

Fair value of Sons total assets


c.

P405,000

Market value of Sons bond:


Book value reported by Son
Bond premium reported following purchase
Market value of bond

d.

P100,000
5,000
P105,000

Shares issued by Papa Corporation:


Par value of stock following acquisition
Par value of stock before acquisition
Increase in par value of shares outstanding
Divide by par value per share
Number of shares issued

e.

f.

P190,000
(120,000)
P 70,000

P5
14,000

Market price per share of stock issued by Papa Corporation


Par value of stock following acquisition
Additional paid-in capital following acquisition

P190,000
262,000

Par value of stock before acquisition


Additional paid-in capital before acquisition
Market value of shares issued in acquisition
Divide by number of shares issued
Market price per share

P120,000
10,000

P452,000
(130,000)
P322,000
14,000
P 23.00

Goodwill reported following the business combination:


Market value of shares issued by Papa
Fair value of Sons assets
Fair value of Sons liabilities:
Accounts payable
P 30,000
Bond payable
105,000
Fair value of liabilities
Fair value of Sons net assets
Goodwill recorded in business combination
Goodwill previously on the books of Papa
Goodwill reported

P322,000
P405,000

(135,000)
(270,000)
P 52,000
30,000
P 82,000

g.

Retained earnings reported by Son at date of combination was P90,000


(P325,000 P30,000 P100,000 P50,000 P55,000)

h.

Papas retained earnings of P120,000 will be reported.

i.

1.

2.

Investment account
Additional paid-in capital
Cash
Goodwill previously computed
Merger costs added to investment account

17,000
9,800
26,800
P82,000
17,000

3.

Total goodwill reported

P99,000

Additional paid-in capital reported following combination


Stock issue costs
Total additional paid-in capital reported

P262,000
(9,800)
P252,200

CHAPTER 15
MULTIPLE CHOICE
15-1:

a
Acquisition cost
Less: Book value of interest acquired (100%)
Difference
Allocation:
Property and equipment
Other assets
Long-term debt
Goodwill

15-2:

P4,000,000
3,200,000
800,000
P(750,000)
150,000
(200,000)

c
Acquisition cost
Less: Book value of interest acquired (P280,000 x 90%)
Difference
Allocation to plant assets (P40,000 x 90%)
Goodwill

15-3:

P 350,000
252,000
98,000
(36,000)
P 62,000

c
Plant assets Pall Company
Plant assets Mall Company
Consolidated

15-4:

( 800,000)
P -0-

P 220,000
180,000
P 400,000

a
Acquisition cost
Less: Book value of interest acquired (P560,000 P70,000)
Difference
Allocation:
Inventory
P 25,000
Property and equipment
( 35,000)
Income from acquisition

P495,000
490,000
5,000
(10,000)
P( 5,000)

15-5:

b
Acquisition cost
Less: Book value of interest acquired (P320,000 x 80%)
Difference
Allocation:
Inventory (P20,000 x 80%)
P(16,000)
Land (P10,000 x 80%)
8,000
Mortgage payable (P5,000 x 80%)
( 4,000)
Goodwill

15-6:

15-7:

15-8:

( 12,000)
P 67,000

a
Inventory (P360,000 + P130,000)

P490,000

Plant and equipment (P500,000 + P420,000)

P920,000

a
Building

P180,000

Land

P 90,000

d
Sons stockholders equity
Minority interest proportionate share
Minority interest in net assets of subsidiary

15-9:

P355,000
256,000
P 79,000

P400,000
20%
P 80,000

d
Acquisition cost
Less: Book value of interest acquired (P145,000 x 75%)
Difference
Allocation to accounts payable (P5,000 x 75%)
Goodwill

P160,000
108,750
51,250
3,750
P 55,000

Therefore:
Total assets (P800,000 + P300,000 + P55,000)
Total liabilities (P250,000 + P155,000 + P160,000 + P5,000)

P1,155,000
570,000

15-10: b (P900,000 x 1%)

15-11: a

Controlling (Parent) interest:


Shares acquired (P120,000 / P120)
Divided shares outstanding (P125,000 /P100)
Parents interest

1,000 shares
1,250
80%

Minority interest in net assets of subsidiary (P200,000 x 20%) P40,000


15-12: a
Goodwill
Book value of interest acquired (P100,000 / 20%) x 80%
Investment cost

P250,000
400,000
P650,000

15-13: b
Net assets on the date of acquisition (P247,095 + P43,605)
Adjustments of assets excluding goodwill:
Inventories
P6,630
Plant and equipment
48,450
Patent
7,650
Net assets at fair value

P290,700

62,730
P353,430

15-14: d (P500,000 + P300,000)


15-15: b
Acquisition cost
Less: Book value of interest acquired (P250,000 x 80%)
Difference
Allocated to plant and equipment (P50,000 x 80%)
Goodwill

P260,000
200,000
60,000
(40,000)
P 20,000

15-16: a (The retained earnings of the parent only).


15-17: a (The stockholders equity of the parent only).
15-18: b (P50,000 + P10,000)
15-19: d (P380,000 + P150,000)

15-20: d
Cash and cash equivalent (P70,000 + P90,000)
Inventory (P100,000 + P60,000)
Property and equipment (P500,000 + P300,000)
Goodwill
Total assets

P 160,000
160,000
800,000
20,000
P1,140,000

15-21: d
Fair value of the reporting unit
Fair value of net assets (excluding goodwill)
Implied goodwill
Carrying value of goodwill (P450,000 P390,000)
Impairment loss

P 485,000
440,000
45,000
60,000
P 15,000

15-22: b
Fair value of the reporting unit
Fair value of the net assets (P590,000 P100,000)
Implied goodwill to be recorded
Carrying value of goodwill
Impairment loss

P 540,000
490,000
50,000
150,000
P 100,000

15-23: a

The amount reported is equal to Primos retained earnings of P567,000

15-24: a

100% [P138, 000 (P320, 000 P140, 000)]

15-25: a

(340,000- 200,000)

15-26: b
Cash
Accounts receivable
Inventories (see 15-25)
Equipment (800,000 - 500,000)
Accounts payable
Fair value of net assets

40,000
20,000
140,000
300,000
(40,000)
460,000

15-27: a
Net asset acquired (320,000 x 70%)
Differential allocated to inventory
Differential allocated to equipment
Differential allocation to goodwill
Minority interest (140,000 x30%)
Amount paid by Parent

224,000
40,000
100,000
10,000
(42,000)
332,000

PROBLEMS
Problem 15-1
a.

b.

Investment in Solo Company stock


Cash
To record acquisition of 90% (90,000 / 100,000)
of the outstanding shares of Solo.

1,080,000
1,080,000

Working paper elimination entries:


(1)

Common stock Solo


Retained earnings Solo
Investment in Solo company stock

400,000
500,000
810,000

Minority interest in net assets of subsidiary


To eliminate Solos equity accounts at date of acquisition.
(2)

90,000

Inventories
30,000
Plant assets
60,000
Goodwill
189,000
Investment in Solo company stock
Minority interest in net assets of subsidiary
To allocate difference
Computation and allocation of difference:
Acquisition cost
Less: Book value of interest acquired
Common stock (P400,000 x90%)
Retained earnings (P500,000 x 90%)
Difference
Allocation:
Inventories
Plant assets
Total
Minority interest (P90,000 x10%)
Goodwill

270,000
9,000

P1,080,000
P360,000
450,000
(30,000)
(60,000)
(90,000)
9,000

810,000
P 270,000

( 81,000)
P 189,000

Problem 15-2
a.

b.

c.

Investment in Straw stock


Cash
To record acquisition of 100% of Straw stock.
Acquisition cost
Less: Book value of interest acquired (100%)
Difference
Allocation (100%:
Inventories
Land
Building
Equipment
Patents
Goodwill

600,000
600,000
P600,000
420,000
180,000
P( 40,000)
( 80,000)
150,000
( 20,000)
( 20,000)

Working paper elimination entries:


(1)

Common stock Straw

100,000

( 10,000)
P170,000

(2)

Retained earnings Straw


Investment in Straw stock
To eliminate equity accounts of Straw at
date of acquisition.

320,000

Inventories
Land
Equipment
Patents
Goodwill
Buildings
Investment in Straw stock
To allocate difference.

40,000
80,000
20,000
20,000
170,000

420,000

150,000
180,000

Problem 15-3
a.

b.

c.

Investment in Soto stock


Cash
To record acquisition of 90% stock of Sotto.

950,000

Acquisition cost
Less: Book value of interest acquired (P900,000 x 90%)
Difference
Allocation:
Current assets
P 50,000
Property and equipment
(100,000)
Long-term debt
( 40,000)
Total
P( 90,000)
Minority interest (10% thereof)
9,000
Goodwill

950,000
P950,000
810,000
140,000

(81,000)
P 59,000

Working paper elimination entries:


(1)

Common stock Sotto


100,000
APIC Sotto
200,000
Retained earnings Sotto
600,000
Investment in Sotto stock
Minority interest in net assets of subsidiary
To eliminate equity accounts of Sotto at date of

810,000
90,000

acquisition.
(2)

Property, plant and equipment


100,000
Goodwill
59,000
Long-term debt
40,000
Current assets
Investment in Sotto stock
Minority interest in net assets of subsidiary
To allocate difference.

45,000
140,000
14,000

Problem 15-4
Paco Company and Subsidiary
Consolidated Balance Sheet
January 2, 2008
Current assets
Property, plant and equipment
Other assets
Total assets

P475,000
285,000
70,000
P830,000

Current liabilities
Mortgage payable
Common stock
Additional paid-in capital
Retained earnings (including income from subsidiary of P20,000)
Total liabilities and stockholders equity

P280,000
85,000
200,000
65,000
200,000
P830,000

Computation of income from acquisition:


Investment cost (20,000 shares x P6)
Less: Book value of interest acquired
Common stock
Retained earnings
Difference
Allocated to property and equipment
Income from acquisition
Problem 15-5

P120,000
P35,000
80,000

115,000
P 5,000
(25,000)
P(20,000)

Under the purchase method, the investment cost is equal to the fair value of stock issued by Palo
(P250,000) plus direct acquisition cost (P10,000) or a total of P260,000. The P20,000 stock issue
cost is treated as a reduction from the additional paid-in capital. The entry to record the
acquisition of stock is as follows:
Investment in Solo stock
Common stock, at par
Additional paid-in capital
Cash (direct acquisition cost)

260,000
100,000
150,000
10,000

Additional paid-in capital


Cash

20,000
20,000

Palo Company and Subsidiary


Consolidated Balance Sheet
December 31, 2008
Cash
Receivables
Inventory
Property and equipment net
Goodwill
Total assets

P 70,000
120,000
170,000
340,000
30,000
P730,000

Current liabilities
Long-term liabilities
Common stock
Additional paid-in capital
Retained earnings, 12/31
Total liabilities and stockholders equity

P 30,000
120,000
210,000
150,000
220,000
P730,000

Computation of goodwill:
Acquisition cost
Less: Book value of interest acquired (P90,000 + P100,000)
Difference
Allocated to equipment
Goodwill

P260,000
190,000
70,000
(40,000)
P 30,000

Problem 15-6
a.
Investment in Seed Company stock
Cash
To record acquisition of 100% of Seed company stock.
Allocation schedule:
Acquisition cost
Less: Book value of interest acquired
Difference
Allocation:
Inventory
Plant assets
Long-term liabilities

350,000
350,000
P350,000
320,000
30,000

P(20,000)
(80,000)
40,000

(60,000)

Income from acquisition

b.

P930,000)

Working paper elimination entries


(1)
Common stock Seed
Additional paid-in capital Seed
Retained earnings Seed
Investment in Seed stock
To eliminate equity accounts of Seed at
date of acquisition.
(2)

100,000
40,000
180,000
320,000

Inventory
20,000
Plant assets
80,000
Long-term debt
Investment in Seed stock
Retained earnings Pill (income from acquisition)
To allocate difference.

40,000
30,000
30,000

Pill Corporation and Subsidiary


Consolidated Working Paper
May 31, 2008 Date of Acquisition
Pill
Corporation

Seed
Company

Assets
Cash
Accounts receivable
Inventories
Investment in Seed company

200,000
700,000
1,400,000
350,000

10,000
60,000
120,000

Plant assets
Total

2,850,000
5,500,000

610,000
800,000

500,000
1,000,000

80,000
400,000

Liabilities & Stockholders


Equity
Current liabilities
Long-term debt
Common stock:
Pill
Seed
Additional paid-in capital
Pill
Seed
Retained earnings
Pill
Seed
Total

Eliminations

& adjustment

Debit

Credit

(2) 20,000
(1)320,000
(2) 30,000
(2) 80,000

210,000
760,000
1,540,000
3,540,000
6,050,000

(2) 40,000

1,500,000

580,000
1,440,000
1,500,000

100,000

(1)100,000

40,000

(1) 40,000

180,000
800,000

(1)180,000
420,000

1,200,000

1,200,000

1,300,000
5,500,000

Consolidated

(2) 30,000

1,330,000

420,000

6,050,000

Problem 15-7
a.
b.

Accounts Receivable
Cash
Investment in Sea Company stock
Common stock ((30,000 shares x P20)
Investment in Sea Company stock
Common stock
Current liabilities

70,000
70,000
600,000
600,000
40,000
30,000
70,000

Pop Corporation and Subsidiary


Working Paper for Consolidated Balance Sheet
April 30, 2008 Date of acquisition
Pop
Corporation
Assets
Cash
Accounts receivable net
Inventories
Investment in Sea Company
Plant assets
Goodwill
Total
Liabilities & Stockholders
Equity
Current liabilities
Long-term debt
Common stock
Pop
Sea
Additional paid-in capital
Retained earnings
Pop
Sea
Minority interest in net assets
Of subsidiary
Total

Sea
Company

Adjustments

& Eliminatio

Debit

Credit

50,000
230,000
400,000
640,000

80,000
270,000
350,000

1,300,000

560,000

2,620,000

1,260,000

380,000
800,000

250,000
600,000

(3) 70,000

100,000
360,000

(1)100,000
(1)360,000

(3) 70,000
(2) 90,000
(1)328,000
(2)312,000
(2)220,000
(2) 80,000

(2) 20,000

560,000
1,420,000
1,070,000

370,000

370,000
(50,000)

(1) 50,000

1,260,000

(1) 82,000
(2) 58,000
920,000

920,000

(1) To eliminate equity accounts of Sea Company on the date of acquisition.


(2) To allocate difference, computed as follows:

(3)

130,000
430,000
840,000
2,080,000
80,000
3,560,000

1,070,000

2,620,000

Consolidated

Acquisition cost
Less: Book value of interest acquired (P410,000 x 80%)
Difference
Allocation:
Inventories
P( 90,000)
Plant assets
(220,000)
Long-term debt
20,000
Total
P(290,000)
Minority interest (20%
58,000
Goodwill
To eliminate intercompany receivables and payables.

P640,000
328,000
312,000

232,000
P 80,000

140,000
3,560,000

Problem 15-8
1. Acquisition cost
Less: Book value of interest acquired
Common stock
APIC
Retained earnings
Difference
Allocation:
Inventory
Land
Building
Equipment
Bonds payable

P500,000
P100,000
200,000
230,000
P( 20,000)
( 10,000)
50,000
60,000
( 50,000)

530,000
( 30,000)

30,000

2. P Company and Subsidiary


Consolidated Working Paper
January 1, 2008 Date of acquisition
P
Company
Debits
Cash
Accounts receivable
Inventory
Land
Building
Equipment
Investment in S Company
Total
Credits
Accounts payable
Bonds payable
Common stock P Company
Common stock S Company
APIC S Company
Retained earnings P Co.
Retained earnings S Co.
Total

S
Company

& Eliminations

Debit

Credit

300,000
200,000
200,000
100,000
600,000
800,000
500,000
2,700,000

50,000
100,000
80,000
50,000
400,000
200,000

150,000

60,000
290,000

(2) 50,000

100,000
200,000

(1)100,000
(1)200,000

230,000
880,000

(1)230,000
640,000

(2) 20,000
(2) 10,000
(2) 30,000

(2) 50,000
(2) 60,000
(1)530,000

880,000

Consolidated
350,000
300,000
300,000
160,000
950,000
940,000
3,000,000
210,000
240,000
1,500,000

1,500,000
1,050,000
2,700,000

(1) To eliminate equity accounts of S Company.


(2) To allocate difference.

Problem 15-9

Adjustments

640,000

1,050,000
3,000,000

1.

2.

Acquisition cost
Less: Book value of interest acquired
Common stock (P100,000 x 80%)
APIC (P200,000 x 80%)
Retained earnings (P230,000 x 80%)
Difference
Allocation
Inventory
Land
Building
Equipment
Bonds payable
Total
Minority interest (20%)
Goodwill

P500,000
P 80,000
160,000
184,000
P (20,000)
(10,000)
50,000
60,000
(50,000)
P 30,000
( 6,000)

424,000
P 76,000

24,000
P100,000

P Company and Subsidiary


Consolidated Working Paper
January 2, 2008 Date of acquisition

Debits
Cash
Accounts receivable
Inventory
Land
Building
Equipment
Investment in S Company
Goodwill
Total
Credits
Accounts payable
Bonds payable
Common stock P Co.
Common stock S Co.
APIC S Co.
Retained earnings P Co.
Retained earnings S Co.
Minority interest in net
Assets of subsidiary
Total

Adjustments

& Eliminations

Debit

Credit

P
Company

S
Company

300,000
200,000
200,000
100,000
600,000
800,000
500,000

50,000
100,000
80,000
50,000
400,000
200,000

2,700,000

880,000

150,000

60,000
290,000

(2) 50,000

100,000
200,000

(1)100,000
(1)200,000

230,000

(1)230,000

880,000

(2) 6,000
716,000

(2) 20,000
(2) 10,000
(2) 50,000
(2) 60,000
(1)424,000
(2) 76,000
(2)100,000

210,000
240,000
1,500,000

1,050,000

1,050,000
(1)106,000
716,000

(1) To eliminate equity accounts of S Company


(2) To allocate difference

Problem 15-10
1.

350,000
300,000
300,000
160,000
950,000
940,000
100,000
3,100,000

1,500,000

2,700,000

Consolidated

Acquisition cost
Less: Book value of interest acquired (100%)
Difference

P542,000
670,000
(128,000)

100,000
3,100,000

Allocation
Inventory
Land
Equipment
Long-term investment in MS
Income from acquisition
2.

P (10,000)
(40,000)
20,000
(15,000)

( 45,000)
P(173,000)

P Company and Subsidiary


Consolidated Working Paper
January 2, 2008 Date of acquisition

Assets
Cash
Accounts receivable
Inventory
Land
Equipment
Investment in S Company
Long-term investment in MS
Total
Liabilities & Stockholders
Equity
Accounts payable
Common Stock P Co.
Common Stock S Co.
APIC P Co.
Retained earnings P Co.
Retained earnings S Co.
Total

P
Company

S
Company

100,000
200,000
150,000
50,000
300,000
542,000
100,000
1,442,000

100,000
150,000
130,000
80,000
200,000

175,000
400,000

115,000

125,000
785,000

200,000

Adjustments

& Eliminations

Debit

Credit

(2) 10,000
(2) 40,000
(2)128,000
(2) 15,000

470,000
785,000

200,000
350,000
290,000
170,000
480,000
240,000
1,730,000

290,000
400,000
(1)200,000

200,000
667,000
1,442,000

(2) 20,000
(1)670,000

Consolidated

(1)470,000
863,000

(2)173,000

200,000
840,000

863,000

1,730,000

(1) To eliminate equity accounts of S Company.


(2) To allocate difference

CHAPTER 16
MULTIPLE CHOICE
16-1:

d, because no impairment of goodwill is recognized.

16-2:

d, consolidated net income will decrease due to amortization of the allocated difference
which is not the goodwill (P60,000 / 10 years).

16-3:

d, computed as follows:
Subsidiarys net income
Amortization of the allocated difference
Minority interest in net income of subsidiary

16-4:

P150,000
( 20,000)
P130,000

c
Acquisition cost (P500,000 + P40,000)
Less: Book value of interest acquired

P540,000
480,000

16-5:

Difference

P 60,000

Cost Method
Acquisition cost
P540,000
Parents share of subsidiarys net income
Dividends received from subsidiary
Amortization of allocated difference (P60,000/20) Investment account balance, Dec. 31, 2008
P540,000

Equity Method
P540,000
120,000
( 48,000)
( 3,000)
P609,000

a
Net assets of Sol, January 2, 2008
Increase in earnings:
Net income
Dividends paid (P60,000 / 75%)
Net assets of Sol, Dec. 31, 2008

P300,000
P160,000
80,000

80,000
P380,000

Minority interest in net assets of subsidiary (P380,000 x 25%) P 95,000


16-6:

a
Punos net income
Dividend income (P40,000 x 90%)
Salas net income
Consolidated net income

16-7:

P145,000
(36,000)
120,000
P229,000

d
Peters net income from own operation
Peters share of Sellers net income
MINIS (P200,000 x 25%)
Consolidated net income attributable to parent

16-8:

Investment in Son, Jan. 1


Pops share of Sons net income (100%)
Dividends received (100%)
Amortization of allocated difference to
Equipment (P38,000 / 10)
Investment in Son, Dec. 31
P648,600
16-9:

P1,000,000
200,000
( 50,000)
P1,150,000

2006
P310,000
150,000
( 60,000)

2007
P396,200
180,000
(60,000)

( 3,800)
( 3,800)
P396,200

a
Sys net income
Amortization of allocated difference
Adjusted net income of Sy

P300,000
( 60,000)
P240,000

Minority interest in net income of subsidiary (P240,000 x 10%) P 24,000

2008
P512,400
200,000
( 60,000)
( 3,800)
P512,400

16-10: a. Under the equity method consolidated retained earnings is equal to the retained
earnings of the parent company.
16-11: c
Retained earnings, Jan. 2, 2008 Puzon
Consolidated net income attributable to parent:
Net income Puzon
P200,000
Net income Suarez
40,000
Dividend income (P20,000 x 80%)
(16,000)
MINIS (P40,000 x 20%)
( 8,000)

P500,000

Dividends paid Puzon


Consolidated retained earnings, Dec. 31, 2008

( 50,000)
P666,000

216,000

16-12: c
Acquisition cost
Less: Book value of interest acquired
Difference
Allocation due to undervaluation of net assets
Goodwill ( not impaired)

P1,700,000
1,260,000
P 440,000
( 40,000)
P 400,000

16-13: d
Net assets of Suazon, Jan. 2, 2008
Increase in earnings (P190,000 P125,000)
Net assets of Suazon, Dec. 31, 2008
Unamortized difference to plant assets (P100,000 P10,000)
Adjusted net assets of Suazon, Dec. 31, 2008

P1,000,000
65,000
P1,065,000
90,000
P1,175,000

Minority interest in net assets of subsidiary (1,175,000 x 20%) P 231,000


16-14: b
Prestos net income from own operations
Prestos share of Storks net income (P80,000 P23,000)
MINIS (P57,000 x 10%)
Consolidated net income attributable to parent

P140,000
57,000
( 5,700)
P191,300

16-15: b
Investment in Siso stock (at acquisition cost)

P600,000

Dividend income (P30,000 x 5%)

P 1,500

16-16 d
Consolidated net income:
Pepes net income from own operations
Sisons adjusted net income:

P210,000

Net income -2008


Amortization of allocated difference
to equipment (P20,000 / 5)
Consolidated net income

P67,000
4,000

63,000
P273,000

Consolidated retained earnings:


Pepes retained earnings, Jan.2, 2007
Consolidated net income attributable to parent 2007
Pepes NI from own operations
P185,000
Sisons adjusted NI;
Net income 2007
P40,000
Amortization -2007
4,000 36,000
MINIS (P36,000 x 30%)
(10,800)
Dividends paid ,2007 - Pepe
Pepes retained earnings, Jan. 2, 2008
Consolidated net income attributable to parent 2008:
Consolidated net income (see above)
P273,000
MINIS (P63,000 x 30%)
( 18,900)
Dividends paid, 2008 Pepe
Consolidated retained earnings, Dec. 31, 2008

P701,000

210,200
( 50,000)
P861,200
254,100
( 60,000)
P1,055,300

16-17: b
Acquisition cost
Less: Book value of interest acquired
Allocated to building
Consolidated retained earnings
Retained earnings, Jan. 1, 2008 Pepe
Consolidated net income attributable to parent:
Net income Precy
Adjusted net income of Susy:
Net income of Susy
P100,000
Amortization (P70,000 / 10) 2
( 3,500)
MINIS (P96,500 x 30%)
Dividends paid Precy
Consolidated retained earnings, Dec. 31, 2008

P700,000
630,000
P 70,000
P550,000
P275,000
96,500
(28,950)

342,550
( 70,000)
P822,550

Minority interest in net assets of subsidiary


Stockholders equity of Susy, June 30, 2008
Increase in earnings- net income (7/1 to 12/31)
Stockholders equity, Dec. 31, 2008
Unamortized difference (P70,000 P3,500)
Adjusted net assets of Susy, Dec. 31, 2008

P 900,000
100,000
P1,000,000
66,500
P1,066,500

Minority interest in net assets of subsidiary (P1,066,500 x 30%)

P 319,950

16-18: a
Goodwill
Acquisition cost
Less: Book value of interest acquired (P1,320,000 P320,000)

P1,200,000
1,000,000

Goodwill (not impaired)

P 200,000

Consolidated retained earnings under the equity method is equal to the retained
earnings of the parent company, P1,240,000.
16-19: b
Net income Pablo
Dividend income (P40,000 x 70%)
Sitos net income
MINIS (P70,000 x 30%)
Consolidated net income attributable to parent

P130,000
(28,000)
70,000
(21,000)
P151,000

16-20: c
Consolidated net income 2008
Net income Ponce
Dividend income (P15,000 x 60%)
Solis net income
MINIS (P40,000 x 40%)
Consolidated net income attributable to parent 2008
Consolidated retained earnings 2008
Retained earnings, Jan. 2, 2007- Ponce
Consolidated net income attributable to parent 2007:
Net income Ponce
Dividend income (P30,000 x 60%)
Solis net income
MINIS (P35,000 x 40%)
Dividends paid, 2007 Ponce
Consolidated retained earnings, Dec. 31, 2007
Consolidated net income attributable to parent 2008
Dividends paid. 2008 Ponce
(30,000)
Consolidated retained earnings, Dec. 31, 2008

P 90,000
(9,000)
40,000
(16,000)
P105,000
P 400,000
P70,000
(18,000)
35,000
( 14,000)

75,000
(25,000)
P450,000
105,000
P525,000

16-21 a
Acquisition cost
Less: Book value of interest acquired (220,000 x 80%)
Difference
Allocated to:
Depreciable assets (30,000 80%)
(37,500)
Minority interest ( 37,500 x 20%)
7,500
Goodwill

P216,000
176,000
40,000

Polo net income from own corporation


Seed net income from own operation:
Net income
Amortization (37,500 10%)
Total
Goodwill impairment lost

P 95,000
35,000
(3,750)

(30,000) = 80%
10,000

31,250
126,250
(8,000)

Consolidated net income


16-22: a
Retained earnings 1/1/08 Polo
Consolidated net income attributed to parent:
Consolidated net income
MINI (35,000 3,750) x 20%
Total
Dividends paid- Polo
Consolidated retained earnings 12/31/08
16-23: a

118,250
P520,000
118,250
6,250

(35,000 3750) x 20%

16-24: a
Seed stockholders equity, January 2, 2008 (80,000 + 140,000)
Undistributed earnings 2008 (35,000 15,000)
Unamortized difference (37,500 - 3750)
Seed stockholders equity (net asset), December 31, 2008
MINAS (273,750 20%)
16-25: a

112,000
632,000
(46,000)
586,000

220,000
20,000
33,750
273,750
54,750

(see no. 16-22)

16-26: a
Acquisition cost
Less: Book value of interest acquired (280,000 x 70%)
Difference
Allocation:
to depreciable assets
(50,000)
MINAS (30%)
15,000
Retained earnings, 1/1/08-Sisa company
Retained earnings, 1/1/07-Sisa company (squeeze)
Increase
Amortization- prior years (50,000 10 years)
Adjusted increase in earnings of Sisa (21,000/30% )
16-27: a
Retained earnings 1/1/08- Pepe
Retained earnings 1/1/08- Sisa
230,000
Adjustment and elimination:
Date of acquisition
(155,000)
Undistributed earnings to MINAS
(21,000)
Amortization- prior year
(5,000)
Consolidated retained earnings 1/1/08
16-28: a
Pepe company net income
Sisa company net income
Dividend income (10,000 x 70%)
Amortization- 2008
Consolidated net income
16-29: a
Consolidated retained earnings 1/1/08(see 16 27)
Consolidated net income attributable to parent:

231,000
196,000
35,000
35,000
230,000
155,000
75,000
(5,000)
70,000
520,000

49,000
569,000
120,000
25,000
(7,000)
(5,000)
133,000
569,000

Consolidated net income (see 16-28)


MINIS (25,000 5,000) 30%
Dividend paid- Pepe company
Consolidated retained earnings 12/31/08

133,000
(6,000)

127,000
(50,000)
646,000

PROBLEMS
Problem 16-1
a.

Since Pasig paid more than the P240,000 fair value of Sibols net assets, all allocations
are based on fair value with the excess of P10,000 assigned to goodwill. The
amortizations of the allocated difference are as follows:
Annual
Allocated to
Allocation
Life
Amortization
Building
Equipment

P 50,000
(20,000)

10 years
5 years

P 5,000
(4,000)

Building:
Allocation, Jan. 1, 2006
Amortization during past years -2006 to 2007 (P5,000 x 2)
Amortization for the current year 2008
Allocation, Dec. 31, 2008

P 50,000
(10,000)
( 5,000)
P 35,000

Equipment
Allocation, Jan. 1, 2006
Amortization during past years 2006 to 2007 (P4,000 x 2)
Amortization for the current year 2008
Allocation, Dec. 31, 2008

P(20,000)
8,000
4,000
P( 8,000)

b.

Since Pasig paid P20,000 less than the P240,000 fair value of Sibols net assets, a
negative difference arises. Under PFRS 3 (Business combination), the allocation of the
negative difference to the non-current assets, excluding long-term investments in
marketable securities is no longer permitted. The negative difference is immediately
amortized in profit or loss (income from acquisition). Therefore, the allocation assigned
to building and equipment is the same as in (a) above.

c.

Same as in (a) above. Except that the negative goodwill amortized to income is P60,000.

d.

Neither allocations nor amortization are found in a pooling of interests.

Problem 16-2
a.

No entry is to be recorded by Holly during 2005 under the cost method.


Allocation schedule Date of acquisition
Difference
Allocation:
Inventory
Land
Equipment
Discount on notes payable
Total

P240,000
P ( 5,000)
(75,000)
(60,000)
(50,000)
P(190,000)

Minority interest (10%)


Goodwill (not impaired)
Amortization of differential:
Inventory sold
Land sold
Equipment (P60,000/15 years)
Discount on notes payable
Total
b.

171,000
P 69,000
P 5,000
75,000
4,000
7,500
P91,500

Working paper elimination entries


(1)

(2)

(3)

(4)

Common stock State


500,000
Premium on common stock State
100,000
Retained earnings State
120,000
Investment in State stock
Minority interest in net assets of subsidiary
To eliminate equity accounts of State on the date
of acquisition.

648,000
72,000

Inventory
5,000
Land
75,000
Equipment
60,000
Discount on notes payable
50,000
Goodwill
69,000
Investment in State stock
Minority interest in net assets of subsidiary
To allocate difference.

240,000
19,000

Cost of goods sold


5,000
Gain on sale of land
75,000
Operating expenses (depreciation)
4,000
Interest expense
7,500
Inventory
5,000
Land
75,000
Equipment
4,000
Discount on notes payable
7,500
To amortize allocated difference.
Minority interest in net asset of subsidiary
2,350
Minority interest in net income of subsidiary
2,350
To recognize minority share in the net income (loss)
of State.
Computed as follows:
Net income
P 68,000
Adjustments for total amortization
91,500
Adjusted net income (loss)
P(23,500)
Minority interest share (P23,500 x 10%)

Problem 16-3
a.

19,000

Consolidated Buildings

P 2,350

b.

c.

d.

Profit Company (at book value)


Simon Corporation (at fair value)
Amortization of differential (P120,000 / 6 years)
Total

P 900,000
560,000
( 20,000)
P1,440,000

Consolidated Retained Earnings, Dec. 31, 2008


Retained earnings, Jan. 1 Profit Company
Consolidated net income (per c below)
Dividends paid Profit Company
Total

P 600,000
380,000
(80,000)
P 900,000

Consolidated net income, Dec. 31, 2008


Total revenues (P700,000 + P400,000)
Total expenses (P400,000 + P300,000)
Amortization
Total

P1,100,000
(700,000)
( 20,000)
P 380,000

Consolidated Goodwill [(P680,000 P480,000)- P120,000]

80,000

Problem 16-4
Allocation Schedule
Acquisition cost
Less: Book value of interest acquired
Difference
Allocation:
Equipment
Buildings
Goodwill (not impaired)

P206,000
140,000
P 66,000
P(40,000)
10,000

(30,000)
P 36,000

a.

Investment in Stag Company 12/31/06 (at acquisition cost)

P 206,000

b.

Minority Interest in Net Assets of Subsidiary (MINAS)

P -0-

c.

Consolidated Net Income


Net income from own operations Pony (P310,000 P198,000) P 112,000
Net income from own operations Stag (P104,000 P74,000)
30,000
Amortization
( 4,500)
Total
P 137,500

d.

Consolidated Equipment
Total book value (P320,000 + P50,000)
Allocation
Amortization (P5,000 x 3 years
Total

P 370,000
40,000
(15,000)
P 395,000

Consolidated Buildings
Total book value
Allocation
Amortization (P500 x 3 years)
Total

P 288,000
( 10,000)
1,500
P 279,500

Consolidated Goodwill (not impaired)

e.

f.

36,000

g.

Consolidated Common Stock (Pony)

P 290,000

h.

Consolidated Retained Earnings


Retained earning, Dec. 31, 2008 Pony
P 410,000
Add: Ponys share of Stags adjusted increase in earnings
Net earnings 2008 (P30,000 P20,000)
P10,000
Amortization
( 4,500)
5,500
Total
P 415,500

Problem 16-5
a.

b.

Retained Earnings, Dec. 31, 2008 Sison


Stockholders equity, Dec. 31, 2008 Sison (P232,000/40%)
Stockholders equity, Jan. 1, 2005 Sison
Increase in earnings
Retained earnings, Jan. 1, 2005 Sison
Retained earnings, Dec. 31, 2008 Sison
Consolidated Retained Earnings Dec. 31, 2008
Retained earnings, Jan. 1, 2005 - Perez
Net income 2005 to 2008
Dividends paid 2005 to 2008
Retained earnings, Dec. 31, 2008
Add: Perez share of adjusted net increase in Sisons
Retained earnings
P80,000
Amortization (P8,333 x 4)
(33,332)
Adjusted
P46,668
Perez interest
60%
Total
Allocation Schedule
Acquisition cost
Less: Book value of interest acquired (P500,000 x 60%)
Difference
Allocation:
Depreciable assets (P50,000 / 60%)
P(83,333)
Minority interest (40%)
33,333
Amortization per year (P83,333/10 years)

P 580,000
(500,000)
P 80,000
200,000
P 280,000
P 600,000
100,000
( 45,000)
P 655,000

28,000
P 683,000
P350,000
300,000
P 50,000
(50,000
P 8,333

Problem 16-6
a.

Working Paper Elimination Entries, Dec. 31, 2008


(1)

(2)

Dividend income
Dividends declared Short
To eliminate intercompany dividends.

10,000

Common stock Short


Retained earnings Short
Investment in Short Company
To eliminate equity accounts of Short at
date of acquisition

100,000
50,000

10,000

150,000

(3)

(4)

b.

Depreciable asset
Investment in Short Company
To allocate difference.

30,000
30,000

Depreciation expense
Depreciable asset
To amortize allocated difference

5,000
5,000

Pony Corporation and Subsidiary


Consolidation Working Paper
December 31, 2008
Pony
Corporation

Short
Company

Income Statement
Sales
Dividend income
Total
Depreciation
Other expenses
Total
Net income carried forward

200,000
10,000
210,000
25,000
105,000
130,000
80,000

120,000
120,000
15,000
75,000
90,000
30,000

Retained Earnings
Retained earnings, Jan. 1
Net income from above
Total
Dividends declared

230,000
80,000
310,000
40,000

50,000
30,000
80,000
10,000

Adjustments

& Eliminations

Debit

Credit

Consolidated
320,000
320,000
45,000
180,000
225,000
95,000

(1) 10,000
(3) 5,000

(2) 50,000
(1) 10,000

230,000
95,000
325,000
40,000

Retained earnings, Dec. 31


Carried forward

270,000

70,000

285,000

Balance Sheet
Cash
Accounts receivable
Inventory
Depreciable asset (net)
Investment in Short stock

15,000
30,000
70,000
325,000
180,000

5,000
40,000
60,000
225,000

20,000
70,000
130,000
575,000
-

Total

620,000

330,000

795,000

Accounts payable
Notes payable
Common stock
Pony
Short
Retained earnings, Dec. 31
From above
Total

50,000
100,000

40,000
120,000

90,000
220,000

(3) 30,000

(4) 5,000
(2)150,000
(3) 30,000

200,000

200,000

270,000
620,000

100,000

(2)100,000

70,000
330,000

195,000

285,000
795,000

195,000

Problem 16-7
a.

Working Paper Elimination Entries


(1)

(2)

(3)

Dividend income
Minority interest in net assets of subsidiary
Dividends declared Sisa

8,000
2,000
10,000

Common stock Sisa


100,000
Retained earnings Sisa
50,000
Investment in Sisa stock
Minority interest in net assets of subsidiary
Minority interest in net income of subsidiary
Minority interest in net assets of subsidiary

120,000
30,000

6,000
6,000

b.

Popo Corporation and Subsidiary


Consolidated Working Paper
December 31, 2008
Popo
Corporation
Income Statement
Sales
200,000
Dividend income
8,000
Total revenue
208,000
Depreciation expense
25,000
Other expenses
105,000
Total expenses
130,000
Net income
78,000
MI in net income of Sub.
Net income carried forward
78,000
Retained Earnings
Retained earnings, 1/1
Net income from above
Total
Dividends declared
Retained earnings, 12/31
Carried forward

Sisa
Company

Adjustments

& Eliminations

Debit

Credit

120,000

320,000
320,000
40,000
180,000
220,000
100,000
( 6,000)
94,000

(1) 8,000
120,000
15,000
75,000
90,000
30,000
(3) 6,000
30,000

230,000
78,000
308,000
40,000

50,000
30,000
80,000
10,000

268,000

70,000

Consolidated

(2) 50,000
(1) 10,000

230,000
94,000
324,000
40,000
284,000

Balance Sheet
Current assets
Depreciable assets
Investment in Sisa stock
Total

173,000
500,000
120,000
793,000

105,000
300,000
405,000

278,000
800,000
1,078,000

Accumulated depreciation
Current liabilities
Long-term debt
Common stock
Retained earnings , 12/31
From above
MI in net assets of Subsidiary

175,000
50,000
100,000
200,000

75,000
40,000
120,000
100,000

250,000
90,000
220,000
200,000

268,000

70,000

Total

793,000

c.

(2)120,000

(2)100,000
(1) 2,000

405,000

166,000

(2) 30,000
(3) 6,000
166,000

284,000
34,000
1,078,000

Consolidated Financial Statements


Popo Corporation and Subsidiary
Consolidated Balance Sheet
December 31, 2008
Assets
Current assets
Depreciable assets
Less: Accumulated depreciation
Total assets
Liabilities and Stockholders Equity
Current liabilities
Long-term debt
Total liabilities
Stockholders Equity
Common stock
Retained earnings, 12/31
Minority interest in net assets of subsidiary
Total liabilities and stockholders equity

P278,000
P800,000
250,000

550,000
P828,000
P 90,000
220,000
P310,000

P200,000
284,000
34,000

518,000
P828,000

Popo Corporation and Subsidiary


Consolidated Income Statement
Year Ended December 31, 2008
Sales

P320,000

Expenses:
Depreciation expense
Other expenses
Consolidated net income
Minority interest in net income of subsidiary
Consolidated net income attributable to parent

P 40,000
180,000

220,000
P100,000
6,000
P 94,000

Popo Corporation and Subsidiary


Consolidated Retained Earnings
Year Ended December 31, 2008
Retained earnings, Jan. 1 Popo
Consolidated net income attributable to parent
Total
Dividends paid Popo
Consolidated retained earnings, Dec. 31

P230,000
94,000
P324,000
40,000
P284,000

Problem 16-8
a.

Palo Corporation and Subsidiary


Consolidation Working Paper
December 31, 2008
Adjustments

& Eliminations

Debit

Credit

Palo
Corporation

Sebo
Company

300,000
19,000
319,000
210,000
25,000
23,000
258,000
61,000

150,000

230,000
61,000
291,000
20,000

50,000
20,000
70,000
10,000

271,000

60,000

272,000

Balance Sheet
Cash
Accounts receivable
Inventory
Buildings and equipment
Investment in Sebo stock

37,000
50,000
70,000
300,000
229,000

20,000
30,000
60,000
240,000

57,000
80,000
130,000
540,000
-

Goodwill
Total

686,000

350,000

Income Statement
Sales
Investment Income
Total revenues
Cost of goods sold
Depreciation expense
Other expenses
Total cost and expenses
Net income carried forward
Retained Earnings
Retained earnings, Jan. 1
Net income from above
Total
Dividends declared
Retained earnings, Dec. 31
carried forward

450,000
450,000
295,000
45,000
48,000
388,000
62,000

(1) 19,000
150,000
85,000
20,000
25,000
130,000
20,000
(2) 50,000
(1) 10,000

(1) 9,000
(2)200,000
(3) 20,000
(3) 20,000

Consolidated

230,000
62,000
292,000
20,000

20,000
827,000

Accumulated depreciation
Accounts payable
Taxes payable
Common stock
Retained earnings, Dec. 31
from above
Total

b.

105,000
40,000
70,000
200,000

65,000
20,000
55,000
150,000

(2)150,000

271,000
686,000

60,000
350,000

239,000

170,000
60,000
125,000
200,000
239,000

Consolidated Financial Statements


Palo Corporation and Subsidiary
Consolidated Income Statement
Year Ended December 31, 2008
Sales
Cost of goods sold
Gross profit
Expenses:
Depreciation expenses
Other expenses
Consolidated net income

P450,000
295,000
155,000
P45,000
48,000

93,000
P 62,000

Palo Corporation and Subsidiary


Consolidated Retained Earnings
Year Ended December 31, 2008
Retained earnings, January 1 Palo
Consolidated net income
Total
Dividends paid Palo
Retained earnings, December 31

P230,000
62,000
292,000
20,000
P272,000

Palo Corporation and Subsidiary


Consolidated Balance Sheet
December 31, 2008
Assets
Cash
Accounts receivable
Inventory
Buildings and equipment
Less: Accumulated depreciation
Goodwill
Total
Liabilities and Stockholders Equity
Accounts payable

P 57,000
80,000
130,000
P540,000
170,000
370,000
20,000
P657,000
P 60,000

272,000
827,000

Taxes payable
Common stock
Retained earnings, Dec. 31
Total

125,000
200,000
272,000
P657,000

Problem 16-9
1.

Acquisition cost
Less: Book value of interest acquired (80%)
Common stock (P300,000 x 80%)
Retained earnings (P400,000 x 80%)
Difference
Allocation:
Inventories
Land
Building
Equipment
Patents
Total
Minority interest (20%)
Goodwill (not impaired)

P756,000
P240,000
320,000
P( 30,000)
( 50,000)
(100,000)
75,000
( 40,000)
P(145,000)
29,000

560,000
P196,000

(116,000)
P 80,000

Working Paper Elimination Entries - December 31, 2006(not required)


(1)

(2)

(3)

(4)

Investment income
Minority interest in net assets of subsidiary
Dividends declared S
Investment in S Company

50,000
54,800

Common stock S
300,000
Retained earnings, Jan. 1 S
400,000
Investment in S Co.
Minority interest in net assets of subsidiary

560,000
140,000

Inventories
30,000
Land
50,000
Building
100,000
Patents
40,000
Goodwill
80,000
Equipment
Investment in S Company
Minority interest in net assets of subsidiary

75,000
196,000
29,000

Cost of goods sold


Inventory
Equipment (P75,000 / 10)
Expenses (amortization)
Buildings (P100,000 / 20)
Patents (P40,000 / 10)

(5)

94,800
10,000

Minority interest in net income of subsidiary


Minority interest in net assets of subsidiary

30,000
30,000
7,500
1,500
5,000
4,000
23,700
23,700

To established minority share in subsidiary net income.


Computed as follows:
Net income S Co.
P150,000
Amortization
31,500
Adjusted net income
P118,500
MINIS (P118,500 x 20%)
P 23,700

2.

P Company and Subsidiary


Consolidated Working Paper
Year Ended December 31, 2008

Income Statement
Sales
Cost of sales
Gross profit
Expenses
Operating income
Investment income
Net /consolidated income
MI interest in net income of
Subsidiary
Net income carried forward
Retained earnings
Retained earnings, 1/1
Net income from above
Total
Dividends declared
Retained earnings, 12/31
Carried forward
Balance Sheet
Cash
Accounts receivable
Inventories
Land
Buildings (net)
Equipment (net)
Patent
Investment in S Co. stock
Goodwill
Total
Accounts payable
Common stock
Additional paid-in capital
Retained earnings, 12/31
from above
MI in net assets of subsidiary
Total

Adjustments

& Eliminations

Debit

Credit

P
Company

S
Company

1,000,000
400,000
600,000
360,000
240,000
94,800
334,800

500,000
150,000
350,000
200,000
150,000
150,000

334,800

150,000

600,000
334,800
934,800
100,000

400,000
150,000
550,000
50,000

834,800

500,000

834,800

200,000
150,000
100,000

100,000
50,000
40,000
150,000
200,000
450,000
-

300,000
200,000
140,000
200,000
295,000
680,500
36,000
-

298,000
810,800

1,500,000
580,000
920,000
561,500
358,500
358,500

(4) 30,000
(4) 1,500
(1) 94,800
(5) 23,700

(23,700)
334,800

(2)400,000

600,000
334,800
934,800
100,000

(1) 50,000

(3) 30,000
(3) 50,000
(3)100,000
(4) 7,500
(3) 40,000

(4) 30,000
(4) 5,000
(3) 75,000
(4) 4,000
(1) 54,800
(2)560,000
(3)196,000

(3) 80,000
1,558,800

1,090,000

124,000
200,000
400,000

190,000
300,000
-

834,800

500,000

1,090,000

80,000
1,931,500
314,000
200,000
400,000

(2)300,000

(1) 10,000
1,558,800

Consolidated

466,200

(2)140,000
(3) 29,000
(5) 23,700
466,200

834,800
182,700
1,931,500

Problem 16-10
a.

Investment in Sally Products Co.


Cash
To record acquisition of 80% stock of Sally.

160,000
160,000

Cash

8,000

Dividend income
To record dividends received from Sally (P10,000 x 80%)
b.

8,000

Working Paper Eliminating Entries Dec. 31, 2008


Allocation schedule:
Acquisition cost
Less: Book value of interest acquired (P150,000 x 80%)
Difference
Allocated to building and equipment
P (50,000)
Minority interest (20%)
10,000
(1)

(2)

(3)

(4)
(5)
(6)

c.

Dividend income
Minority interest in net assets of subsidiary
Dividends declared Sally

P160,000
120,000
40,000
(40,000)

8,000
2,000
10,000

Common stock Sally


100,000
Retained earnings, 1/1 Sally
50,000
Investment in Sally Products
Minority interest in net assets of subsidiary

120,000
30,000

Building and equipment


50,000
Investment in Sally Products
Minority interest in net assets of subsidiary

40,000
10,000

Depreciation expense
Accumulated depreciation Bldg
Accounts payables
Cash and receivables

5,000
5,000
10,000
10,000

Minority interest in net income of subsidiary


5,000
Minority interest in net assets of subsidiary
Computed as follows:
Net income Sally
P30,000
Amortization
(5,000)
Adjusted net income
P25,000
MINIS (P25,000 x 20%)
P 5,000

5,000

Pilar Corporation and Subsidiary


Consolidation Working Paper
December 31, 2008
Pilar

Sally Wood

Adjustments

& Eliminations

Consoli-

Corporation

Products

Income Statement
Sales
Dividend income
Total revenue

200,000
8,000
208,000

100,000

Cost of goods sold


Depreciation expense
Inventory losses
Total cost and expenses
Net /consolidated income

120,000
25,000
15,000
160,000
48,000

50,000
15,000
5,000
70,000
30,000

Credit

100,000

170,000
45,000
20,000
235,000
65,000

(4) 5,000

(6) 5,000
48,000

30,000

298,000
48,000
346,000
30,000

90,000
30,000
120,000
10,000

316,000

110,000

81,000
260,000
80,000
500,000
160,000

65,000
90,000
80,000
150,000

1,081,000

385,000

205,000
60,000
200,000
300,000
316,000

105,000
20,000
50,000
100,000
110,000

dated
300,000
300,000

(1) 8,000

MI interest in net income of


subsidiary (MINIS
Net income carried forward

Debit

(5,000)
60,000

Retained earnings statement

Retained earnings, 1/1


Net income from above
Total
Dividends declared
Retained earnings, 12/31
carried forward
Balance Sheet
Cash and receivables
Inventory
Land
Buildings and equipment
Investment in Sally
Total
Accumulated depreciation
Accounts payable
Notes payable
Common stock
Retained earnings from above
MI in net assets if subsidiary

Total

1,081,000

(2) 50,000
(1) 10,000

368,000
(5) 10,000
(3) 50,000
(2)120,000
(3) 40,000

385,000

338,000
60,000
398,000
30,000

136,000
350,000
160,000
700,000
1,346,000

(4) 5,000
(5) 10,000
(2)100,000
(1) 2,000

(2) 30,000
(3) 10,000
(6) 5,000

230,000

230,000

315,000
70,000
250,000
300,000
368,000
43,000

1,346,000

Problem 16-11
a.

Eliminating entries:
E(1)

E(2)

Dividend Income
Dividends Declared
Eliminate dividend income from subsidiary.
Common Stock Star Company
Retained Earnings, January 1

20,000
20,000
150,000
50,000

E(3)

Differential
Investment in Star Company Stock
Eliminate investment balance at date
of acquisition.

20,000

Goodwill
Retained Earnings, January 1
Differential
Assign differential at beginning of year

8,000
12,000

220,000

20,000

Porno Corporation and Star Company


Consolidated Workingpaper
December 31, 2008
Light
_____Item_____
Credit Consolidated
Income Statement
Sales
Dividend income
Credits
Cost of goods sold
Depreciation expense
Other expenses
Debits
Net income, carry forward
Retained Earnings Statement
Retained earnings, Jan. 1
Net income, from above
Dividends declared
Retained earnings, Dec. 31,
carry forward

Balance Sheet
Cash
Accounts receivable
Inventory
Buildings and equipment
Investment in Star Company
stock
Differential
Goodwill
Debits
Accumulated depreciation
Accounts payable`
Taxes payable
Common stock

Star

Eliminations
Corporation
Company

350,000
20,000
370,000
270,000
25,000
21,000
(316,000)
54,000

200,000
200,000
135,000
20,000
10,000
(165,000)
35,000

262,000

60,000

54,000
316,000
(20,000)

35,000
95,000
(20,000)

296,000

75,000

46,000
55,000
75,000
300,000

30,000
40,000
65,000
240,000

__

20,000

____

550,000
_______
550,000
405,000
45,000
31,000
(481,000)
69,000

(2) 50,000
(3) 12,000
20,000
___

(1) 20,000

82,000

20,000

260,000
69,000
329,000
(20,000)
309,000

76,000
95,000
140,000
540,000

220,000
-

(1) 20,000

Debit

(2)220,000
(2) 20,000 (3) 20,000
(3) 8,000

696,000

375,000

8,000
859,000

130,000
20,000
50,000

85,000
30,000
35,000

215,000
50,000
85,000

Light Corporation
Star Company
Retained earnings, from above
Credits

200,000
296,000
696,000

200,000
150,000
75,000
375,000

(2)150,000
82,000
260,000

20,000
260,000

CHAPTER 17
MULTIPLE CHOICE
17-1:

B
Consolidated sales
Sales Papa
Sales San
Elimination of inter-company sales
Consolidated sales
Consolidated cost of goods sold
Cost of goods sold Papa
Cost of goods sold San
Eliminations:
Realized profit in beginning inventory
Unrealized profit in ending inventory
Intercompany purchases
Consolidated cost of goods sold

17-2:

( 4,000)
10,000
( 50,000)
P 636,000

60,000
( 10,000)
P 50,000
20%
P 10,000

d
Net income from own operation Pat
Pats share of adjusted net income of Susan:
Net income Susan
P200,000
Realized profit in beginning inventory
(P112,000 x 50%/150%)
37,500
Unrealized profit in ending inventory
(P33,000 x 50%/150%)
(11,000)
Consolidated net income
Attributable to minority interest (P226,500 x 30%)
Attributable to parent

17-4:

P 490,000
190,000

c
Net income Sisa
Unrealized profit in ending inventory upstream
Adjusted net income Sisa
Minority interest proportionate share
Minority interest in net income of subsidiary

17-3:

P 900,000
500,000
( 50,000)
P 1,350,000

P 200,000

226,500
P 426,500
67,950
P 358,550

b
Net income from own operations- Patton
Unrealized profit in ending inventory DS (P200,000 x .25)
Realized income
Solis net loss
Consolidated net income

P 300,000
(50,000)
250,000
(150,000)
P 100,000

309,000
859,000

17-5:

d
Pardos share of Santos net income (P300,000 x 75%)
Unrealized profit in ending inventory Upstream
(P200,000 x 25%/125%) x 75%
Realized profit in beginning inventory Upstream
(P150,000 x 25%/125%) x 75%
Investment income account balance, Dec. 31, 2008

17-6:

17-7:

d
Net income from own operation Puzon
Suazons adjusted net income:
Net income
Unrealized profit in ending inventoryUpstream (P25,000 x 40%)
Consolidated net income
MINIS (P100,000 x 25%)
Attributable to parent

( 30,000)
22,500
P 217,500
P 200,000

P110,000
( 10,000)

100,000
P 300,000
(25.000)
P 275,000

2008
P 500,000

2009
P 550,000

b
Net income from own operation Pat
Unrealized profit in ending inventory:
2008 (P20,000 x .40)
2009 (P30,000 x .50)
Realized profit in beginning inventory
Realized income
Sun net income
Consolidated net income

17-8:

P 225,000

(8,000)
492,000
200,000
P 692,000

(15,000)
8,000
543,000
225,000
P 768,000

a
Net income from own operation Pip
Adjusted net income of Sol:
Net income
P 250,000
Realized profit in beginning inventoryUpstream (P40,000 x 40%)
16,000
Unrealized profit in ending inventoryUpstream (P70,000 x 30%)
( 21,000)
Consolidated net income - 2008

a
Net income from own operations Popo
Unrealized profit in ending inventory Downstream
Realized separate net income Popo
Popos share of Sottos adjusted net income:
Net income
P 360,000
Realized profit in beginning inventoryUpstream
10,000
MINIS (P370,000 x 5%)
Attributable to parent
17-10: a

P 400,000

245,000
P 645,000

17-9:

Stockholders equity Sands, Dec. 31, 2008


Unamortized difference (P1,000,000 P200,000)
Adjusted stockholders equity (net assets) Sands

P 500,000
( 15,000)
P 485,000

370,000
( 18,500)
P 836,500
P5,500,000
800,000
P6,300,000

Minority interest in net assets of subsidiary (P6,300,000 x 40%) P2,520,000


17-11: d
Gross profit rate Short (P110,000 / P200,000)

55%

Inventories
Inventory from outsiders Power
Inventory from outsiders Short
Powers inventory acquired from Short at cost:
[P5,000 (P5,000 x 55%)}
Consolidated ending inventories
Investment income
Powers share of Shorts net income (P50,000 x 75%)
Unrealized profit in ending inventory upstream
(P5,000 x 55%) x 75%
Realized profit in beginning inventory upstream
(P10,000 x 55%) x 75%
Investment income, Dec. 31, 2008

P 5,000
25,000
2,250
P 32,250
P 37,500
( 2,063)
4,125
P 39,562

Investment in Short Company


Acquisition cost (P80,000 x 75%)
Unrealized profit in ending inventory
Realized profit in beginning inventory
Investment in Short Company, Dec. 31, 2008

P 60,000
( 2,063)
4,125
P 62,062

Minority interest in net assets of subsidiary


Stockholders equity, Dec. 31, 2006 Short
Realized profit in beginning inventory (P10,000 x 55%)
Unrealized profit in ending inventory (P5,000 x 55%)
Adjusted net assets of Short, Dec. 31, 2006
Minority interest
MINAS

P 80,000
5,500
( 2,750)
P 82,750
25%
P 20,687.50

17-12: b
Gross profit rate of Sit (P200,000 / P500,000)
Net income from own operations Pit
Adjusted net income of Sit:
Net income
P 75,000
Realized profit in beginning inventoryUpstream (P40,000 x 40%)
16,000
Unrealized profit in ending inventoryUpstream (P25,000 x 40%)
( 10,000)
Consolidated net income
MINIS (P281,000 x 10%)
Attributable to parent
17-13: b
Gross profit of Sir (P120,000 / P400,000)
Consolidated cost of sales
Cost of sales Pig
Cost of sales Sir
Eliminations:
Realized profit in beginning inventory (P70,000 x 30%)
Unrealized profit in ending inventory (P60,000 x 30%)

40%
P 200,000

81,000
P 281,000
( 8,100)
P 272,900
30%
P 600,000
280,000
( 21,000)
18,000

Intercompany purchases
Consolidated cost of sales
Consolidated net income
Net income from own operations Pig
Pigs share of Sirs adjusted net income:
Net income
Realized profit in beginning inventory
Unrealized profit in ending inventory
Consolidated net income
MINIS (P83,000 x 10%)
Attributable to parent

(200,000)
P 677,000
P 200,000
P 80,000
21,000
(18,000)

83,000
283,000
(8,300)
P 274,700

17-14: a
2006
Pal Corp net income
150,000
Intercompany profit in ending inventory:
2006
(14,000)
2007
2008
Pal net income from own operation
136,000
Solo net income from own operation
100,000
Consolidated net income
236,000
MINIS:
2006(100,000 14,000) x 40%
34,400
2007(90,000 +14,000 21,000) 40%
2008(160,000 + 21,000 24,000) 40%
Consolidated NI attributable to Parent 201,600

2007
240,000
14,000
(21,000)
233,000
90,000
323,000

2008
300,000
21,000
( 24,000)
297,000
160,000
427,000

33,200
289,800

17-15: a
Acquisition cost
Less: book value of interest acquired (400,000 x 60%)
Difference
Allocated to Equipment
( 20,000)
MINAS (40%)
8,000
Total sales
Intercompany sales (30,000 + 80,000)
Consolidated sales

62,800
394,200
252,000
240,000
12,000
(12,000)
600,000
(110,000)
490,000

17-16: c
Total cost of goods sold (250,000 +120,000)
370,000
Adjustments due to intercompany sale:
COGS charged for intercompany sale (20,000 + 50,000) 70,000
COGS charged by: Star (30,000 6,000)
24,000
Polo (80,000 20,000)
60,000
Total
154,000
Cost of goods sold for consolidated entity:
20,000 x (24,000/30,000)
(16,000)
50,000 x (60,000/80,000)
(37,500) (100,500)
Consolidated cost of goods sold
269,500
17-17: c
Polo Corp. net income from own operation (105,000 25,000)

80,000

Unrealized profit in ending inventory-DS (6,000 x 10/30)


(2,000)
Adjusted Polo Corp. net income from own operation
Star Corp. net income from own operation:
Net income
Unrealized profit in EI-US (20,000 x 30/80)
Amortization (20,000/10 years)
Consolidated net income
MINIS (35,500 x 40%)
Attributable to Parent

78,000
45,000
(7,500)
(2,000)

17-18: a
Pepsi net income from own operation
Sarsi net income
90,000
Unrealized profit in EI (45,000 x 60/180)
(15,000)
Consolidated net income
MINIS (75,000 x 30%)
Consolidated net income attributable to Parent-2007
17-19: a
Inventory-Pepsi
P 30,000
Less: unrealized profit in books of Sarsi:
(135,000 90,000) x (30,000/135,000)
(10,000)
Inventory-Sarsi
P110,000
Less: unrealized profit in books of Pepsi:
(280,000 140,000) x (110,000/280,000)
(55,000)
Consolidated inventory 12/31/08
17-20: a
Cost of goods sold on sale of inventory on hand-1/1/08:
[45,000 x (120,000/180,000)]
Cost of goods sold on purchases from Sarsi- 2008
[(135,000 30,000) x (90,000/135,000)]
Cost of goods sold on purchases from Pepsi- 2008
[(280,000 110,000) x (140,000/280,000)]
Consolidated cost of goods sold-2008
17-21: b
Pepsi net income
Sarsi net income
Realized profit in beginning inventory - 2008
Unrealized profit in ending inventory- Sarsi
Unrealized profit in ending inventory- Pepsi
Consolidated net income

35,500
113,500
(14,200)
99,300
160,000
75,000
235,000
(22,500)
212,500

20,000
55,000
75,000

30,000
70,000
85,000
185,000

220,000
85,000
15,000
(10,000)
(55,000)
255,000

CHAPTER 18
MULTIPLE CHOICE
18-1:

a
Equipment at original cost
Accumulated depreciation:
Time of sale

P500,000
P250,000

Current depreciation based on


Original cost (P500,000/10 years
18-2:

50,000

b
Net income Sol
Unrealized gain on sale of computer, Dec. 31
Adjusted net income
Minority interest proportionate share
Minority interest in net income of subsidiary (MINIS)

18-3:

P100,000
( 30,000)
P 70,000
30%
P 21,000

b
Net income from own operations Prime
Unrealized gain Downstream
Realized net income Prime
Second Company net income
Consolidated net income

18-4:

P300,000

2005
P200,000
(30,000)
P170,000
100,000
P270,000

2006
P250,000
__P250,000
150,000
P400,000

c
Net income Saw
Unrealized loss-Upstream
Realized loss ((P12,000 / 5) x 6/12
Adjusted net income Saw

P100,000
12,000
( 1,200)
P110,800

Minority interest in net income of subsidiary (P110,800 x 25%) P 27,700


18-5:

c
Equipment at original cost
Accumulated depreciation:
Time of sale
Current depreciation (P900,000/10)

18-6:

P 450,000

P 30,000
90,000
P120,000

a
Original cost
Amount debited to Truck account
Selling price of the truck Amount paid

18-8:

P360,000
90,000

a
Adjusted net income Susie (P12,000 / 40%)
Add back: Unrealized gain Upstream
Net income of Susie 2008

18-7:

P1,000,000

P100,000
(48,000)
P 52,000

c
Net income Po

P200,000

Unrealized gain, Dec. 31 DS


Net income from own operation Po
Net income of So
Consolidated net income, Dec. 31, 2008
MINIS (P180,000 x 20%)
Attributable to parent
18-9:

(30,000)
270,000
180,000
P350,000
(36,000)
P314,000

b
Stockholders equity, Jan. 1, 2008 Sy
Increase in earnings 2008 (P65,000 P30,000)
Stockholders equity, Dec. 31, 2008 Sy

P1,000,000
35,000
P1,035,000

Minority interest in net assets of subsidiary (P1,035,000 x 20%) P 207,000


18-10: b
Consolidated net income attributable to parent:
Net income Pink
Unrealized gain, July 1- Downstream
Realized gain, Dec. 31 (P50,000 / 10) x 6/12
Realized net income Pink
Sodas adjusted net loss:
Net loss
P(40,000)
Unrealized loss, 1/1 Upstream
15,000
Realized loss, 12/31 (P15,000/5)
( 3,000)
Consolidated net income, Dec. 31, 2008
MI in net loss of subsidiary (P28,000 x 20%)
Attributable to parent

Minority interest in net assets of subsidiary


Net assets, Jan. 1, 2008 (P1,240,000 / 80%)
Decrease in earnings:
Net loss
Dividends paid
Net assets, Dec. 31, 2008
Unrealized loss, Dec. 31 (Upstream)
Adjusted net assets, Dec. 31, 2008

P300,000
( 50,000)
2,500
P252,500

(28,000)
P224,500
5,600
P230,100

P1,550,000
P40,000
30,000

( 70,000)
P1,480,000
( 12,000)
P1,468,000

Minority interest in net assets of subsidiary (P1,468,000 x 20%) P 293,60


18-11: a
Net assets, Dec. 31, 2008
Minority interest, Dec. 31, 2008
Add: MI share of unrealized profit in ending inventory -Upstream
(P36,000 x 20%) x 20%
MI share of unrealized gain on sale of equipment- Upstream
(P60,000 x 20%) (P12,000 / 5)
Minority interest before adjustment

P188,960
1,440
9,600
P200,000

Net assets Steve, Dec.31, 2008 (P200,000 / 20%)

P1,000,000

Investment in Steve Company stock Equity method


Acquisition cost:
Net assets, Dec. 31, 2008
Less: net income steve
MINIS
P36,960
MI share of unrealized profit in ending
Inventory Upstream
1,440
MI share of unrealized gain on sale of
Equipment Upstream
9,600
MINIS per book
P48,000
Divided by
20%
Net assets, Jan. 1, 2008
Parents proportionate share
Book value of interest acquired
Add: difference
Purchase price (acquisition cost)
Add: Investment income
Peters share of Steve net income (P240,000 x 80%)
Unrealized profit in ending inventory Downstream
(P24,000 x 20%/120%) x 100%
Unrealized profit in beginning inventory Upstream
(P36,000 x 25/125%) x 80%
Unrealized gain on sale of equipment Upstream
(P48,000 9,600)
Investment in Steve Company, Dec. 31, 2008

P1,000,000

240,000
P 760,000
x 80%
P 608,000
20,000
P 628,000
P 192,000
( 4,000)
( 5,760)
( 38,400)
P 771,840

18-12: a
Net income from own operations Pipe
Pipes share of Smokers adjusted net income:
Net income
Unrealized gain, July 1, 2008 Upstream
Realized gain, Dec. 31, 2008 (P50,000/5)x
Consolidated net income, Dec. 31, 2008

P400,000
P100,000
(50,000)
5,000

55,000
P455,000

18-13: d
Net income from operations Parent
Parents share of adjusted net income of Sub:
Net income
Unrealized gain Upstream
Realized gain: 2007 (P9,000/3) x
2008 (P9,000/3)
Adjusted net income
Consolidated net income
MINIS
Attributable to parent

2007
P100,000

2008
P120,000

P 60,000
( 9,000)
750
P 51,750
P151,750
(10,350)
P141,400

P 75,000
3,000
P 78,000
P198,000
(15,600)
P182,400

18-14: d
Investment in Sili Company stock Equity method
Acquisition cost
Investment income net of dividends 2005 to 2007:

P500,000

Increase in earnings (P500,000 P200,000) x 75%


Investment income, Dec. 31, 2007:
Share of Silis net income (P60,000 x 75%)
45,000
Unrealized gain on sale of land Downstream
(15,000)
Unrealized loss on sale of building Downstream
10,000
Realized loss on sale of building (P10,000 / 5) x 75% ( 1,500)
Investment income, Dec. 31, 2008:
Share of Silis net income (P70,000 x 75%)
52,500
Realized loss (P10,000 / 5)
(2,000)
Dividends received:
2007: (P10,000 x 75%)
7,500
2008: (P20,000 x 75%)
15,000
Investment in Sili Company stock, Dec. 31, 2008

225,000

38,500
50,500
(22,500)
P791,500

18-15: a
Investment in Saw Company stock, Dec. 31, 2008
Acquisition cost
Investment income 2002 to 2006:
Increase in earnings (P500,000 P300,000) x 90%
Investment income 2007 (see above)
Investment income 2008:
Powers share of Saws net income (P120,000 x 90%) P108,000
Realized loss on sale of warehouse (P20,000/2) x 90%
(9,000)
Dividends received:
2007: ( P20,000 x 90%)
P 18,000
2008: ( P30,000 x 90%)
27,000
Investment in Saw Company stock account balance 12/31/08

P550,000
180,000
101,250
99,000
(45,000)
P885,250

PROBLEMS
Problem 18-1
Computation of the missing amounts in the working paper eliminations for P Corporation and S
Company:
(1)
P640 (P3,200 x 20%)
(2)
P2,560 (P3,200 x 80%)
(3)
P1,600 (P800 x 2)
(4)
P320 (P1,600 x 20%)
(5)
P1,280 (P1,600 x 80%)
(6)
P3,200 (P800 x 4)
Problem 18-2
a.
Consolidated Net Income
Net income from own operations P Company
Unrealized gain on sale of equipment, Dec. 31 Downstream
Adjusted net income P Co,
S Company net income
Consolidated net income

P200,000
(30,000)
P170,000
180,000
P350,000

b.

Minority interest in net income of subsidiary (P180,000 x 20%)

P 36,000

c.

Minority Interest in Net Assets of Subsidiary:


Stockholders equity, Jan. 1, 2008 S Company
Increase in earnings 2008 (P180,000 P60,000)
Stockholders equity, Dec. 31, 2008 S Company
Minority interest
Minority interest in net assets of subsidiary

P 900,000
120,000
P1,020,000
x 20%
P 204,000

Problem 18-3
Pony Corporation and Subsidiary
Consolidated Income Statement
Year Ended December 31, 2008
Sales (P500,000 + P300,000)
Gain on sale of machinery (schedule 1)
Total revenue

P800,000
20,000
820,000

Cost of sales P200,000 + P130,000)


Gross profit
Expenses:
Depreciation (P50,000 +P30,000 P5,000)
P 75,000
Other expenses (P80,000 + P140,000)
220,000
Consolidated net income
Attributable to minority interest (P190,000 + P5,000) +10,000) x 25%
Attributable to parent

330,000
490,000
295,000
785,000
(28,750)
P266,250

Schedule 1:
Selling price Dec. 28, 2008
Book value (P65,000 5) x3
Gain on sale
Unrealized gain (P25,000 P15,000)
Total gain

P36,000
26,000
10,000
10,000
P20,000

Problem 18-4
a.

b.

Consolidated Net Income


Net income from own operations P Company
Adjusted net income of S Company:
Net income S
Unrealized gain, 4/1/08 - Upstream
Realized gain, 12/31/08 (P30,000/5) x 9/12
Consolidated net income
MINIS (P124,500 x 20%)
Attributable to parent
Minority Interest in Net Assets of Subsidiary
Stockholders equity , Jan. 1, 2008 S Company
Increase in adjusted earnings 2008:
Net earnings (P150,000 P50,000)
Unrealized gain 12/31 (P30,000 P4.500)
Stockholders equity, Dec. 31, 2008
Minority interest
MINAS

P300,000
P150,000
( 30,000)
4,500

124,500
424,500
(24,900)
P399,600

P800,000
P100,000
(25,500)

74,500
P874,500
x 20%
P174,900

Problem 18-5
a.

Consolidated Net Income - 2008


Net income from own operations BJ
Gain on sale of machine, July 1 - Downstream
Realized gain, Dec. 31 (P50,000 / 10) x 6/12
Adjusted net income BJ
Net income (loss) of DK:
Net income (loss) DK
Loss on sale of truck , Jan. 1 - Upstream
Realized loss, Dec. 31 (P15,000 / 5)

P300,000
(50,000)
2,500
P252,500
P(40,000)
15,000
( 3,000)

(28,000)

Consolidated net income


b.

c.

P224,500

Minority Interest in Net Income of Subsidiary


Net loss from own operations DK
Upstream loss on sale of truck
Realized loss on sale of truck
Adjusted net loss
Minority interest
MI in net loss of subsidiary
Minority Interest in Net Assets of Subsidiary
Net assets, Jan. 1, 2006 (P1,240,000 / 80%)
Increase in earnings (loss) -2006 (P40,000 + P30,000)
Net assets, Dec. 31, 2006
Unrealized loss Upstream (P15,000 P3,000)
Adjusted net assets
Minority interest
MINAS

P (40,000)
15,000
( 3,000)
P ( 28,000)
x 20%
P ( 5,600)
P1,550,000
(70,000)
P1,480,000
12,000
P1,492,000
x 20%
P 298,400

Problem 18-6
Texas Company and Subsidiary
Consolidated Income Statement
Year Ended December 31, 2008
Sales
Cost of goods sold
Gross profit
Expenses (P200,000 + P100,000 P8,000 )
Consolidated net income
Attributable to minority interest (P150,000 x 25%)
Attributable to parent

P1,500,000
650,000
850,000
292,000
P 558,000
37,500
P 520,500

Adjustment for expenses (depreciation) = P40,000 / 5 years.


Problem 18-7
a.

Leo Company and Subsidiary


Consolidated Balance Sheet Working Paper
December 31, 2007
Leo
Company
101,000
80,000
150,000
400,000
141,000

Taurus
Corporation
20,000
40,000
90,000
300,000

Total debits

872,000

450,000

Accumulated depreciation
Accounts payable
Notes payable
Common stock
Retained earnings

135,000
90,000
200,000
100,000
347,000

85,000
25,000
90,000
200,000
50,000

Cash and receivables


Inventory
Land
Building and equipment
Investment in stock Taurus

MI in net assets in Subsidiary

Adjustments

& Eliminations

Debit

Credit

(2) 10,000
(3) 9,000
(3) 15,000

(1)150,000
(2) 6,000

Consolidated
121.000
120,000
250,000
709,000
1,200,000

(3) 24,000

244,000
115,000
290,000
100,000
347,000

(1)100,000
(2) 4,000

104,000

(1)200,000
(1) 50,000

Total

872,000

450,000

1,200,000

(1) To eliminate equity accounts of subsidiary


(2) To intercompany gain on sale of land.
(3) To eliminate intercompany gain on sale of equipment debited to Investment account and restore equipment to
its original book value.

b.

Leo Company and Subsidiary


Consolidated Balance Sheet
December 31, 2008
Cash and receivables
Inventory
Land
Building and equipment
Less: Accumulated depreciation
Total assets

P121,000
120,000
250,000
P709,000
244,000
465,000
P956,000

Accounts payable
Notes payable
Common stock stock
Retained earnings
Minority interest in net assets of subsidiary
Total liabilities and equity

P115,000
290,000
100,000
347,000
104,000
P956,000

Problem 18-8
a.

Working Paper Elimination Entries Dec. 31, 2008


(1)

(2)

(3)

(4)

(5)

Dividend income
Minority interest in net assets of subsidiary
Dividends declared Jupiter
To eliminate intercompany dividends

4,000
1,000
5,000

Common stock Jupiter


100,000
Retained earnings Jupiter
50,000
Investment in Jupiter stock
Minority interest in net assets of subsidiary
To eliminate equity accounts of Jupiter as of the
date of acquisition
Goodwill
Investment in Jupiter Stock
To allocate difference to goodwill

40,000
40,000

Retained earnings Jan. 1


8,000
Minority interest in net assets of subsidiary
2,000
Land
To eliminate unrealized gain on sale of land Upstream.

Gain on sale of equipment


Building and equipment

120,000
30,000

20,000
5,000

10,000

Accumulated depreciation
To eliminate gain on sale of equipment
(6)

(7)

(8)

b.

Accumulated depreciation
Depreciation
To adjust excess depreciation

25,000
2,000
2,000

Accounts payable
7,000
Accounts receivable
To eliminate intercompany payables and receivables.

7,000

Minority interest in net income of subsidiary


6,000
Minority interest in net assets of subsidiary
(P40,000 10,000) x 20%

6,000

Vincent Company and Subsidiary


Consolidation Working Paper
December 31, 2008

Income Statement
Sales

Vincent
Company

Jupiter
Company

240,000

120,000

Adjustments

& Eliminations

Debit

Credit

Consolidated
360,000

Gain on sale of equipment


Dividend income
Total revenues
Cost of goods sold
Depreciation
Other expenses
Total cost and expenses
Net/consolidated income
MI in net income of subsidiary
Net income carried forward
Retained Earnings Statement
Retained earnings, Jan.1
Net income from above
Total
Dividends declared
Retained earnings, Dec. 31
Carried forward
Balance Sheet
Cash and receivables
Inventory
Land
Buildings and equipment
Investment in Jupiter stock
Goodwill
Total
Accumulated depreciation
Accounts payable
Bonds payable
Common stock
Retained earnings from above
MI in net assets of subsidiary
Total

20,000
4,000
264,000
140,000
25,000
15,000
180,000
84,000

(5) 20,000
(1) 4,000
120,000
60,000
15,000
5,000
80,000
40,000

84,000

40,000

294,000

105,000

84,000
378,000
30,000

40,000
145,000
5,000

348,000

140,000

113,000
260,000
80,000
500,000
160,000

35,000
90,000
80,000
150,000

1,113,000

355,000

205,000
60,000
200,000
300,000
348,000

45,000
20,000
50,000
100,000
140,000

(6) 2,000

(8) 6,000

(2) 50,000
(4) 8,000

341,000

(1) 5,000

(7) 7,000
(4) 10,000
(5) 5,000
(2)120,000
(3) 40,000

355,000

(6) 2,000
(7) 7,000

(5) 25,000

(2)100,000
(1) 1,000
(4) 2,000

(2) 30,000
(8) 6,000

245,000

245,000

Consolidated Financial Statements


Vincent Company and Subsidiary
Consolidated Balance Sheet
December 31, 2008
Assets
Cash and receivables
Inventory
Land
Buildings and equipment
Less: Accumulated depreciation
Goodwill
Total assets

P 141,000
350,000
150,000
P655,000
273,000

141,000
350,000
150,000
655,000
40,000
1,336,000

c.

96,000
437,000
30,000
407,000

(3) 40,000

1,113,000

360,000
200,000
38,000
20,000
258,000
102,000
(6,000)
96,000

382,000
40,000
P1,063,000

273,000
73,000
250,000
300,000
407,000
33,000
1,336,000

Liabilities and Stockholders equity


Liabilities
Accounts payable
Bonds payable
Total liabilities
Stockholders Equity
Common stock
Retained earnings
Minority interest in net assets of subsidiary
Total liabilities and stockholders equity

73,000
250,000
P 323,000
P300,000
407,000
33,000

740,000
P1,063,000

Vincent Company and Subsidiary


Consolidated Income Statement
Year Ended December 31, 2008
Sales
Cost of goods sold
Gross profit
Expenses: Depreciation
Other expenses
Consolidated net income
Attributable to minority interest
Attributable to parent

P 360,000
200,000
160,000
P 38,000
20,000

58,000
102,000
6,000
P 96,000

Vincent Company and Subsidiary


Consolidated Retained Earnings
Year Ended December 31, 2008
Retained earnings, Jan. 1 Vincent
Retained earnings, Jan. 1 Jupiter
Total
Consolidated net income attributable to parent
Dividends declared Vincent
Consolidated retained earnings

P 294,000
47,000
341,000
96,000
( 30,000)
P 407,000

Problem 18-9
(a)

P100,000 (the common stock of Phantom only)

(b)

P140,000

P250,000 (P593,000 P343,000)

(d)

accounts payable
P100,000 (P126,000 P35,000) + [(P25,000 + P85,000) - P101,000]

(e)

(f)

Purchase price, Jan. 1, 2008


Undistributed earnings from 1/1/05 to 1/1/08:
(P80,000 P30,000) x 60%
Undistributed income for 2008 (P30,000 P20,000) x 60%
Total
Adjustments:

P105,000
30,000
6,000
P141,000

Unrealized gain on sale of land Downstream (g)


Unrealized gain on sale of equipment Upstream
(P9,000 P3,000) x 60%
Adjusted Investment account balance, Dec. 31, 2008

(7,000)
(3,600)
P 70,400

(g)

P7,000 (P70,000 + P90,000) P153,000

(h)

(i)

P510,000 [P345,000 + P150,000 + (P60,000 P45,000)]

(j)

P278,000 = P180,000 + P80,000 + [(P60,000/5) x 4 ]


Less [(P45,000 / 3) x 2 years]

(k)

Retained earnings, Dec. 31, 2008


Less: Share of unrealized profit on sale of equipment:
Gain record [P45,000 (P60,000 x 3/5)]
Realized in 2008 (P9,000 / 3)
Unrealized
Phantoms interest
Consolidated retained earnings

(l)

P380,000
P9,000
3,000
P6,000
x 60%

Net income Shadow, 2008 (P250,000 P220,000)


Realized gain on sale of building c Dec. 31, 2006 Upstream
Adjusted net income
Minority interest
Minority interest in net income of subsidiary

3,600
P376,400
P 30,000
3,000
P 33,000
x 40%
P 13,200

Problem 18-10
Supporting computations
(1)

Allocation schedule (purchase price)


Less: Book value of interest acquired (P350,000 x 60%)
Difference
Allocated to patents (P120,000 x 60%)
Goodwill

P 372,000
210,000
P 162,000
( 72,000)
P 90,000

Amortization of patents (P120,000 / 12)

P 10,000

(2)

Unrealized gain on intercompany sale of building Upstream, Jan. 1, 2006:


Unrealized gain at date of sale (P80,000 P30,000)
P 50,000
Realized gain (P50,000 / 5) x 2 years
(20,000)
Unrealized gain as of Jan. 1, 2008
P 30,000

(3)

Realized profit from intercompany sale of inventory Downstream, 1/1/08:


Remaining inventory as of Dec. 31, 2007
P 50,000
Gross profit rate on sales 2007 (P30,000 / P150,000)
x 20%
Realized profit as of Jan. 1, 2008
P 10,000

(4)

Unrealized profit from intercompany sale of inventory Downstream, 12/31/08


Remaining inventory as of Dec. 31, 2008
P 40,000

Gross profit rate on sales 2008 (P48,000 / P160,000)


Unrealized profit as of Dec. 31, 2008

x 30%
P 12,000

Consolidated balances 2008


a.

b.

Cost of goods Sold


Cost of goods sold Apex
Cost of goods sold Small
Intercompany sale of inventory 2008
Realized profit on beginning inventory
Unrealized profit on ending inventory
Consolidated

P 460,000
205,000
(160,000)
( 10,000)
12,000)
P 507,000

Operating Expenses
Operating expenses Apex
Operating expenses Small
Amortization (No. 1 above)
Excess depreciation (P50,000 / 5 years)
Consolidated

P 170,000
70,000
10,000
(10,000)
P 240,000

c.

Consolidated Net Income


Sales (after elimination of intercompany sales)
P 840,000
Cost of goods sold (a)
(507,000)
Operating expenses (b)
(240,000)
Minority interest in net income of subsidiary:
Net income Small
P25,000
Realized gain on sale of building Upstream
10,000
Adjusted net income
P35,000
Minority interest
x 40%
( 14,000)
Attributable to parent
P 79,000
note: the amortization for patent is not deducted w/c is wrong.
d.
Consolidated Retained Earnings, Jan. 1, 2008
Retained earnings, Jan. 1, 2008 Apes
P 690,000
Amortization of patents 2002 to 2007 (P10,000 x 6)
(60,000)
Unrealized profit on inventory, 2007 Downstream
(10,000)
Unrealized gain on sale of building, 1/1/08 - Upstream (P30,000 x 60%)
(18,000)
Consolidated retained earnings, Jan. 1, 2008
P 602,000
note: book value is treated in different way. no RE increase from date of consolidation. the
date is different for the sale of building.
e.
Consolidated Inventory
Inventory Apex
P 233,000
Inventory Small
229,000
Unrealized profit in inventory Dec. 31, 2008
( 12,000)
Consolidated inventory
P 450,000
f.

Consolidated Building
Buildings Apex
Buildings Small

P 308,000
202,000

g.

Unrealized gain, Jan. 1, 2006


Realized gain, 2006 2008 (P10,000 x 3 )
Consolidated buildings

(50,000)
30,000
P 490,000

Consolidated Patents
Patents Small
Allocation
Amortization, 2002 2008 (P10,000 x 7)
Consolidated patents (net)

P 20,000
120,000
( 70,000)
P 70,000

h.

Consolidated Common Stock = P300,000 (Apex common stock)

i.

Minority Interest in Net Assets of Subsidiary


Stockholders equity Small, Dec. 31, 2008 (P100,000 + P420,000)
Unrealized gain on sale of building, Dec. 31,2008 Upstream
Adjusted net assets, Dec. 31, 2008
Minority interest
Minority interest in net assets of subsidiary
the patent(net) in comsolidation is not included but must be.

P 520,000
(20,000)
P 500,000
x 40%
P
200,000

Problem 18-11
a.

Working Paper Elimination Entries


(1)

(2)

(3)

(4)

(5)

(6)

Retained earnings Jan. 1


Investment in Duke
To adjust Investment account for unrealized profit
in inventory on Dec. 31, 2005 (P10,000 x 60%)

6,000

Income from Duke Company


Minority interest in net assets of subsidiary
Dividends declared Duke
Investment in Duke
To eliminate intercompany dividends.

84,000
24,000

6,000

60,000
48,000

Common stock Duke


320,000
APIC Duke
90,000
Retained earnings, 1/1 Duke
620,000
Investment in Duke (60%)
Minority interest in net assets of subsidiary (40%)
To eliminate equity accounts of Duke as of beginning of year.
Goodwill
Investment in Duke
To allocate difference
Impairment loss
Goodwill
To reduce goodwill for impairment.
Sales
Cost of goods sold
To eliminated intercompany sales

618,000
412,000

100,000
100,000
5,000
5,000
200,000
200,000

(7)

(8)

(9)

(10)

(11)

b.

c.

d.

Investment in Duke
Minority interest in net assets of subsidiary
Cost of goods sold
To eliminate realized profit in beginning inventory Upstream

6,000
4,000
10,000

Cost of goods sold


12,000
Inventory
To eliminate unrealized profit in ending inventory Upstream

12,000

Investment in Duke
Land
To eliminate gain on sale of land Downstream

40,000
40,000

Liabilities
Accounts receivable
To eliminate intercompany debt.

40,000

Minority interest in net income of subsidiary


Minority interest in net assets of subsidiary
(P140,000 + 10,000 P12,000 P5,000) x 40%

53,200

40,000

53,200

Minority Interest in Net Income of Subsidiary


Net income Duke
Realized profit in beginning inventory Upstream
Unrealized profit in ending inventory Upstream
Impairment loss
Adjusted net income Duke
Minority interest
MINIS

P140,000
10,000
( 12,000)
( 5,000)
P133,000
x 40%
P 53,200

Minority Interest in Net Assets of Subsidiary


Stockholders equity, 1/1/08 Duke (P320,000 + P90,000 + 620,000)
Increase in earnings 2008 (P140,000 P60,000)
P80,000
Unrealized profit in ending inventory
(12,000)
Realized profit in beginning inventory
10,000
Goodwill impairment loss
( 5,000)

P1,030,000

73,000

Adjusted net assets, 12/31/08


Minority interest
MINAS

P1,103,000
x 40%
P 441,200

Consolidated Net Income


Net income from own operations Baron (P284,000 P84,000)
Unrealized gain on sale of land
Adjusted net income- Baron
Adjusted net income of Duke (P133,000 x 60%)
Consolidated net income

P 200,000
(10,000)
P 190,000
133,000
P 323,000

Problem 18 12
Pluto Corporation and Subsidiary Star Corporation
Comparative Consolidated Income Statement
Years Ended December 31, 2007 and 2008
.
.
Sales
Cost of goods sold

December 31
2008
P800,000
442,000

2007
P660,000
368,000

.
.
.

Gross profit
Operation expenses
Consolidated net income
Minority interest in net income of subsidiary
Attributable to equity holders of Pluto

358,000
178,000
180,000
10,000
P170,000

292,000
138,000
154,000
10,000
P144,000

.
.
.

Supporting computations:
.
.
Consolidated sales:
Combined sales
Less: intercompany sales
Consolidated sales

P850,000
(50,000)
P800,000

P700,000
(40,000)
P660,000

Consolidated cost of goods sold:


Combined costs of good sold
Intercompany sales
Unrealized profit in ending inventory
Unrealized profit in beginning inventory
Consolidated cost of goods sold

P490,000
(50,000)
10,000
(8,000)
P442,000

P400,000
(40,000)
8,000
P368,000

.
.

Consolidated operating expenses


Combined operating expenses
Realized gain on sale of equipment (P10,000/.2)
Consolidated operating expenses

P180,000
(2,000)
P178,000

P140,000
(2,000)
P138,000

.
.

Minority interest in net income of subsidiary


Star Companys reported net income
Gain on upstream sale of land
Unrealized gain in upstream, inventory sales
Realized net income
Minority interest
Minority interest in net income of subsidiary

P65,000
(5,000)
(10,000)
P50,000
20%
P10,000

P50,000

2008

.
.

2007

.
P50,000
20%
P10,000

CHAPTER 19
Multiple Choice
19-1:

d.
Direct exchange rate:
December 1
December 31
Decrease in forex rate
Forex gain (200,000 yen x P0.08)

19-2:

19-3:

1 2.22 yen =
1 2.70 yen =

P
P
P

0.45
0.37
0.08
16,000

c.
Forex rate, December 1
Forex rate, December 31
Increase in forex rate

Forex gain (1,500,000 yen x P0.02)

d.

.
.

0.45
0.47
0.02
30,000

.
.

September 30:
Forex rate, September 1
Forex rate, September 30
Decrease in forex rate

P
P

Forex gain (200,000 hkg.$ x P0.02)

December 31:
Forex rate, October 1
Forex rate, December 30
Increase in forex rate

Forex loss (200,000 hkg.$ x P0.03)

19-4:

Forex loss on notes payable:


Peso equivalent, September 1, 2004
Peso equivalent, December 31, 2004
Forex loss on principal
Add: Forex loss on interest
Based on P 3,2000,000
Based on P 300,000,000 (P3,000,000x10%x4/12)
Forex loss
Total forex loss (P 8,000 + P220,000)

Forex gain (loss), 2004


Transaction date ($10,000 x P55.5555)
Balance sheet ($10,000 x P 58.8235)
Forex loss (increase)
Forex gain (loss), 2005
Balance sheet date ($10,000 x P58.8235)
Settlement data ($10,000 x P 50.00)
Forex gain (decrease)

(6,000)

P
P

600,000
608,000
(8,000)

P 3,000,000
3,200,000
P (200,000)
P 120,000
100,000

20,000
P (220,000)
P (228,000)

55.5555
58.8235
50.0000

555,555
588,235
P ( 32,680)
P 588,235
500,000
P
88,235

b.
Adjusted value of accounts receivable, 6/30
Peso equivalent, 7/27
Forex loss

19-7:

5.59
5.62
0.03

a.
Direct forex rate Transaction date (P 1 $0.018)
Direct forex rate Balance sheet date (P 1 $0.017)
Direct forex rate Settlement date (P 1 $0.020)

19-6:

4,000

c.
Forex loss on importation of merchandise:
Peso equivalent, January 10, 2004
Peso equivalent, April 20, 2004
Forex loss (increase)

19-5:

5.61
5.59
0.02

P
P

315,000
300,000
(15,000)

a.
2004
Forex rate, 11/5/04
Forex rate, 12/31/04

0.4295
0.4245

Decrease in forex rate


Payable in foreign currency
Forex gain

P
P

2005
Forex rate, 12/31/04
Forex rate, 1/15/05
Decrease in forex rate
Payable in foreign currency
Forex loss

19-8:

a. (1000,000 FC x P 0.85)

19-9:

c. (50,000 FC x P 0.6498)

P
P
P

0.0050
50,000
250
0.4245
0.4345
0.0100
50,000
(500)

19-10: b
Forward rate, 3/31/04
Selling spot rate, 4/30/04
Decrease

0.25
0.22
P

0.03
Forward contract receivable
Forex loss

100,000 FC
3,000

19-11: d. forex gain (loss) on purchase commitments is based on the changes in the forward rates.
Forward rates December 31, 2004
90-day forward rate

.0055
.0055

On December 31, 2004, no changes in forward rates occurred, so no forex gains (losses) are to be
recognized on December 31, 2004 under both transactions.

19-12: b.
Forward contract receivable (P100,000 Baht x P1.650)
Spot rate (100,000 Baht x P1.600)
Forex loss

P 165,000
160,000
P (5,000)

19-13: d.
Import transaction Based on spot rates:
12/31/04: Forex loss [1,000,000 Francs x (P6.01 P6.16)]
Forward Contract Based on forward rates:
12/31/04: Forex gain [1,000,000 Francs x (P6.06 P6.07)]

P (150,000)
P

Net forex loss

10,000

P (140,000)

19-14: b.
12/31/04: Forex gain [$5,000 x (P56.50 P56.60)]
3/31/04 : Forex loss:
Forward contract receivable ($5,000 x P56.60)
Settlement at spot rate ($5,000 x P56.32)
Net forex loss

P
P 283,000
281,600

500
(1,400)

(900)

19-15: a.
Increase in forward rates:
Forward contract receivable, 11/1/04 (10,000 fc x P.78)
Forward contract receivable, 12/31/04 (10,000 fc x P82)
Forex loss

P
P

7,800
8,200
(400)

19-16: b. Increase in forward rates [100,000 x (P.90 P.93)]


19-17: c
Gain from increase in intrinsic value of put option
Loss from decrease in fair value of available for sale securities
Loss from decrease in time value of the option
Net loss on hedging activity 12/31/07

100
(100)
(60)
(60)

19-18: a

19-19: a
12/01/08:
12/31/08:

A$ 70,000/P42,000= 1.667 A$ to P1.00


A$ 70,000/P41,700= 1.679 A$ to P1.00

19-20: a, A$70,000 x P.57 (December 31 forward rate)


19-21: a, The balance will not change, because it is denominated in Philippine peso.
19-22: a
P82,000/KRW 400,000 = P.205
The P82,000 is the amount of the peso payable to bank. This amount is computed
using the forward rate.

Problems
Problem 19-1

Accounts
Receivable
Case 1

NA

Case 2

P 38,000

Accounts
Payable
P 160,000 (a)
NA

Foreign
Currency
Transactions
Exchange Loss

Foreign
Currency
Transactions
Exchange Gain

NA

P 20,000 (b)

NA

P 2,000 (d)

Case 3

NA

P 13,500 (e)

P 1,500 (f)

NA

Case 4

P 6,250 (g)

NA

P 1,250 (h)

NA

(a)
(b)
(c)
(d)
(e)
(f)
(g)
(h)

$40,000 x P4.00
$40,000 x (P4.00 P4.50)
$20,000 x P1.90
$20,000 x (P1.90 P1.80)
$30,000 x P.45
$30,000 x (P.45 P.40)
$2,500,000 x P.0025
$2,500,000 x (P.0025 P.003)

Problem 19-2
a.

May 1

June 20

July 1

August 10

Inventory (or purchases)


Accounts payable
Foreign purchases denominated in
Philippine pesos.

800,000

Accounts payable
Cash
Settlement.

800,000

Accounts receivable
Sales
Foreign sales denominated in
Philippine pesos.

500,000

Cash

500,000

800,000

800,000

500,000

Accounts receivable
Collections.

b.

May 1

June 20

July 1

August 10

500,000

Inventory (or purchases)


800,000
Accounts payable
Foreign purchases denominated in yen:
P800,000 / P.40 = 2,000,000 yen
Foreign currency transaction loss
Accounts payable
P900,000 = 2,000,000 yen x P.45
800,000 = 2,000,000 yen x P.40
P100,000

100,000

Accounts payable
Cash or foreign currency
Settlement denominated in yen.

900,000

100,000

900,000

Accounts receivable
500,000
Sales
Foreign sale denominated in Hongkong $
P500,000 / P5.20 = 96,154 Hkg $
Accounts receivable
Foreign currency transaction gain
P501,924 = 96,154 Hkg. $ x P 5.22
500,000 = 96,154 Hkg. $ x P 5.20
P 1,924
Cash or foreign currency
Accounts receivable
Collections

800,000

500,000

1,924
1,924

501,924
501,924

Problem 19-3
a.

No net exposure between November 1 and March 1. Michael, Inc. has hedged its foreign currency
purchase commitment with a forward contract to receive an equal number of foreign currency
units.

b.

November 1:

December 31:

Forward contract receivable


3,076,800
Forward contract payable
To record forward contract at forward rate:
240,000 Ringgit x P12.82
Forex loss
4,800
Forward contract receivable
To record forex loss for the decrease in
forward rate, P240,000 x P.02

3,076,800

4,800

December 31:

March 1:

Firm commitment for merchandise


4,800
Forex gain
To record increase in fair value of the
Purchase commitment, and resultant
gain or the decrease in the forward rate.

4,800

Forward contract payable


3,076,800
Cash
To record settlement of forward contract.

3,076,800

Cash (240,000 x P12.86)


3,086,400
Forex loss (240,000 x P.02)
4,800
Forward contract receivable
To record receipt of 240,000 Ringgit when
the spot rate is P12.86.

3,091,200

Firm commitment for merchandise


Forex gain
To record change in value of the firm
commitment.

4,800

Purchases (240,000 x P12.82)


3,076,800
Firm commitment for merchandise
Cash
To record purchases of merchandise.

4,800

9,600
3,086,400

Problem 19-4
June 1:

Purchases
460,000
Accounts payable
To record purchases ( 1,000,000 x P.46).
Forward contract receivable (fc)
480,000
Forward contract payable
To record purchase of 1,000,000 for delivery
in 60 days at forward rate of P.48.

June 30:

Forex loss
20,000
Accounts payable
To record forex loss for the increase in spot
rate, 1,000,000 x (P.46 P.48)
Forward contract receivable
20,000
Forex gain
To record forex gain for the increase
in forward rate, 1,000,000 x (P.48 P.50).

August 1:

Accounts payable
Forex loss ( 1,000,000 x P.03)

480,000
30,000

460,000

480,000

20,000

20,000

Cash ( 1,000,000 x P.51)


To record settlement.

510,000

Cash ( 1,000,000 x P.51)


510,000
Forex gain
Forward contract receivable
To record receipt of 1,000,000 at spot rate

10,000
500,000

Forward contract payable


480,000
Cash
To record settlement of forward contract.

480,000

Accounts receivable
1,280,000
Sales
To record sale (100,000 Rial x P12.80).

1,280,000

Problem 19-5
December 1:

Forward contract receivable


1,240,000
Forward contract payable (fc)
To record forward contract to sell 100,000 Rial
at a 90-day forward rate of P12.40.
December 31:

Forex loss
10,000
Accounts receivable
To adjust receivable for the decrease in spot rate
and record forex loss, 100,000 Rial x (P12.80 P 12.70).
Forex loss
20,000
Forward contract payable (FC)
To record forex gain for the increase in forward rate,
100,000 Rial x (P12.40 P12.60).

March 1:

Cash
1,290,000
Forex gain(100,000 Rial x P.20)
Accounts receivable
To record collection of accounts receivable at spot rate.
Forward contract payable (FC)
Forex loss
Cash (100,000 Rial x P12.60)
To record delivery of 100,000 Rial.

1,240,000

10,000

20,000

20,000
1,270,000

1,260,000
30,000
1,290,000

Cash
1,240,000
Forward contract receivable
To record collection for forward contract.

1,240,000

Problem 19-6
October 1:

December 31:

Forward contract receivable


Forward contract payable (fc)
(15,000 Baht x P1.16)

17,400

Forex loss
Forward contract payable (fc)
15,000 Baht x (P1.16 P1.17).

150

17,400

150

Firm commitment for materials


Forex gain
To record increase in fair value of sales
commitment.
April 1:

150
150

Cash
17,400
Forward contract receivable
To record collection of forward contract.
Forward contract payable
17,550
Forex gain
Cash /fc (15,000 Baht x P1.16)
To record delivery of 15,000 Baht at forward rate
of P1.16.
Forex loss
Firm commitment for materials
Cash/fc (15,000 Baht x P1.18)
Sales
To record sales.

17,400

150
17,400

150
150
17,700
17,700

Problem 19-7
Contract 1:
October 1:

December 31:

April 1:

Forward contract receivable (fc)


160,000
Forward contract payable
To record forward contract to buy 400,000 at P40.
Forward contract receivable (fc)
4,000
Forex gain
To record forex gain for the increase in forward
rate of P.01.
Cash ( 400,000 x P.43)
172,000
Forward contract receivable (fc)
Forex gain
To record receipt of 400,000 at spot rate of
P.43.
Forward contract payable
160,000
Cash
To record payment of forward contract.

160,000

4,000

164,000
2,000

160,000

Contract 2:
December 1:

December 31:

March 1:

Forward contract receivable


9,200
Forward contract payable (fc)
To record forward contract to sell 2 million Rupiah
at P.0046.
Forward contract payable
200
Forex gain
To record forex gain for the decrease in forward
Rate by P.0001.
Cash
9,200
Forward contract receivable
To record settlement of forward contract.
Forward contract payable (fc)

9,000

9,200

200

9,200

Forex loss
800
Cash
To record payment of 2 million Rupiah at spot
rate of P.0049.

9,800

Problem 19-8
1.

Investment in Siam
Cash
To record purchase of 40% of Siam Company.

1,920,000
1,920,000

Cash

123,200
Investment in Siam
To record dividends from Siam for 20 x 1 (P308,000 x 40%)

123,200

Investment in Siam
243,200
Other comprehensive income-translation adjustment 128,800
Income from Siam
372,000
To record income from Siam for 20x1 computed as follows:
Share of reported income (P930,000 x 40%)
P 372,000
Share of equity adjustment (P322,000 x 40%)
128,800
2a.

Cash (fc)

1,860,000

Loans payable (fc)


To record loan of 1,200,000 NT dollar at P1.55.
b.

c.

1,860,000

Loan payable (fc)


60,000
Other comprehensive income-translation adjustment
To adjust loan to current rate (P1.55 P1.50) x 1,200,000.
Interest expense
91,500
Interest payable (fc)
Forex gain
To accrue interest expense (1,200,000 x 10% x year x P1.525)
And record interest payable (1,200,000 x 10% x year x P 1.50).

60,000

90,000
1,500

Problem 19-9
1.

Cash (fc)

168,000
Accounts receivable (fc)
Forex gain
To record collection of 100,000 Baht from Queens Company.

167,000
1,000

Forward contract payable (fc)


167,000
Forex loss
1,000
Cash (fc)
To record delivery of 100,000 Baht in settlement of the
forward contract denominated in Baht.

168,000

Cash

164,000

Forward contract receivable


To record receipt of Phil. Pesos in settlement of the
forward contract receivable.
2.

Forward contract payable


Cash
To record payment of forward contract payable.

164,000

76,000
76,000

Cash (fc)
Forex loss

75,000
500

Forward contract receivable (fc)


To record collection of forward contract receivable:
(10,000,000 Rupiah x P.00750)
Accounts payable (fc)
Cash (fc)
Forex gain
To record payment of accounts payable to Indon Co.
(1,000,000 Rupiah x P.00750)

75,500

75,500
75,000
500

Problem 19-10
1.

Schedule of forward contract items at December 31, 2004 balance shee t.


Current assets:
Forward contract receivable (Siam hedge: in Phil. pesos)
Forward contract receivable (Indon hedge: 10,000,000 x P.0077)
Forward contract receivable (Speculation in Yen: 200,000 x P.670)
Change in value of firm commitment

P 168,000
77,000
134,000
1,000

Current liabilities:
Accounts payable (Indon account: 10,000,000 x P.0077)
P 77,000
Forward contract payable (Siam hedge: 100,000 Baht x P1.690)
169,000
Forward contract payable (Speculation in Yen: payable in Phil. pesos)
130,000

2.

Forex gain or loss for 2004:


Indon:
Siam:

P2,000 loss on account payable offset by P2,000 gain on


Forward contract receivable
Forex loss is offset by the change in the value of firm
commitment

Speculation: The speculation is accounted for at the forward rate


throughout the life of the contract. Therefore, the forward
contract receivable is adjusted to P 134,000 (the rate for
60-day futures at December 31 and the P4,000 gain is
recognized).
Forex gain for 2004 in the income statement

4,000
P 4,000

Problem 19-11
a. Entry to record the purchase of the call options on November 30, 2007
November 30, 2007
Call Options
Cash
Purchase call options for 10,000 barrels
of oil at a premium of P2 per barrel for
March 1, 2008. The options are at the money

20,000
20,000

of P30 per barrel; therefore, the entire


P20,000 is time value
b. Adjusting entry on December 31, 2007:
December 31, 2007
Loss on hedge activity
Call options

14,000
14,000

Record the decrease in the time value


of the options to current earnings.
Call options
Other comprehensive income
Record the increase in the intrinsic value
of the options to other comprehensive income.

10,000
10,000

c. Entries to record March 1, 2008, expiration of options, the sales of option, and the purchase
of oil.
March 1, 2008
Loss on hedge activity
Call options
Record the decrease in the time value
of the options to current earnings.
The options have expired.
Call options
Other comprehensive income
Record the increase in the intrinsic value
of the options to other comprehensive income.

6,000

Cash
Call options
Record the sale of the call options.

30,000

Oil inventory
Cash
Record the purchase of 10,000 barrels
of oil at the spot price of P33 per barrel.

330,000

6,000

20,000
20,000

30,000

330,000

d. June 1, 2008, entries to record the sale of the oil and other entries:
June 1, 2008
Cash
Sales
Record the sale of 10,000 barrels
of oil at P34 per barrel

340,000

Cost of goods sold


Oil inventory
Recognize the cost of the oil sold.

330,000

Other comprehensive income- reclassification


Cost of goods sold

30,000

340,000

330,000

30,000

Reclassify into earnings the other


comprehensive income from the cash flow hedge.

CHAPTER 20
MULTIPLE CHOICE
20-1:

b
Bad debt expense (S$ 6,000 x P28.20)
P169,200
Amortization of patents (S$ 4,000 x P28.20)
112,800
Rent expense (S$ 10,000 x P28.20)
282,000
Total
P564,000
Average rate (P28.20) is used to translate all expenses since this is a reasonable
estimation.

20-2:

b
Machinery [(24,000 Ringgit 10) x P10.42]
Equipment [(12,000 Ringgit 10) x P10.42]
Total depreciation

20-3:

d
Accounts receivable
Prepaid expenses
Property and equipment (net)
Total

20-4:

P 25,008
12,504
P 37,512

P120,000
55,000
275,000
P450,000

a
Depreciation expense (H$ 12,000 x P5.80)
Bad debts (H$ 8,000 x P5.80)
Rent (H$ 20,000 x P5.80)
Total

P 69,600
46,400
116,000
P232,000

Average rate for the year is used in translating depreciation expense because this is more
reasonable estimation than the rate when the related asset was acquired (P4.80).
20-5:

d
[25,000 LCU x (1 2)]

20-6:

20-7:

d
Long-term receivable: [1,500,000 LCU x (1 1.5 LCU)]

P1,000,000

Long-term debt: [2,400,000 LCU x (1 1.5 LCU)]

P1,600,000

b (NT Dollar 10,000 x P1.70)

20-8:

20-9:

b
Beginning inventory
Purchases
Goods available for sale
Ending inventory
Cost of goods sold

40,000 Rupee
300,000
340,000
30,000
310,000 Rupee

Translated cost of goods sold (310,000 Rupee x P.5745)

P178,095

c
Net assets, 1/1/05
Increase in net assets:
Net income, 2005 (30,000 20,000)
Net assets 12/31/05
Net assets at current rate
Translation adjustment, 2005 (positive)

NZ Dollar
20,000

Rate
P15

Phil Peso
P300,000

10,000
30,000
30,000

P19

190,000
P490,000
630,000
P140,000

P21

20-10: b
Equipment [800,000 x (1 50)]

P16,000

Accumulated depreciation [560,000 x (1 50)]

P11,200

Depreciation [80,000 x (1 50)]

P 1,600

20-11: a (25,000 Rupee x P1.24)


20-12: d (5,000 Rupee x P1.30)
20-13: c
Investment cost, Jan. 1, 2005
Less: Book and fair value of net assets acquired
(300,000 Rp x P1.20)
Goodwill
Goodwill
Impairment
Balance

Pesos
P42,000
4,340 (3,500 Rp x P1.24)
P37,660

P402,000
360,000
P 42,000
Rupee
35.000 (P42,000 / P1.20)
3,500
31,500

Translated balance (31,500 Rp x P1.32)


Less: umimpaired goodwill
Translation adjustment

P41,580
37,660
P 3,920

20-14: b
Translation adjustment from translating the trial balance
Translation adjustment from translating goodwill (per 20-13)
Total translation adjustment
20-15: b

P12,000 Cr
3,920 Cr
P15,920

Investment in Subsidiary account, Jan. 1, 2005


Share in subsidiary net income [(800,000 yen x 70%) x P.57]
Translation adjustment (P25,000 x 70%)
Share of subsidiary dividends [(50,000 yen x 70%) x P.59]
Investment in Subsidiary account, December 31, 2005

P1,600,000
319,200
17,500
( 20,650)
P1,916,050

20-16: d
20-17: a
Initial inventory transfer date:
Selling price
Cost
Profit
Balance sheet date (75,000 x 1.70)
20-18: a

Phil Peso

Thailand Baht

P120,0001.60
(80,000)
40,000

75,000 B

127,500

75,000 B

(P127,500 40,000)

20-19: a
Net asset beginning
Net income
Net asset translated at rate:
During the year
At end of year

Yen
200,000
200,000
400,000
400,000

Exchange Rate
.44
.46
.48

Translation adjustment (credit)


20-20: a

Phil Peso
88,000
92,000
180,000
192,000
(12,000)

(70,000 rupee x P1.50)

20-21: c
Investment cost
Book value of interest acquired (1,100,000 x 1.10) x .80
Goodwill

P1,210,000
968,000
242,000

PROBLEMS
Problem 20-1
a.
Pilipino Company
Translation Working Paper
December 31, 2005
Cash
Accounts receivable
Inventory
Plant and equipment

Yen
40,000
120,000
100,000
700,000

Exchange Rate
.40 CR
.40 CR
.40 CR
.40 CR

Phil. Pesos
16,000
48,000
40,000
280,000

Cost of sales
Operating expenses
Depreciation expenses
Total
Accumulated Other Comprehensive Income Translation Adjustment
Total debits
Accumulated depreciation
Accounts payable
Common stock
Retained earnings, Jan. 1
Sales
Total credits

360,000
140,000
60,000
1,520,000

.425 AR
.425 AR
.425 AR

153,000
59,500
25,500
622,000
25,000
647,000

240,000
80,000
200,000
400,000
600,000
1,5200,000

.40
.40
.44
.44
.425

CR
CR
HR
HR
AR

P96,000
32,000
88,000
176,000
255,000
647,000

CR Current Rate
AR Average Rate
HR Historical Rate

b. Proof of Translation Adjustment


Yen
Net assets at beginning of year
Adjustment for changes in net assets
Position during year
Net income for the year
Net assets translated at rates in effect
For those items
Net assets at end of year
Change in translation adjustment during year
(to OCI) net decrease (debit)

600,000

Translation
Rate
.44

Phil. Pesos

40,000

.425

17,000

640,000

.40

281,000
256,000

264,000

25,000

Accumulated OCI translation adjustment,1/1

-0-

Accumulated OCI translation adjustment,


Dec. 31 (debit)

25,000

Problem 20-2
(1)

Trial Balance Translation

Cash
Accounts receivable (net)
Receivable from Davao
Inventory
Plant and equipment
Cost of goods sold
Depreciation expense
Operating expenses
Dividends paid
Total debits

Thailand
Baht
7,000
20,000
5,000
25,000
100,000
70,000
10,000
30,000
15,000
282,000

Translation
Rate
1.60 CR
1.60 CR
1.60 CR
1.60 CR
1.60 CR
1.50 AR
1.50 AR
1.50 AR
1.54 HR

Philippine
Pesos
11,200
32,000
8,000
40,000
160,000
105,000
15,000
45,000
23,000
439,300

Accumulated depreciation
Accounts payable
Bonds payable

10,000
12,000
50,000

1.60 CR
1.60 CR
1.80 CR

16,000
19,200
80,000

Common stock
Sales
Total
Accumulated other comprehensive
Income Translation adj. (credit)
Total credits

60,000
150,000
282,.000

1.46 HR
1.50 AR

87,600
225,000
427,800
11,500
439,300

CR Current Rate
AR Average Rate
HR Historical Rate

(2)

Proof of Translation Adjustment

Net assets at beginning of year


Adjustments for changes in net
asset position during year:
Net income for year (sch. 1)
Dividends paid
Net assets translated at:
Rates during year
Rates at end of year
Change in OCI translation adj.
during year Net increase
Accumulated OCI translation
adjustment Jan. 1
Change in OCI translation
adjustment, Dec. 31 (credit)

Thailand
Baht
60,000

Translation
Rate
1.46

Philippine
Pesos
87,600

40,000
(15,000)

1.50
1.54

60,000
(23,100)

85,000

1.60

124,500
136,000
11,500
-011,500

Schedule 1:
Sales
Cost of goods sold
Depreciation expense
Operating expenses
Net income

150,000 Thailand Baht


( 70,000)
( 10,000)
( 30,000)
40,000 Thailand Baht

(b)
The change in the translation adjustment of P11,500 is included as a credit in the other
comprehensive income on the Statement of Comprehensive Income. The other comprehensive
income is then accumulated and reported in the stockholders equity section of the consolidated
balance sheet as presented below:
Net assets

P136,000

Common stock
Retained earnings, Dec. 31
Accumulated Other Comprehensive Income
Total

P 87,600
36,900
11,500
P136,000

Problem 20-3
a.

Translation Work paper


Exchange

Philippine

Brunei $
1.600
2,500
4,000
35,000
17,000
7,000
3,000
1,500
71,600

Cash
Accounts receivable
Inventory
Plant and equipment
Cost of sales
Operating expenses
Depreciation expense
Dividends
Total debits
Accumulated depreciation
Accounts payable
Common stock
Retained earnings, Jan. 1
Sales
Total
Accumulated OCI Translation
Adjustment
Total credits

9,000
2,600
20,000
10,000
30,000
71,600

Rate
33 CR
33 CR
33 CR
33 CR
31 AR
31 AR
31 AR
32 HR

Pesos
52,800
82,500
132,500
1,155,000
527,000
217,000
93,000
48,000
2,307,300

31
33
30
30
31

297,000
85,800
600,000
300,000
930,000
2,212,800

AR
CR
HR
HR
AR

94,500
2,307,300

Proof of Translation Adjustment (not required)


Brunei $
Net assets at beginning of year
Adjustment for net assets position
during the year:
Net income
Dividends paid
Net assets translated at rates
in effect for those items
Net assets at end of year
Change in translation adjustment during
Year to OCI net increase (credit)
Accumulated OCI translation adj. 1/1
Accumulated OCI translation
Adjustment 12/31 (credit)

b.
Jan. 2:

Oct. 15:

Translation
rate

30,000

30

Philippine
Pesos
900,000

3,000
(1,500)

31
32

93,000
(48,000)

31,500

33

945,000
1,039,500
94,500
-094,500

Parent Company entries affecting Investment in Moslem Co. (equity method)


Investment in Moslem Co.
Cash
To record investment cost.
Cash

900,000
900,000
48,000

Investment in Moslem Co.

48,000

To record dividends received


Dec. 31:

Investment in Moslem Co.


Investment income
To record equity in income of Moslem

93,000

Investment in Moslem Co.


Other Comprehensive Income Translation
adjustment
To record parents share of change in translation
Adjustment

94,500

93,000

94,500

Problem 20-4
UK Company
Translation Working Paper
Year Ended December 31, 2005
In Pounds
Income Statement
Sales
Cost of sales
Depreciation expense
Other expenses
Net income carried forward
Retained Earnings Statement
Balance, 1/1
Net income from above
Balance, 12/31
Balance Sheet
Cash
Accounts receivable
Inventories, at cost
Prepaid expenses
Property, plant and equipment (net)
Total assets
Accounts payable
Current portion of long-term debt
Long-term debt
Capital stock
Retained earnings from above
Total
Cumulative translation adjustment:
Balance, 1/1
Current translation adjustment

90,000
(80,000)
(1,500)
(5,750)
2,750

Exchange
Rate
P67.50
67.50
67.50
67.50

2,500
2,750
5,250

In
Phil. Pesos

(A)
(A)
(A)
(A)

6,075,000
(5,400,000)
(101,250)
(388,125)
185,625

B
F

119,500
185,625
305,125

2,500
4,000
5,500
750
9,000
21,750

67.60
67.60
67.60
67.60
67.60

(C)
(C)
(C)
(C)
(C)

169,000
270,400
371,800
50,700
608,400
1,470,300

3,500
500
7,500
5,000
5,250

67.60
67.60
67.60
67.20

(C)
(C)
(C)
(H)

236,600
33,800
507,000
336,000
1,418,525

50,000
1,775

Balance, 12/31
Total liabilities and stockholders equity

51,775
1,470,300

21,750

Translation Code:
C = Current rate
H = Historical rate
A = Average rate
B = Balance in Philippine pesos at the beginning of the year.
F = Per Income Statement

Problem 20-5
Goodluck Corporation
Foreign Exchange Translation Worksheet
Year Ended December 31, 2005

Cash
Marketable securities
Accounts receivable
Inventories
Property, plant and equip-net
Cost of goods sold
Depreciation expense
Other expenses
Totals

Trial
Balance
(In Pounds)
15,000
25,000
60,000
80,000
420,000
150,000
40,000
10,000
800,000

Accounts payable
Current portion of LT debt
Long-term debt
Sales
Other revenues
Capital stock
Retained earnings, 1/1
FC translation adjustment
Balance, 1/1
Current year
Net income

50,000
40,000
120,000
200,000
50,000
250,000
90,000

Totals

800,000

Translation Code:
A = Average rate
B = Current rate
H = Historical rate
G = Given
B = Balancing amount

Problem 20-6

Exchange
Rate
0.95 C
0.95 C
0.95 C
0.95 C
0.95 C
0.90 A
0.90 A
0.90 A
0.95
0.95
0.95
0.90
0.90
0,87
G
G
B
B

C
C
C
A
A
H

Trial
Balance
(In Pesos)
14,250
23,750
57,000
76,000
399,000
135,000
36,000
9,000
750,000
47,500
38,000
114,000
180,000
45,000
217,500
70,000

Income
Statement
(In Pesos)

135,000
36,000
9,000
180,000

570,000
47,500
38,000
114,000

180,000
45,000
217,500
70,000

1,500
36,500
750,000

Balance
Sheet
(In Pesos)
14,250
23,750
57,000
76,000
399,000

(45,000)

1,500
36,500
45,000

180,000

570,000

a. Direct and indirect exchange rates


Direct A$
P.03333=1
P.02857=1
P .025=1

January 1, 2007
December 31, 2007
December 31, 2008

Indirect
A$30=P1
A$35=P1
A$40=P1

The peso strengthened during 2007 because the number of A$ one Phil. Peso could acquire
at the end of the year (35) is greater than the number of A$ that could be acquired at the
beginning of the year (30); therefore, the value of the peso has increased relative to the A$
during 2007. The peso continued to strengthen during 2008.
b. Translated December 31, 2007, balance sheet:
Subsidiarys
Trial Balance
_ (in A$)__
A$ 100,000
400,000
680,000
1,000,000
R 2,230,000

Cash
Receivables
Inventory
Fixed assets
Total
Accumulated other
comprehensive income
translated adjustment (debit)
Total debits

Current payables
Long-term debt
Common stock
Retained earnings
Total credits

A$ 260,000
1,250,000
500,000
220,000
A$2,230,000

Direct
Translated
Exchange
Trial Balance
Rate
( in $)___
P.02857P 2,857
P.02857 12,857
P.02857 19,428
P.02857 28,570
P 63,712

2,903
P 66,615
P.02857P 7,428
P.02857 35,713
P.03333 16,665
P.03333 6,809
P 66,615

P.03333= average of beginning and ending exchange rates, rounded to 4 decimal points:
P.030945= [(P.03333 + P.02856) /2]
(Not required: Proof of translation adjustment (debit) of P 2,903)
___A$___
A$ 500,000

Net assets, 1/1/07


Adjustment for changes in
net assets during year:
Net income
220,000
Net assets translated at:
Rates during year
Rates at end of year
A$ 720,000
Change in translation
Adjustment during year (debit)

Translation
_ Rate_
P.03333
P.03095
P.02857

*Difference of P1 (P 2,904 P 2,903) due to rounding of exchange rates.

_Dollars_
P 16,665
6,809
P 23,474
(20,570)
P 2,904*

c. Translated December 31, 2008, balance sheet:

Cash
Receivables
Inventory
Fixed assets
Accumulated other
comprehensive incometranslation adjustment (debit)
Total debits

Subsidiarys
Trial Balance
(in A$)
A$
80,000
550,000
720,000
900,000
A$ 2,250,000

Direct
Exchange
__Rate
P.025
P.025
P.025
P.025

Translated
Trial Balance
(in P)__
P 2,000
13,750
18,000
22,500__
P56,250
5,635___
P61,885

(a)The retained earnings in pesos would begin with the December 31, 2007, peso
balance (P6,809) that would be carried forward. To this would be added 2008s net income
of A$90,000, which is the change in retained earnings in A$ multiplied by the 2008
exchange rate of P.02679 [(P.02857 + P.025/2)] which equals P2, 411. Therefore, translated
retained earnings on December 31, 2008, is P9, 220 (P9, 220= P6, 809 + P2, 411)
(Not required: Proof of translation adjustment (debit) of P5, 635)
Australian
Dollar
A$ 720,000

Net assets, 1/1/08


Adjustment for changes in
net assets during year:
Net income
90,000
Net assets translated at:
rates during year
Other comprehensive incomerate at end of year
A$ 810,000
Change in other comprehensive
income- translation
adjustment during year (debit)
Accumulated other comprehensive
income- translation adjustment, 1/1/08
Accumulated other comprehensive
income- translation adjustment, 12/31/08 (debit)

Translation
_ Rate
P.02857
P.02679

Pesos___
P20, 570
2,411___
P22, 981

P.025

(20,250)__
P2, 731
2,904___
P5, 635

e. The P2, 731 change in the accumulated other comprehensive income- translation
adjustment during 2008 would be reported as a component of other comprehensive
income on 2008 statement of other comprehensive income.

CHAPTER 21
MULTIPLE CHOICE

21-1

21.2

21.3

21.4

21.5

21.6

21.7

21.8

21.9

21.10

21.11

21.12

21.13

21.14

21.15

a
Excess of income over expenses
Depreciation
Increase in due from national government agencies
Increase in prepaid rent
Increase in accounts payable
Cash provided by operating activities

P 200
70
( 10)
( 15)
30
P 275

PROBLEMS
Problem 21-1
1.

Memo entry in the RAOPS, RAOMO, RAOCO and RAOFE.

2.

Cash National Treasury MDS


Subsidy Income from National Government

3.

2,000,000

Memo entry in the RAOPS, RAOMO, RAOCO and RAOFE.

2,000,000

4.
5.
6.

7.

8.

9.

10.
11.

Office equipment
Accounts payable

50,000

Cash Disbursing Officer


Cash National Treasury MDS

40,000

Salaries and wages Regular


Personal economic relief allowance (PERA)
Additional compensation
Due to BIR
Due to GSIS
Due to Pag-ibig
Due to Philhealth
Cash Disbursing Officer

44,000
3,000
3,000

Due to GSIS
Due to Pag-ibig
Due to Philhealth
Cash National Treasury MDS

5,500
400
600

Life and retirement contribution


Pag-ibig contribution
Philhealth contribution
Cash National Treasury MDS

5,500
400
600

Electricity
Telephone expense Landline
Accounts payable
Due to BIR
Cash National Treasury MDS

5,000
4,000
50,000

Due to BIR
Subsidy income from national government

4,500

Cash Collecting Officer


Sales revenue
Permit fees
Miscellaneous income

12.

Cash in Bank Local currency Current account


Cash Collecting Officer
Problem 21-2

50,000
40,000

3,500
5,500
400
600
40,000

6,500

6,500

5,000
54,000
4,500
90,000
40,000
30,000
20,000
90,000
90,000

Building
1.

Memo entry in RAOCO.

2.

Advances to contractor
Cash National Treasury MDS

240

Construction in progress Other Public Infrastructure


Advances to contractor
Accounts payable

400

Accounts payable
Due to BIR

160

3.

4.

240
240
160
40

Cash National Treasury MDS


5.
6.

7.
8.

120

Construction in progress Other Public Infrastructure


Accounts payable

400

Accounts payable
Due to BIR
Cash National Treasury

400

Due to BIR
Subsidy income from national government
Office Building
Construction in progress OPI

400
40
360
80
80
800
800

Repairs of Building
1.

Memo entry in RAOCO.

2.

Construction materials inventory


Accounts payable

70

Accounts payable
Due to BIR
Cash National Treasury MDS

70

Construction in progress Other Public Infrastructure


Construction materials inventory

60

3.

4.

70
7
63
60

5.

Memo entry in the RAOCO

6.

Cash Disbursing Officer


Cash National Treasury MDS

36

Construction in progress Other Public Infrastructure


Due to BIR
Cash Disbursing Officer

40

Due to BIR
Cash National Treasury MDS

47

7.

8.
9.

Office building
Construction in progress OPI

36
4
36
47
100
100

Land:
1.

Memo entry in the RAOCO, P100.

2.

Land

100
Accounts payable

100

3.

Accounts payable
Due to BIR
Cash National Treasury MDS

100
10
90

Problem 21-3
(a)

Journal Entries:

1.

Memo entry in the Registry of Obligations and Allotments.

2.

Cash National Treasury MDS


Subsidy income from national government

2,500
2,500

3.

Memo entry in Registry of Obligations and Allotments.

4.

Office equipment
Accounts payable

120

IT equipment and software


Accounts payable

30

Prepaid rent
Cash National Treasury MDS

60

Electricity expense
Cash National Treasury MDS

50

Telephone expense Landline


Cash National Treasury MDS

40

5.
6.
7.
8.

120
30
60
50
40

9.
10.

11.
12.
13.

14.

(b)

Petty cash fund


Cash National Treasury MDS

45

Accounts payable
Due to BIR
Cash National Treasury MDS

120

Accounts payable
Cash National Treasury MDS

30

Due to BIR
Subsidy income from national government

12

Cash Collecting Officer


Other service income
Sales revenue

50

Cash in bank LCCA


Cash Collecting Officer

50

45
12
108
30
12
10
40
50

Pre-closing Trial Balance

Petty cash fund


Cash National Treasury MDS
Cash in Bank Local Currency Current Account
Prepaid rent
Office equipment
IT equipment and software
Other service income
Sales revenue
Subsidy income from national government
Electricity expense
Telephone expense landline
Total

45
2,167
50
60
120
30
10
40
2,512
50
40
2,562

2,562

Adjusting Entries
(1)

(2)

Depreciation Office equipment & software


Depreciation IT equipment
Accumulated depreciation Office equipment
Accumulated depreciation IT equip & software

20
5

Rent expense
Prepaid rent

30

20
5
30

Closing Entries:
(1)

Unused National Clearing Account (NCA)


Subsidy income from national government
Cash National Treasury MDS
NCA received during the year
Less: MDS check issued

2,167
2,167
2,500
333

Unused NCA
(2)

(3)

(4)
(5)

Income accounts:
Other service income
Sales revenue
Subsidy income from national government
Income and expense summary
Expense accounts:
Income and expense summary
Electricity expense
Telephone expense landline

2,167
10
40
345
395
90
50
40

Income and expense summary


Retained operating surplus

305

Retained operating surplus

305

305

Government equity

305

Problem 21-4
Agency VV
Statement of Income and Expenses
Year Ended December 31, 2008
Income:
Subsidy income from national government
Less: Reversion of unused NCA

P1,700
800

Less: Expenses
Salaries and wages Regular
Personnel Economic Relief Allowance
Additional compensation
Life and retirement insurance contribution
Pag-ibig contribution
Philhealth contgribution
Traveling expense Local
Office supplies expense
Electricity expense
Telephone expense landline
Janitorial services
Security services
Repairs and maintenance Office building
Depreciation Office building
Depreciation office equipment
Depreciation furniture and fixtures
Depreciation IT equipment and software
Net income over expenses

P 320
40
40
60
10
10
35
60
75
45
30
35
65
15
10
5
5

P900

860
P 40

Agency VV
Balance Sheet
As of December 31, 2008
ASSETS
Current Assets
Cash:
Cash in vault
Cash collecting officer
Cash disbursing officer
Petty cash fund
Cash in bank LCCA
Receivables:
Accounts receivable
Less: Allowance for doubtful accounts
Inventories:
Office supplies inventory
Other current assets
Long-term investment:
Investment in stock
Property, Plant and Equipment:
Land
Office building
Less: accumulated depreciation
Office equipment
Less: accumulated depreciation
Furniture and fixtures
Less: accumulated depreciation
IT equipment and software
Less: accumulated depreciation
Total assets
LIABILITIES AND EQUITY
Liabilities
Current liabilities
Accounts payable
Due to BIR
Due to GSIS
Due to Pag-ibig
Due to Philhealth
Other payables

P 200
500
1,000
150
350

P2,200

P 120
20

100
30
15
400

600
650
50
250
20
110
10
190
25

600
230
100
165

185
50
30
25
25
15

1,695
4,440

330

Equity:
Government equity
Total liabilities and equity

4,110
4,440
CHAPTER 22
Multiple Choice

22-1:

b.

22-2:

d.

(P500,000 P300,000)

The total tuition fees for educational and general purposes.


22-3:

d. (P1,240,000 P160,000)

22-4:

a.
Unrestricted cash contribution received from donors are to be reported as increase in net cash
provided by operation.

22-5:

d.
The remaining contribution of P5,000 on December 31, 2004.

22-6:

b.
Unregistered pledges from donors are treated as revenues at the time of the pledge.

22-7:

d.
Patient revenues
Nursing services
Professional services
Total revenues
Less:
Staff discounts
Allowances
Third party payors
Net revenues

P 5,000,000
1,000,000
500,000
P 6,500,000
P50,000
230,000
800,000

1,080,000
P 5,420,000

Bad debts are treated in the usual manner as expense.


22-8:

a.
As of July 31, 2004, all of the funds are properly includible in the Plan Funds, for a total of
P900,000.

22-9:

c. (P800,000 P110,000)

22-10: d.
Patient revenues (net of charity care)
Less: contractual adjustments
Net patient service revenues
22-11: c.

P 600,000
200,000
P 400,000

22-12: c.
The contributed services are debited to Salary Expense account and credited to Contribution
Revenue account.
22-13: c.
The net effect on unrestricted net assets of spending P10,000 on research is zero.
22-14: b. (P5,000,000 + P50,000)
The P1,000,000 contribution from the donor, who stipulated that the contribution be invested
Indefinitely, should be reported as permanently restricted revenue.
22-15: c.
22-16: b.
Both are treated as a financing activity on the statement of cash flows.
22-17: a.
Cash flows from operating activities would include both the cash received from patient service
Revenue of P300,000 and the cash received from gift shop sales of P25,000.
22-18: b.
Cash received from patient revenue (collection of receivables) and from tuition revenue are both
included in the amount reported for cash flows from operating activities. The other cash receipts
would be reported as increases in cash flows provided by financing activities.
22-19: b.
Expirations of donor restrictions on temporarily restricted net assets should be reported on the
Statement of operations as net assets released from restrictions.
22-20: c.
Current funds revenues include (1) all unrestricted gifts and other unrestricted resources earned
during the reporting period, and (2) restricted current funds to the extent that such funds were
expended for current operating purpose. Therefore, the amount that should be included in current
funds revenue is:
Unrestricted gifts received:
Expended
Not expended
Restricted gifts received
Expended
Total

P600,000
75,000
100,000
P775,000

Problems
Problem 22-1
1.

Pledges receivable
Allowance for uncollectible pledges

300,000
10,000

Contribution revenue
2.

Cash

270,000
260,000

Pledges receivable
3.

4.
5.

260,000

Cash
Fund raising expense
Fund raising revenue

40,000
5,000

Investment
Cash

35,000

Cash

45,000
35,000
5,000

Sales public revenue


6.

7.

Salaries
Employee fringe benefits
Payroll taxes
Supplies
Telephone
Utilities
Rent
Conference, conventions and meetings
Cost of sales to public
Miscellaneous
Cash
Utilities
Salaries

5,000
90,000
15,000
16,000
7,000
1,500
6,000
10,000
5,000
1,000
3,000
154,500
1,000
5,000

Accounts payable or accrued expense payable


8.

Fund Balance - Unrestricted


Fund balance Restricted to purchases
of new equipment.

6,000
10,000
10,000

Problem 22-2
(1)

Accounts receivable
Patient service revenues
To record gross patient service revenue for the month
at full rates.

80,000
80,000

Accounts receivable
Patient service revenues
To record receivable from Social Medicare.

2,500

Contractual adjustments

6,000

2,500

Accounts receivable
To record contractual adjustments allowed.

(2)

(3)

6,000

Doubtful accounts
Allowance for doubtful accounts
To provide allowances for doubtful accounts.

8,000

Salaries expense
Contribution revenues
To record donated services (10,000 200).

9,800

Pledges receivable
Contribution revenues
To record pledges received from donors.

5,000

Cash

3,500

8,000

9,800

5,000

Pledges receivable
To record pledges collected.

3,500

Provisions for doubtful pledges


Allowance for doubtful pledges
To provide doubtful pledges.
(4)

800
800

Cash

3,000

Fund balance
To record receipt of cash from restricted fund.

3,000

Plant assets
Cash
To record acquisition of new surgical equipment.

3,000
3,000

Problem 22-3
Plant Fund Ledger Account:
(1)

(2)

Equipment

50,000
Fund Balance
To record acquisitions of computers from unrestricted fund.
Buildings
Cash
Mortgage notes payable
To record construction of new building financed in
part by 5% mortgage note payable.

Quasi-Endowment Fund Ledger Account:

50,000

2,000,000
250,000
1,750,000

(3)

Cash

110,000

Investments
Payable to Unrestricted fund
To record sale of investments at a gain, the use of
which is unrestricted.

100,000
10,000

Unrestricted Fund Ledger Account:


(1)

(2)

Undesignated fund balance


Cash
To record acquisitions of computers to be carried in
Plant Fund.
Cash

50,000
50,000

2,000,000

Contribution revenues
To record receipt of unrestricted gift.
(3)

2,000,000

Receivable from quasi-endowment fund


Investment income
To record investment gain receivable.

10,000
10,000

Problem 22-4
Nonprofit Trade Association
Statement of Activities
Year Ended June 30, 2004
Revenues and Gains:
Membership dues
Conferences and meetings
Publications and advertising sales
Special assessments
Investment income, including net gains
Total
Expenses:
Member services
Conferences and meetings
Technical services
Communications
General administration
Membership development
Increase in unrestricted net assets
Net assets, beginning of year
Net assets, end of year

P184,000
321,000
143,000
50,000
11,000
P709,000
P 56,000
166,000
218,000
61,000
154,000
27,000

682,000
27,000
285,000
P312,000

Nonprofit Trade Association


Statement of Financial Position
June 30, 2004
ASSETS
Current assets
Cash
P 7,000
Short-term investments
Accounts receivable (net)
Publications inventory
Total current assets
Long-term investments
Plant assets (net)
Other assets
Total assets

217,000
25,000
61,000
310,000
120,000
33,000
28,000
P491,000

LIABILITIES AND NET ASSETS


Current liabilities
Accounts payable and accrued liabilities
Deferred membership dues
Total current liabilities
Net assets (unrestricted)
Total liabilities and net assets

P 48,000
131,000
179,000
312,000
P491,000

Problem 22-5
Children Association
Statement of Activities
Year Ended December 31, 2004
Changes in unrestricted net assets:
Revenues and gains:
Contributions
Membership dues
Program service fees
Investment income
Total unrestricted revenues and gains
Expenses:
Programs
Management and general expenses
Fund raising
Increase in unrestricted net assets
Changes in temporarily restricted net assets:
Contributions
Expenses:
Management and general expenses
Fund raising expenses
Increase in temporarily restricted net assets
Increase in net assets
Net assets, beginning of year (P12,000 + P26,000 + P3,000)
Net assets, end of year

P320,000
25,000
30,000
10,000
P385,000
P270,000
47,000
8,000

325,000
P 60,000
P 15,000

P 4,000
1,000

5,000
P 10,000
P 70,000
41,000
P111,000

Children Association
Statement of Financial Position
December 31, 2004
ASSETS
Cash (P40,000 + P9,000)
Bequest and interest receivable (P5,000 + P1,000)
Pledges receivable (net) (P12,000 P3,000)
Investments, at cost
Total assets
LIABILITIES AND NET ASSETS
Liabilities
Accounts payable and accrued liabilities (P50,000 + P1,000)
Deferred revenues
Total liabilities
Net assets:
Unrestricted (P38,000 + P60,000)
Temporarily restricted (P3,000 + P10,000)
Total net assets
Total liabilities and net assets

P 40,000
6,000
9,000
100,000
P164,000
P 51,000
2,000
P 53,000
P 98,000
13,000
P111,000
P164,000

Problem 22-6
San Pedro Hospital
Statement of Financial Position
June 30, 2004
ASSETS
Current assets
Cash
Accounts receivable (net of allowance of P5,000)
Inventories
Prepaid expenses
Total current assets
Investments
Property, plant and equipment (net of accumulated depreciation of P140,000)
Total assets
LIABILITIES AND NET ASSETS
LIABILITIES
Current liabilities:
Accounts payable
Accrued expenses
Deferred revenues
Current portion of long-term debt
Total current liabilities
Mortgage payable
Total liabilities
NET ASSETS
Unrestricted
Temporarily restricted
Permanently restricted
Total net assets
Total liabilities and net assets

P 222,000
20,000
50,000
10,000
P 302,000
660,000
160,000
P1,122,000

45,000
17,000
11,000
24,000
P 97,000
125,000
P 222,000
P 148,000
232,000
520,000
P 900,000
P1,122,000

PROBLEMS
Problem 17-1
a.

b.

Consolidated Net Income


Net income from own operations P Company
S Co. adjusted net income:
Net income S
Unrealized profit in ending inventory
Upstream (P9,000 x 50/150)
Realized profit in beginning inventoryUpstream (P6,000 x 50/150)
Consolidated net income
Minority Interest in Net Income of Subsidiary
Adjusted net income - S Co.

P200,000
P30,000
(3,000)
2,000

29,000
P229,000
P 29,000

Minority interest
Minority interest in net income of subsidiary

x 30%
P 8,700

Problem 17-2
a.

b.

c.

Consolidated Net Income


Net income from own operations P Co.
Realized profit in beginning inventory Downstream
(P10,500 x 40/140)
Adjusted net income
S Company adjusted net income:
Net income S
Unrealized profit in ending inventoryUpstream (P8,000 x 25%)
Consolidated net income

P100,000
3,000
P103,000
P90,000
(2,000)

88,000
P191,000

Minority Interest in Net Income of Subsidiary


Adjusted net income S Co.
Minority interest
MINIS

P88,000
x 20%
P17,600

Minority Interest in Net Assets of Subsidiary


Stockholders equity , Jan. 1, 2008 S Company
Increase in earnings 2008 (P90,000 P35,000)
Unrealized profit in ending inventory Upstream
Stockholders equity, Dec. 31, 2008 S Company
Minority interest
MINAS

P350,000
55,000
(2,000)
P403,000
x 20%
P 80,600

Problem 17-3
a.

b.

Net Assets, Dec. 31, 2008 S Co.


Minority interest per consolidated balance sheet, 12/31
Unrealized profit in ending inventory Upstream
(P36,000 x 25/125) x 20%
Minority interest per books S Co.
Divided by
Net assets- S Co.
Price Paid
Net assets S Co., Dec. 31, 2008
Net income S Co.
Net assets S Co., Jan. 1, 2008
Parents interest
Book value of interest acquired
Difference
Price paid

Problem 17-4

P158,560
1,440
P160,000
20%
P800,000
P800,000
(160,000)
P640,000
x 80%
P512,000
20,000
P532,000

The computation of the selected consolidation balances are affected by the inter-company profit
in downstream intercompany sales as computed below:
Unrealized profit in ending inventory, Dec. 31, 2007 Downstream
Intercompany profit (P120,000 P72,000)
Inventory left at year end
Unrealized profit, Dec. 31, 20057

P 48,000
x 30%
P 14,400

Unrealized profit in ending inventory, Dec. 31, 2008 Downstream


Intercompany profit (P250,000 P200,000)
Inventory left at year end
Unrealized profit, Dec. 31, 2008

P 50,000
x 20%
P 10,000

a.

b.

c.

Consolidated Sales
Apo
Bicol
Intercompany sales 2008
Total
Cost of goods sold
Apos book value
Bicols book value
Intercompany sales-2008
Realized profit in beginning inventory 2008
Unrealized profit in ending inventory 2008
Consolidated cost of goods sold
Operating expenses
Apo
Bicol
Total

d.

Dividend Income 0 (eliminated)

e.

Minority Interest in Net Income of Subsidiary (P100,000 x 30%)

f.

Inventory
Apo
Bicol
Unrealized profit in ending inventory, Dec. 31, 2008
Consolidated inventory
Minority Interest in Net Assets of Subsidiary
Stockholders equity , Jan. 1, 2008 Bicol
Increase in earnings in 2008 (P100,000 P50,000)
Stockholders equity, Dec. 31, 2008 Bicol
Minority interest
MINAS

g.

P800,000
600,000
(250,000)
P1,150,000
P 535,000
400,000
(250,000)
( 14,400)
10,000
P 680,600
P 100,000
100,000
P 200,000

P 30,000
P 298,000
700,000
(10,000)
P 988,000
P 950,000
50,000
P1,000,000
x 30%
P 300,000

Problem 17-5
P Company and Subsidiary
Consolidated Income Statement
Year Ended December 31, 2008
Sales (P2,000,000 + P1,000,000 P600,000)
Cost of goods sold (Schedule 1)
Gross profit
Expenses

P2,400,000
704,000
1,696,000
600,000

Income before income tax


Provision for income tax
Consolidated net income after income tax
Attributable to minority interest (Schedule 2)
Attributable to parent

1,096,000
440,000
656,000
44,000
P 612,000

Schedule 1:
Cost of sales P Company
Purchases from S Company
Intercompany profit in beginning inventory (P60,000 x 25%)
Intercompany profit in ending inventory (P76,000 x 25%)
Total
Cost of sales S Company
Consolidated cost of sales

P 800,000
(600,000)
( 15,000)
19,000
P 204,000
500,000
P 704,000

Schedule 2:
Net income S Company
Realized profit in beginning inventory Upstream
Unrealized profit in ending inventory Upstream
Adjusted net income
Minority interest
MINIS

P 180,000
15,000
(19,000)
P 176,000
x 25%
P 44,000

Problem 17-6
a.

Working Paper Eliminating Entries


(1)

(2)

(3)

(4)

Dividend income
Minority interest in net assets of subsidiary (20%)
Dividends declared- D (P32,000 / 80%)
To eliminate intercompany dividends.

32,000
8,000
40,000

Common stock S
90,000
Retained earnings S
220,000
Investment in S Co. stock
Minority interest in net assets of subsidiary
To eliminate equity accounts of S on the date of
acquisition.
Minority interest in net assets of subsidiary
Retained earnings, Jan. 1
Cost of goods sold
To eliminate realized profit in beginning inventory

4,000
16,000

Sales

150,000

20,000

Cost of goods sold


Inventory, Dec. 31 (P45,000 x 33.33%)
To eliminated intercompany sales and unrealized
profit in ending inventory.
(5)

Minority interest in net income of subsidiary


Minority interest in net assets of subsidiary
To establish minority interest in net income of S Co.
computed as follows:

248,000
62,000

135,000
15,000

8,000
8,000

Sales
Cost and expenses (P140,000 +P20,000)
Net income
Realized profit in beginning inventory Upstream
Unrealized profit in ending inventory Upstream
Adjusted net income
Minority interest
MINIS
b.

c.

P200,000
160,000
40,000
20,000
(15,000)
P 45,000
x 20%
P 9,000

Consolidated Net Income


Net income from own operations (P250,000 P205,000)
S Company adjusted net income
Consolidated net income

P 45,000
45,000
P 90,000

Minority Interest in Net Assets of Subsidiary (MINAS)


Stockholders equity, Dec. 31, 2008 S Company
Adjusted net income 2008
Adjusted net assets, Dec. 31, 2008 S Co.
Minority interest
MINAS

P 310,000
45,000
P 355,000
x 20%
P 71,000

Problem 17-7
a.

b.

Consolidated Sales
Reported total sales (P600,000 + P510,000)
Intercompany sales (P140,000 + P240,000)
Consolidated sales

P1,170,000
(380,000)
P 790,000

Consolidated Cost of Goods Sold


Cost of goods sold:
Pato (P660,000 / 140%)
Sales (P510,000 / 120%
Amount to be eliminated (P128,000 + P232,000) see entry below
Total

P 471,429
425,000
( 360,000)
P 536,429

Elimination of intercompany sales and intercompany profit in inventory:


Downstream Sales
Sales
Inventory (P42,000 x 40/140)
Cost of goods sold
Upstream Sales
Sales
Inventory (P48,000 x 20/120)
Cost of goods sold
c.

140,000
12,000
128,000
240,000

Consolidated Net Income


Net income from own operations Pato
Unrealized profit in ending inventory Downstream
Adjusted net income Pato
Add: Adjusted net income of Sales Co.
Net income
P20,000
Unrealized profit in ending inventory Upstream
(8,000)
Consolidated net income

8,000
232,000
P 70,000
(12,000)
P 58,000
12,000
P 70,000

d.

Consolidated Inventory, Dec. 31, 2008


Inventory reported Pato
Inventory reported Sales
Unrealized profit in ending inventory (P8,000 + P12,000)
Consolidated inventory

P 48,000
42,000
(20,000)
P 70,000

Problem 17-8
a.

b.

c.

Unrealized Profit in Beginning Inventory


Beginning inventory - Downstream
Gross profit rate (P240,000/ P400,000)
Unrealized profit in beginning inventory

P 100,000
x 60%
P 60,000

Unrealized Profit in Ending Inventory


Ending inventory Downstream (P200,000 x 80%)
Gross profit rate
Unrealized profit in ending inventory

P 160,000
x 60%
P 96,000

Intercompany Sales
Sales P Company
Sales S Company
Intercompany sales 2008
Consolidated sales

P2,000,000
1,000,000
(400,000)
P2,600,000

Intercompany Cost of Sales


Cost of sales P Company
Cost of sales S Company
Intercompany purchases
Intercompany profit in beginning inventory
Intercompany profit in ending inventory
Consolidated cost of sales

P 800,000
600,000
(400,000)
( 60,000)
96,000
P1,036,000

Parents interest (40,000 shares / 50,000 shares)


P Company Entries 2008:
(1)
Investment in S Company stock
Income from subsidiary
To record Ps share of S Co. income
(P120,000 x 80%)
(2)

Cash

80%
96,000
96,000

48,000

Investment in S Company stock


To record dividends received from S
(P60,000 x 80%)
(2)

Income from subsidiary


Investment in S Company stock

48,000

36,000
36,000

To adjust income from subsidiary for intercompany


profit in :
Ending inventory
(96,000)
Beginning inventory
60,000
Net adjustment
( 36,000)

d.

Working Paper Eliminating Entries:

(1)

Income from subsidiary


Minority interest in net assets of subsidiary
(P60,000 x 20%)
Dividends declared S
Investment in S Company stock
To eliminate intercompany dividends.

(2)

(3)

(4)

(5)

(6)

60,000
12,000
60,000
12,000

Common stock S Co.


500,000
Retained earnings S Co.
860,000
Investment in S Company stock
Minority interest in net assets of subsidiary
To eliminate equity accounts of S Company as of
beginning of year.
Goodwill
Investment in S Company stock
To allocate difference to goodwill.

60,000
60,000

Retained earnings Jan. 1


60,000
Cost of sales
To eliminate realized profit in beginning inventoryDownstream.
Cost of sales
96,000
Inventories
To eliminate unrealized profit in ending inventoryDownstream.
Sales

(8)

e.

60,000

96,000

400,000

Cost of sales
To eliminate intercompany sales.
(7)

1,088,000
272,000

Accounts payable
50,000
Accounts receivable
To eliminate intercompany payables and receivables.
Minority interest in net income of subsidiary
24,000
Minority interest in net assets of subsidiary
To establish minority share of S net income
(P120,000 x 20%)

Consolidated Net Income


Net Income from own operations P Company (P480,000 P60,000)
Realized profit in beginning inventory

400,000

50,000

24,000

P420,000
60,000

Unrealized profit in ending inventory


Adjusted net income P Compay
S Company net income
Consolidated net income

( 96,000)
P384,000
120,000
P504,000

Das könnte Ihnen auch gefallen